The Ultimate BMAT Guide

1464
1

Transcript of The Ultimate BMAT Guide

1

2

TABLE OF CONTENTSe Basics

General Advice

SECTION 1

SECTION 1: Critical inking

Critical inking Questions

SECTION 1: Problem Solving Questions

SECTION 1: Data Analysis

Data Analysis Questions

SECTION 2

SECTION 2: Biology

Biology Questions

SECTION 2: Chemistry

Chemistry Questions

SECTION 2: Physics

Physics Questions

SECTION 2: Maths

Maths Questions

SECTION 3

Annotated Essays

3

Mock Paper A

Mock Paper B

Mock Paper C

ANSWERS

Mock Papers Answer Key

Worked Answers

Mock Paper Answers

e

4

5

6

7

8

9

10

11

12

13

14

SECTION 1

THIS IS THE FIRST SECTION of the BMAT and as you walk in, it isinevitable that you will feel nervous. Make sure that you have been to thetoilet because once it starts you cannot simply pause and go. Take a fewdeep breaths and calm yourself down. Remember that panicking will nothelp and may negatively affect your marks- so try and avoid this as muchas possible.

You have one hour to answer 35 questions in section 1. e questionsfall into three categories:- Problem solving- Data handling- Critical thinking

Whilst this section of the BMAT is renowned for being difficult toprepare for, there are powerful shortcuts and techniques that you can useto save valuable time on these types of questions.

You have approximately 100 seconds per question; this may sound likea lot but given that you’re often required to read and analyse passages orgraphs- it can often not be enough. Nevertheless, this section is not astime pressured as section 2 so most students usually nish the majority ofquestions in time. However, some questions in this section are very trickyand can be a big drain on your limited time. e people who fail tocomplete section 1 are those who get bogged down on a particularquestion.

15

erefore, it is vital that you start to get a feel for which questions aregoing to be easy and quick to do and which ones should be left till the end.e best way to do this is through practice and the questions in this bookwill offer extensive opportunities for you to do so.

16

SECTION 1: CRITICAL THINKING

BMAT CRITICAL THINKING QUESTIONS REQUIRE you to understandthe constituents of a good argument and be able to pick them apart. emajority of BMAT Critical thinking questions tend to fall into 3 majorcategories:

1. Identifying Conclusions2. Identifying Assumptions + Flaws3. Strengthening and Weakening arguments

Having a good grasp of language and being able to lter unnecessaryinformation quickly and efficiently is a vital skill in medical school – yousimply do not have the time to sit and read vast numbers of textbookscover to cover, you need to be able to lter the information and realisewhich part is important and this will contribute to your success in yourstudies. Similarly, when you have quali ed and are on the wards, you needto be able to pick out key information from patient notes and makehealthcare decisions from them, so getting to grips with verbal reasoninggoes a long way and do not underestimate its importance.

Only use the PassageYour answer must only be based on the information available in the

passage. Do not try and guess the answer based on your generalknowledge as this can be a trap. For example, if the passage says thatspring is followed by winter, then take this as true even though you knowthat spring is followed by summer.

17

18

Take your timeUnlike the problem solving questions, critical thinking questions are

less time pressured. Most of the passages are well below 300 words andtherefore don’t take long to read and process (unlike the UKCAT in whichyou should skim read passages). us, your aim should be to understandthe intricacies of the passage and identify key information so that youdon’t miss key information and lose easy marks.

Identifying ConclusionsStudents struggle with these type of questions because they confuse a

premise for a conclusion. For clarities sake:- A Conclusion is a summary of the arguments being made and is usuallyexplicitly stated or heavily implied.- A Premise is a statement from which another statement can be inferredor follows as a conclusion.

Hence a conclusion is shown/implied/proven by a premise. Similarly, apremise shows/indicates/establishes a conclusion. Consider for example:My mom, being a woman, is clever as all women are clever.

Premise 1: My mom is a woman + Premise 2: Women are clever =Conclusion: My mom is clever.

is is fairly straightforward as it’s a very short passage and theconclusion is explicitly stated. Sometimes the latter may not happen.Consider: My mom is a woman and all women are clever.

Here, whilst the conclusion is not explicitly being stated, bothpremises still stand and can be used to reach the same conclusion.

You may sometimes be asked to identify if any of the options cannot be“reliably concluded”. is is effectively asking you to identify why anoption cannot be the conclusion. ere are many reasons why but themost common ones are:

1. Over-generalising: My mom is clever therefore all women are clever.2. Being too speci c: All kids like candy thus my son also likes candy.

19

3. Confusing Correlation vs. Causation: Lung cancer is much more likelyin patients who drink water. Hence, water causes lung cancer.

4. Confusing Cause and Effect: Lung cancer patients tend to smoke so itfollows that having lung cancer must make people want to smoke.

Note how conjunctives like hence, thus, therefore and it follows giveyou a clue as to when a conclusion is being stated. More examples of theseinclude: “it follows that, implies that, whence, entails that”.

Similarly, words like “because, as indicated by, in that, given that, dueto the fact that” usually identify premises.

Assumptions + Flaws:Other types of critical thinking questions may require you to identify

assumptions and aws in a passage’s reasoning. Before proceeding it isuseful to de ne both:- An assumption is a reasonable assertion that can be made on the basis ofthe available evidence.- A aw is an element of an argument which is inconsistent to the rest ofthe available evidence. It undermines the crucial components of theoverall argument being made.

Consider for example: My mom is clever because all doctors are clever.Premise 1: Doctors are clever. Assumption: My mom is a doctor.

Conclusion: My mom is clever.Note that the conclusion follows naturally even though there is only

one premise because of the assumption. e argument relies on theassumption to work. us, if you are unsure if an option you have is anassumption or not, just ask yourself:1) Is it in the passage? If the answer is no then proceed to ask:2) Does the conclusion rely on this piece of information in order to work? – Ifthe answer is yes – then you’ve identi ed an assumption.

You may sometimes be asked to identify aws in an argument – it isimportant to be aware of the types of aws to look out for. In general, these

20

are broadly similar to the ones discussed earlier in the conclusion section(over-generalising, being too speci c, confusing cause and effect,confusing correlation and causation). Remember that an assumption mayalso be a aw.

For example consider again: My mom is clever because all doctors areclever.

What if the mother was not actually a doctor? e argument wouldthen breakdown as the assumption would be incorrect or �awed.

Strengthening and Weakening Arguments:You may be asked to identify an answer option that would most

strengthen or weaken the argument being made in the passage. Normally,you’ll also be told to assume that each answer option is true. Before we candiscuss how to strengthen and weaken arguments, it is important tounderstand “what constitutes a good argument:

1. Evidence: Arguments which are heavily based on value judgementsand subjective statements tend to be weaker than those based onfacts, statistics and the available evidence.

2. Logic: A good argument should ow and the constituent partsshould t well into an overriding view or belief.

3. Balance: A good argument must concede that there are other viewsor beliefs (counter-argument). e key is to carefully dismantle theseideas and explain why they are wrong.

us, when asked to strengthen an argument, look for options thatwould: Increase the evidence basis for the argument, support or add apremise, address the counter-arguments.

21

Similarly, when asked to weaken an argument, look for options thatwould: decrease the evidence basis for the argument or create doubt overexisting evidence, undermine a premise, strengthen the counter-arguments.

In order to be able to strengthen or weaken arguments, you mustcompletely understand the passage’s conclusion. en you can starttesting the impact of each answer option on the conclusion to see whichone strengthens or weakens it the most i.e. is the conclusionstronger/weaker if I assume this information to be true and included inthe passage.

Often you’ll have to decide which option strengthens/weakens thepassage most – and there really isn’t an easy way to do this apart from lotsof practice. ankfully, you have plenty of time for these questions.

22

CRITICAL THINKINGQUESTIONS

Question 1-6 are based on the passage below:People have tried to elucidate the differences between the different

genders for many years. Are they societal pressures or genetic differences?In the past it has always been assumed that it was programmed into ourDNA to act in a certain more masculine or feminine way but now evidencehas emerged that may show it is not our genetics that determines the waywe act, but that society pre-programmes us into gender identi cation.Whilst it is generally acknowledged that not all boys and girls are thesame, why is it that most young boys like to play with trucks and diggerswhilst young girls prefer dollies and pink?

e society we live in has always been an important factor in ouridentity, take cultural differences; the language we speak the food we eat,the clothes we wear. All of these factors in uence our identity. Newresearch nds that the people around us may prove to be the biggestin uence on our gender behaviour. It shows our parents buying genderedtoys may have a much bigger in uence than the genes they gave us. Girlsare being programmed to like the same things as their mothers and thishas lasting effects on their personality. Young girls and boys are forced intotheir gender stereotypes through the clothes they are bought, the hairstylethey wear and the toys they play with.

23

e power of society to in uence gender behaviour explains the caseswhere children have been born with different external sex organs to thosethat would match their sex determining chromosomes. Despite thein uence of their DNA they identify to the gender they have always beentold they are. Once the difference has been detected, how then are theyever to feel comfortable in their own skin? e only way to prevent societyhaving such a large in uence on gender identity is to allow children toexpress themselves, wear what they want and play with what they wantwithout fear of not tting in.

Question 1:What is the main conclusion from the rst paragraph?

1. Society controls gender behaviour.2. People are different based on their gender.3. DNA programmes how we act.4. Boys do not like the same things as girls because of their genes.

Question 2:Which of the following, if true, points out the aw in the rst

paragraph’s argument?

1. Not all boys like trucks.2. Genes control the production of hormones.3. Differences in gender may be due to an equal combination of society

and genes.4. Some girls like trucks.

Question 3:According to the statement, how can culture affect identity?

1. Culture can in uence what we wear and how we speak.2. Our parents act the way they do because of culture.

24

3. Culture affects our genetics.4. Culture usually relates to where we live.

Question 4:Which of these is most implied by the statement?

1. Children usually identify with the gender they appear to be.2. Children are programmed to like the things they do by their DNA.3. Girls like dollies and pink because their mothers do.4. It is wrong for boys to have long hair like girls.

Question 5:What does the statement say is the best way to prevent gender

stereotyping?

1. Mothers spending more time with their sons.2. Parents buying gender-neutral clothes for their children.3. Allowing children to act how they want.4. Not telling children if they have different sex organs.

Question 6:What, according to the statement is the biggest problem for children

born with different external sex organs to those which match their sexchromosomes?

1. ey may have other problems with their DNA.2. Society may not accept them for who they are.3. ey may wish to be another gender.4. ey are not the gender they are treated as which can be distressing.

Questions 7-11 are based on the passage below:

25

New evidence has emerged that the most important factor in a child’sdevelopment could be their napping routine. It has come to light thatregular napping could well be the deciding factor for determining toddlers’memory and learning abilities. e new countrywide survey of 1000toddlers, all born in the same year showed around 75% had regular 30-minute naps. Parents cited the bene ts of their child having a regularroutine (including meal times) such as decreased irritability, and statedthe only downfall of occasional problems with sleeping at night. Researchindicating that toddlers were 10% more likely to suffer regular night-timesleeping disturbances when they regularly napped supported the parent’sview.

ose who regularly took 30-minute naps were more than twice aslikely to remember simple words such as those of new toys than their non-napping counterparts, who also had higher incidences of memoryimpairment, behavioural problems and learning difficulties. Toddlers whoregularly had 30 minute naps were tested on whether they were able recallthe names of new objects the following day, compared to a control groupwho did not regularly nap. ese potential links between napping andmemory, behaviour and learning ability provides exciting new evidence inthe eld of child development.

Question 7:If in 100 toddlers 5% who did not nap were able to remember a new

teddy’s name, how many who had napped would be expected toremember?

26

1. 82. 93. 104. 12

27

Question 8:Assuming that the incidence of night-time sleeping disturbances is the

same in for all toddlers independent of all characteristics other thannapping, what is the percentage of toddlers who suffer regular night-timesleeping disturbances as a result of napping?

28

1. 7.5%2. 10%3. 14%4. 20%5. 50%

29

Question 9:Using the information from the passage above, which of the following

is the most plausible alternative reason for the link between memory andnapping?

1. Children who have bad memory abilities are also likely to havetrouble sleeping.

2. Children who regularly nap, are born with better memories.3. Children who do not nap were unable to concentrate on the memory

testing exercises for the study.4. Parents who enforce a routine of napping are more likely to conduct

memory exercises with their children.

Question 10:Which of the following is most strongly indicated?

30

1. Families have more enjoyable meal times when their toddlersregularly nap.

2. Toddlers have better routines when they nap.3. Parents enforce napping to improve their toddlers’ memory ability.4. Napping is important for parents’ routines.

31

Question 11:Which of the following, if true, would strengthen the conclusion that

there is a causal link between regular napping and improved memory intoddlers?

1. Improved memory is also associated with regular mealtimes.2. Parents who enforce regular napping are more inclined to include

their children in studies.3. Toddlers’ memory development is so rapid that even a few weeks can

make a difference to performance.4. Among toddler playgroups where napping incidence is higher and

more consistent memory performance is signi cantly improvedcompared to those that do not.

Question 12:Tom’s father says to him: ‘You must work for your A-levels. at is the

best way to do well in your A-level exams. If you work especially hard forGeography, you will de nitely succeed in your Geography A-level exam’.

Which of the following is the best statement Tom could say to prove aaw in his father’s argument?

1. ‘It takes me longer to study for my History exam, so I shouldprioritise that.’

2. ‘I do not have to work hard to do well in my Geography A-level.’3. ‘Just because I work hard, does not mean I will do well in my A-levels.’4. ‘You are putting too much importance on studying for A-levels.’5. ‘You haven’t accounted for the fact that Geography is harder than my

other subjects.’

Question 13:Today the NHS is increasingly struggling to be nancially viable. In the

future, the NHS may have to reduce the services it cannot afford. e NHS

32

is supported by government funds, which come from those who pay tax inthe UK. Recently the NHS has been criticised for allowing fertilitytreatments to be free, as many people believe these are not important andshould not be paid for when there is not enough money to pay the doctorsand nurses.

Which of the following is the most accurate conclusion of thestatement above?

1. Only taxpayers should decide where the NHS spends its money.2. Doctors and nurses should be better paid.3. e NHS should stop free fertility treatments.4. Fertility treatments may have to be cut if nances do not improve.

Question 14:‘We should allow people to drive as fast as they want. By allowing

drivers to drive at fast speeds, through natural selection the mostdangerous drivers will kill only themselves in car accidents. ese peoplewill not have children, hence only safe people will reproduce andeventually the population will only consist of safe drivers.’

Which one of the following, if true, most weakens the aboveargument?

1. Dangerous drivers harm others more often than themselves bydriving too fast.

2. Dangerous drivers may produce children who are safe drivers.3. e process of natural selection takes a long time.4. Some drivers break speed limits anyway.

Question 15:In the winter of 2014 the UK suffered record levels of rainfall, which led

to catastrophic damage across the country. ousands of homes weredamaged and even destroyed, leaving many homeless in the chaos that

33

followed. e Government faced harsh criticism that they had failed toadequately prepare the country for the extreme weather. In such cases theGovernment assess the likelihood of such events happening in the futureand balance against the cost of advance measures to reduce the impactshould they occur versus the cost of the event with no preparativedefences in place. Until recently, for example, the risk of acts of terrortaking was low compared with the vast cost anticipated should they occur.However, the risk of ooding is usually low, so it could be argued that thecosts associated with anti- ooding measures would have been pre-emptively unreasonable. Should the Government be expected to preparefor every conceivable threat that could come to pass? Are we to put inplace expensive measures against a seismic event as well as a possibleextra-terrestrial invasion?

Which of the following best expresses the main conclusion of thestatement above?

1. e Government has an obligation to assess risks and costs ofpossible future events.

2. e Government should spend money to protect against potentialextra-terrestrial invasions and seismic events.

3. e Government should have spent money to protect againstpotential oods.

4. e Government was justi ed in not spending heavily to protectagainst ooding.

5. e Government should assist people who lost their homes in theoods.

Question 16:Sadly the way in which children interact with each other has changed

over the years. Where once children used to play sports and gamestogether in the street, they now sit alone in their rooms on the computer

34

playing games on the Internet. Where in the past young children learnedhuman interaction from active games with their friends this is no longerthe case. How then, when these children are grown up, will they be able tosocially interact with their colleagues?

Which one of the following is the conclusion of the above statement?

1. Children who play computer games now interact less outside ofthem.

2. e Internet can be a tool for teaching social skills.3. Computer games are for social development.4. Children should be made to play outside with their friends to develop

their social skills for later in life.5. Adults will in the future play computer games as a means of

interaction.

Question 17:Between 2006 and 2013 the British government spent £473 million on

Tami u antiviral drugs in preparation for a u pandemic, despite therebeing little evidence to support the effectiveness of the drug. e antiviralswere stockpiled for a u pandemic that never fully materialised. Only150,000 packs were used during the swine u episode in 2009, and it isunclear if this improved outcomes. erefore this money could have beenmuch better spent on drugs that would actually bene t patients.

Which option best summarises the author’s view in the passage?

1. Drugs should never be stockpiled, as they may not be used.2. Spending millions of pounds on drugs should be justi ed by strong

evidence showing positive effects.3. We should not prepare for u pandemics in the future.4. e recipients of Tami u in the swine u pandemic had no difference

in symptoms or outcomes to patients who did not receive the

35

antivirals.

Question 18:High BMI and particularly central weight are risk factors associated

with increased morbidity and mortality. Many believe the development ofcheap, easily accessible fast-food outlets is partly responsible for theincrease in rates of obesity. An unhealthy weight is commonly associatedwith a generally unhealthy lifestyle, such a lack of exercise. e best way totackle the growing problem of obesity is for the government to taxunhealthy foods so they are no longer a cheap alternative.

Why is the solution given, to tax unhealthy foods, not a logicalconclusion from the passage?

1. Unhealthy eating is not exclusively con ned to low-income families.2. A more general approach to unhealthy lifestyles would be optimal.3. People do not only choose to eat unhealthy food because it is

cheaper.4. People need to take personal responsibility for their own health.

Question 19:As people are living longer, care in old age is becoming a larger burden.

Many people require carers to come into their home numerous times a dayor need full residential care. It is not right that the NHS should bespending vast funds on the care of people who are sufficiently wealthy tofund their own care. Some argue that they want their savings kept to giveto their children; however this is not a right, simply a luxury. It is not rightthat people should be saving and depriving themselves of necessary care,or worse, making the NHS pay the bill, so they have money to pass on totheir offspring. People need to realise that there is a nancial cost to livinglonger.

36

Which of the following statements is the main conclusion of the abovepassage?

1. We need to take a personal responsibility for our care in old age.2. Caring for the elderly is a signi cant burden on the NHS.3. e reason people are reluctant to pay for their own care is that they

want to pass money onto their offspring.4. e NHS should limit care to the elderly to reduce their costs.5. People shouldn’t save their money for old age.

Question 20:ere is much interest in research surrounding production of human

stem cells from non-embryo sources for potential regenerative medicine,and a huge nancial and personal gain at stake. In January 2014, a teamfrom Japan published two papers in Nature that claimed to have developedtotipotent stem cells from adult mouse cells by exposure to an acidicenvironment. However, there has since been much controversysurrounding these papers. Problems included: inability by other teams toreplicate the results of the experiment, an insufficient protocol describedin the paper and issues with images in one of the papers. It was dishonestof the researchers to publish the papers with such problems, and arequirement of a paper is a sufficiently detailed protocol, so that anothergroup could replicate the experiment.

Which statement is most implied?

1. Research is fuelled mainly by nancial and personal gains.2. e researchers should take responsibility for publishing the paper

with such aws.3. Rivalry between different research groups makes premature

publishing more likely.

37

4. e discrepancies were in only one of the papers published inJanuary 2014.

Question 21:e placebo effect is a well-documented medical phenomenon in

which a patient’s condition undergoes improvement after being given anineffectual treatment that they believe to be a genuine treatment. It isfrequently used as a control during trials of new drugs/procedures, withthe effect of the drug being compared to the effect of a placebo, and if thedrug does not have a greater effect than the placebo, then it is classed asineffective. However, this analysis discounts the fact that the drugtreatment still has more of a positive effect than no action, and so we areclearly missing out on the potential to improve certain patient conditions.It follows that where there is a demonstrated placebo effect, buttreatments are ineffective, we should still give treatments, as there willtherefore be some bene t to the patient.

Which of the following best expresses the main conclusion of thispassage?

1. In situations where drugs are no more effective than a placebo, weshould still give drugs, as they will be more effective than not takingaction.

2. Our current analysis discounts the fact that even if drug treatmentshave no more effect than a placebo, they may still be more effectivethan no action.

3. e placebo effect is a well-recognised medical phenomenon.4. Drug treatments may have negative side effects that outweigh their

bene t to patients.5. Placebos are better than modern drugs.

Question 22:

38

e speed limit on motorways and dual carriageways has been 70mphsince 1965, but this is an out-dated policy and needs to change. Since 1965,car brakes have become much more effective, and many safety featureshave been introduced into cars, such as seatbelts (which are nowcompulsory to wear), crumple zones and airbags. erefore, it is clear thatcars no longer need to be restricted to 70mph, and the speed limit can besafely increased to 80mph without causing more road fatalities.

Which of the following best illustrates an assumption in this passage?

1. e government should increase the speed limit to 80mph.2. If the speed limit were increased to 80mph, drivers would not begin

to drive at 90mph.3. e safety systems introduced reduce the chances of fatal road

accidents for cars travelling at higher speeds.4. e roads have not become busier since the 70mph speed limit was

introduced.5. e public want the speed limit to increase.

Question 23:Despite the overwhelming scienti c proof of the theory of evolution,

and even acceptance of the theory by many high-ranking religiousministers, there are still sections of many major religions that do notaccept evolution as true. One of the most prominent of these in westernsociety is the Intelligent Design movement, which promotes the religious-based (and scienti cally discredited) notion of Intelligent Design as ascienti c theory. Intelligent Design proponents often point to complexissues of biology as proof that god is behind the design of human beings,much as a watchmaker is inherent in the design of a watch.

One part of anatomy that has been identi ed as supposedlysupporting Intelligent Design is ngerprints, with some proponentsarguing that they are a mark of individualism created by God, with no

39

apparent function except to identify each human being as unique. is isincorrect, as ngerprints do have a well documented function – namelychannelling away of water to improve grip in wet conditions – in whichhairless, smooth skinned hands otherwise struggle to grip smooth objects.e individualism of ngerprints is accounted for by the complexity ofthousands of small grooves. Development is inherently affected bystochastic or random processes, meaning that the body is unable touniformly control its development to ensure that ngerprints are the samein each human being. Clearly, the presence of individual ngerprints doesnothing to support the so-called-theory of Intelligent Design.

Which of the following best illustrates the main conclusion of thispassage?

1. Fingerprints have a well-established function.2. Evolution is supported by overwhelming scienti c proof.3. Fingerprints do not offer any support to the notion of Intelligent

Design.4. e individual nature of ngerprints is explained by stochastic

processes inherent in development that the body cannot uniformlycontrol.

5. Intelligent design is a credible and scienti cally rigorous theory.

Question 24:High levels of alcohol consumption are proven to increase the risk of

many non-infectious diseases, such as cancer, atherosclerosis and liverfailure. James is a PhD student, and is analysing the data from a large-scalestudy of over 500,000 people to further investigate the link between heavyalcohol consumption and health problems. In the study, participants wereasked about their alcohol consumption, and then their medical historywas recorded. His analysis displays surprising results, concluding thatthose with high alcohol consumption have a decreased risk of cancer.

40

James decides that those carrying out the study must have incorrectlyrecorded the data.

Which of the following is NOT a potential reason why the study hasproduced these surprising results?

1. Previous studies were incorrect, and high alcohol consumption doeslower the risk of cancer.

2. e studies didn’t take account of other cancer risk factors incomparing those with high and low alcohol consumption.

3. James has made some errors in his analysis, and thus his conclusionsare erroneous.

4. e participants involved in the study did not truthfully report theiralcohol consumption, leading to false conclusions being drawn.

5. e studies control group data was mixed up with the test groupdata.

Question 25:A train is scheduled to depart from Newcastle at 3:30pm. It stops at

Durham, Darlington, York, Sheffield, Peterborough and Stevenage beforearriving at Kings Cross station in London, where the train completes itsjourney. e total length of the journey between Newcastle and KingsCross was 230 miles, and the average speed of the train during the journey(including time spent stood still at calling stations) is 115mph. erefore,the train will complete its journey at 5:30pm.

Which of the following is an assumption made in this passage?

1. e various stopping points did not increase the time taken tocomplete the journey.

2. e train left Newcastle on time.3. e train travelled by the most direct route available.4. e train was due to end its journey at Kings Cross.

41

5. ere were no signalling problems encountered on the journey.

Question 26:ere have been many arguments over the last couple of decades

about government expenditure on healthcare in the various devolvedregions of the UK. It is often argued that, since spending on healthcare perperson is higher in Scotland than in England, that therefore the people inScotland will be healthier. However, this view fails to take account of thedifferent needs of these 2 populations of the UK. For example, one majorfactor is that Scotland gets signi cantly colder than England, and coldweakens the immune system, leaving people in Scotland at much higherrisk of infectious disease. us, Scotland requires higher levels ofhealthcare spending per person simply to maintain the health of thepopulace at a similar level to that of England.

Which of the following is a conclusion that can be drawn from thispassage?

1. e higher healthcare spending per person in Scotland does notnecessarily mean people living in Scotland are healthier.

2. Healthcare spending should be increased across the UK.3. Wales requires more healthcare spending per person simply to

maintain population health at a similar level to England.4. It is unfair on England that there is more spending on healthcare per

person in Scotland.5. Scotland’s healthcare budget is a controversial topic.

Question 27:Vaccinations have been hugely successful in reducing the incidence of

several diseases throughout the 20th century. One of the most spectacularachievements was arguably the global eradication of Smallpox, once adeadly worldwide killer, during the 1970s. Fortunately, there was a highly

42

effective vaccine available for Smallpox, and a major factor in itseradication was an aggressive vaccination campaign. Another disease thatis potentially eradicable is Polio. However, although there is a highlyeffective vaccine for Polio available, attempts to eradicate it have so farbeen unsuccessful. It follows that we should plan and execute anaggressive vaccination campaign for Polio, in order to ensure that thisdisease too is eradicated.

Which of the following is the main conclusion of this passage?

1. Polio is a potentially eradicable disease.2. An aggressive vaccination campaign was a major factor in the

eradication of smallpox.3. Both Polio and smallpox have been eradicated by effective

vaccination campaigns.4. We should execute an aggressive vaccination campaign for Polio.5. e eradication of smallpox remains one of the most spectacular

achievements of medical science.

Question 28:e Y chromosome is one of 2 sex chromosomes found in the human

genome, the other being the X chromosome. As the Y chromosome is onlyfound in males, it can only be passed from father to son. Additionally, the Ychromosome does not exchange sections with other chromosomes (ashappens with most chromosomes), meaning it is passed on virtuallyunchanged through the generations. All of this makes the Y chromosome afantastic tool for genetic analysis, both to identify individual lineages andto investigate historic population movements. One famous achievement ofgenetic research using the Y chromosome provides further evidence of itsutility, namely the identi cation of Genghis Khan as a descendant of up to8% of males in 16 populations across Asia.

43

Which of the following best illustrates the main conclusion of thispassage?

1. e Y chromosome is a fantastic tool for genetic analysis.2. Research using the Y chromosome has been able to identify Genghis

Khan as the descendant of up to 8% of men in many Asianpopulations.

3. e Y chromosome does not exchange sections with otherchromosomes.

4. e Y chromosome is a sex chromosome.5. Genghis Khan had a staggering number of children.

Question 29:In order for a bacterial infection to be cleared, a patient must be

treated with antibiotics. Rachel has a minor lung infection, which isthought by her doctor to be a bacterial infection. She is treated withantibiotics, but her condition does not improve. erefore, it must not be abacterial infection.

Which of the following best illustrates a aw in this reasoning?

1. It assumes that a bacterial infection would de nitely improve aftertreatment with antibiotics.

2. It ignores the other potential issues that could be treated byantibiotics.

3. It assumes that antibiotics are necessary to treat bacterial infections.4. It ignores the actions of the immune system, which may be sufficient

to clear the infection regardless of what has caused it.5. It assumes that antibiotics are the only option to treat a bacterial

infection.

Question 30:

44

e link between smoking and lung cancer has been well establishedfor many decades by overwhelming numbers of studies and conclusiveresearch. e answer is clear and simple, that the single best measure thatcan be taken to avoid lung cancer is to not smoke, or to stop smoking ifone has already started. However, despite the overwhelming evidence andclear answers, many smokers continue to smoke, and seek to minimisetheir risk of lung cancer by focusing on other, less important risk factors,such as exercise and healthy eating. is approach is obviously severely

awed, and the fact that some smokers feel this is a good way to reducetheir risk of lung cancer shows that they are delusional.

Which of the following best illustrates the main conclusion of thispassage?

1. Many smokers ignore the largest risk factor, and focus on improvingless important risk factors by eating healthily and exercising.

2. Some smokers are delusional.3. e biggest risk factor of lung cancer is smoking.4. Overwhelming studies have proven the link between smoking and

lung cancer.5. e government should ban smoking in order to reduce the

incidence of lung cancer.

Question 31:e government should invest more money into outreach schemes in

order to encourage more people to go to university. ese schemes allowstudents to meet other people who went to university, which they may notalways be able to do otherwise, even on open days.

Which of the following is the best conclusion of the above argument?

1. Outreach schemes are the best way to encourage people to go touniversity.

45

2. People will not go to university without seeing it rst.3. e government wants more people to go to university.4. Meeting people who went to a university is a more effective method

than university open days.5. It is easier to meet people on outreach schemes than on open days.

Question 32:e illegal drug cannabis was recently upgraded from a class C drug to

class B, which means it will be taken less in the UK, because people willknow it is more dangerous. It also means if people are caught, possessingthe drug they will face a longer prison sentence than before, which willalso discourage its use.

Which TWO statements if true, most weaken the above argument?

1. Class C drugs are cheaper than class B drugs.2. Upgrading drugs in other countries has not reduced their use.3. People who take illegal drugs do not know what class they are.4. Cannabis was not the only class C drug before it was upgraded.5. Even if they are caught possessing class B drugs, people do not think

they will go to prison.

Question 33:Schools with better sports programmes such as well-performing

football and netball teams tend to have better academic results, lessbullying and have overall happier students. us, if we want schools tohave the best results, reduce bullying and increase student happiness,teachers should start more sports clubs.

Which one of the following best demonstrates a aw in the aboveargument?

1. Teachers may be too busy to start sports clubs.

46

2. Better academic results may be a precondition of better sportsteams.

3. Better sports programmes may prevent students from spending timewith their family.

4. Some sports teams may be seen to encourage internal bullying.5. Sport teams that do not perform well lead to increase bulling.

Question 34:e legal age for purchasing alcohol in the UK is 18. is should be

lowered to 16 because the majority of 16 year olds drink alcohol anywaywithout any fear of repercussions. Even if the police catch a 16-year-oldbuying alcohol, they are unable to enforce any consequences. If thedrinking limit was lowered the police could spend less time trying to catchunderage drinkers and deal with other more important crimes. ere is noevidence to suggest that drinking alcohol at 16 is any more dangerousthan at 18.

Which one of the following, if true, most weakens the aboveargument?

1. Most 16 year olds do not drink alcohol.2. If the legal drinking age were lowered to 16, more 15 year olds would

start purchasing alcohol.3. Most 16 year olds do not have enough money to buy alcohol.4. Most 16 year olds are able to purchase alcohol currently.

Question 35:ere has been a recent change in the way the government helps small

businesses. Whilst previously small businesses were given non-repayablegrants to help them grow their pro ts, they can now only receivegovernment loans that must be repaid with interest when the businessturns a certain amount of pro t. e government wants to support small

47

businesses but studies have shown they are less likely to prosper under thenew scheme as they have been deterred from taking government moneyfor fear of loan repayments.

Which one of the following can be concluded from the passage above?

1. Small businesses do not want government money.2. e government cannot afford to give out grants to small businesses

anymore.3. All businesses avoid accumulating debt.4. e action of the government is more likely to do more harm than

good to small businesses.5. Big businesses do not need government money.

Questions 36-41 are based on the passage below:Despite the numerous safety measures in place within the practice of

medicine, these can fail when the weaknesses in the layers of defencealigns to create a clear path leading to often disastrous results. is isknown as the ‘Swiss cheese model of accident causation’. One suchoccurrence occurred where the wrong kidney was removed from a patientdue to a failure in the line of defences designed to prevent such an incidentoccurring.

When a kidney is diseased it is removed to prevent furthercomplications, this operation, a ‘nephrectomy’, is regularly performed byexperienced surgeons. Where normally the consultant who knew thepatient would have conducted the procedure, in this case he passed theresponsibility to his registrar, who was also well experienced but had notmet the patient previously. e person who had copied out the patient’snotes had poor handwriting had accidentally written the ‘R’ for ‘right’ insuch a way that it was read as an ‘L’ and subsequently copied, and notnoticed by anyone who further reviewed the notes.

48

e patient had been put asleep before the registrar had arrived and sohe proceeded without checking the procedure with the patient, as henormally would have done. e nurses present noticed this error but saidnothing, fearing repercussions for questioning a senior professional. Amedical student was present whom, having met the patient previously inclinical, tried to alert the registrar to the mistake he was about to make.e registrar shouted at the student that she should not interrupt surgery;she did not know what she was talking about and asked her to leave.Consequently the surgery proceeded with the end result being that thepatient’s healthy left kidney was removed, leaving them with only theirdiseased right kidney, which would eventually lead to the patient’sunfortunate death. Frightening as these cases appear what is perhapsscarier is the thought of how those reported may be just the ‘tip of theiceberg’.

When questioned about his action to allow his registrar to perform thesurgery alone, the consultant had said that it was normal to allow capableregistrars to do this. ‘While the public perception is that medicalknowledge steadily increases over time, this is not the case with manydoctors reaching their peak in the middle of their careers.’ He had foundthat his initial increasing interest in surgery had enhanced his abilities, butwith time and practice the similar surgeries had become less exciting andso his lack of interest had correlated with worsening outcomes, thusjustifying his decision to devolve responsibility in this case.

Question 36:Which of the following, if true, most weakens the argument above?

1. If incidences are severe enough to occur they will be reported.2. Doctors undergo extensive training to reduce risks.3. ousand of operations happen every year with no problems.4. Some errors are unavoidable.

49

5. e patient could have passed away even if the operation had been acomplete success.

Question 37:Which one of the following is the overall conclusion of the statement?

1. e error that occurred was a result of the failure of safetyprecautions in place.

2. Surgeries should only be performed by surgeons who know theirpatients well.

3. e human element to medicine means errors will always occur.4. e safety procedures surrounding surgical procedures need to be

reviewed.5. Some doctors are overcon dent.

Question 38:Which of the following is attributed as the original cause of the error?

50

1. e medical student not having asserted herself.2. e poor handwriting in the chart.3. e hierarchical system of medicine.4. e registrar not having met the patient.5. e patient being asleep.6. e lack of the surgical skill possessed by the registrar.7. e registrar’s poor attitude.

51

Question 39:What does the ‘tip of the iceberg’ refer to in the passage?

52

1. Problems we face every day.2. e probable large numbers of medical errors that go unreported.3. e difficulties of surgery.4. Reported medical errors.5. Problems within the NHS.

53

You may use the graphs below once, more than once, or not at all.Question 40:Which graph best describes the consultants’ performance versus

emotional arousal over his career?

54

1. A2. B3. C4. D5. E6. F

55

Question 41:Which graphs best describe the medical knowledge acquired over

time?

Question 42:Sadly, in recent times, the lack of exercise associated with sedentary

lifestyles has increased in the developed world. e lack of opportunity forexercise is endemic and these countries have also seen a rise of diseasessuch as diabetes even in young people. In these developed countries,bodily changes such as increased blood pressure, that are usuallyassociated with old age, are rapidly increasing. ese are however stilluncommon in undeveloped countries, where most people are physicallyactive throughout the entirety of their lives.

Which one of the following can be concluded from the passage above?

1. Exercise has a greater effect on old people than young people.2. Maintenance of good health is associated with lifelong exercise.3. Changes in lifestyle will be necessary to cause increased life

expectancies in developed countries.4. Exercise is only bene cial when continued into old age.5. Obesity and diabetes are the result of lack of exercise.

56

Questions 43 - 45 are based on the passage below:‘Midwives should now encourage women to, as often as possible, give

birth at home. Not only is there evidence to suggest that normal births athome are as safe those as in hospital, but it removes the medicalisation ofchildbirth that emerged over the years. With the increase in availability ofhealth resources we now, too often, use services such as a full medicalteam for a process that women have been completing single-handedly forthousands of years. Midwives are extensively trained to assist womenduring labour at home and capable enough to assess when there is aproblem that requires a hospital environment. Expensive hospital birthsmust and should move away from being standard practice, especially in anera where the NHS has far more demands on its services that it cancurrently afford.’

Question 43:Which one of the following is the most appropriate conclusion from

the statement?

1. People are over dependent on healthcare.2. Some women prefer to have their babies in hospital.3. Having a baby in hospital can actually be more risky than at home.4. Childbirth has been over medicalised.5. Encouraging women to have their babies at home may relieve some

of the nancial pressures on the NHS.6. We should have more midwives than doctors.

Question 44:Which one of the following if true most weakens the argument

presented in the passage above?

1. Some women are scared of home births.2. Home births are associated with poorer outcomes.

57

3. Midwives do not like performing home visits.4. Some home births result in hospital births anyway.

Question 45:Which one of the following describes what the statement cites as the

cause for the ‘medicalisation of childbirth’?

1. Women fear giving birth without a full medical team present.2. Midwives are incapable of aiding childbirth without help.3. Giving birth at home is not as safe as it used to be.4. Excessive availability of health services.5. Women only used to give birth at home because they could not do so

at hospital.

Question 46:We need to stop focussing so much attention on the dangers of res. In

2011 there were only 242 deaths due to exposure to smoke, re and ames,while there were 997 deaths from hernias. We need to think moreproportionally as these statistics show that campaigns such as ‘ re kills’are not necessary as comparison with the risk from the death from herniasclearly shows that res are not as dangerous as they are perceived to be.

Which of the following statements identify a weakness in the aboveargument?

1. More people may die in res if there were no campaigns about theirdanger and how to prevent them.

2. e smoke of a re is more dangerous than it ames.3. ere may be more people with hernias than those in res.4. 1 only5. 2 only6. 3 only7. 1 and 2 only

58

8. 1 and 3 only9. 2 and 3 only

10. 1, 2 and 3

Question 47:A survey of a school was taken to nd out whether there was any

correlation between the sports students played and the subjects they liked.e ndings were as follows: some football players liked Maths and someof them liked History. All students liked English. None of the basketballplayers liked History, but all of them, as well as some rugby players likedChemistry. All rugby players like Geography.

Based on the ndings, which one of the below must be true?

1. Some of the footballers liked Maths and History.2. Some of the rugby players liked three subjects.3. Some rugby players liked History.4. Some of the footballers liked English but did not like Maths and

History.5. Some basketball players like more than 3 subjects.

Question 48:e control of illegal drug use is becoming increasingly difficult. New

‘legal highs’ are being manufactured which are slightly changedmolecularly from illegal compounds so they are not technically illegal.ese new ‘legal drugs’ are being brought onto the street at a rate of atleast one per week, and so the authorities cannot keep up. Some healthprofessionals therefore believe that the legality of drugs is becoming lessrelevant as to the potentially dangerous side effects. e fact that thesenew compounds are legal may however mean that the public are notaware of their equally high risks.

Which of the following are implied by the argument?

59

1. Some health professionals believe there is no value in making drugsillegal.

2. e major problem in controlling illegal drug use is the rapidmanufacture of new drugs that are not classi ed as illegal.

3. e general public are not worried about the risks of legal or illegalhighs.

4. ere is no longer a good correlation between risk of drug taking andthe legal status of the drug.

60

1. 1 only2. 2 only3. 1 and 44. 2 and 45. 2 and 36. 1,2,3 and 4

61

Question 49:WilderTravel Inc. is a company which organises wilderness travel

holidays, with activities such as trekking, mountain climbing, safari toursand wilderness survival courses. ese activities carry inherent risks, sothe directors of the company are drawing up a set of health regulations,with the aim of minimising the risks by ensuring that nobody participatesin activities if they have medical complications meaning that doing so mayendanger them. ey consider the following guidelines:

‘Persons with pacemakers, asthma or severe allergies are at signi cantrisk of heart attack in low oxygen environments’. People undertakingmountain climbing activities with WilderTravel frequently encounterenvironments with low oxygen levels. e directors therefore decide thatin order to ensure the safety of customers on WilderTravel holidays, onestep that must be taken is to bar those with pacemakers, asthma orallergies from partaking in mountain climbing.

Which of the following best illustrates a aw in this reasoning?

1. Participants should be allowed to assess the safety risks themselves,and should not be barred from activities if they decide the risk isacceptable.

2. ey have assumed that all allergies carry an increased risk of heartattack, when the guidelines only say this applies to those with severeallergies.

3. e directors have failed to consider the health risks of people withthese conditions taking part in other activities.

4. People with these conditions could partake in mountain climbingwith other holiday organisers, and thus be exposed to danger of heartattack.

Question 50:

62

St John’s Hospital in Northumbria is looking to recruit a newconsultant cardiologist, and interviews a series of candidates. einterview panel determines that 3 candidates are clearly more quali ed forthe role than the others, and they invite these 3 candidates for a secondinterview. During this second interview, and upon further examination oftheir previous employment records, it becomes apparent that Candidate 3is the most pro cient at surgery of the 3, whilst Candidate 1 is the best atpatient interaction and explaining the risks of procedures. Candidate 2,meanwhile, ranks between the other 2 in both these aspects.

e hospital director tells the interviewing team that the hospitalalready has a well-renowned team dedicated to patient interaction, butthe surgical success record at the hospital is in need of improvement. edirector issues instructions that therefore, it is more important that thenew candidate is pro cient at surgery, and patient interaction is less of aconcern.

Which of the following is a conclusion that can be drawn from theDirectors’ comments?

1. e interviewing team should hire Candidate 2, in order to achieve abalance of good patient relations with good surgical records.

2. e interviewing team should hire Candidate 1, in order to ensuregood patient interactions, as these are a vital part of a doctor’s work.

3. e interviewing team should ignore the hospital director and assessthe candidates further to see who would be the best t.

4. e interviewing team should hire Candidate 3, in order to ensurethat the new candidate has excellent surgical skills, to boost thehospital’s success in this area.

Question 51:Every winter in Britain, there are thousands of urgent callouts for

ambulances in snowy conditions. e harsh conditions mean that

63

ambulances cannot drive quickly, and are delayed in reaching patients.ese delays cause many injuries and medical complications, which couldbe avoided with quicker access to treatment. Despite this, very fewambulances are equipped with winter tyres or special tyre coverings tohelp the ambulances deal with snow. Clearly, if more ambulances were

tted with winter tyres, then we could avoid many medical complicationsthat occur each winter.

Which of the following is an assumption made in this passage?

1. Fitting winter tyres would allow ambulances to reach patients morequickly.

2. Ambulance trusts have sufficient funding to equip their vehicles withwinter tyres.

3. Many medical complications could be avoided with quicker access tomedical care.

4. ere are no other alternatives to winter tyres that would allowambulances to reach patients more quickly in snowy conditions.

Question 52:Vaccinations have been one of the most outstanding and in uential

developments in medical history. Despite the huge successes, however,there is a strong anti-vaccination movement active in some countries,particularly the USA, who claim vaccines are harmful and ineffective.

ere have been several high-pro le events in recent years where anti-vaccine campaigners have been refused permission to enter countries forcampaigns, or have had venues refuse to host them due to the nature oftheir campaigns. Many anti-vaccination campaigners have claimed this isan affront to free speech, and that they should be allowed to entercountries and obtain venues without hindrance. However, although freespeech is desirable, an exception must be made here because the anti-

64

vaccination campaign spreads misinformation to parents, causingvaccination to rates to drop.

When this happens, preventable infectious diseases often begin toincrease, causing avoidable deaths of innocent members of thecommunity, particularly so in children. us, in order to protect innocentpeople, we must continue to block the anti-vaccine campaigners fromspreading misinformation freely by pressuring venues not to host anti-vaccination campaigners.

Which of the following best illustrates the principle that this argumentfollows?

1. Free speech is always desirable, and must not be compromised underany circumstances.

2. e right of innocent people to protection from infectious diseases ismore important than the right of free speech.

3. e right of free speech does not apply when the party speaking islying or spreading misinformation.

4. Public health programmes that achieve signi cant success inreducing the incidence of disease should be promoted.

Question 53:In order for a tumour to grow larger than a few centimetres, it must

rst establish its own blood supply by promoting angiogenesis. Roger hasa tumour in his abdomen, which is investigated at the Royal GeneralHospital. During the tests, they detect newly formed blood vessels in thetumour, showing that it has established its own blood supply. us, weshould expect the tumour to grow signi cantly, and become larger than afew centimetres. Action must be taken to deal with this.

Which of the following best illustrates a aw in this reasoning?

65

1. It assumes that the tumour in Roger’s abdomen has established itsown blood supply.

2. It assumes that a blood supply is necessary for a tumour to growlarger than a few centimetres.

3. It assumes that nothing can be done to stop the tumour once a bloodsupply has been established.

4. It assumes that a blood supply is sufficient for the tumour to growlarger than a few centimetres.

Question 54:In this year’s Great North Run, there are several dozen people running

to raise money for the Great North Air Ambulance (GNAA), as part of alarge national fundraising campaign. If the runners raise £500,000 betweenthem, then the GNAA will be able to add a new helicopter to its eet.However, the runners only raise a total of £420,000. us, the GNAA willnot be able to get a new helicopter.

Which of the following best illustrates a aw in this passage?

1. It has assumed that the GNAA will not be able to acquire a newhelicopter without the runners raising £500,000.

2. It has assumed that that GNAA wishes to add a new helicopter to itseet.

3. It has assumed that the GNAA does not have better things to spendthe money on.

4. It has assumed that some running in the Great North Run are raisingmoney for the GNAA.

Question 55:Many courses, spanning Universities, colleges, apprenticeship

institutions and adult skills courses should be subsidised by thegovernment. is is because they improve the skills of those attending

66

them. It has been well demonstrated that the more skilled people are, themore productive they are economically. us, government subsidies ofmany courses would increase overall economic productivity, and lead toincreased growth.

Which of the following would most weaken this argument?

1. e UK already has a high level of growth, and does not need toaccelerate this growth.

2. Research has demonstrated that higher numbers of people attendingadult skills courses results in increased economic growth.

3. Research has demonstrated that the cost of many courses (to thosetaking them) has little effect on the number of people undertakingthe courses.

4. Employers often seek to employ those with greater skill-sets, andappoint them to higher positions.

Question 56:Pluto was once considered the 9th planet in the solar system. However,

further study of the planet led to it being reclassi ed as a dwarf planet in2006. One key factor in this reclassi cation was the discovery of manyobjects in the solar system with similar characteristics to Pluto, whichwere also placed into this new category of ‘Dwarf Planet’. Someastronomers believe that Pluto should remain classi ed as a planet, alongwith the many entities similar to Pluto that have been discovered.Considering all of this, it is clear that if we were to reclassify Pluto as aplanet, and maintain consistency with classi cation of astronomicalentities, then the number of planets would signi cantly increase.

Which of the following best illustrates the main conclusion of thispassage?

67

1. If Pluto is classi ed as a planet, then many other entities should alsobe planets, as they share similar characteristics.

2. Some astronomers believe Pluto should be classi ed as a planet.3. Pluto should not be classi ed as a Planet, as this would also require

many other entities to be classi ed as planets to ensure consistency.4. If Pluto is to be classi ed as a planet, then the number of objects

classi ed as planets should increase signi cantly.

Question 57:2 trains depart from Birmingham at 5:30 pm. One of the trains is

heading to London, whilst the other is heading to Glasgow. e distancefrom Birmingham to Glasgow is three times larger than the distance fromBirmingham to London, and the train to London arrives at 6:30 pm. us,the train to Glasgow will arrive at 8:30pm.

Which of the following is an assumption made in this passage?

1. Both trains depart at the same time.2. Both trains depart from Birmingham.3. Both trains travel at the same speed.4. e train heading to Glasgow has to travel three times as far as the

train heading to London.

Question 58:Carcinogenesis, oncogenesis and tumorigenesis are various names

given to the generation of cancer, with the term literally meaning ‘creationof cancer’. In order for carcinogenesis to happen, there are several stepsthat must occur. Firstly, a cell (or group of cells) must achieve immortality,and escape senescence (the inherent limitation of a cell’s lifespan). enthey must escape regulation by the body, and begin to proliferate in anautonomous way. ey must also become immune to apoptosis and othercell death mechanisms. Finally, they must avoid detection by the immune

68

system, or survive its responses. If a single one of these steps fails to occur,then carcinogenesis will not be able to occur.

Which of the following is a conclusion that can be reliably drawn fromthis passage?

1. Several steps are essential for carcinogenesis.2. If all the steps mentioned occur, then carcinogenesis will de nitely

occur.3. e immune system is unable to tackle cells that have escaped

regulation by the body.4. ere are various mechanisms by which carcinogenesis can occur.5. e terminology for the creation of cancer is confusing.

Question 59:P53 is one of the most crucial genes in the body, responsible for

detecting DNA damage and halting cell replication until repair can occur.If repair cannot take place, P53 will signal for the cell to kill itself. eseactions are crucial to prevent carcinogenesis, and a loss of functional P53is identi ed in over 50% of all cancers. e huge importance of P53towards protecting the cell from damaging mutations has led to itdeservedly being known as ‘the guardian of the genome’. e implicationsof this name are clear – any cell that has a mutation in P53 is at seriousrisk of developing a potentially dangerous mutation.

Which of the following CANNOT be reliably concluded from thispassage?

1. P53 is responsible for detecting DNA damage.2. Most cancers have lost functional P53.3. P53 deserves its name ‘guardian of the genome’.4. A cell that has a mutation in P53 will develop damaging mutations.5. None of the above.

69

Question 60:Sam is buying a new car, and deciding whether to buy a petrol or a

diesel model. He knows he will drive 9,000 miles each year. He calculatesthat if he drives a petrol car, he will spend £500 per 1,000 miles on fuel, butif he buys a diesel model he will only spend £300 per 1,000 miles on fuel. Hecalculates, therefore, that if he purchases a Diesel car, then this year hewill make a saving of £1800, compared to if he bought the petrol car.

Which of the following is NOT an assumption that Sam has made?

1. e price of diesel will not uctuate relative to that of petrol.2. e cars will have the same initial purchase cost.3. e cars will have the same costs for maintenance and garage

expenses.4. e cars will use the same amount of fuel.5. All of the above are assumptions.

Question 61:In the UK, cannabis is classi ed as a Class B drug, with a maximum

penalty of up to 5 years imprisonment for possession, or up to 14 years forpossession with intent to supply. e justi cation for drug laws in the UKis that classi ed drugs are harmful, addictive, and destructive to people’slives. However, available medical evidence indicates that cannabis isrelatively safe, non-addictive and harmless. In particular, it is certainlyshown to be less dangerous than alcohol, which is freely sold andadvertised in the UK. e fact that alcohol can be freely sold andadvertised, but cannabis, a less harmful drug, is banned highlights thegross inconsistencies in UK drugs policy.

Which of the following best illustrates the main conclusion of thispassage?

1. Cannabis is a less dangerous drug than alcohol.

70

2. Alcohol should be banned, so we can ensure consistency in the UKdrug policy.

3. Cannabis should not be banned, and should be sold freely, in order toensure consistency in the UK drug policy.

4. e UK government’s policy on drugs is grossly inconsistent.5. Alcohol should not be advertised in the UK.

Question 62:Every year in Britain, there are thousands of accidents at people’s

homes such as burns, broken limbs and severe cuts, which cause a largenumber of deaths and injuries. Despite this, very few households maintaina sufficient rst aid kit equipped with bandages, burn treatments, splintsand saline to clean wounds. If more households stocked sufficient rst aidsupplies, many of these accidents could be avoided.

Which of the following best illustrates a aw in this argument?

1. It ignores the huge cost associated with maintaining good rst aidsupplies, which many households cannot afford.

2. It implies that presence of rst aid equipment will lead to feweraccidents.

3. It ignores the many accidents that could not be treated even if rstaid supplies were readily available.

4. It neglects to consider the need for trained rst aid persons in orderfor rst aid supplies to help in reducing the severity of injuries causedby accidents.

Question 63:Researchers at SmithJones Inc., an international drug rm, are

investigating a well-known historic compound, which is thought to reducelevels of DNA replication by inhibiting DNA polymerases. It is proposedthat this may be able to be used to combat cancer by reducing the

71

proliferation of cancer cells, allowing the immune system to combat thembefore they spread too far and become too damaging. Old experimentshave demonstrated the effectiveness of the compound via monitoringDNA levels with a dye that stains DNA red, thus monitoring the levels ofDNA present in cell clusters. ey report that the compound is observed toreduce the rate at which DNA replicates. However, it is known that ifresearchers use the wrong solutions when carrying out these experiments,then the amount of red staining will decrease, suggesting DNA replicationhas been inhibited, even if it is not inhibited. As several researcherspreviously used this wrong solution, we can conclude that theseexperiments are awed, and do not re ect what is actually happening.

Which of the following best illustrates a aw in this argument?

1. From the fact that the compound inhibits DNA replication, it cannotbe concluded that it has potential as an anticancer drug.

2. From the fact that the wrong solutions were used, it cannot beconcluded that the experiments may produce misleading results.

3. From the fact that the experiments are old, it cannot be concludedthat the wrong solutions were used.

4. From the fact that the compound is old, it cannot be concluded thatit is safe.

Question 64:Rotherham football club are currently top of the league, with 90 points.

eir closest competitors are South Shields football club, with 84 points.Next week, the teams will play each other, and after this, they each have 2games left before the end of the season. Each win is worth 3 points, a drawis worth 1 point, and a loss is worth 0 points. us, if Rotherham beatSouth Shields, they will win the league (as they will then be 9 points clear,and South Shields would only be able to earn 6 more points).

72

In the match of Rotherham vs. South Shields, Rotherham are winninguntil the 85th minute, when Alberto Simeone scores an equaliser for SouthShields, and South Shields then go on to win the match. us, Rotherhamwill not win the league.

Which of the following best illustrates a aw in this passage’sreasoning?

1. It has assumed that Alberto Simeone scored the winning goal forSouth Shields.

2. It has assumed that beating South Shields was necessary forRotherham to win the league, when in fact it was only sufficient.

3. Rotherham may have scored an equaliser later in the game, and notlost the match.

4. It has failed to consider what other teams might win the league.

Question 65:Oakville Supermarkets is looking to build a new superstore, and a

meeting of its directors has been convened to decide where the best placeto build the supermarket would be. e Chairperson of the Board suggeststhat the best place would be Warrington, a town that does not currentlyhave a large supermarket, and would thus give them an excellent share ofthe shopping market.

However, the CEO notes that the population of Warrington has beensteadily declining for several years, whilst Middlesbrough has recentlybeen experiencing high population growth. e CEO therefore argues thatthey should build the new supermarket in Middlesbrough, as they wouldthen be within range of more people, and so of more potential customers.

Which of the following best illustrates a aw in the CEO’s reasoning?

1. Middlesbrough may already have other supermarkets, so the newsuperstore may get a lower share of the town’s shoppers.

73

2. Despite the recent population changes, Warrington may still have alarger population than Middlesbrough.

3. Middlesbrough’s population is projected to continue growing, whilstWarrington’s is projected to keep falling.

4. Many people in Warrington travel to Liverpool or Manchester, 2nearby major cities, in order to do their shopping.

Question 66:Global warming is a key challenge facing the world today, and the

changes in weather patterns caused by this phenomenon have led to thedestruction of many natural habitats, causing many species to becomeextinct. Recent data has shown that extinctions have been occurring at afaster rate over the last 40 years than at any other point in the earth’shistory, exceeding the great Permian mass extinction, which wiped out96% of life on earth. If this rate continues, over 50% of species on earth willbe extinct by 2100. It is clear that in the face of this huge challenge,conservation programmes will require signi cantly increased levels offunding in order to prevent most of the species on earth from becomingextinct.

Which of the following are assumptions in this argument?

1. e rate of extinctions seen in the last 40 years will continue to occurwithout a step-up in conservation efforts.

2. Conservation programmes cannot prevent further extinctionswithout increased funding.

3. Global warming has caused many extinction events, directly orindirectly.

4. 1 only5. 2 only6. 3 only7. 1 and 2

74

8. 1 and 39. 2 and 3

10. 1,2 and 3

Question 67:After an election in Britain, the new government is debating what

policy to adopt on the railway system, and whether it should be entirelyprivatised, or whether public subsidies should be used to supplementcosts and ensure that sufficient services are run. Studies in Austria, whichhas high public funding for railways, have shown that the rail service isused by many people, and is highly thought of by the population. However,this is clearly down to the fact that Austria has many mountainous andhigh-altitude areas, which experience signi cant amounts of snow and ice.is makes many roads impassable, and travelling by road difficult. us,rail is often the only way to travel, explaining the high passenger numbersand approval ratings. us, the high public subsidies clearly have no effect.

Which of the following, if true, would weaken this argument?

1. France also has high public subsidy of railways, but does not havelarge areas where travel by road is difficult. e French railway alsohas high passenger numbers and approval ratings.

2. Italy also has high public subsidy of railways, but the local populationdislike using the rail service, and it has poor passenger numbers.

3. ere are many reasons affecting the passenger numbers andapproval ratings of a given country’s rail serviced.

4. 1 only5. 2 only6. 3 only7. 1 and 28. 1 and 39. 2 and 3

75

Question 68:In 2001-2002, 1,019 patients were admitted to hospital due to obesity.

is gure was more than 11 times higher by 2011-12 when there were11,736 patients admitted to hospital with the primary reason foradmission being obesity. Data has shown higher percentages of both menand women were either obese or overweight in 2011 compared to 1993,with male obesity climbing from 58% to 65%, and female from 49% to 58%.Rates of adult obesity have increased even more steeply within this period– 13% to 24% for men and 16% to 26% for women.

Studies in 2011 found that nearly a third of children between 2 – 15years were either overweight or obese, however this was not signi cantlydifferent from 2010. Lifestyles are also becoming less healthy, with adecline in both children and adults eating the recommended number offruit and vegetables each day and taking the recommended amount ofexercise each week. e ease of availability of fast-food outlets may bepartly to blame for the rising number of obese people. Education isrequired to teach people the importance of a healthy lifestyle, howeverpeople must take some personal responsibility for their health.

Using only information from the passage, which of the followingstatements is correct?

1. In 2011, there was a higher proportion of obese men than women.2. Obesity rates are rising steeply for both males and females of all age

groups.3. Responsibility needs to be taken by both individuals and local

authorities to effectively tackle the epidemic.4. e main reason people eat fast food is because it’s cheaper in times

of reducing income.

Question 69:

76

Tobacco companies sell cigarettes despite being fully aware thatcigarettes cause signi cant harm to the wellbeing of those that smokethem. Diseases caused or aggravated by smoking cost billions of poundsfor the NHS to treat each year. is is despicable behaviour from thetobacco companies. Tobacco companies should be taxed, and the moneyraised put towards funding the NHS.

Which of the following conclusions CANNOT be drawn from theabove?

1. ere is a connection between lung cancer and smoking.2. ere is a connection between liver disease and smoking.3. ere is a connection between oral cancer and smoking.4. All smokers drink excessively.5. All of the above.

Question 70:Investigations in the origins of species suggest that humans and the

great apes have the same ancestors. is is suggested by the high degree ofgenetic similarity between humans and chimpanzees (estimated at 99%).At the same time there is an 84% homology between the human genomeand that of pigs. is raises the interesting question of whether it would bepossible to use pig or chimpanzee organs for the treatment of humandisease.

Which conclusion can be reasonably drawn from the above article?

1. Pigs and chimpanzees have a common ancestor.2. Pigs and humans have a common ancestor.3. It can be assumed that chimpanzees will develop into humans if

given enough time.4. ere seems to be great genetic homology across a variety of species.

77

5. Organs from pigs or chimpanzees present a good alternative forhuman organ donation.

Question 71:Poor blood supply to a part of the body can cause damage of the

affected tissue - i.e. lead to an infarction. ere are a variety of known riskfactors for vascular disease. Diabetes is a major risk factor. Other riskfactors are more dependent on the individual as they represent individualchoices such as smoking, poor dietary habits as well as little to no exercise.In some cases infarction of the limbs and in particular the feet can becomevery bad and extensive with patches of tissue dying. is is known asnecrosis and is marked by affected area of the body turning black. Necrotictissue is usually removed in surgery.

Which of the following statements CANNOT be concluded from theinformation in the above passage?

1. Smoking causes vascular disease.2. Diabetes causes vascular disease.3. Vascular disease always leads to infarctions.4. Necrotic tissue must be removed surgically.5. Necrotic tissue only occurs following severe infarction.6. All of the above.

Question 72:People who can afford to pay for private education should not have

access to the state school system. is would allow more funding forstudents from lower income backgrounds. More funding will providebetter resources for students from lower income backgrounds, and willhelp to bridge the gap in educational attainment between students fromhigher income and lower income backgrounds.

78

Which of the following statements, if true, would most strengthen theabove argument?

1. Educational attainment is a signi cant factor in determining futureprospects.

2. Providing better resources for students has been demonstrated tolead to an increase in educational attainment.

3. Most people who can afford to do so choose to purchase privateeducation for their children.

4. A signi cant gap exists in educational attainment between studentsfrom high income and low-income backgrounds.

5. Most schools currently receive a similar amount of funding relativeto the number of students in the school.

Question 73:Increasing numbers of people are choosing to watch lms on DVD in

recent years. In the past few years, cinemas have lost customers, causingthem to close down. Many cinemas have recently closed, removing animportant focal point for many local communities and causing damage tothose communities. erefore, we should ban DVDs in order to help localcommunities.

Which of the following best states an assumption made in thisargument?

1. e cinemas that have recently closed have done so because ofreduced pro ts due to people choosing to watch DVDs instead.

2. Cinemas being forced to close causes damage to local communities.3. DVDs are improving local communities by allowing people to meet

up and watch lms together.4. Sales of DVDs have increased due to economic growth.5. Local communities have called for DVDs to be banned.

79

Question 74:Aeroplanes are the fastest form of transport available. An aeroplane

can travel a given distance in less time than a train or a car. John needs totravel from Glasgow to Birmingham. If he wants to arrive as soon aspossible, he should travel by aeroplane.

Which of the following best illustrates a aw in this argument?

1. One day, there could be faster cars built that could travel as fast asaeroplanes.

2. Travelling by air is often more expensive.3. It ignores the time taken to travel to an airport and check in to a

ight, which may mean he will arrive later if travelling by aeroplane.4. John may not own a car, and thus may not have any option.5. John may not be legally allowed to make the journey.

Question 75:During autumn, spiders frequently enter people’s homes to escape the

cold weather. Many people dislike spiders and seek ways to prevent themfrom entering properties, leading to spider populations falling as theystruggle to cope with the cold weather. Studies have demonstrated thatwhen spider populations fall, the population of ies rises. Higher numbersof ies are associated with an increase in food poisoning cases. erefore,people must not seek to prevent spiders from entering their homes.

Which of the following best illustrates the main conclusion of thisargument?

1. People should not dislike spiders being present in their homes.2. People should seek methods to prevent ies from entering their

homes.3. People should actively encourage spiders to occupy their homes to

increase biodiversity.

80

4. People should accept the presence of spiders in their homes toreduce the incidence of food poisoning.

5. Spiders should be cultivated and used as a biological pest control tocombat ies.

Question 76:Each year, thousands of people acquire infections during prolonged

stays at hospital. Concurrently, bacteria are becoming resistant toantibiotics at an ever-increasing rate. In spite of this, progressively lesspharmaceutical companies are investing in research into new antibiotics,and the number of antibiotics coming onto the market is decreasing. As aresult, the number of antibiotics that can be used to treat infections isfalling. If pharmaceutical companies were pressured into investing in newantibiotic research, many lives could be saved.

Which of the following best illustrates a aw in this argument?

1. It assumes the infections acquired during stays at hospital areresulting in deaths.

2. It ignores the fact that many people never have to stay in hospital.3. It does not take into account the fact that antibiotics do not produce

much pro t for pharmaceutical companies.4. It ignores the fact that some hospital-acquired infections are caused

by organisms that cannot be treated by antibiotics, such as viruses.5. It assumes that bacterial resistance to antibiotics has not been

happening for some time.

Question 77:Katherine has shaved her armpits most of her adult life, but has now

decided to stop. She explains her reasons for this to John, saying she doesnot like the pressures society puts on women to be shaven in this area.John listens to her reasons, but ultimately responds ‘just because you

81

explain why I should nd your hairiness attractive, it does not mean I will.I nd you unattractive, as I do not like girls with hair on their arm pits.’

What assumption has John made?

1. at just because he nds Katherine unattractive, he would ndother girls with unshaven arm pits unattractive.

2. at Katherine is trying to make John nd her armpit hair attractive.3. at Katherine will never conceal her armpit hair.4. Katherine must be wrong, because she is a woman.5. at Katherine thinks women should stop shaving.

Question 78:Medicine has improved signi cantly over the last century. Better

medicine causes a reduction in the death rate from all causes. However, aspeople get older, they suffer from infectious disease more readily.

Many third world countries have a high rate of deaths from infectiousdisease. Sunita argues that this high death rate is caused by bettermedicine, which has given an ageing population, thus giving a high rate ofdeaths from infectious disease as elderly people suffer from infectiousdisease more readily. Sunita believes that better medicine is thus indirectlyresponsible for this high death rate from infectious disease.

However, this cannot be the case. In third world countries, mostpeople do not live to old age, often dying from infectious disease at ayoung age. erefore, an ageing population cannot be the reason behindthe high rate of death from infectious disease. As better medicine causes areduction in the death rate from all causes, it is clear that better medicinewill lead to a reduction in the death rate from infectious disease in thirdworld countries.

Which of the following best states the main conclusion of thisargument?

82

1. We can expect that improvements in medicine seen over the lastcentury will improve.

2. Better medicine is not responsible for the increased prevalence ofinfectious disease in third world countries.

3. Better medicine has caused the overall death rate of third worldcountries to increase.

4. Better medicine will cause a decrease in the rate of death frominfectious disease in third world countries.

5. As people get older, they suffer from infectious disease more readily.

Question 79:Bristol and Cardiff are 2 cities with similar demographics, and located

in a roughly similar area of the country. Bristol has higher demand forhousing than Cardiff. erefore, a house in Bristol will cost more than asimilar house in Cardiff.

Which of the following best illustrates an assumption in the statementabove?

1. House prices will be higher if demand for housing is higher.2. People can commute from Cardiff to Bristol.3. Supply of housing in Cardiff will not be lower than in Bristol.4. Bristol is a better place to live.5. Cardiff has sufficient housing to provide for the needs of its

communities.

Question 80:Jellicoe Motors is a small motor company in Sheffield, employing 3

people. e company is hiring a new mechanic and interviews severalcandidates. New research into production lines has indicated that havingemployees with a good ability to work as part of a team boosts a

83

company’s productivity and pro ts. erefore, Jellicoe motors should hirea candidate with good team-working skills.

Which of the following best illustrates the main conclusion of thisargument?

1. Jellicoe Motors should not hire a new mechanic.2. Jellicoe motors should hire a candidate with good team-working

skills in order to boost their productivity and pro ts.3. Jellicoe motors should hire several new candidates in order to form a

good team, and boost their productivity.4. If Jellicoe motors does not hire a candidate with good team-working

skills, they may struggle to be pro table.5. Jellicoe motors should not listen to the new research.

Question 81:Research into new antibiotics does not normally hold much pro t for

pharmaceutical rms. As a consequence many rms are not investing inantibiotic research, and very few new antibiotics are being produced.However, with bacteria becoming increasingly resistant to currentantibiotics, new ones are desperately needed to avoid running the risk ofthousands of deaths from bacterial infections. erefore, the UKgovernment must provide nancial incentives for pharmaceuticalcompanies to invest in research into new antibiotics.

Which of the following best expresses the main conclusion of thisargument?

1. If bacteria continue to become resistant to antibiotics, there could bethousands of deaths from bacterial infections.

2. Pharmaceutical rms are not investing in new antibiotic researchdue to a lack of potential pro t.

84

3. If the UK government invests in research into new antibiotics,thousands of lives will be saved.

4. e pharmaceutical rms should invest in areas of research that arepro table, and ignore antibiotic research.

5. e UK government must provide nancial incentives forpharmaceutical rms to invest into antibiotic research if it wishes toavoid risking thousands of deaths from bacterial infections.

Question 82:People in developing countries use far less water per person than

those in developed countries. It is estimated that at present, people in thedeveloping world use an average of 30 litres of water per person per day,whilst those in developed countries use on average 70 litres of water perperson per day. It is estimated that for the current world population, anaverage water usage of 60 litres per person per day would be sustainable,but any higher than this would be unsustainable.

e UN has set development targets such that in 20 years, people livingin developing countries will be using the same amount of water per personper day as those living in developed countries. Assuming the worldpopulation stays constant for the next 20 years, if these targets are met theworld’s population will be using water at an unsustainable rate.

Which of the following, if true, would most weaken the argumentabove?

1. e prices of water bills are dropping in developed countries like theUK.

2. e level of water usage in developed countries is falling, and may bebelow 60 litres per person per day in 20 years.

3. e population of all developing countries is less than the populationof all developed countries.

85

4. Climate change is likely to decrease the amount of water available forhuman use over the next 20 years.

5. e UN’s development targets are unlikely to be met.

Question 83:In this Senior Management post we need someone who can keep a

cool head in a crisis and react quickly to events. e applicant says hesuffers from a phobia about ying, and panics especially when an aircraftis landing and that therefore he would prefer not to travel abroad onbusiness if it could be avoided. He is obviously a very nervous type ofperson who would clearly go to pieces and panic in an emergency and failto provide the leadership qualities necessary for the job. erefore thisperson is not a suitable candidate for the post.

Which of the following highlights the biggest aw in the argumentabove?

1. It falsely assumes that phobias are untreatable or capable of beingeliminated.

2. It falsely assumes that the person appointed to the job will need totravel abroad.

3. It falsely assumes that a speci c phobia indicates a general tendencyto panic.

4. It falsely assumes that people who stay cool in a crisis will be goodleaders.

5. It fails to take into account other qualities the person might have forthe post.

Question 84:ere are signi cant numbers of people attending university every

year, as many as 45% of 18 year olds. As a result, there are many moregraduates entering the workforce with better skills and better earning

86

potential. Going to university makes economic sense and we shouldencourage as many people to go there as possible.

Which of the following highlights the biggest aw in the argumentabove?

1. ere are no more university places left.2. Students can succeed without going to university.3. Not all degrees equip students with the skills needed to earn higher

salaries.4. Some universities are better than others.

Question 85:Young people spend too much time watching television, which is bad

for them. Watching excessive amounts of TV is linked to obesity, socialexclusion and can cause eye damage. If young people were to spend justone evening a week playing sport or going for a walk the bene ts would bemanifold. ey would lose weight, feel better about themselves and itwould be a sociable activity. Exercise is also linked to strong performanceat school and so young people would be more likely to perform well intheir exams.

Which of the following highlights the biggest aw in the argumentabove?

1. Young people can watch sport on television.2. ere are many factors that affect exam performance.3. Television does not necessarily have any damaging effect.4. Television and sport are not linked.

Question 86:Campaigners pushing for legalisation of cannabis have many

arguments for their cause. Most claim there is little evidence of anyadverse affects to health caused by cannabis usage, that many otherwise

87

law-abiding people are users of cannabis and that in any case, prohibitionof drugs does not reduce their usage. Legalising cannabis would alsoreduce crime associated with drug trafficking and would provide anadditional revenue stream for the government.

Which of the following best represents the conclusion of the passage?

1. Regular cannabis users are unlikely to have health problems.2. Legalising cannabis would be good for cannabis users.3. ere are multiple reasons to legalise cannabis.4. Prohibition is an effective measure to reduce drugs usage.5. Drug associated crime would reduce if cannabis was legal.

Question 87:Mohan has been offered a new job in Birmingham, starting in several

months with a xed salary. In order to ensure he can afford to live inBirmingham on his new salary, Mohan compares the prices of somehouses in Birmingham. He nds that a 2 bedroomed house will cost£200,000. A 3 bedroomed house will cost £250,000. A 4 bedroomed housewith a garden will cost £300,000.

Mohan’s bank tells him that if he is earning the salary of the job he hasbeen offered, they will grant him a mortgage for a house costing up to£275,000. After a month of deliberation, Mohan accepts the job anddecides to move to Wolverhampton. He begins searching for a house tobuy. He reasons that he will not be able to purchase a 4-bedroomed house.

Which of the following is NOT an assumption that Mohan has made?

1. A house in Wolverhampton will cost the same as a similar house inBirmingham.

2. A different bank will not offer him a mortgage for a more expensivehouse on the same salary.

88

3. e salary for the job could increase, allowing him to purchase amore expensive house.

4. A 4-bedroomed house without a garden will not cost less than a 4-bedroomed house with a garden.

5. House prices in Birmingham will not have fall in the time betweennow and Mohan purchasing a house.

Question 88:We should teach the Holocaust in schools. It is important that young

people see what it was like for Jewish people under Nazi rule. If we exposethe harsh realities to impressionable people then this will help improvetolerance of other races. It will also prevent other such terrible eventshappening again.

Which is the best conclusion?

1. We should teach about the Holocaust in schools.2. e Holocaust was a tragedy.3. e Nazis were evil.4. We should not let terrible events happen again.5. Educating people is the best solution to the world’s problems.

Question 89:e popular series ‘Game of rones’ should not be allowed on

television because it shows scenes of a disturbing nature, in particularscenes of rape. Children may nd themselves watching the programme onTV, and then going on to commit the terrible crime of rape, mimickingwhat they have watched.

Which of the following best illustrates a aw in this argument?

1. Children may also watch the show on DVD.2. Adults may watch the show on television.

89

3. Watching an action does not necessarily lead to recreating the actionyourself.

4. ere are lots of non-violent scenes in the show.

Question 90:e TV series ‘House of Cards’ teaches us all a valuable lesson: the

world is not a place that rewards kind behaviour. e protagonist of theseries, Frank Underwood, uses intrigue and guile to achieve his goals, andthrough clever political tactics he is able to climb in rank. If he were to bekinder to people, he would not be able to be so successful. Success ispredicated on his refusal to conform to conventional morality. e TVseries should be shown to small children in schools, as it could teach themhow to achieve their dreams.

Which of the following is an assumption made in the argument?

1. Children pay attention to school lessons.2. e TV series is sufficiently entertaining.3. One cannot both obey a moral code and succeed.4. Frank Underwood is a likable character.

Question 91:Freddy makes lewd comments on a female passer-by’s body to his

friend, Neil, loud enough for the woman in question to hear. Neil isuncomfortable with this, and states that it is inappropriate for Freddy todo so, and that Freddy is being sexist. Freddy refutes this, and Neil retortsthat Freddy would not make these comments about a man’s body. Freddyreplies by saying ‘it is not sexist, I am a feminist, I believe in equality formen and women.’

Which of the following describes a aw made in Freddy’s logic?

1. A self-proclaimed feminist could still say a sexist thing.2. e female passer-by in question felt uncomfortable.

90

3. Neil, too, considers himself a feminist.4. It would still not be OK to make lewd comments at male passers-by.5. Lewd comments are always inappropriate.

Question 92:e release of CO2 from consumption of fossil fuels is the main reason

behind global warming, which is causing signi cant damage to manynatural environments throughout the world. One signi cant source of CO2

emissions is cars, which release CO2 as they use up petrol. In order totackle this problem, many car companies have begun to design cars withengines that do not use as much petrol. However, engines which use lesspetrol are not as powerful, and less powerful cars are not attractive to thepublic. If a car company produces cars which are not attractive to thepublic, they will not be pro table.

Which of the following best illustrates the main conclusion of thisargument?

1. Car companies which produce cars that use less petrol will not bepro table.

2. e public prefer more powerful cars.3. Car companies should prioritise pro ts over helping the

environment.4. Car companies should seek to produce engines that use less petrol

but are still just as powerful.5. e public are not interested in helping the environment.

91

SECTION 1: PROBLEM SOLVINGQUESTIONS

SECTION 1 PROBLEM SOLVING QUESTIONS are arguably the hardestto prepare for. However, there are some useful techniques you can employto solve some types of questions much more quickly:

Construct EquationsSome of the problems in Section 1 are quite complex and you’ll need to

be comfortable with turning prose into equations and manipulating them.For example, when you read “Mark is twice as old as Jon” – this shouldimmediately register as M = 2J. Once you get comfortable formingequations, you can start to approach some of the harder questions in thisbook (and past papers) which may require you to form and solvesimultaneous equations. Consider the example:

Nick has a sleigh that contains toy horses and clowns and counts 44heads and 132 legs in his sleigh. Given that horses have one head and fourlegs, and clowns have one head and two legs, calculate the differencebetween the number of horses and clowns.

1. 02. 53. 224. 285. 1326. More information is needed.

92

To start with, let C= Clowns and H= Horses.For Heads:; For Legs:is now sets up your two equations that you can solve

simultaneously.us, ;erefore,Substitute back in to giveus the difference between horses and clownsIt’s important you are able to do these types of questions quickly (and

without resorting to trial & error as they are commonplace in section 1.DiagramsWhen a question asks about timetables, orders or sequences, draw out

diagrams. By doing this, you can organise your thoughts and help makesense of the question.

“Mordor is West of Gondor but East of Rivendale. Lorien is midwaybetween Gondor and Mordor. Erebus is West of Mordor. Eden is not East ofGondor.”

Which of the following cannot be concluded?

1. Lorien is East of Erebus and Mordor.2. Mordor is West of Gondor and East of Erebus.3. Rivendale is west of Lorien and Gondor.4. Gondor is East of Mordor and East of Lorien5. Erebus is West of Mordor and West of Rivendale.

Whilst it is possible to solve this in your head, it becomes much moremanageable if you draw a quick diagram and plot the positions of eachtown:

93

Now, it’s a simple case of going through each option and seeing if it iscorrect according to the diagram. You can now easily see that Option E-Erebus cannot be west of Rivendale.

Don’t feel that you have to restrict yourself to linear diagrams like thiseither – for some questions you may need to draw tables or even Venndiagrams. Consider the example:

Slifers and Osiris are not legendary. Krakens and Minotaurs arelegendary. Minotaurs and Lords are both divine. Humans are neitherlegendary nor divine.

1. Krakens may be only legendary or legendary and divine.2. Humans are not divine.3. Slifers are only divine.4. Osiris may be divine.5. Humans and Slifers are the same in terms of both qualities.

Constructing a Venn diagram allows us to quickly see that theposition of Osiris and Krakens aren’t certain. us, A and D must be true.

94

Humans are neither so B is true. Krakens may be divine so A is true. Ecannot be concluded as Slifers are divine but are humans are not. us, Eis False.

Spatial Reasoningere are usually 1-2 spatial reasoning questions every year. ey

usually give nets for a shape or a patterned cuboid and ask which optionsare possible rotations. Unfortunately, they are extremely difficult toprepare for because the skills necessary to solve these types of questionscan take a very long time to improve. e best thing you can do to prepareis to familiarise yourself with the basics of how cube nets work and whatthe effect of transformations are e.g. what happens if a shape is re ected ina mirror etc.

It is also a good idea to try to learn to draw basic shapes like cubesfrom multiple angles if you can’t do so already. Finally, remember that ifthe shape is straightforward like a cube, it might be easier for you to drawa net, cut it out and fold it yourself to see which of the options are possible.

Problem Solving QuestionsQuestion 93:Pilbury is south of Westside, which is south of Harrington. Twotown is

north of Pilbury and Crewville but not further north than Westside.Crewville is:

1. South of Westside, Pilbury and Harrington but not necessarilyTwotown.

2. North of Pilbury, and Westside.3. South of Westside and Twotown, but north of Pilbury.4. South of Westside, Harrington and Twotown but not necessarily

Pilbury.5. South of Harrington, Westside, Twotown and Pilbury.

Question 94:

95

e hospital coordinator is making the rota for the ward for next week;two of Drs Evans, James and Luca must be working on weekdays, none ofthem on Sundays and all of them on Saturdays. Dr Evans works 4 days aweek including Mondays and Fridays. Dr Luca cannot work Monday orursday. Only Dr James can work 4 days consecutively, but he cannot do5.

What days does Dr James work?

1. Saturday, Sunday and Monday.2. Monday, Tuesday, Wednesday, ursday and Saturday.3. Monday, ursday Friday and Saturday.4. Tuesday, Wednesday, Friday and Saturday.5. Monday, Tuesday, Wednesday, ursday and Friday.

Question 95:Michael, a taxi driver, charges a call out rate and a rate per mile for taxi

rides. For a 4 mile ride he charges £11, and for a 5 mile ride, £13.How much does he charge for a 9-mile ride?

96

1. £152. £173. £194. £205. £21

97

Question 96:Goblins and trolls are not magical. Fairies and goblins are both

mythical. Elves and fairies are magical. Gnomes are neither mythical normagical.

Which of the following is FALSE?

1. Elves may be only magical or magical and mythical.2. Gnomes are not mythical.3. Goblins are only mythical.4. Trolls may be mythical.5. Gnomes and goblins are the same in terms of both qualities.

Question 97:Jessica runs a small business making bespoke wall tiles. She has just

had a rush order for 100 tiles placed that must be ready for today at 7pm.e client wants the tiles packed all together, a process which will take 15minutes. Only 50 tiles can go in the kiln at any point and they must be putin the kiln to heat for 45 minutes. e tiles then sit in the kiln to coolbefore they can be packed, a process which takes 20 minutes. While tilesare in the kiln Jessica is able to decorate more tiles at a rate of 1 tile perminute.

What is the latest time Jessica can start making the tiles?

98

1. 2:55pm2. 3:15pm3. 3:30pm4. 3:45pm

99

Question 98:Pain nerve impulses are twice as fast as normal touch impulses. If Yun

touches a boiling hot pan this message reaches her brain, 1 metre away, in1 millisecond.

What is the speed of a normal touch impulse?

100

1. 5 m/s2. 20 m/s3. 50 m/s4. 200m/s5. 500 m/s

101

Question 99:A woman has two children Melissa and Jack, yearly, their birthdays are

3 months apart, both being on the 22nd. e woman wishes to continuethe trend of her children’s names beginning with the same letter as themonth they were born. If her next child, Alina is born on the 22nd 2 monthsafter Jack’s birthday, how many months after Alina is born will Melissahave her next birthday?

102

1. 2 months2. 4 months3. 5 months4. 6 months5. 7 months

103

Question 100:Policemen work in pairs. PC Carter, PC Dirk, PC Adams and PC Bryan

must work together but not for more than seven days in a row, which PCAdams and PC Bryan now have. PC Dirk has worked with PC Carter for 3days in a row. PC Carter does not want to work with PC Adams if it can beavoided.

Who should work with PC Bryan?

1. PC Carter2. PC Dirk3. PC Adams4. Nobody is available under the guidelines above.

Question 101:My hair-dressers charges £30 for a haircut, £50 for a cut and blow-dry,

and £60 for a full hair dye. ey also do manicures, of which the rst costs£15, and includes a bottle of nail polish, but are subsequently reduced by£5 if I bring my bottle of polish. e price is reduced by 10% if I book andpay for the next 5 appointments in advance and by 15% if I book at leastthe next 10.

I want to pay for my next 5 cut and blow-dry appointments, as well asfor my next 3 manicures. How much will it cost?

104

1. £1702. £2553. £2604. £2855. £305

105

Question 102:Alex, Bertha, David, Gemma, Charlie, Elena and Frankie are all

members of the same family consisting of three children, two of whom,Frankie and Gemma are girls. No other assumption of gender based onname can be established. ere are also four adults. Alex is a doctor and isDavid’s brother. One of them is married to Elena, and they have twochildren. Bertha is married to David; Gemma is their child.

Who is Charlie?

1. Alex’s daughter2. Frankie’s father3. Gemma’s brother4. Elena’s son5. Gemma’s sister

Question 103:At 14:30 three medical students were asked to examine a patient’s

heart. Having already watched their colleague, the second two studentswere twice as fast as the rst to examine. During the 8 minutes break afterthe nal student had nished, they were told by their consultant that theyhad taken too long and so should go back and do the examinations again.e second time all the students took half as long as they had taken the

rst time with the exception of the rst student who, instead took thesame time as his two colleagues’ second attempt. Assuming there was aone minute change over time between each student and they were

nished by 15:15, how long did the second student take to examine therst time?

106

1. 3 minutes2. 4 minutes3. 6 minutes4. 7 minutes5. 8 minutes

107

Question 104:I pay for 2 chocolate bars that cost £1.65 each with a £5 note. I receive

8 coins change, only 3 of which are the same.Which TWO coins do I not receive in my change?

108

1. 1p2. 2p3. 5p4. 10p5. 20p6. £27. £1

109

Question 105:Two 140m long trains are running at the same speed in opposite

directions. If they cross each other in 14 seconds then what is speed ofeach train?

110

1. 10 km/hr2. 18 km/hr3. 32 km/hr4. 36 km/hr5. 42 km/hr

111

Question 106:Anil has to re ll his home’s swimming pool. He has four hoses which

all run at different speeds. Alone, the rst would completely ll the poolwith water in 6 hours, the second in two days, the third in three days andthe fourth in four days.

Using all the hoses together, how long will it take to ll the pool to thenearest quarter of an hour?

1. 4 hours 15 minutes2. 4 hours 30 minutes3. 4 hours 45 minutes4. 5 hours5. 5 hours 15 minutes

Question 107:An ant is stuck in a 30 cm deep ditch. When the ant reaches the top of

the ditch he will be able to climb out straight away. e ant is able to climb3 cm upwards during the day, but falls back 2 cm at night.

How many days does it take for the ant to climb out of the ditch?

112

1. 272. 283. 294. 305. 31

113

Question 108:When buying his ingredients a chef gets a discount of 10% when he

buys 10 or more of each item, and 20% discount when he buys 20 or more.On one order he bought 5 sausages and 10 Oranges and paid £8.50. Onanother, he bought 10 sausages and 10 apples and paid £9, on a third hebought 30 oranges and paid £12.

How much would an order of 2 oranges, 13 sausages and 12 applescost?

114

1. £12.522. £12.763. £13.524. £13.765. £13.80

115

Question 109:My hairdressers encourage all of its clients to become members. By

paying an annual member fee, the cost of haircuts decreases. VIPmembership costs £125 annually with a £10 reduction on haircuts.Executive VIP membership costs £200 for the year with a £15 reductionper haircut. At the moment I am not a member and pay £60 per haircut. Iknow how many haircuts I have a year, and I work out that by becoming amember on either programme it would work out cheaper, and I wouldsave the same amount of money per year on either programme.

How much will I save this year by buying membership?

116

1. £102. £153. £254. £305. £50

117

Question 110:If criminals, thieves and judges are represented below:

Assuming that judges must have clean record, all thieves are criminalsand all those who are guilty are convicted of their crimes, which of one ofthe following best represents their interaction?

118

119

Question 111:e months of the year have been made into number codes. e code

is comprised of three factors, including two of these being related theletters that make up the name of the month. No two months would havethe same rst number. But some such as March, which has the code 3513,have the same last number as others, such as May, which has the code5313. October would be coded as 10715 while February is 286.

What would be the code for April?

120

1. 1542. 4413. 4514. 5145. 541

121

Question 112:A mother gives yearly birthday presents of money to her children

based on the age and their exam results. She gives them £5 each plus £3 forevery year they are older than 5, and a further £10 for every A* theyachieved in their results. Josie is 16 and gained 9 A*s in her results.Although Josie’s brother Carson is 2 years older he receives £44 less a yearfor his birthday.

How many more A*s did Josie get than Carson?

122

1. 22. 33. 44. 55. 10

123

Question 113:Apples are more expensive than pears, which are more expensive than

oranges. Peaches are more expensive than oranges. Apples are lessexpensive than grapes.

Which two of the following must be true?

1. Grapes are less expensive than oranges.2. Peaches may be less expensive than pears.3. Grapes are more expensive than pears.4. Pears and peaches are the same price.5. Apples and peaches are the same price.

Question 114:What is the minimum number of straight cutting motions needed to

slice a cylindrical cake into 8 equally sized pieces?

124

1. 22. 33. 44. 55. 66. 8

125

Question 115ree friends, Mark, Russell and Tom had agreed to meet for lunch at

12 PM on Sunday. Daylight saving time (GMT+1) had started at 2 AM thesame day, where clocks should be put forward by one hour. Mark’s phoneautomatically changes the time but he does not realise this so when hewakes up he puts his phone forward an hour and uses his phone to timehis arrival to lunch. Tom puts all of his clocks forward one hour at 7 AM.Russell forgets that the clocks should go forward, wakes at 10 AM doesn’tchange his clocks. All of the friends arrive on time as far as they areconcerned.

Assuming that none of the friends realise any errors before arriving,which TWO of the following statements are FALSE?

1. Tom arrives at 12 PM (GMT +1).2. All three friends arrive at the same time.3. ere is a 2 hour difference between when the rst and last friend

arrive.4. Mark arrives late.5. Mark arrives at 1 PM (GMT+3).6. Russell arrives at 12 PM (GMT+0).

Question 116:A class of young students has a pet spider. Deciding to play a practical

joke on their teacher, one day during morning break one of the studentsput the spider in their teachers’ desk. When rst questioned by the headteacher, Mr Jones, the ve students who were in the classroom duringmorning break all lied about what they saw. Realising that the studentswere all lying, Mr Jones called all 5 students back individually and,threatened with suspension, all the students told the truth. UnfortunatelyMr Jones only wrote down the student’s statements not whether they hadbeen told in the truthful or lying questioning.

126

e students’ two statements appear below:

127

Archie: “It wasn’t Edward. ““It was Bella.”Darcy: “It was Charlotte”“It was Bella”Edward: “It was Darcy”“It wasn’t Archie”Charlotte: “It was Edward.”“It wasn’t Archie”Bella: “It wasn’t Charlotte.”“It wasn’t Edward.”

128

Who put the spider in the teacher’s desk?

129

1. Edward2. Bella3. Darcy4. Charlotte5. More information needed.

130

Question 117:Dr Massey wants to measure out 0.1 litres of solution. Unfortunately

the lab assistant dropped the 200 ml measuring cylinder, and so thescientist only has a 300 ml and a half litre-measuring beaker. Assuming hecannot accurately use the beakers to measure anything less than their fullcapacity, what is the minimum volume he will have to use to be able toensure he measures the right amount?

131

1. 100 ml2. 200 ml3. 300 ml4. 400 ml5. 500 ml6. 600 ml

132

Question 118:Francis lives on a street with houses all consecutively numbered

evenly. When one adds up the value of all the house numbers it totals 870.In order to determine Francis’ house number:

1. e relative position of Francis’ house must be known.2. e number of houses in the street must be known.3. At least three of the house numbers must be known.

133

1. 1 only2. 2 only3. 3 only4. 1 and 25. 2 and 3

134

Question 119:ere were 20 people exercising in the cardio room of a gym. Four

people were about to leave when suddenly a man collapsed on one of themachines. Fortunately a doctor was on the machine beside him. Emergingfrom his office, one of the personal trainers called an ambulance. In the 5minutes that followed before the two paramedics arrived, half of thepeople who were leaving, left upon hearing the commotion, and eightpeople came in from the changing rooms to hear the paramedicspronouncing the man dead.

How many living people were left in the room?

135

1. 252. 263. 274. 285. 296. 30

136

Question 120:A man and woman are in an accident. ey both suffer the same

trauma, which causes both of them to lose blood at a rate of 0.2Litres/minute. At normal blood volume the man has 8 litres and thewoman 7 litres, and people collapse when they lose 40% of their normalblood volume.

Which TWO of the following are true?

1. e man will collapse 2 minutes before the woman.2. e woman collapses 2 minutes before the man.3. e total blood loss is 5 litres.4. e woman has 4.2 litres of blood in her body when she collapses.5. e man’s blood loss is 4.8 litres when he collapses.6. Blood loss is at a rate of 2 litres every 12 minutes.

Question 121:Jenny, Helen and Rachel have to run a distance of 13 km. Jenny runs at

a pace of 8 kmph, Helen at a pace of 10 kmph, and Rachel 11 kmph.If Jenny sets off 15 minutes before Helen, and 25 minutes before

Rachel, what order will they arrive at the destination?

137

1. Jenny, Helen, Rachel.2. Helen, Rachel, Jenny.3. Helen, Jenny, Rachel.4. Rachel, Helen, Jenny.5. Jenny, Rachel, Helen.6. None of the above.

138

Question 122:On a speci c day at a GP surgery 150 people visited the surgery and

common complaints were recorded as a percentage of total patients. Eachpatient could use their appointment to discuss up to 2 complaints. 56%

u-like symptoms, 48% pain, 20% diabetes, 40% asthma or COPD, 30%high blood pressure.

Which statement must be true?

1. A minimum of 8 patients complained of pain and u-like symptoms.2. No more than 45 patients complained of high blood pressure and

diabetes.3. ere were a maximum of 21 patients who did not complain about

u-like symptoms or high blood pressure.4. ere were actually 291 patients who visited the surgery.5. None of the above.

Question 123:All products in a store were marked up by 15%. ey were

subsequently reduced in a sale with quoted saving of 25% from the higherprice. What is the true reduction from the original price?

139

1. 5%2. 10%3. 13.75%4. 18.25%5. 20%6. None of the above.

140

Question 124:A recipe states it makes 12 pancakes and requires the following

ingredients: 2 eggs, 100g plain our, and 300ml milk. Steve is cookingpancakes for 15 people and wants to have sufficient mixture for 3pancakes each.

What quantities should Steve use to ensure this whilst using wholeeggs?

1. 2½ eggs, 125g plain our, 375ml milk2. 3 eggs , 150g plain our, 450 ml milk3. 7½ eggs, 375g plain our, 1125 ml milk4. 8 eggs, 400g plain our, 1200 ml milk5. 12 eggs, 600g plain our, 1800 ml milk6. None of the above.

Question 125:Spring Cleaning cleaners buy industrial bleach from a warehouse and

dilute it twice before using it domestically. e rst dilution is by 9:1 andthen the second, 4:1.

If the cleaners require 6 litres of diluted bleach, how much warehousebleach do they require?

141

1. 30 ml2. 120 ml3. 166 ml4. 666 ml5. 1,200 ml6. None of the above

142

Question 126:During a GP consultation in 2015, Ms Smith tells the GP about her

grandchildren. Ms Smith states that Charles is the middle grandchild andwas born in 2002. In 2010, Bertie was twice the age of Adam and that in2015 there are 5 years between Bertie and Adam. Charles and Adam areseparated by 3 years.

How old are the 3 grandchildren in 2015?

1. Adam = 16, Bertie = 11, Charles = 132. Adam = 5, Bertie = 10, Charles = 83. Adam = 10, Bertie = 15, Charles = 134. Adam = 10, Bertie = 20, Charles = 135. Adam = 11, Bertie = 10, Charles = 86. More information needed.

Question 127:Kayak Hire charges a xed at rate and then an additional half-hourly

rate. Peter hires the kayak for 3 hours and pays £14.50, and his friend Kevinhires 2 kayaks for 4hrs30mins each and pays £41. How much would

Tom pay to hire one kayak for 2 hours?

143

1. £82. £10.503. £154. £33.205. £35.706. None of the above.

144

Question 128:A ticketing system uses a common digital display of numbers 0 – 9.

e number 7 is showing. However, a number of the light elements are notcurrently working.

Which set of the following digits is possible?

145

1. 3, 4, 72. 0, 1, 93. 2, 7, 84. 0, 5, 95. 3, 8, 96. 3, 4, 9

146

Question 129:A team of 4 builders take 12 days of 7 hours work to complete a house.

e company decides to recruit 3 extra builders.How many 8 hour days will it take the new workforce to build a house?

147

1. 2 days2. 6 days3. 7 days4. 10 days5. 12 days6. More information needed

148

Question 130:All astragalus are fabacaea as are all gummifer. Acacia are not

astragalus. Which of the following statements is true?

1. Acacia are not fabacaea.2. No astragalus are also gummifer.3. All fabacae are astragalus or gummifer.4. Some acacia may be fabacaea.5. Gummifer are all acacia.6. None of the above.

Question 131:e Smiths want to reupholster both sides of their seating cushions

(dimensions shown on diagram). e fabric they are using costs £10/m,can only be bought in whole metre lengths and has a standard width of1m. Each side of a cushion must be made from a single piece of fabric. eseamstress changes a at rate of £25 per cushion. How much will it costthem to reupholster 4 cushions?

1. £ 202. £ 803. £ 1104. £ 1305. £ 1506. £ 200

149

Question 132:Lisa buys a cappuccino from either Milk or Beans Coffee shops each

day. e quality of the coffee is the same but she wishes to work out therelative costs once the loyalty scheme has been taken into account. InMilk, a regular cappuccino is £2.40, and in Beans, £2.15. However, theloyalty scheme in Milk gives Lisa a free cappuccino for every 9 she buys,whereas Beans use a points system of 10 points per full pound spent (eachpoint is worth 1p) which can be used to cover the cost of a full cappuccino.

If Lisa buys a cappuccino each day of September, which coffee shopwould work out cheaper, and by how much?

150

1. Milk, by £4.602. Beans by £6.303. Beans, by £4.604. Beans, by £2.455. Milk, by £2.456. Milk, by £6.25

151

Question 133:Paula needs to be at a meeting in Notting Hill at 11am. e route

requires her to walk 5 minutes to the 283 bus which takes 25 minutes, andthen change to the 220 bus which takes 14 minutes. Finally she walks for 3minutes to her meeting. If the 283 bus comes every 10 minutes, and the220 bus at 0 minutes, 20 minutes and 40 minutes past the hour, what is thelatest time she can leave and still be at her meeting on time?

152

1. 09.452. 09.583. 10.014. 10.055. 10.106. 10.15

153

Question 134:Two trains, a high speed train A and a slower local train B, travel from

Manchester to London. Train A travels the rst 20 km at 100 km/hr andthen at an average speed of 150km/hr. Train B travels at a constant averagespeed of 90 km/hr. If train B leaves 20 minutes before train A, at whatdistance will train A pass train B?

154

1. 75 km2. 90 km3. 100 km4. 120 km5. 150 km

155

Question 135:e university gym has an upfront cost of £35 with no contract fee, but

classes are charged at £3 each. e local gym has no joining fee and is £15per month. What is the minimum number of classes I need to attend in a12 month period to make the local gym cheaper than the university gym?

156

1. 402. 483. 494. 505. 556. 60

157

Question 136:“All medicines are drugs, but not all drugs are medicines”, goes a well-

known saying. If we accept this statement as true, and consider that allantibiotics are medicines, but no herbal drugs are medicines, then whichof the following is de nitely FALSE?

158

1. Some herbal drugs are not medicines.2. All antibiotics are drugs.3. Some herbal drugs are antibiotics.4. Some medicines are antibiotics

159

Question 137:Sonia has been studying the paths taken by various trains travelling

between London and Edinburgh on the East coast. Trains can stop at thefollowing stations: Newark, Peterborough, Doncaster, York, Northallerton,Darlington, Durham and Newcastle.

She notes the following:- All trains stop at Peterborough, York, Darlington and Newcastle.- All trains which stop at Northallerton also stop at Durham.- Each day, 50% of the trains stop at both Newark and Northallerton.- All designated “Fast” trains make less than 5 stops. All other trains make5 stops or more.- On average, 16 trains run each day.

Which of the following can be reliably concluded from theseobservations?

1. All trains, which are not designated “fast” trains, must stop atDurham.

2. No more than 8 trains on any 1 day will stop at Northallerton.3. No designated “Fast” trains will stop at Durham.4. It is possible for a train to make 5 stops, including Northallerton.5. A train which stops at Newark will also stop at Durham.

Question 138:Rakton is 5 miles directly north of Blueville. Gallford is 8 miles directly

south of Haston. Lepstone is situated 5 miles directly east of Blueville, and5 miles directly west of Gallford.

Which of the following CANNOT be reliably concluded from thisinformation?

1. Lepstone is South of Rakton2. Haston is North of Rakton

160

3. Gallford is East of Rakton4. Blueville is East of Haston5. Haston is North of Lepstone

Question 139e Eastminster Parliament is undergoing a new set of elections. ere

are 600 seats up for election, each of which will be elected separately bythe people living in that constituency. 6 parties win at least 1 seat in theelection, the Blue Party, the Red party, the Orange party, the Yellow party,the Green party and the Purple party. In order to form a government, aparty (or coalition) must hold over 50% of the seats. After the election, apolitical analysis committee produces the following report:- No party has gained more than 45% of the seats, so nobody is able to forma government by themselves.- e red and the blue party each gained over 40% of the seats.- No other party gained more than 4% of the seats.- e green party gained the 4th highest number of seats.

e red party work out that if they collaborate with the green partyand the orange party, between the 3 of them, they will have enough seatsto form a coalition government.

What is the minimum number of seats that the green party couldhave?

161

1. 52. 63. 74. 85. 96. 10

162

Questions 140-144 are based on the following information:A grandmother wants to give her 5 grandchildren £100 between them

for Christmas this year. She wants to grade the money she gives to eachgrandchild exactly so that the older children receive more than theyounger ones. She wants share the money such that she will give the 2nd

youngest child as much more than the youngest, as the 3rd youngest getsthan the 2nd youngest, as the 4th youngest gets from the 3rd youngest andso on. e result will be that the two youngest children together will getseven times as less money than the three oldest.

M is the amount of money the youngest child receives, and D thedifference between the amount the youngest and 2nd youngest childrenreceive.

Question 140:What is the expression for the amount the oldest child receives?

163

1. M2. M + D3. 2M4. 4M2

5. M + 4D6. None of the above.

164

Question 141:What is the correct expression for the total money received?

165

1. 5M = £1002. 5D + 10M = £1003. D = M/1004. 5M + 10D = £1005. M = 2D/11

166

Question 142:“e two youngest children together will get seven times less money than

the three oldest.”Which one of the following best expresses the above statement?

167

1. 7(3M + 9D) = 2M + D2. 7D = M3. 7(2M + D)= 3M + 9D4. 2(7M + D) = 3M + 9D

168

Question 143Using the statement in the previous question, what is the correct

expression for M?

169

1. 2D/112. 2/113. 10D/114. 120/11

170

Question 144:Express £100 in terms of D.

171

1. £100= 120D/112. £100= 120D/103. £100= 120/11D4. £100= 21D5. £100= 5M+10D

172

Question 145:Four young girls entered a local baking competition. ough a bit

burnt, Ellen’s carrot cake did not come last. e girl who baked a Madeirasponge had practiced a lot, and so came rst, while Jaya came third withher entry. Aleena did better than the girl who made the Tiramisu, and thegirl who made the Victoria sponge did better than Veronica.

Which TWO of the following were NOT results of the competition?

1. Veronica made a tiramisu2. Ellen came second3. Aleena made a Victoria sponge4. e Victoria sponge came in 3rd place5. e carrot cake came 3rd

Question 146:In a young children’s football league of 5 teams were; Celtic Changers,

Eire Lions, Nordic Nesters, Sorten Swipers and the Whistling Winners. Oneof the boys playing in the league, after being asked by his parents, said thatwhile he could remember the other teams’ total points he could notremember his own, the Eire Lions, score. He said that all the teams playedeach other and when teams lost they were given 0 points, when they drew,1 point, and 3 for a win. He remembered that the Celtic Changers had atotal of 2 points; the Sorten Swipers had 5; the Nordic Nesters had 8, andthe Whistling Winners 1.

How many did the boy’s team score?

173

1. 12. 43. 84. 105. 116. None of the above.

174

Question 147:T is the son of Z, Z and J are sisters, R is the mother of J and S is the son

of R.Which one of the following statements is correct?

1. T and J are cousins2. S and J are sisters3. J is the maternal uncle of T4. S is the maternal uncle of T5. R is the grandmother of Z.

Question 148:John likes to shoot bottles off a shelf. In the rst round he places 16

bottles on the shelf and knocks off 8 bottles. 3 of the knocked off bottlesare damaged and can no longer be used, whilst 1 bottle is lost. He puts theundamaged bottles back on the shelf before continuing. In the secondround he shoots six times and misses 50% of these shots. He damages twobottles with every shot which does not miss. 2 bottles also fall off the shelfat the end. He puts up 2 new bottles before continuing. In the nal round,John misses all his shots and in frustration, knocks over gets angry andknocks over 50% of the remaining bottles.

How many bottles were left on the wall after the nal round?

175

1. 22. 33. 44. 55. 66. More information needed.

176

Questions 149 - 155 are based on the information below:All lines are named after a station they serve, apart from the Oval and

Rectangle lines, which are named for their recognisable shapes. Trains runin both directions.➢ ere are express trains that run from end to end of the St Mark’s andStraightly lines in 5 and 6 minutes respectively.➢ It takes 2 minutes to change between St Mark’s and both Oval andRectangle lines, 1 minute between Rectangle and Oval.➢ It takes 3 minutes to change between the Straightly and all other lines,except with the St Mark’s line which only takes 30 seconds➢ e Straightly line is a fast line and takes only 2 minutes between stopsapart from to and from Keyton, which only takes 1 minute, and to andfrom Lime St which takes 3 minutes.➢ e Oval line is much slower and takes 4 minutes between stops, apartfrom between Baxton and Marven, and also Archite and West Quays,which takes 5 minutes.➢ e Rectangle line a reliable line; never running late but as aconsequence is much slower taking 6 minutes between stops.➢ e St Mark’s line is fast and takes 2 and half minutes between stations.➢ If a passenger reaches the end of the line, it takes three minutes tochange onto a train travelling back in the opposite direction.

177

Question 149:Assuming all lines are running on time, how long does it take to go

from St Mark’s to Archite on the St Mark’s line?

178

1. 5 minutes2. 6 minutes3. 7.5 minutes4. 10 minutes5. 12.5 minutes

179

Question 150:Assuming all lines are running on time, what’s the shortest time it will

take to go from Buft to Straightly?

180

1. 6 minutes2. 10 minutes3. 12 minutes4. 14 minutes5. 16 minutes

181

Question 151:What is the shortest time it will take to go from Baxton to Pop Lane?

182

1. 11 minutes2. 12 minutes3. 13 minutes4. 14 minutes5. 15 minutes

183

Question 152Which station, even at the quickest journey time, is furthest in terms

of time from Cambridge Square?

184

1. Catensey2. Buft3. Woopole4. Westland

185

Questions 153-155 use this additional information:On a difficult day there are signal problems whereby all lines except

the reliable line are delayed, such that train travel times between stationsare doubled. ese delays have caused overcrowding at the platformswhich means that while changeover times between lines are still the same,passengers always have to wait an extra 5 minutes on all of the platformsbefore catching the next train.

Question 153At best, how long will it now take to go from Westland to Marven?

186

1. 25 minutes2. 29 minutes3. 30 minutes4. 33 minutes5. 35 minutes

187

Question 154:ere is a bus that goes from Baxton to Archite and takes 27-31

minutes. Susan lives in Baxton and needs to get to her office in Archite asquickly as possible. With all the delays and lines out of service,

How should you advise Susan best to get to work?

1. Baxton to Archite via Barts using the Rectangle line.2. Baxton to Woopole on the Rectangle line, then Oval to Archite via

Keyton.3. It is not possible to tell between the fastest two options.4. Baxton to Woopole on the Rectangle line, then Oval to Archite via

Keyton.5. Baxton to Archite on the Oval line.6. Baxton to Archite using the bus.

Question 155:In addition to the delays the Oval line signals fail completely, so the

line falls out of service. How long will it now take to go from St Mark’s toWest Quays as quickly as possible?

188

1. 35 minutes2. 30 minutes3. 33 minutes4. 29 minutes5. 30.5 minutes6. None of the above.

189

Question 156:In an unusual horserace, only 4 horses, each with different racing

colours and numbers competed. Simon’s horse wore number 1. Lila’s horsewasn’t painted yellow nor blue, and the horse that wore 3, which waswearing red, beat the horse that came in third. Only one horse wore thesame number as the position it nished in. Arthur’s horse beat Simon’shorse, whereas Celia’s horse beat the horse that wore number 1. e horsewearing green, Celia’s, came second, and the horse wearing blue worenumber 4. Which one of the following must be true?

1. Simon’s horse was yellow and placed 3rd.2. Celia’s horse was red.3. Celia’s horse was in third place.4. Arthur’s horse was blue.5. Lila’s horse wore number 4.

Question 157:Jessie plants a tree with a height of 40 cm. e information lea et

states that the plant should grow by 20% each year for the rst 2 years, andthen 10% each year thereafter.

What is the expected height at 4 years?

190

1. 58.08 cm2. 64.89 cm3. 69.696 cm4. 89.696 cm5. 82.944 cm6. None of the above

191

Question 158A company is required to pay each employee 10% of their wage into a

pension fund if their annual total wage bill is above £200,000. However,there is a legal loophole that if the company splits over two sites, the£200,000 bill is per site. e company therefore decides to have an eastsite, and a west site.

Which employees should be grouped at the same site to minimise thecost to the company?

192

1. John, Nicola, Luke2. Nicola, Victoria, Daniel3. Nicola, Daniel, Luke4. John, Daniel, Emma5. Luke, Victoria, Emma

193

Question 159:A bus takes 24 minutes to travel from White City to Hammersmith

with no stops. Each time the bus stops to pick up and/or drop offpassengers, it takes approximately 90 seconds. is morning, the buspicked up passengers from 5 stops, and dropped off passengers at 7 stops.

What is the minimum journey time from White City to Hammersmiththis morning?

194

1. 28 minutes2. 34 minutes3. 34.5 minutes4. 36 minutes5. 37.5 minutes6. 42 minutes

195

Question 160:Sally is making a Sunday roast for her family and is planning her

schedule regarding cooking times. e chicken takes 15 minutes toprepare, 75 minutes to cook, and needs to stand for exactly 5 minutes aftercooking. e potatoes take 18 minutes to prepare, 5 minutes to boil, then50 minutes to roast, and must be roasted immediately after boiling, andthen served immediately. e vegetables require only 5 minutespreparation time and 8 minutes boiling time before serving, and can bekept warm to be served at any time after cooking. Given that the cookercan only be cooking two items at any given time and Sally can prepare onlyone item at a time, what should Sally’s schedule be if she wishes to servedinner at 4pm and wants to start cooking each item as late as possible?

196

1. Chicken 2.25, potatoes 2.47, vegetables 2.422. Chicken 2.25, potatoes 2.47, vegetables 3.473. Chicken 2.35, potatoes 3.47, vegetables 2.474. Chicken 2.35, potatoes 2.47, vegetables 3.475. Chicken 2.45, potatoes 3.47, vegetables 2.476. Chicken 2.45, potatoes 2.47, vegetables 3.47

197

Question 161:e Smiths have 4 children whose total age is 80. Paul is double the age

of Jeremy. Annie is exactly half way between the ages of Jeremy and Paul,and Rebecca is 2 years older than Paul. How old are each of the children?

198

A. Paul 23, Jeremy 12, Rebecca 26, Annie 19.B. Paul 22, Jeremy, 11, Rebecca 24, Annie 16.C. Paul 24, Jeremy 12, Rebecca 26, Annie 18.D. Paul 28, Jeremy 14, Rebecca 30, Annie 21.E. More information needed.

199

Question 162:Sarah has a jar of spare buttons that are a mix of colours and sizes. e

jar contains the following assortment of buttons:

Sarah wants to use a 25mm diameter button, but doesn’t mind if it iscream or yellow. What is the maximum number of buttons she will have toremove in order to guarantee to pick a suitable button on the nextattempt?

1. 2102. 2183. 2194. 2395. None of the above

Question 163:

200

Ben wants to optimise his score with one throw of a dart. 50% of thetime he hits a segment to either side of the one he is aiming at. With this inmind, which segment should he aim for?

[Ignore all double/triple modi ers]

201

1. 152. 163. 174. 185. 196. 20

202

Question 164:Victoria is completing her weekly shop, and the total cost of the items

is £8.65. She looks in her purse and sees that she has a £5 note, and a largeamount of change, including all types of coins. She uses the £5 note, andpays the remainder using the maximum number of coins possible in orderto remove some weight from the purse. However, the store has certainrules she has to follow when paying:- No more than 20p can be paid in “bronze” change (the name given to anycombination of 1p pieces and 2p pieces)- No more than 50p can be paid using any combination of 5p pieces and10p pieces.- No more than £1.50 can be paid using any combination of 20p pieces and50p pieces.

Victoria pays the exact amount, and does not receive any change.Under these rules, what is the maximum number of coins that Victoria canhave paid with?

203

1. 302. 313. 364. 415. 46

204

Question 165:I look at the clock on my bedside table, and I see the following digits:

However, I also see that there is a glass of water between me and theclock, which is in front of 2 adjacent gures. I know that this means these2 gures will appear reversed. For example, 10 would appear as 01, and 20would appear as 05 (as 5 on a digital clock is a reversed image of a 2). Somenumbers, such as 3, cannot appear reversed because there are no numberswhich look like the reverse of 3.

Which of the following could be the actual time?

205

1. 15:522. 21:253. 12:554. 12:225. 21:52

206

Question 166:Slavica has invaded Worsid, whilst Nordic has invaded Lorkdon.

Worsid, spotting an opportunity to bolster its amount of land and naturalresources, invades Nordic. Each of these countries is either a dictatorshipor a democracy. Slavica is a dictatorship, but Lorkdon is a democracy. 10years ago, a treaty was signed which guaranteed that no democracy wouldinvade another democracy. No dictatorship has both invaded anotherdictatorship and been invaded by another dictatorship.

Assuming the aforementioned treaty has been upheld, what style ofgovernment is practiced in Worsid?

1. Worsid is a Dictatorship.2. Worsid is a Democracy.3. Worsid does not practice either of these forms of government.4. It is impossible to tell.

Question 167:Sheila is on a shift at the local supermarket. Unfortunately, the till has

developed a fault, meaning it cannot tell her how much change to giveeach customer. A customer is purchasing the following items, at thefollowing costs:- A packet of grated cheese priced at £3.25- A whole cucumber, priced at 75p- A sh pie mix, priced at £4.00- 3 DVDs, each priced at £3.00

Sheila knows there is an offer on DVDs in the store at present, in which3 DVDs bought together will only cost £8.00.e customer pays with a £50note.

How much change will Sheila need to give the customer?

207

1. £42. £333. £344. £365. £38

208

Question 168:Ryan is cooking breakfast for several guests at his hotel. He is frying

most of the items using the same large frying pan, to get as much foodprepared in as little time as possible. Ryan is cooking Bacon, Sausages, andeggs in this pan. He calculates how much room is taken up in the pan byeach item. He calculates the following:- Each rasher of bacon takes up 7% of the available space in the pan- Each sausage takes up 3% of the available space in the pan.- Each egg takes up 12% of the available space in the pan.

Ryan is cooking 2 rashers of bacon, 4 sausages and 1 egg for eachguest. He decides to cook all the food for each guest at the same time,rather than cooking all of each item at once.

How many guests can he cook for at once?

209

1. 12. 23. 34. 45. 5

210

Question 169:SafeEat Inc. is a national food development testing agency. e

Manchester-based laboratory has a system for recording all the laboratoryemployees’ birthdays, and presenting them with cake on their birthday, inorder to keep staff morale high. Certain amounts of petty cash are setaside each month in order to fund this.40% of the staff have their birthdayin March, and the secretary works out that £60 is required to fund thebirthday cake scheme during this month.

If all birthdays cost £2 to provide a cake for, how many people work atthe laboratory?

211

1. 452. 603. 754. 1005. 150

212

Question 170:Many diseases, such as cancer, require specialist treatment, and thus

cannot be treated by a general practitioner. Instead, these diseases mustbe referred to a specialist after an initial, more generalised, medicalassessment. Bob has had a biopsy on the 1st of August on a lump found inhis abdomen. e results show that it is a tumour, with a slight chance ofbecoming metastatic, so he is referred to a waiting list for specialistradiotherapy and chemotherapy. e average waiting time in the UK forsuch treatment is 3 weeks, but in Bob’s local district, high demand meansthat it takes 50% longer for each patient to receive treatment. As he is alower risk case, with a low risk of metastasis, his waiting time is extendedby another 20%.

How many weeks will it be before Bob receives specialist treatment?

213

1. 4.52. 4.63. 5.04. 5.15. 5.46. 5.6

214

Question 171:In a class of 30 seventeen year old students, 40% drink alcohol at least

once a month. Of those who drink alcohol at least once a month, 75%drink alcohol at least once a week. 1 in 3 of the students who drink alcoholat least once a week also smoke marijuana. 1 in 3 of the students whodrink alcohol less than once a month also smoke marijuana.

How many of the students in total smoke marijuana?

215

1. 32. 43. 64. 95. 106. 15

216

Question 172:Complete the following sequence of numbers: 1, 4, 10, 22, 46, …

217

1. 842. 923. 944. 965. 100

218

Question 173:If the mean of 5 numbers is 7, the median is 8 and the mode is 3, what

must the two largest numbers in the set of numbers add up to?

219

1. 142. 213. 244. 265. 356. More information needed.

220

Question 174:Ahmed buys 1kg bags of potatoes from the supermarket. 1kg bags have

to weigh between 900 and 1100 grams. In the rst week, there are 10potatoes in the bag. e next week, there are only 5. Assuming that thepotatoes in the bag in week 1 are all the same weight as each other, andthe potatoes in the bag in week 2 are all the same weight as each other,what is the maximum possible difference between the heaviest andlightest potato in the two bags?

221

1. 50g2. 70g3. 90g4. 110g5. 130g

222

Question 175:A football tournament involves a group stage, then a knockout stage.

In the group stage, groups of four teams play in a round robin format (i.e.each team plays every other team once) and the team that wins the mostmatches in each group proceeds through to a knockout stage. In addition,the single best performing second place team across all the groups gains aplace in the knockout stage. In the knockout stage, sets of two teams playeach other and the one that wins proceeds to the next round until thereare two teams left, who play the nal.

If we start with 60 teams, how many matches are played altogether?

223

1. 752. 903. 1004. 1055. 165

224

Question 176:e last 4 digits of my card number are 2 times my PIN number, plus

200. e last 4 digits of my husband’s card number are the last four digitsof my card number doubled, plus 200. My husband’s PIN number is 2 timesthe last 4 digits of his card number, plus 200. Given that all these numbersare 4 digits long, whole numbers, and cannot begin with 0, what is thelargest number my PIN number can be?

225

1. 1,0742. 1,1743. 2,3484. 4,0965. 9,9996. More information needed.

226

Question 177:All women between 50 and 70 in the UK are invited for breast cancer

screening every 3 years. Patients at Doddinghurst Surgery are invited forscreening for the rst time at any point between their 50th and 53rdbirthday. If they ignore an invitation, they are sent reminders every 5months. We can assume that a woman is screened exactly 1 month aftershe is sent the invitation or reminder that she accepts. e next invitationfor screening is sent exactly 3 years after the previous screening.

If a woman accepts the screening on the second reminder each time,what is the youngest she can be when she has her 4th screening?

227

1. 602. 613. 624. 635. 646. 65

228

Question 178:Ellie gets a pay rise of k thousand pounds on every anniversary of

joining the company, where k is the number of years she has been at thecompany. She currently earns £40,000, and she has been at the companyfor 5.5 years. What was her salary when she started at the company?

229

1. £25,0002. £27,0003. £28,0004. £30,0005. £31,0006. £32,000

230

Question 179:Northern Line trains arrive into Kings Cross station every 8 minutes,

Piccadilly Line trains every 5 minutes and Victoria Line trains every 2minutes. If trains from all 3 lines arrived into the station exactly 15minutes ago, how long will it be before they do so again?

231

1. 24 minutes2. 25 minutes3. 40 minutes4. 60 minutes5. 65 minutes6. 80 minutes

232

Question 180:If you do not smoke or drink alcohol, your risk of getting Disease X is 1

in 12. If you smoke, you are half as likely to get Disease X as someone whodoes not smoke. If you drink alcohol, you are twice as likely to get DiseaseX. A new drug is released that halves anyone’s total risk of getting DiseaseX for each tablet taken. How many tablets of the drug would someone whodrinks alcohol have to take to reduce their risk to the same level assomeone who smoked but did not take the drug?

233

1. 02. 13. 24. 35. 46. 5

234

Questions 181 – 183 refer to the following information:ere are 20 balls in a bag. 1/2 are red. 1/10 of those that are not red

are yellow. e rest are green except 1, which is blue.Question 181:If I draw 2 balls from the bag (without replacement), what is the most

likely combination to draw?

235

1. Red and green2. Red and yellow3. Red and red4. Blue and yellow

236

Question 182:If I draw 2 balls from the bag (without replacement), what is the least

likely (without being impossible) combination to draw?

237

1. Blue and green2. Blue and yellow3. Yellow and yellow4. Yellow and green

238

Question 183:How many balls do you have to draw (without replacement) to

guarantee getting at least one of at least three different colours?

239

1. 52. 123. 134. 175. 186. 19

240

Question 184:A general election in the UK resulted in a hung parliament, with no

single party gaining more than 50% of the seats. us, the main politicalparties are engaged in discussion over the formation of a coalitiongovernment. e results of this election are shown below:

ere are a total of 699 seats, meaning that in order to form agovernment, any coalition must have at least 350 seats between them.Several of the party leaders have released statements about who they areand are not willing to form a coalition with, which are summarised asfollows:

e Conservative party and Labour are not willing to take part in acoalition together.e Liberal Democrats refuse to take part in any coalition which alsoinvolves UKIP.e Labour party will only form a coalition with UKIP if the Greenparty are also part of this coalition.e Conservative party are not willing to take part in any coalitionwith UKIP unless the Liberal Democrats are also involved.

241

Considering this information, what is the minimum number of partiesrequired to form a coalition government?

242

A) 2B) 3C) 4D) 5E) 6

243

Question 185:On Tuesday, 360 patients attend appointments at Doddinghurst

Surgery. Of the appointments that are booked in, only 90% are attended. Ofthe appointments that are booked in, 1 in 2 are for male patients, theremaining appointments are for female patients. Male patients are threetimes as likely to miss their booked appointment as female patients.

How many male patients attend appointments at DoddinghurstSurgery on Tuesday?

244

1. 302. 603. 1304. 1505. 170

245

Question 186:Every A Level student at Greentown Sixth Form studies Maths.

Additionally, 60% study Biology, 50% study Economics and 50% studyChemistry. e other subject on offer at Greentown Sixth Form is Physics.Assuming every student studies 3 subjects and that there are 60 studentsaltogether, how many students study Physics?

246

1. 152. 243. 304. 405. 606. More information needed

247

Question 187:100,000 people are diagnosed with chlamydia each year in the UK. An

average of 0.6 sexual partners are informed per diagnosis. Of these, 80%have tests for chlamydia themselves. Half of these tests come backpositive.

Assuming that each of the people diagnosed has had an average of 3sexual partners (none of them share sexual partners or have sex with eachother) and that the likelihood of having chlamydia is the same for thosepartners who are tested and those who are not, how many of the sexualpartners who were not tested (whether they were informed or not) havechlamydia?

248

1. 120,0002. 126,0003. 136,0004. 150,0005. 240,0006. 252,000

249

Question 188:In how many different positions can you place an additional tile to

make a straight line of 3 tiles?

1. 62. 73. 84. 95. 106. 117. 12

Question 189:Harry is making orange squash for his daughter’s birthday party. He

wants to have a 200ml glass of squash for each of the 20 children attendingand a 300ml glass of squash for him and each of 3 parents who are helpinghim out. He has 1,040ml of the concentrated squash.

What ratio of water: concentrated squash should he use in the dilutionto ensure he has the right amount to go around?

250

1. 2:12. 3:13. 4:14. 5:15. 6:16. 5:2

251

Question 190:4 children, Alex, Beth, Cathy and Daniel are each sitting on one of the 4

swings in the park. e swings are in a straight line. One possiblearrangement of the children is, left to right, Alex, Beth, Cathy, Daniel.

How many other possible arrangements are there?

252

1. 52. 123. 234. 245. 646. 256

253

Question 191:A delivery driver is looking to make deliveries in several towns. He is

given the following map of the various towns in the area. e lines indicateroads between the towns, along with the lengths of these roads.

e delivery driver’s vehicle has a black box which records the distancetravelled and locations visited. At the end of the day, the black boxrecording shows that he has travelled a total of 14 miles. It also shows thathe has visited one town twice, but has not visited any other town morethan once. Which of the following is a possible route the driver could havetaken?

1. Snayton Baleford Grassford Snayton Kirkelby2. Baleford Kirkelby Hondale Grassford Baleford Snayton3. Kirkelby Hondale Baleford Grassford Snayton4. Baleford Hondale Grassford Baleford Hondale Kirkelby5. Snayton Baleford Kirkelby Hondale Grassford6. None of the above.

Question 192:Ellie, her brother Tom, her sister Georgia, her mum and her dad line

up in height order from shortest to tallest for a family photograph. Ellie isshorter than her dad but taller than her mum. Georgia is shorter than bothher parents. Tom is taller than both his parents.

254

If 1 is shortest and 5 is tallest, what position is Ellie in the line?

255

1. 12. 23. 34. 45. 5

256

Question 193:Miss Briggs is trying to arrange the 5 students in her class into a

seating plan. Ashley must sit on the front row because she has pooreyesight. Danielle disrupts anyone she sits next to apart from Caitlin, soshe must sit next to Caitlin and no-one else. Bella needs to have a teachingassistant sat next to her. e teaching assistant must be sat on the lefthand side of the row, near to the teacher. Emily does not get on with Bella,so they need to be sat apart from one another. e teacher has 2 tableswhich each sit 3 people, which are arranged 1 behind the other.

Who is sitting in the front right seat?

257

1. Ashley2. Bella3. Caitlin4. Danielle5. Emily

258

Question 194:My aunt runs the dishwasher twice a week, plus an extra time for each

person who is living in the house that week. When her son is away atuniversity, she buys a new pack of dishwasher tablets every 6 weeks, butwhen her son is home she has to buy a new one every 5 weeks. How manypeople are living in the house when her son is home?

259

1. 22. 33. 44. 55. 66. 7

260

Question 195:Dates can be written in an 8 digit form, for example 26-12-2014. How

many days after 26-12-2014 would be the next time that the 8 digits weremade up of exactly 4 different integers?

261

1. 62. 83. 104. 165. 246. 30

262

Question 196:Redtown is 4 miles east of Greentown. Bluetown is 5 miles north of

Greentown. If every town is due North, South, East or West of at least twoother towns, and the only other town is Yellowtown, how many miles awayfrom Yellowtown is Redtown, and in what direction?

263

1. 4 miles east of Yellowtown.2. 5 miles south of Yellowtown.3. 5 miles north of Yellowtown.4. 4 miles west of Yellowtown.5. 5 miles west of Yellowtown.6. None of the above.

264

Question 197:Jenna pours wine from two 750ml bottles into glasses. e glasses hold

250ml, but she only lls them to 4/5 of capacity, except the last glass,where she puts whatever she has left. How full is the last glass comparedto its capacity?

265

1. 1/52. 2/53. 3/54. 4/55. 5/5

266

Question 198:ere are 30 children in Miss Ellis’s class. Two thirds of the girls in Miss

Ellis’s class have brown eyes, and two thirds of the class as a whole havebrown hair. Given that the class is half boys and half girls, what is thedifference between the minimum and maximum number of girls thatcould have brown eyes and brown hair?

267

1. 02. 23. 54. 75. 106. More information needed.

268

Question 199:A biased die with the numbers 1 to 6 on it is rolled twice. e resulting

numbers are multiplied together, and then their sum subtracted from thisresult to get the ‘score’ of the dice roll. If the probability of getting anegative (non-zero) score is 0.75, what is the probability of rolling a 1 on athird throw of the die?

269

1. 0.12. 0.23. 0.34. 0.45. 0.56. More information needed.

270

Questions 200 - 202 are based on the following information:Fares on the number 11 bus are charged at a number of pence per stop

that you travel, plus a at rate. Emma, who is 21, travels 15 stops and pays£1.70. Charlie, who is 43, travels 8 stops and pays £1.14. Children (under16) pay half the adult at rate plus a quarter of the adult charge “per stop”.

Question 200:How much does 17 year old Megan pay to travel 30 stops to college?

271

1. £0.852. £2.403. £2.904. £3.405. More information needed.

272

Question 201:How much does 14 year old Alice pay to travel 25 stops to school?

273

1. £0.502. £0.753. £1.254. £2.505. More information needed.

274

Question 202:James, who is 24, wants to get the bus into town. e town stop is the

25th stop along a straight road from his house, but he only has £2.Assuming he has to walk past the stop nearest his house, how many

stops will he need to walk past before he gets to the stop he can afford tocatch the bus from?

275

1. 42. 63. 74. 85. 96. 10

276

Questions 203 -205 are based on the following information:Emma mounts and frames paintings. Each painting needs a mount

which is 2 inches bigger in each dimension than the painting, and awooden frame which is 1 inch bigger in each dimension than the mount.Mounts are priced by multiplying 50p by the largest dimension of themount, so a mount which is 8 inches in one direction and 6 in the otherwould be £4. Frames are priced by multiplying £2 by the smallestdimension of the frame, so a frame which is 8 inches in one direction and 6in the other would be £12.

Question 203:How much would mounting and framing a painting that is 10 x 14

inches cost?

277

1. £82. £263. £274. £345. £42

278

Question 204:How much more would mounting and framing a 10 x 10 inch painting

cost than mounting and framing an 8 x 8 inch painting?

279

1. £ 3.002. £ 4.003. £ 5.004. £ 6.005. £ 7.00

280

Question 205:What is the largest square painting that can be framed for £40?

281

1. 12 inches2. 13 inches3. 14 inches4. 15 inches5. 16 inches

282

Question 206:If the word ‘CREATURES’ is coded as ‘FTEAWUTEV’, which itself

would be coded as ‘HWEAYUWEX’. What would be the second coding ofthe word ‘MAGICAL’?

283

1. QCKIGAN2. OCIIEAN3. PAJIFAN4. RALIHAQ5. RCIMGEP

284

Question 207:Jane’s mum has asked Jane to go to the shops to get some items that

they need. She tells Jane that she will pay her per kilometre that she cycleson her bike to get to the shop, plus a at rate payment for each place shegoes to. Jane receives £6 to go to the grocers, a distance of 5 km, and £4.20to go the supermarket, a distance of 3km.

How much would she earn if she then cycles to the library to changesome books, a distance of 7 km?

285

1. £7.502. £7.703. £7.804. £8.005. £8.106. £8.20

286

Question 208:In 2001-2002, 1,019 patients were admitted to hospital due to obesity.

is gure was more than 11 times higher by 2011-12 when there were11,736 patients admitted to hospital with the primary reason foradmission being obesity.

If the rate of admissions due to obesity continues to increase at thesame linear rate as it has from 2001/2 to 2011/12, how many admissionswould you expect in 2031/32?

287

1. 22,4532. 23,4373. 33,1704. 134,9645. 269,9286. 300,000

288

Question 209:A shop puts its dresses on sale at 20% off the normal selling price.

During the sale, the shop makes a 25% pro t over the price at which theybought the dresses. What is the percentage pro t when the dresses aresold at full price?

289

1. 36%2. 42.5%3. 56.25%4. 64%5. 77%6. 80%

290

Question 210:e ‘Keys MedSoc committee’ is made up of 20 students from each of

the 6 years at the university. However, the president and vice-president aresabbatical roles (students take a year out from studying). ere must be atleast two general committee students from each year, as well as thespecialist roles. Additionally, the social and welfare officers must be pre-clinical students (years 1-3) but not rst years, and the treasurer must be aclinical student (years 4-6).

Which TWO of the following statements must be true?

1. ere can be a maximum of 13 preclinical (years 1-3) students on thecommittee.

2. ere must be a minimum of 6 2nd and 3rd years.3. ere is an unequal distribution of committee members over the

different year groups.4. ere can be a maximum of 10 clinical (years 4-6) students on the

committee.5. ere can be a maximum of 2 rst year students on the committee.6. General committee members are equally spread across the 6 years.

291

1. 1 and 42. 2 and 33. 2 and 44. 3 and 65. 4 and 56. 4 and 6

292

Question 211:Friday the 13th is superstitiously considered an ‘unlucky’ day. If 13th

January 2012 was a Friday, when would the next Friday the 13th be?

293

1. March 20122. April 20123. May 20124. June 20125. July 20126. August 20127. September 20128. January has the only Friday 13th in 2012.

294

Question 212:A farmer has 18 sheep, 8 of which are male. Unfortunately, 9 sheep die,

of which 5 were female. e farmer decides to breed his remaining sheepin order to increase the size of his herd. Assuming every female gives birthto two lambs, how many sheep does the farmer have after all the femaleshave given birth once?

295

1. 102. 143. 154. 165. 19

296

Question 213:Piyanga writes a coded message for Nishita. Each letter of the original

message is coded as a letter a speci c number of characters further on inthe alphabet (the speci c number is the same for all letters). Piyanga’scoded message includes the word “PJVN”. What could the original wordsay?

297

1. CAME2. DAME3. FAME4. GAME5. LAME

298

Question 214:A number of people get on the bus at the station, which is considered

the rst stop. At each subsequent stop, 1/2 of the people on the bus get offand then 2 people get on. Between the 4th and 5th stop after the station,there are 5 people on the bus.

How many people got on at the station?

299

1. 42. 63. 204. 245. 30

300

Question 215:I have recently moved into a new house, and I am looking to repaint

my new living room. e price of several different colours of paint isdisplayed in the table below. A small can contains enough to paint 10 m2 ofwall. A large can contains enough to paint 25 m2 of wall.

I decide to paint my room a mixture of blue and white, and I purchasesome small cans of blue paint and white paint. e cost of blue paintaccounts for 50% of the total cost. I paint a total of 100 m2 of wall space.

I use up all the paint. How many m2 of wall space have I painted blue?

301

1. 10 m2

2. 20 m2

3. 40 m2

4. 50 m2

5. 80 m2

302

Question 216:Cakes usually cost 42p at the bakers. e bakers want to introduce a

new offer where the amount in pence you pay for each cake is discountedby the square of the number of cakes you buy. For example, buying 3 cakeswould mean each cake costs 33p. Isobel says that this is not a good offerfrom the baker’s perspective as it would be cheaper to buy several cakesthan just 1. How many cakes would you have to buy for the total cost to fallbelow 40p?

303

1. 22. 33. 44. 55. 6

304

Question 217:e table below shows the percentages of students in two different

universities who take various courses. ere are 800 students in UniversityA and 1200 students in University B. Biology, Chemistry and Physics arecounted as “Sciences”.

Assuming each student only takes one course, how many morestudents in University A than University B study a “Science”?

305

1. 102. 253. 604. 2505. 600

306

Question 218:Traveleasy Coaches charge passengers at a rate of 50p per mile

travelled, plus an additional charge of £5.00 for each international bordercrossed during the journey. Europremier Coaches charge £15 for everyjourney, plus 10p per mile travelled, with no charge for crossinginternational borders. Sonia is travelling from France to Germany, crossing1 international border. She nds that both companies will charge the sameprice for this journey.

How many miles is Sonia travelling?

307

1. 102. 203. 254. 355. 40

308

Question 219:Lauren, Amy and Chloe live in different cities across England. ey

decide to meet up together in London and have a meal together. Laurendeparts from Southampton at 2:30pm, and arrives in London at 4pm.Amy’s journey lasts twice as long as Lauren’s journey and she arrives inLondon at 4:15pm. Chloe departs from Sheffield at 1:30pm, and herjourney lasts an hour longer than Lauren’s journey.

Which of the following statements is de nitely true?

1. Chloe’s journey took the longest time.2. Amy departed after Lauren.3. Chloe arrived last.4. Everybody travelled by train.5. Amy departed before Chloe.

Question 220:Emma is packing to go on holiday by aeroplane. On the aeroplane, she

can take a case of dimension 50cm by 50cm by 20cm, which, when fullypacked, can weigh up to 20kg. e empty suitcase weighs 2kg. In hersuitcase, she needs to take 3 books, each of which is 0.2m by 0.1m by 0.05min size, and weighs 1000g. She would also like to take as many items ofclothing as possible. Each item of clothing has volume 1500cm3 andweighs 400 g.

Assuming each item of clothing can be squashed so as to ll any shapegap, how many items of clothing can she take in her case?

309

1. 282. 313. 344. 375. 40

310

Question 221:Alex is buying a new bed and mattress. ere are 5 bed shops Alex can

buy the bed and mattress he wants from, each of which sells the bed andmattress for a different price as follows:➢ Bed Shop A: Bed £120, Mattress £70➢ Bed Shop B: All beds and mattresses £90 each➢ Bed Shop C: Bed £140, Mattress £60. Mattress half price when you buy abed and mattress together.➢ Bed Shop D: Bed £140, Mattress £100. Get 33% off when you buy a bedand mattress together.➢ Bed Shop E: Bed £175. All beds come with a free mattress.

Which is the cheapest place for Alex to buy the bed and mattressfrom?

311

1. Bed Shop A2. Bed Shop B3. Bed Shop C4. Bed Shop D5. Bed Shop E

312

Question 222:In Joseph’s sock drawer, there are 21 socks. 4 are blue, 5 are red, 6 are

green and the rest are black. How many socks does he need to take fromthe drawer in order to guarantee he has a matching pair?

313

1. 32. 43. 54. 65. 7

314

Question 223:Printing a magazine uses 1 sheet of card and 25 sheets of paper. It also

uses ink. Paper comes in packs of 500 and card comes in packs of 60 whichare twice the price of a pack of paper. Each ink cartridge prints 130 sheetsof either paper or card. A pack of paper costs £3. Ink cartridges cost £5each.

How many complete magazines can be printed with a budget of £300?

315

1. 2102. 2203. 2304. 2405. 250

316

Question 224:Rebecca went swimming yesterday. After a while she had covered one

fth of her intended distance. After swimming six more lengths of thepool, she had covered one quarter of her intended distance. How manylengths of the pool did she intend to complete?

317

1. 402. 723. 804. 1005. 120

318

Question 225:As a special treat, Sammy is allowed to eat ve sweets from his very

large jar which contains many sweets of each of three avours – Lemon,Orange and Strawberry. He wants to eat his ve sweets in such a way thatno two consecutive sweets have the same avour.

In how many ways can he do this?

319

1. 322. 483. 724. 1085. 162

320

Question 226:Granny and her granddaughter Gill both had their birthday yesterday.

Today, Granny’s age in years is an even number and 15 times that of Gill. In4 years’ time Granny’s age in years will be the square of Gill’s age in years.

How many years older than Gill is Granny today?

321

1. 422. 493. 564. 605. 64

322

Question 227:Pierre said, “Just one of us is telling the truth”. Qadr said, “What Pierre

says is not true”. Ratna said, “What Qadr says is not true”. Sven said,“What Ratna says is not true”. Tanya said, “What Sven says is not true”.

How many of them were telling the truth?

323

1. 02. 13. 24. 35. 4

324

Question 228:Two entrants in a school’s sponsored run adopt different tactics.

Angus walks for half the time and runs for the other half, whilst Brucewalks for half the distance and runs for the other half. Both competitorswalk at 3 mph and run at 6 mph. Angus takes 40 minutes to complete thecourse.

How many minutes does Bruce take?

325

1. 302. 353. 404. 455. 50

326

Question 229:Dr Song discovers two new alien life forms on Mars. Species 8472 have

one head and two legs. Species 24601 have four legs and one head. Dr Songcounts a total of 73 heads and 290 legs in the area. How many members ofSpecies 8472 are present?

327

1. 02. 13. 724. 735. 1456. More information needed.

328

Question 230:A restaurant menu states that:“All chicken dishes are creamy and all vegetable dishes are spicy. No

creamy dishes contain vegetables.”Which of the following must be true?

1. Some chicken dishes are spicy.2. All spicy dishes contain vegetables.3. Some creamy dishes are spicy.4. Some vegetable dishes contain tomatoes.5. None of the above

Question 231:Simon and his sister Lucy both cycle home from school. One day,

Simon is kept back in detention so Lucy sets off for home rst. Lucy cyclesthe 8 miles home at 10 mph. Simon leaves school 20 minutes later thanLucy. How fast must he cycle in order to arrive home at the same time asLucy?

329

1. 10 mph2. 14 mph3. 17 mph4. 21 mph5. 24 mph

330

Question 232:Dr. Whu buys 2000 shares in a company at a rate of 50p per share. He

then sells the shares for 58p per share. Subsequently he buys 1000 sharesat 55p per share then sells them for 61p per share. ere is a charge of £20for each transaction of buying or selling shares. What is Dr. Whu’s totalpro t?

331

1. £1402. £1603. £1804. £2005. £220

332

Question 233:Jina is playing darts. A dartboard is composed of equal segments,

numbered from 1 to 20. She takes three throws, and each of the darts landsin a numbered segment. None land in the centre or in double or triplesections. What is the probability that her total score with the three darts isodd?

333

1. 1/4

2. 1/3

3. 1/2

4. 3/5

5. 2/3

334

Question 234:John Morgan invests £5,000 in a savings bond paying 5% interest per

annum. What is the value of the investment in 5 years’ time?

335

1. £6,2502. £6,3153. £6,3814. £6,4425. £6,570

336

Question 235:Joe is 12 years younger than Michael. In 5 years the sum of their ages

will be 62. How old was Michael two years ago?

337

1. 202. 243. 264. 305. 32

338

Question 236:A book has 500 pages. Vicky tears every page out that is a multiple of 3.

She then tears out every remaining page that is a multiple of 6. Finally, shetears out half of the remaining pages. If the book measures 15 cm x 30cmand is made from paper of weight 110 gm-2, how much lighter is the booknow than at the start?

339

1. 1,648 g2. 1,698 g3. 1,722 g4. 1,790 g5. 1,848 g

340

Question 237:A farmer is fertilising his crops. e more fertiliser is used, the more

the crops grow. Fertiliser costs 80p per kilo. Fertilising at a rate of 0.2 kgm-

2 increases the crop yield by £1.30 m-2. For each additional 100g of fertiliserabove 200g, the extra yield is 30% lower than the linear projection of thestated rate. At what rate of fertiliser application is it no longer costeffective to increase the dose

341

1. 0.5 kgm-2

2. 0.6 kgm-2

3. 0.7 kgm-2

4. 0.8 kgm-2

5. 0.9 kgm-2

342

Question 238:Pet-Star, Furry Friends and Creature Cuddles are three pet shops,

which each sell food for various types of pets.

Given the information above, which of the following statements canwe state is de nitely not true?

1. Regardless of which of these shops you use, the most expensiveanimal to provide food for will be a dog.

2. If I own a mixture of cats and rabbits, it will be cheaper for me toshop at Pet-star.

3. If I own 3 cats and a dog, the cheapest place for me to shop is at Pet-star

4. Furry Friends sells the cheapest food for the type of pet requiring themost food

5. If I only have one pet, Creature Cuddles will not be the cheapestplace to shop regardless of which type of pet I have.

Question 239:I record my bank balance at the start of each month for six months to

help me see how much I am spending each month. My salary is paid on the10th of each month. At the start of the year, I earn £1000 a month but fromMarch inclusive I receive a pay rise of 10%.

343

In which month did I spend the most money?

1. January2. February3. March4. April5. May

Question 240:Amy needs to travel from Southtown station to Northtown station,

which are 100 miles apart. She can travel by 3 different methods: train,aeroplane or taxi. e tables below show the different times for these 3methods. e taxi takes 1 minute to cover a distance of 1 mile. Aeroplanepassengers must be at the airport 30 minutes before their ight.Southtown airport is 10 minutes travelling time from Southtown stationand Northtown airport is 30 minutes travelling time from Northtownstation.

If Amy wants to arrive by 1700 and wants to set off as late as possible,what method of travel should she choose and what time will she leaveSouthtown station?

344

1. Flight, 15302. Train, 16003. Taxi, 15204. Train, 15005. Flight, 1610

Question 241:

In the multiplication grid below, a, b, c and d are all integers. Whatdoes d equal?

1. 182. 243. 304. 405. 45

Question 242:A sixth form college has 1,500 students. 48% are girls. 80 of the girls are

mixed race.If an equal proportion of boys and girls are mixed race, how many

mixed race boys are there in the college to the nearest 10?

345

1. 502. 603. 704. 805. 90

346

Question 243:Christine is a control engineer at the Browdon Nuclear Power Plant.

On Wednesday, she is invited to a party on the Friday, and asks hermanager if she can take the Friday off. She acknowledged that this willmean she will have worked less than the required number of hours thisweek, and offers to make this up by working extra hours next week. Hermanager suggests that instead, she works 5 hours this Sunday, and 3 extrahours next ursday to make up the required hours. Christine accepts thisproposal. Christine’s amended schedule for the week is shown below:

How many hours was Christine supposed to have worked this week, ifshe had completed her usual Friday shift?

347

1. 342. 353. 364. 385. 406. 42

348

Question 244:Leonidas notes that the time on a normal analogue clock is 0340.

What is the smaller angle between the hands on the clock?

349

1. 110°2. 120°3. 130°4. 140°5. 150°

350

Question 245:Sheila is on a shift at the local supermarket. Unfortunately, the till has

developed a fault, meaning it cannot tell her how much change to giveeach customer. A customer is purchasing the following items, at thefollowing costs:- A packet of grated cheese priced at £3.25- A whole cucumber, priced at 75p- A sh pie mix, priced at £4.00- 3 DVDs, each priced at £3.00

Sheila knows there is an offer on DVDs in the store at present, in which3 DVDs bought together will only cost £8.00.e customer pays with a £50note. How much change will Sheila need to give the customer?

1. £332. £343. £354. £365. £37

351

SECTION 1: DATA ANALYSIS

DATA ANALYSIS QUESTIONS SHOW A great variation in type anddifficulty. e best way to improve with these questions is to do lots ofpractice questions in order to familiarise yourself with the style ofquestions.

Options FirstDespite the fact that you may have lots of data to contend with, the

rule about looking at the options rst still stands in this section. is willallow you to register what type of calculation you are required to make andwhat data you might need to look at for this. Remember, Options QuestionData/Passage.

Working with NumbersPercentages frequently make an appearance in this section and it’s

vital that you’re able to work comfortably with them. For example, youshould be comfortable increasing and decreasing by percentages, andworking out inverse percentages too. When dealing with complexpercentages, break them down into their components. For example, 17.5%= 10% + 5% + 2.5%.

Graphs and TablesWhen you’re working with graphs and tables, it’s important that you

take a few seconds to check the following before actually extracting datafrom it.

352

Get into the habit of doing this whenever you are faced with data andyou’ll nd it much easier to approach these questions under time pressure.

353

DATA ANALYSIS QUESTIONS

Questions 246 to 248 are based on the following passage:It has recently been questioned as to whether the recommended ve

fruit and vegetables a day is sufficient or if it would be more bene cial toeat 7 fruit and vegetable portions each day. A study at UCL looked at thefruit and vegetables eating habits of 65,000 people in England. Analysis ofthe data showed that eating more portions was bene cial and vegetablesseemed to have a greater protective effect than fruit. e study howeverdid not distinguish whether vegetables themselves have a greaterprotective effect, or whether these people tend to eat an overall healthierdiet. A meta-analysis carried out by researchers across the world complieddata from 16 studies which encompassed over 800,000 participants, ofwhom 56,423 had died.

ey found a decline in death of around 5% from all causes for eachadditional portion of fruit or vegetables eaten, however they recorded nofurther decline for people who ate over 5 portions. Rates of cardiovasculardisease, heart disease or stroke, were shown to decline 4% for each portionup to ve, whereas the number of portions of fruit and vegetables eatenseemed to have little impact on cancer rates. e data from these studiespoints in a similar direction, that eating as much fruit and vegetables a dayis preferable, but that ve portions is sufficient to have a signi cant impacton reduction in mortality. Further studies need to look into the slightdiscrepancies, particularly why the English study found vegetables moreprotective, and if any speci c cancers may be affected by fruit and

354

vegetables even if the general cancer rates more greatly depend on otherlifestyle factors.

Question 246:Which of the following statements is correct?

1. e UCL study found no additional reduction in mortality in thosewho eat 7 rather than 5 portions of fruit and vegetables a day.

2. People who eat more fruit and vegetables are assumed to have anoverall healthier diet which is what gives them the bene cial effect.

3. e meta analysis found fruit and vegetables are more protectiveagainst cancer than cardiovascular disease

4. e English study showed fruit had more protective effects thanvegetables.

5. e meta-analysis found no additional reduction in mortality inthose who eat 7 rather than 5 portions of fruit and vegetables a day.

6. e meta-analysis suggests people who eat 7 portions would have a10% lower risk of death from any cause than those who eat 5portions.

7. Fruit and vegetables are not protective against any speci c cancers.

Question 247:If rates of death were found to be 1% lower in the UCL study than the

meta-analysis, approximately how many people died in the UCL study?

355

1. 3,0002. 3,2003. 3,9004. 4,5505. 5,200

356

Question 248Which statement does the article MOST agree with?

1. Eating more fruit and vegetables does not particularly lower the riskof any speci c cancers.

2. e UCL research suggests that the guideline should be 7 fruit andvegetables a day for England.

3. e results found by the UCL study and the meta-analysis werecontradictory.

4. Many don’t eat enough vegetables due to cost and taste.5. Fruit and vegetables are only protective against cardiovascular

disease.6. e UCL study and meta-analysis use a similar sample of

participants.7. People should aim to eat 7 portions of fruit and vegetables a day.

Questions 249-251 relate to the following table regarding averagealcohol consumption in 2010.

.

357

Question 249:Which of the following countries had the highest total beer and wine

consumption for 2010?

1. Belarus2. Lithuania3. Ireland4. France5. Andorra

Question 250:Which country has the greatest difference for spirit consumption in

2010 and 2020 projection, assuming percentages stay the same?

1. Russia2. Belarus3. Lithuania4. Grenada5. Ireland

Question 251:It was later found that some of the percentages of types of alcohol

consumed had been mixed up. If the actual amount of beer consumed byeach person in the Czech Republic was on average 4.9L, which countrywere the percentage gures mixed up with?

1. Lithuania2. Grenada3. Russia4. France5. Ireland6. Belarus

358

7. Andorra

Questions 252-255 are based on the following information:e table below shows the incidence of 6 different types of cancer in

Australia:

Question 252:Supposing there are 10 million men and 10 million women in

Australia, how many percentage points higher is the incidence of canceramongst women than amongst men?

359

1. 0.007 %2. 0.07 %3. 0.093 %4. 0.7 %5. 0.93 %

360

Question 253:Now suppose there are 11.5 million men and 10 million women in

Australia. Assuming all men are equally likely to get each type of cancerand all women are equally likely to get each type of cancer, how many ofthe types of cancer are you more likely to develop if you are a man than ifyou are a woman?

361

1. 12. 23. 34. 4

362

Question 254:Suppose that prostate, bladder and breast cancer patients visit

hospital 1 time during the rst month of 2015 and patients for all othercancers visit hospital 2 times during the rst month of 2015. 10% of cancerpatients in Australia are in Sydney, and patients in Sydney are not more orless likely to have certain types of cancer than other patients.

How many hospital visits are made by patients in Sydney with these 6cancers during the rst month of 2015?

363

1. 10,3002. 18,4003. 19,5004. 28,7005. 195,0006. 287,000

364

Question 255:Which of the graphs correctly represents the combined proportion of

men versus women with bladder cancer?

Questions 256 – 258 are based on the following information:Units of alcohol are calculated by multiplying the alcohol percentage

by the volume of liquid in litres, for example a 0.75 L bottle of wine whichis 12% alcohol contains 9 units. 1 pint = 570 ml.

Question 256:Which standard drink has the most units of alcohol in?

1. Vodka2. Beer3. Cocktail4. Wine

365

Question 257:Some guidance suggests the recommended maximum number of units

of alcohol per week for women is 14. In a week, Hannah drinks 4 standarddrinks of wine, 3 standard drinks of beer, 2 standard cocktails and 5standard vodkas. is guidance states the recommended maximumnumber of units per week for men is 21. In a week, Mark drinks 2 standarddrinks of wine, 6 standard drinks of beer, 3 standard cocktails and 10standard vodkas.

Who has exceeded their recommended maximum number of units bymore and by how many units more have they exceeded it by than the otherperson?

1. Hannah, by 1 unit2. Hannah, by 0.5 units3. Both by the same4. Mark, by 0.5 units5. Mark, by 1 unit

Question 258:How many different combinations of drinks that total 4 units are there

(the same combination in a different order doesn’t count).

1. 22. 33. 44. 55. 6

Questions 259-261 relate to the table below which showsinformation about Greentown’s population:

366

Question 259:How many males under 20 are there in Greentown?

1. 2,6502. 2,7003. 2,7304. 2,7505. 2,850

Question 260:How many females aged 40-59 are there in Greentown?

1. Between 3,000 and 4,0002. Between 4,000 and 5,0003. Between 5,000 and 6,0004. Between 6,000 and 7,000

Question 261:Which is the approximate ratio of females:males in the age group that

has the highest ratio of males:females?

1. 1.4:12. 1.9:13. 1:1.94. 1:1.4

367

]Questions 262-264 relate to the follow graph:

e graph below shows the average temperatures in London (toptrace) and Newcastle (bottom trace).

Question 262:If the average monthly temperature is the same in every year, how

many times during the period May 2007 to September 2013 inclusive is theaverage temperature the same in 2 consecutive months in Newcastle?

1. 202. 243. 254. 30

Question 263:In how many months in the period speci ed in the previous question

is the average temperature in London AND Newcastle lower than theprevious month?

1. 192. 213. 25

368

4. 32

Question 264:To the nearest 0.5 degrees Celsius, what is the average temperature

difference between Newcastle and London?

369

1. 1.5ºC2. 2ºC3. 2.5 ºC4. 3 ºC

370

Questions 265-267 concern the following data:

e pie chart to the right shows sales of ice cream across the fourquarters of a year from January to December. Sales are lowest in themonth of February. From February they increase in every subsequentmonth until they get to the maximum sales and from that point theydecrease in every subsequent month until the end of the year.

Question 265:In which month are the sales highest?

371

1. June2. July3. August4. Cannot tell

372

Question 266:If total sales of ice cream were £354,720 for the year, how much of this

was taken during Q1?

373

1. 29,4802. £29,5603. £29,6504. £29,7205. £29,800

374

Question 267:Assuming total sales revenue (i.e. before costs are taken off) is

£180,000, and that each tub of ice cream is sold for £2 and costs themanufacturer £1.50 in total production and transportation costs, howmuch pro t is made during Q2?

375

1. £15,0002. £30,0003. £45,0004. £60,000

376

Question 268:Data on the amount families spend on food per month to the nearest

£100 was collected for families with 1, 2 and 3 children. e percentage offamilies with different spending sizes is displayed below:

Which of the following statements is de nitely true?

1. More families with 1 child than families with 2 children spent £300 amonth on food.

2. e overall fraction of families spending £600 was 1/6.3. All of the families with 2 children spent under £4000 on food per year.4. e fraction of families with 1 child spending £400 on food per

month is the same as the fraction of families with 3 childrenspending this amount.

5. e average amount spent on food by families with 2 children is £410a month.

Questions 269-272 are based on the passage below:A big secondary school recently realised that there were a large

number of incidences of bullying occurring that were going unnoticed byteachers. It is possible that some believe bullying to be as much a part ofstudent life as lessons and homework. In order to tackle the problem, theschool emailed out a questionnaire to all students’ parents and askedthem to question their children about where they had experienced or seenbullying in school. ose children that answered yes were then asked if

377

they had told their teachers about it, and asked why they did not if theyhad not. ose that had told their teacher were asked whether they hadseen the teacher act upon the information and whether the bullying hadstopped as a result.

Of the 2500 school students surveyed 2210 lled in the onlinequestionnaire. e results were that, 1121 students, almost exactly half(50.7%) had seen bullying in school. Only 396 (35%) of these students told ateacher about the bullying. Of the students who told a teacher, 286 did notwitness any action following sharing of the information and of those thatdid, 60% did not notice any direct action with the bully involved.

From those students who did not report the bullying, 146 gave thereason that they didn’t think it was important. 427 cited fears of beingfound out. 212 students said they did not tell because they didn’t think theteachers would do anything about it even if they did know. Assume that allthe students who lled out the survey did so honestly.

Question 269:To the nearest integer, what percentage of students did not respond?

378

1. 10%2. 12%3. 18%4. 8%5. 5%

379

Question 270:If a student saw bullying occur and did not tell a teacher about it, what

is the probability that the reasoning for this is that they thought it to beunimportant?

380

1. 0.12. 0.153. 0.24. 0.355. 0.13

381

Question 271:After reporting the bullying, how many students saw the teacher act

on the information directly with the bully?

1. 662. 443. 1784. 1045. 118

Question 272:Which of the following does the questionnaire indicate is the best

explanation for why students at the school did not report bullying?

1. Students do not think bullying happens at their school.2. Students think the teachers will do nothing with the information.3. Students think that bullying is a part of school life.4. e student’s were worried about others nding out.

Question 273:e obesity epidemic is growing rapidly with reports of a three-fold

rise in the period from 2007 to 2012. e rates of hospital admission havealso been found to vary massively across different areas of England withthe highest rates in the North-East (56 per 100,000 people), and the lowestrates in the East of England (12 per 100,000). During almost every yearfrom 2001-12, there were around twice as many women admitted forobesity as men. e reason for this is however unclear and does not implythere are twice as many obese women as men.

What was the approximate number of admissions per 100 000 womenin the North-East in 2011-12?

382

1. 182. 263. 374. 565. 626. 74

383

Question 274:Health professionals are becoming increasingly worried by the decline

in exercise being taken by both children and adults. Around only 40% ofadults take the recommended amount of exercise which is 150 minutesper week. As well as falling rates of exercise, a shockingly low number ofindividuals eat ve portions of fruit and vegetables a day. Figures forchildren aged 5-15 fell to only 16% for boys, and 20% for girls in 2011. Datafor adults was only slightly better with 29% of women and 24% of meneating the recommended number of portions.

Using a gure of 8 million children between 5-15 years (equal ratio ofgirls to boys) in England in 2011, how many more girls than boys ate 5portions of fruit and vegetables a day?

384

1. 80,0002. 120,0003. 160,0004. 320,0005. 640,000

385

Question 275:e table below shows the leading causes of death in the UK.

Using information from the table only, which of the followingstatements is correct?

1. More women died from cancers than men.2. More than 30,000 women died due to respiratory causes.3. Dementia and Alzheimer’s is more common in women than men.4. No cause of death is of the same ranking for both men and women.

Question 276 is based on the passage below:e government has recently released a campaign lea et saying that

last year waiting times in NHS A&E departments decreased 20% comparedto the year before. e opposition has criticised this statement, saying thatthere are several de nitions which can be described as “waiting times”,and the government’s campaign lea et does not make it clear what theymean by “waiting times in A&E”.

386

e NHS watchdog has recently released the following guresdescribing different aspects of A&E departments, and the change from lastyear:

Question 276:Assuming these gures are correct, which criterion of assessment have

the government described as “waiting times in A&E” on their campaignlea et?

1. Number of people waiting for over 4 hours in A&E.2. Number of people waiting for under 4 hours in A&E.3. Number of high-priority cases waiting longer than 1 hour.4. Average time spent before being seen in A&E.5. Average time between dialling 999 and receiving treatment in A&E.6. Average waiting time for those seen in less than 4 hours.

Questions 277– 279 refer to the following information:e table below shows the nal standings at the end of the season,

after each team has played all the other teams twice each (once at home,once away). e teams are listed in order of how many points they gotduring the season. Teams get 3 points for a win, 1 point for a draw and 0points for a loss. No team got the same number of points as another team.Some of the information in the table is missing.

387

Question 277:How many points did Rovers get?

1. 02. 33. 64. 95. More information needed.

Question 278:How many games did Athletic lose?

1. 02. 13. 24. 35. More information needed.

Question 279:How many more points did United get than Rangers?

1. 72. 153. 23

388

4. 255. More information needed.

Questions 280-282 use information from the graph recordingA&E attendances and response times for NHS England from 2004 to2014. Type 1 departments are major A&E units, type 2&3 are urgent carecentres or minor injury units. e old target (2004 – June 2010) was 97.5%;the new target (July 2010 – 2015) is 95%.

Question 280:Which of the following statements is FALSE?

1. ere has been an overall increase in total A&E attendances from2004-2014.

2. e number of attendances in type 1 departments has been fairlyconstant from 2004-2014.

389

3. e new target of 4 hours waiting time has only been reached in twoquarters by type 1 departments.

4. e change in attendances is largely due to an in increase peoplegoing to type 2&3 departments.

Question 281:What percentage has the number of total attendances changed from

Q1 2004-5 to Q1 2008-9?

390

1. +5%2. –5%3. +10%4. –10%5. +15%6. –15%

391

Question 282:If the new target was achieved by type 1 departments 4 times, in what

percentage of the quarters was the target missed?

1. 25%2. 60%3. 75%4. 90%

Questions 283-284 relate to the following data:Ranjna is travelling from Manchester to Bali. She is required to make a

stopover in Singapore for which he wants to allow at least 2 hours. It takes14 hours to y from Manchester to Singapore, and 2 hours from Singaporeto Bali. e table below shows the departure times in local time[Manchester GMT, Singapore GMT + 8, Bali GMT + 8]:

Question 283:What is the latest ight Ranjna can take from Manchester to ensure

she arrives at Bali Airport by ursday 22:00?

1. 18:00 Tuesday2. 14:00 Wednesday3. 18:00 Wednesday4. 20:00 Wednesday5. 02:30 ursday

392

6. 08:30 ursday

Question 284:Ranjna takes the 08:00 ight from Manchester to Singapore on

Monday. She allows 1 hour to clear customs and collect her luggage at BaliAirport and another 45 minutes for the taxi to her hotel. At what time willshe arrive at the hotel?

1. 16.45 Monday2. 04:15 Tuesday3. 10:30 Tuesday4. 12:15 Tuesday5. 12:30 Tuesday6. 20:30 Tuesday

Question 285:

e graph below represents the percentage of adult smokers in the UKfrom 1974 to 2010. e top trace represents men and the bottom tracerepresents women. e middle trace is for both men and women.

393

Which of the following statements can be concluded from the graph?

1. e 2007 smoking ban increased the rate in decline of smokers.2. ere has been a constant reduction in percentage of smoker since

1974.3. e highest rate in decline in smoking for women was 2004-2006.4. From 1974 to 2010, the smoking rate in men decreased by a half.5. ere has always been a signi cant difference between the smoking

habits of men and women.

Question 286:e name, age, height, weight and IQ of 11 people were recorded below

in a table and a scatter plot. However, the axis labels were left out bymistake. Scale breaks are permitted.

Which variants are possible for the X and Y axis?

394

Question 287:A group of students looked at natural variation in height and arm span

within their group and got the following results:

ey then drew a scatter plot, but forgot to include names for eachpoint. ey also forgot to plot one student.

395

Which student is represented by the point marked with a *?

1. Alice2. Sarah3. Matthew4. Adam5. Emily6. Michael

Questions 288 - 294 are based on the following information:e rectangle represents women. e circle represents those that have

children. e triangle represents those that work, and the square thosethat went to university.

Question 288:What is the number of non-working women who have children and

who did not go to university?

396

1. 32. 53. 64. 75. 9

397

Question 289:What is the total number of women who have children and work?

398

1. 12. 23. 34. 115. 14

399

Question 290:How many women were surveyed in total?

400

1. 492. 513. 584. 675. 856. None of the above.

401

Question 291:What is the number of people who went to university and had

children?

402

1. 52. 113. 134. 165. 186. None of the above.

403

Question 292:What is the total number of people who went to university, or have

children but not both?

404

1. 182. 283. 354. 415. 536. None of the above.

405

Question 293:e total number of men who went to university and had children

was?

406

1. 32. 43. 54. 125. 136. 18

407

Question 294:Which of the following people were not surveyed? Choose TWO

options.

1. A non-working woman who went to university but did not havechildren.

2. A working man who went to university and has children.3. A working woman who had children but did not go to university.4. A non-working man who did not have children and did not go to

university.5. A working woman who went to university but did not have children.

Question 295:Savers“R”Us is national chain of supermarkets. e price of several

items in the supermarket is displayed below:

is week the supermarket has a sale on, with 50% off the normal priceof all meat products. Alfred visits the supermarket during this sale andpurchases a beef roasting joint, a 10 pack of sausages and a lamb shoulder,paying with a £20 note.

How much change does Alfred get?

408

1. £1.502. £5.003. £10.754. £11.755. £12.506. None of the above.

409

Question 296:e local football league table is shown below, but the number of goals

scored against Wilmslow is missing. Each team played the other teams inthe league once at home and once away during the season.

How many goals must Wilmslow have conceded?

410

1. 82. 93. 104. 115. 126. 14

411

Question 297:e heights and weights of three women with BMI’s 21, 22 and 23 were

measured. If Julie and Lydia had different weights but the same height of154 cm, and the weight of Emma, Lydia and Julie combined was 345 lbs,what was Emma’s height?

1. 158 cm2. 162 cm3. 160 cm4. 164 cm5. 165 cm

Question 298:

412

e measurements for different types of sh appear below:Which sh is shown by the point marked X?

413

1. Silver nn2. Starbug3. Lobe n4. Blondeye5. Eringill

414

e following graphs are required for questions 299-300:e graph below shows the price of crude oil in US Dollars during

2014:

e graph below shows total oil production, in millions of barrels perday:

415

Question 299:What was approximate total oil production in 2014?

1. 1,750 million barrels2. 2,146 million barrels3. 2,300 million barrels4. 2,700 million barrels5. 3,500 million barrels

Question 300:How much did oil sales total in July 2014?

416

1. $0.56 Billion2. $16.9 Billion3. $17.4 Billion4. $21.1 Billion

417

SECTION 2

SECTION 2 IS UNDOUBTEDLY THE most time-pressured section of theBMAT. is section tests GCSE biology, chemistry, physics and maths. Youhave to answer 27 questions in 30 minutes. e questions can be quitedifficult and it’s easy to get bogged down. However, it’s also the section inwhich you can improve the most quickly in so it’s well worth spendingtime on it.

Although the vast majority of questions in section 2 aren’t particularlydifficult, the intense time pressure of having to do one question everyminute makes this section the hardest in the BMAT. As with section 1, thetrick is to identify and do the easy questions whilst leaving the hard onesfor the end.

In general, the biology and chemistry questions in the BMAT requirethe least amount of time per question whilst the maths and physics aremore time-draining as they usually consist of multi-step calculations.

Gaps in Knowledge

418

e BMAT only tests GCSE level knowledge. However, there is a largevariation in content between the GCSE exam boards meaning that youmay not have covered some topics that are examinable. is is more likelyif you didn’t carry on with Biology or physics to AS level (e.g. Newtonianmechanics and parallel circuits in physics; hormones and stem cells inbiology). If you fall into this category, you are highly advised to go throughthe BMAT Speci cation and ensure that you have covered all examinabletopics. An electronic copy of this can be obtained from the official BMATwebsite at www.admissionstestingservice.org/bmat.

e questions in this book will help highlight any particular areas ofweakness or gaps in your knowledge that you may have. Upon discoveringthese, make sure you take some time to revise these topics before carryingon – there is little to be gained by attempting section 2 questions withhuge gaps in your knowledge.

MathsBeing con dent with maths is extremely important for section 2. Many

students nd that improving their numerical and algebraic skills usuallyresults in big improvements in their section 1 and 2 scores. Rememberthat maths in section 2 not only comes up in the maths question but alsoin physics (manipulating equations and standard form) and chemistry(mass calculations). So if you nd yourself consistently running out of timein section 2, spending a few hours on brushing up your basic maths skillsmay do wonders for you.

419

SECTION 2: BIOLOGY

THANKFULLY, THE BIOLOGY QUESTIONS TEND to be fairlystraightforward and require the least amount of time. You should be ableto do the majority of these within the 60 second limit (often far less). ismeans that you should be aiming to make up time in these questions. Inthe majority of cases – you’ll either know the answer or not i.e. they testadvanced recall so the trick is to ensure that there are no obvious gaps inyour knowledge.

Before going onto to do the practice questions in this book, ensure youare comfortable with the following commonly tested topics:➢ Structure of animal, plant and bacterial cells➢ Osmosis, Diffusion and Active Transport➢ Cell Division (mitosis + meiosis)➢ Family pedigrees and Inheritance➢ DNA structure and replication➢ Gene Technology & Stem Cells➢ Enzymes – Function, mechanism and examples of digestive enzymes➢ Aerobic and Anaerobic Respiration➢ e central vs. peripheral nervous system➢ e respiratory cycle including movement of ribs and diaphragm➢ e Cardiac Cycle➢ Hormones➢ Basic immunology➢ Food chains and food webs

420

➢ e carbon and nitrogen cycles

421

BIOLOGY QUESTIONS

Question 301:In relation to the human genome, which of the following are correct?

1. e DNA genome is coded by 4 different bases.2. e sugar backbone of the DNA strand is formed of glucose.3. DNA is found in the nucleus of bacteria.

422

1. 1 only2. 2 only3. 3 only4. 1 and 25. 1 and 36. 2 and 37. 1, 2 and 3

423

Question 302:Animal cells contain organelles that take part in vital processes.

Which of the following is true?

1. e majority of energy production by animal cells occurs in themitochondria.

2. e cell wall protects the animal cell membrane from outsidepressure differences.

3. e endoplasmic reticulum plays a role in protein synthesis.

424

1. 1 only2. 2 only3. 3 only4. 1 and 25. 2 and 36. 1 and 37. 1, 2 and 3

425

Question 303:With regards to animal mitochondria, which of the following is

correct?

1. Mitochondria are not necessary for aerobic respiration.2. Mitochondria are the sole cause of sperm cell movement.3. e majority of DNA replication happens inside mitochondria.4. Mitochondria are more abundant in fat cells than in skeletal muscle.5. e majority of protein synthesis occurs in mitochondria.6. Mitochondria are enveloped by a double membrane.

Question 304:In relation to bacteria, which of the following is FALSE?

1. Bacteria always lead to disease.2. Bacteria contain plasmid DNA.3. Bacteria do not contain mitochondria.4. Bacteria have a cell wall and a plasma membrane.5. Some bacteria are susceptible to antibiotics.

Question 305:In relation to bacterial replication, which of the following is correct?

1. Bacteria undergo sexual reproduction.2. Bacteria have a nucleus.3. Bacteria carry genetic information on circular plasmids.4. Bacterial genomes are formed of RNA instead of DNA.5. Bacteria require gametes to replicate.

Question 306Which of the following are correct regarding active transport?

426

1. ATP is necessary and sufficient for active transport.2. ATP is not necessary but sufficient for active transport.3. e relative concentrations of the material being transported have

little impact on the rate of active transport.4. Transport proteins are necessary and sufficient for active transport.5. Active transport relies on transport proteins that are powered by an

electrochemical gradient.

Question 307:Concerning mammalian reproduction, which of the following is

FALSE?

1. Fertilisation involves the fusion of two gametes.2. Reproduction is sexual and the offspring display genetic variation.3. Reproduction relies upon the exchange of genetic material.4. Mammalian gametes are diploid cells produced via meiosis.5. Embryonic growth requires carefully controlled mitosis.

Question 308:Which of the following apply to Mendelian inheritance?

1. It only applies to plants.2. It treats different traits as either dominant or recessive.3. Heterozygotes have a 25% chance of expressing a recessive trait.

427

1. 1 only2. 2 only3. 3 only4. 1 and 25. 1 and 36. 2 and 37. All of the above

428

Question 309:Which of the following statements are correct?

1. Hormones are secreted into the blood stream and act over longdistances at speci c target organs.

2. Hormones are substances that almost always cause muscles tocontract.

3. Hormones have no impact on the nervous or enteric systems.4. Hormones are always derived from food and never synthesised.5. Hormones act rapidly to restore homeostasis.

Question 310:With regard to neuronal signalling in the body, which of the following

are true?

1. Neuronal transmission can be caused by both electrical and chemicalstimulation.

2. Synapses ultimately result in the production of an electrical currentfor signal transduction.

3. All synapses in humans are electrical and unidirectional.

429

1. 1 only2. 2 only3. 3 only4. 1 and 25. 1 and 36. 2 and 37. 1, 2 and 3

430

Question 311:What is the primary reason that pH is controlled so tightly in

humans?

1. To allow rapid protein synthesis.2. To allow for effective digestion throughout the GI tract.3. To ensure ions can function properly in neural signalling.4. To prevent changes in electrical charge in polypeptide chains.5. To prevent changes in core body temperature.

Question 312:Which of the following statements are correct regarding bacterial cell

walls?

1. It confers bacteria protection against external environmentalstimuli.

2. It is an evolutionary remnant and now has little functionalsigni cance in most bacteria.

3. It is made up primarily of glucose in bacteria.

431

1. Only 12. Only 23. Only 34. 1 and 25. 2 and 36. 1 and 37. 1, 2 and 3

432

Question 313:Which of the following statements are correct regarding mitosis?

1. It is important in sexual reproduction.2. A single round of mitosis results in the formation of 2 genetically

distinct daughter cells.3. Mitosis is vital for tissue growth, as it is the basis for cell

multiplication.

433

1. Only 12. Only 23. Only 34. 1 and 25. 2 and 36. 1 and 37. 1, 2 and 3

434

Question 314:Which of the following is the best de nition of a mutation?

1. A mutation is a permanent change in DNA.2. A mutation is a permanent change in DNA that is harmful to an

organism.3. A mutation is a permanent change in the structure of intra-cellular

organelles caused by changes in DNA/RNA.4. A mutation is a permanent change in chromosomal structure caused

by DNA/RNA changes.

Question 315:In relation to mutations, which of the following are correct?

1. Mutations always lead to discernible changes in the phenotype of anorganism.

2. Mutations are central to natural processes such as evolution.3. Mutations play a role in cancer.

435

1. Only 12. Only 23. Only 34. 1 and 25. 2 and 36. 1 and 37. 1, 2 and 3

436

Question 316:Which of the following is the most accurate de nition of an antibody?

1. An antibody is a molecule that protects red blood cells from changesin pH.

2. An antibody is a molecule produced only by humans and has apivotal role in the immune system.

3. An antibody is a toxin produced by a pathogen to damage the hostorganism.

4. An antibody is a molecule that is used by the immune system toidentify and neutralize foreign objects and molecules.

5. Antibodies are small proteins found in red blood cells that helpincrease oxygen carriage.

Question 317:Which of the following statements about the kidney are correct?

1. e kidneys lter the blood and remove waste products from thebody.

2. e kidneys are involved in the digestion of food.3. In a healthy individual, the kidneys produce urine that contains high

levels of glucose.

437

1. Only 12. Only 23. Only 34. 1 and 25. 2 and 36. 1 and 37. 1, 2 and 3

438

Question 318:Which of the following statements are correct?

1. Hormones are slower acting than nerves.2. Hormones act for a very short time.3. Hormones act more generally than nerves.4. Hormones are released when you get a scare.5. 1 only6. 1 and 3 only7. 2 and 4 only8. 1, 3 and 4 only9. 1, 2, 3 and 4

Question 319:Which statements about homeostasis are correct?

1. Homeostasis is about ensuring the inputs within your body exceedthe outputs to maintain a constant internal environment.

2. Homeostasis is about ensuring the inputs within your body are lessthan the outputs to maintain a constant internal environment.

3. Homeostasis is about balancing the inputs within your body with theoutputs to ensure your body uctuates with the needs of the externalenvironment.

4. Homeostasis is about balancing the inputs within your body with theoutputs to maintain a constant internal environment.

439

1. 1 only2. 2 only3. 3 only4. 4 only5. 1 and 3 only6. 2 and 4 only7. 2 and 3 only

440

Question 320:Which of the following statement is true?

1. ere is more energy and biomass each time you move up a trophiclevel.

2. ere is less energy and biomass each time you move up a trophiclevel.

3. ere is more energy but less biomass each time you move up atrophic level.

4. ere is less energy but more biomass each time you move up atrophic level.

5. ere is no difference in the energy or biomass when you move up atrophic level.

Question 321:Which of the following statements are true about asexual

reproduction?

1. ere is no fusion of gametes.2. ere are two parents.3. ere is no mixing of chromosomes.4. ere is genetic variation.

441

1. 1 and 3 only2. 1 and 4 only3. 2 and 3 only4. 3 and 4 only5. 2 and 4 only6. 1, 2, 3 and 4

442

Question 322:Put the following in the order which they occur when Jonas sees a

bowl of chicken and moves towards it.

443

1. Retina2. Motor neuron3. Sensory neuron4. Brain5. Muscle

444

1. 1 - 3 - 4 - 5 - 22. 1 - 2 - 3 - 4 - 53. 5 - 1 - 3 - 2 - 44. 1 - 3 - 2 - 4 - 55. 1 - 3 - 4 - 2 - 56. 4 - 1 - 3 - 2 - 5

445

Question 323:What path does blood take from the kidney to the liver?

446

1. Pulmonary artery2. Inferior vena cava3. Hepatic artery4. Aorta5. Pulmonary vein6. Renal vein

447

1. 2 - 1 - 4 - 3 - 5 - 62. 1 - 2 - 3 - 4 - 5 - 63. 6 - 2 - 5 - 1 - 4 - 34. 6 - 2 - 1 - 5 - 4 - 35. 3 - 2 - 1 - 4 - 6 - 56. 3 - 6 - 2 - 4 - 1 – 5

448

Question 324:Which of the following statements are true about animal cloning?

1. Animals cloned from embryo transplants are genetically identical.2. e genetic material is removed from an unfertilised egg during

adult cell cloning.3. Cloning can cause a reduced gene pool.4. Cloning is only possible with mammals.

449

1. 1 only2. 2 only3. 3 only4. 4 only5. 1 and 2 only6. 1, 2 and 3 only7. 1, 2, 3 and 4

450

Question 325:Which of the following statements are true with regard to evolution?

1. Individuals within a species show variation because of differences intheir genes.

2. Bene cial mutations will accumulate within a population.3. Gene differences are caused by sexual reproduction and mutations.4. Species with similar characteristics never have similar genes.

451

1. 1 only2. 1 and 4 only3. 2 and 3 only4. 2 and 4 only5. 3 and 4 only6. 1, 2 and 3 only

452

Question 326:Which of the following genetic statements are correct?

1. Alleles are a similar version of different cells.2. If you are homozygous for a trait, you have three alleles the same for

that particular gene.3. If you are heterozygous for a trait, you have two different alleles for

that particular gene.4. To show the characteristic that is caused by a recessive allele, both

carried alleles for the gene have to be recessive.

453

1. 1 only2. 2 only3. 3 only4. 4 only5. 1 and 2 only6. 3 and 4 only7. 1, 2, and 3 only

454

Question 327:Which of the following statements are correct about meiosis?

1. e DNA content of a gamete is half that of a human red blood cell.2. Meiosis requires ATP.3. Meiosis only takes place in reproductive tissue.4. In meiosis, a diploid cell divides in such a way so as to produce two

haploid cells.

455

1. 1 only2. 3 only3. 1 and 2 only4. 2 and 3 only5. 2 and 4 only6. 1, 2, 3 and 4

456

Question 328:Put the following statements in the correct order of events for when

there is too little water in the blood.

1. Urine is more concentrated2. Pituary gland releases ADH3. Blood water level returns to normal4. Hypothalamus detects too little water in blood5. Kidney affects water level

457

1. 1 - 2 - 3 - 4 - 52. 5 - 4 - 3 - 2 - 13. 4 - 2 - 5 - 1 - 34. 3 - 2 - 4 - 1 - 55. 5 - 2 - 3 - 4 - 16. 4 - 2 – 1- 5 - 3

458

Question 329:e pH of venous blood is 7.35. Which of the following is the likely pH

of arterial blood?

459

1. 4.42. 5.23. 6.54. 7.05. 7.46. 7.95

460

Question 330:Which of the following are true of the cytoplasm?

1. e vast majority of the cytoplasm is made up of water.2. All contents of animal cells are contained in the cytoplasm.3. e cytoplasm contains electrolytes and proteins.

461

1. 1 only2. 2 only3. 3 only4. 1 and 2 only5. 1 and 3 only6. 1, 2 and 3

462

Question 331:ATP is produced in which of the following organelles?

1. e golgi apparatus2. e rough endoplasmic reticulum3. e mitochondria4. e nucleus

463

1. 1 only2. 2 only3. 3 only4. 4 only5. 1 and 26. 2 and 3 only7. 3 and 4 only8. 1, 2, 3 and 4

464

Question 332:e cell membrane:

1. Is made up of a phospholipid bilayer which only allows activetransport across it.

2. Is not found in bacteria.3. Is a semi-permeable barrier to ions and organic molecules.4. Consists purely of enzymes.

Question 333:Cells of the Polyommatus atlantica butter y of the Lycaenidae family

have 446 chromosomes. Which of the following statements about a P.atlantica butter y are correct?

1. Mitosis will produce 2 daughter cells each with 223 pairs ofchromosomes

2. Meiosis will produce 4 daughter cells each with 223 chromosomes3. Mitosis will produce 4 daughter cells each with 446 chromosomes4. Meiosis will produce 2 daughter cells each with 223 pairs of

chromosomes

465

1. 1 and 2 only2. 1 and 3 only3. 2 and 3 only4. 3 and 4 only5. 1, 2 and 3 only6. 1, 2, 3 and 4

466

Questions 334-336 are based on the following information:Assume that hair colour is determined by a single allele. e R allele is

dominant and results in black hair. e r allele is recessive for red hair.Mary (red hair) and Bob (black hair) are having a baby girl.

Question 334:What is the probability that she will have red hair?

467

1. 0% only2. 25% only3. 50% only4. 0% or 25%5. 0% or 50%6. 25% or 50%

468

Question 335:Mary and Bob have a second child, Tim, who is born with red hair.

What does this con rm about Bob?

1. Bob is heterozygous for the hair allele.2. Bob is homozygous dominant for the hair allele.3. Bob is homozygous recessive for the hair allele.4. Bob does not have the hair allele.

Question 336:Mary and Bob go on to have a third child. What are the chances that

this child will be born homozygous for black hair?

469

1. 0%2. 25%3. 50%4. 75%5. 100%

470

Question 337:Why does air ow into the chest on inspiration?

1. Atmospheric pressure is smaller than intra-thoracic pressure duringinspiration.

2. Atmospheric pressure is greater than intra-thoracic pressure duringinspiration.

3. Anterior and lateral chest expansion decreases absolute intra-thoracic pressure.

4. Anterior and lateral chest expansion increases absolute intra-thoracic pressure.

471

1. 1 only2. 2 only3. 2 and 34. 1 and 45. 1 and 36. 2 and 4

472

Question 338:Which of the following components of a food chain represent the

largest biomass?

473

1. Producers2. Decomposers3. Primary consumers4. Secondary consumers5. Tertiary consumers

474

Question 339:Concerning the nitrogen cycle, which of the following are true?

1. e majority of the Earth’s atmosphere is nitrogen.2. Most of the nitrogen in the Earth’s atmosphere is inert.3. Bacteria are essential for nitrogen xation.4. Nitrogen xation occurs during lightning strikes.

475

1. 1 and 22. 1 and 33. 2 and 34. 2 and 45. 3 and 46. 1, 2, 3 and 4

476

Question 340:Which of the following statement are correct regarding mutations?

1. Mutations always cause proteins to lose their function.2. Mutations always change the structure of the protein encoded by the

affected gene.3. Mutations always result in cancer.

477

1. Only 12. Only 23. Only 34. 1 and 25. 2 and 36. 1 and 37. 1, 2 and 38. None of the above

478

Question 341:Which of the following is not a function of the central nervous system?

479

1. Coordination of movement2. Decision making and executive functions3. Control of heart rate4. Cognition5. Memory

480

Question 342:Which of the following control mechanisms are involved in

modulating cardiac output?

1. Voluntary control.2. Sympathetic control to decrease heart rate.3. Parasympathetic control to increase heart rate.

481

1. Only 12. Only 23. Only 34. 1 and 25. 2 and 36. 1 and 37. 1, 2 and 38. None of the above

482

Question 343:Vijay goes to see his GP with fatty, smelly stools that oat on water.

Which of the following enzymes is most likely to be malfunctioning?

483

1. Amylase2. Lipase3. Protease4. Sucrase5. Lactase

484

Question 344:Which of the following statements concerning the cardiovascular

system is correct?

1. Oxygenated blood from the lungs ows to the heart via thepulmonary artery.

2. All arteries carry oxygenated blood.3. All animals have a double circulatory system.4. e superior vena cava contains oxygenated blood5. All veins have valves.6. None of the above.

Question 345:Which part of the GI tract has the least amount of enzymatic digestion

occurring?

485

1. Mouth2. Stomach3. Small intestine4. Large intestine5. Rectum

486

Question 346:Oge touches a hot stove and immediately moves her hand away.

Which of the following components are NOT involved in this reaction?

487

1. ermo-receptor2. Brain3. Spinal Cord4. Sensory nerve5. Motor nerve6. Muscle

488

1. 1 only2. 2 only3. 3 only4. 1 and 2 only5. 1, 2 and 3 only6. 3, 4, 5 and 6

489

Question 347:Which of the following represents a scenario with an appropriate

description of the mode of transport?

1. Water moving from a hypotonic solution outside of a potato cell,across the cell wall and cell membrane and into the hypertoniccytoplasm of the potato cell→ Osmosis.

2. Carbon dioxide moving across a respiring cell’s membrane anddissolving in blood plasma →Active transport.

3. Reabsorption of amino acids against a concentration gradient in theglomeruluar apparatus → Diffusion.

490

1. 1 only2. 2 only3. 3 only4. 1 and 2 only5. 2 and 3 only6. 1 and 3 only7. 1, 2 and 3

491

Question 348:Which of the following equations represents anaerobic respiration?

1. Carbohydrate + Oxygen → Energy + Carbon Dioxide + Water2. Carbohydrate → Energy + Lactic Acid + Carbon dioxide3. Carbohydrate → Energy + Lactic Acid4. Carbohydrate → Energy + Ethanol + Carbon dioxide

492

1. 1 only2. 2 only3. 3 only4. 4 only5. 1 and 26. 1 and 37. 1 and 48. 2 and 4 only9. 3 and 4 only

493

Question 349:Which of the following statements regarding respiration are correct?

1. e mitochondria are the centres for both aerobic and anaerobicrespiration.

2. e cytoplasm is the main site of anaerobic respiration.3. For every two moles of glucose that is respired aerobically, 12 moles

of CO2 are liberated.4. Anaerobic respiration is more efficient than aerobic respiration.

494

1. 1 and 22. 1 and 43. 2 and 34. 2 and 45. 3 and 4

495

Question 350:Which of the following statements are true?

1. e nucleus contains the cell’s chromosomes.2. e cytoplasm consists purely of water.3. e plasma membrane is a single phospholipid layer.4. e cell wall prevents plants cells from lysing due to osmotic

pressure.

496

1. 1 and 22. 1 and 43. 1, 3 and 44. 1, 2 and 35. 1, 2 and 46. 2, 3 and 4

497

Question 351:Which of the following statements are true about osmosis?

1. If a medium is hypertonic relative to the cell cytoplasm, the cell willgain water through osmosis.

2. If a medium is hypotonic relative to the cell cytoplasm, the cell willgain water through osmosis.

3. If a medium is hypotonic relative to the cell cytoplasm, the cell willlose water through osmosis.

4. If a medium is hypertonic relative to the cell cytoplasm, the cell willlose water through osmosis.

5. e medium’s tonicity has no impact on the movement of water.

498

1. 1 only2. 2 only3. 1 and 34. 2 and 45. 5 only

499

Question 352:Which of the following statements are true about stem cells?

1. Stem cells have the ability to differentiate into other mature types ofcells.

2. Stem cells are unable to maintain their undifferentiated state.3. Stem cells can be classi ed as embryonic stem cells or adult stem

cells.4. Stem cells are only found in embryos.

500

1. 1 and 32. 3 and 43. 2 and 34. 1 and 25. 2 and 4

501

Question 353:Which of the following are NOT examples of natural selection?

1. Giraffes growing longer necks to eat taller plants.2. Antibiotic resistance developed by certain strains of bacteria.3. Pesticide resistance among locusts in farms.4. Breeding of horses to make them run faster.

502

1. 1 only2. 4 only3. 1 and 34. 1 and 45. 2 and 4

503

Question 354:Which of the following statements are true?

1. Enzymes stabilise the transition state and therefore lower theactivation energy.

2. Enzymes distort substrates in order to lower activation energy.3. Enzymes decrease temperature to slow down reactions and lower the

activation energy.4. Enzymes provide alternative pathways for reactions to occur.

504

1. 1 only2. 1 and 23. 1 and 44. 2 and 45. 3 and 4

505

Question 355:Which of the following are examples of negative feedback?

506

1. Salivating whilst waiting for a meal.2. rowing a dart.3. e regulation of blood pH.4. e regulation of blood pressure.

507

1. 1 only2. 1 and 23. 3 and 44. 2, 3, and 45. 1, 2, 3 and 4

508

Question 356:Which of the following statements about the immune system are true?

1. White blood cells defend against bacterial and fungal infections.2. White blood cells can temporarily disable but not kill pathogens.3. White blood cells use antibodies to ght pathogens.4. Antibodies are produced by bone marrow stem cells.

509

1. 1 and 32. 1 and 43. 2 and 34. 2 and 45. 1, 2, and 36. 1, 3, and 4

510

Question 357:e cardiovascular system does NOT:

1. Deliver vital nutrients to peripheral cells.2. Oxygenate blood and transports it to peripheral cells.3. Act as a mode of transportation for hormones to reach their target

organ.4. Facilitate thermoregulation.5. Respond to exercise by increasing cardiac output to exercising

muscles.

Question 358:Which of the following statements is correct?

1. Adrenaline can sometimes decrease heart rate.2. Adrenaline is rarely released during ight or ght responses.3. Adrenaline causes peripheral vasoconstriction.4. Adrenaline only affects the cardiovascular system.5. Adrenaline travels primarily in lymphatic vessels.6. None of the above.

Question 359:Which of the following statements is true?

1. Protein synthesis occurs solely in the nucleus.2. Each amino acid is coded for by three DNA bases.3. Each protein is coded for by three amino acids.4. Red blood cells can create new proteins to prolong their lifespan.5. Protein synthesis isn’t necessary for mitosis to take place.6. None of the above.

Question 360:

511

A solution of amylase and carbohydrate is present in a beaker, wherethe pH of the contents is 6.3. Assuming amylase is saturated, which of thefollowing will increase the rate of production of the product?

512

1. Add sodium bicarbonate2. Add carbohydrate3. Add amylase4. Increase the temperature to 100o C

513

1. 1 only2. 2 only3. 3 only4. 4 only5. 1 and 26. 1 and 37. 1, 2 and 38. 1, 3 and 4

514

Question 361:Celestial Necrosis is a newly discovered autosomal recessive disorder.

A female carrier and a male with the disease produce two boys. What isthe probability that neither boy’s genotype contains the celestial necrosisallele?

515

1. 100%2. 75%3. 50%4. 25%5. 0%

516

Question 362:Which among the following has no endocrine function?

517

1. e thyroid2. e ovary3. e pancreas4. e adrenal gland5. e testes6. None of the above.

518

Question 363:Which of the following statements are true?

1. Increasing levels of insulin cause a decrease in blood glucose levels.2. Increasing levels of glycogen cause an increase in blood glucose

levels.3. Increasing levels of adrenaline decrease the heart rate.

519

1. 1 only2. 2 only3. 3 only4. 1 and 25. 2 and 36. 1 and 37. 1, 2 and 3

520

Question 364:Which of the following rows is correct?

Questions 365-367 are based on the following information:e pedigree below shows the inheritance of a newly discovered

disease that affects connective tissue called Nafram syndrome. Individual1 is a normal homozygote.

Question 365:What is the inheritance of Nafram syndrome?

521

1. Autosomal dominant2. Autosomal recessive3. X-linked dominant4. X-linked recessive5. Co-dominant

522

Question 366:Which individuals must be heterozygous for Nafram syndrome?

523

1. 1 and 22. 8 and 93. 2 and 54. 5 and 65. 6 and 86. 6 and 10

524

Question 367:Taking N to denote a diseased allele and n to denote a normal allele,

which of the following are NOT possible genotypes for 6’s parents?

1. NN x NN2. NN x Nn3. Nn x nn4. Nn x Nn5. nn x nn

525

1. 1 and 22. 1 and 33. 2 and 34. 2 and 55. 3 and 46. 4 and 5

526

Question 368:Which of the following correctly describes the passage of urine

through the body?

Question 369:Which of the following best describes the passage of blood from the

body, through the heart, back to the body?

1. Aorta → Left Ventricle → Left Atrium → Inferior Vena Cava → RightAtrium → Right Ventricle → Lungs → Aorta

2. Inferior vena cava → Left Atrium → Left Ventricle → Lungs → RightAtrium → Right Ventricle → Aorta

3. Inferior vena cava → Right Ventricle → Right Atrium → Lungs → LeftAtrium → Left Ventricle → Aorta

4. Aorta → Left Atrium → Left Ventricle → Lungs → Right Atrium → RightVentricle → Inferior Vena Cava

5. Right Atrium → Left Atrium → Inferior vena cava → Lungs → LeftAtrium → Right Ventricle → Aorta

6. None of the above.

Question 370:Which of the following best describes the events during inspiration?

527

Questions 371-372 are based on the following information:DNA is made up of the four nucleotide bases: adenine, cytosine,

guanine and thymine. A triplet repeat or codon is a sequence of threenucleotides which code for an amino acid. While there are only 20 aminoacids there are 64 different combinations of the four DNA nucleotidebases. is means that more than one combination of 3 DNA nucleotidessequences code for the same amino acid.

Question 371:Which property of the DNA code is described above?

528

1. e code is unambiguous.2. e code is universal.3. e code is non-overlapping.4. e code is degenerate.5. e code is preserved.6. e code has no punctuation.

529

Question 372:Which type of mutation does the described property protect against

the most?

1. An insertion - where a single nucleotide is inserted.2. A point mutation - where a single nucleotide is replaced for another.3. A deletion - where a single nucleotide is deleted.4. A repeat expansion - where a repeated trinucleotide sequence is

added.5. A duplication - where a piece of DNA is abnormally copied.

Question 373:Which row of the table below describes what happens when external

temperature decreases?

Question 374:Which of the following processes involve active transport?

1. Reabsorption of glucose in the kidney.2. Movement of carbon dioxide into the alveoli in the lungs.3. Movement of chemicals in a neural synapse.4. 1 only5. 2 only

530

6. 3 only7. 1 and 28. 1 and 39. 2 and 3

10. 1, 2 and 3

Question 375:Which of the following statements is correct about enzymes?

1. All enzymes are made up of amino acids only.2. Enzymes can sometimes slow the rate of reactions.3. Enzymes have no impact on reaction temperatures.4. Enzymes are heat sensitive but resistant to changes in pH.5. Enzymes are unspeci c in their substrate use.6. None of the above.

531

SECTION 2: CHEMISTRY

MOST STUDENTS DON’T STRUGGLE WITH BMAT chemistry as they’llbe studying it at A2. However, there are certain questions that even goodstudents tend to struggle with under time pressure e.g. balancingequations and mass calculations. It is essential that you’re able to do thesequickly as they take up by far the most time in the chemistry questions.

Balancing EquationsFor some reason, most students are rarely shown how to formally

balance equations – including those studying it at A-level. Balancingequations intuitively or via trial and error will only get you so far in theBMAT as the equations you’ll have to work with will be fairly complex. Toavoid wasting valuable time, it is essential you learn a method that willallow you to solve these in less than 60 seconds on a consistent basis. emethod shown below is the simplest way and requires you to be able to doquick mental arithmetic (which is something you should be aiming foranyway). e easiest way to do learn it is through an example:

e following equation shows the reaction between Iodic acid,hydrochloric acid and copper Iodide:

a HIO3 + b CuI2 + c HCl d CuCl3 + e ICl + f H2OWhat values of a, b, c, d, e and f are needed in order to balance the

equation?

532

Step 1: Pick an element and see how many atoms there are on the left and right sides.

Step 2: Form an equation to represent this. For Cu: b = d

Step 3: See if any of the answer options don’t satisfy b=d. In this case, for option E, b is 8 and d is 10. isallows us to eliminate option E.

Once you’ve eliminated as many options as possible, go back to step 1 and pick another element.

For Hydrogen (H): a + c = 2fen see if any of the answer options don’t satisfy a + c = 2f.

➢ Option A: 5 + 25 is equal to 2 x 15➢ Option B: 5 + 20 is not equal to 2 x 15➢ Option C: 5 + 20 is not equal to 2 x 15➢ Option D: 2 + 10 is not equal to 2 x 15

is allows us to eliminate option B, C and D. E has already beeneliminated. us, the only solution possible is A.

is method works best when you get given a table above as thisallows you to quickly eliminate options. However, it is still a viable methodeven if you don’t get this information.

Chemistry CalculationsEquations you MUST know:

Atomic Mass = Mass/Moles

533

Amount (mol) = Concentration (mol/dm3) x Volume (dm3)

Avogadro’s Constant:One mole of anything contains 6 x 1023 of it e.g. 5 Moles of water

contain 5 x 6 x 1023 number of water molecules.Abundances:e average atomic mass takes the abundances of all isotopes into

account. us:Ar = (Abundance of Isotope 1) x (Mass of Isotope 1) + (Abundance of

Isotope 2) x (Mass of Isotope 2) +…It’s easier to understand this by working through examples e.g.

questions 406, 412 and 439.

534

CHEMISTRY QUESTIONS

Question 376:Which of the following most accurately de nes an isotope?

1. An isotope is an atom of an element that has the same number ofprotons in the nucleus but a different number of neutrons orbitingthe nucleus.

2. An isotope is an atom of an element that has the same number ofneutrons in the nucleus but a different number of protons orbitingthe nucleus.

3. An isotope is any atom of an element that can be split to producenuclear energy.

4. An isotope is an atom of an element that has the same number ofprotons in the nucleus but a different number of neutrons in thenucleus.

5. An isotope is an atom of an element that has the same number ofprotons in the nucleus but a different number of electrons orbiting it.

Question 377:Which of the following is an example of a displacement reaction?1. Fe + SnSO4 → FeSO4 + Sn2. Cl2 + 2KBr → Br2 + 2KCl3. H2SO4 + Mg → MgSO4 + H2

4. Pb(NO3)2 + 2NaCl → PbCl2 + 2NaNO3

535

1. 1 only2. 1 and 2 only3. 2 and 3 only4. 3 and 4 only5. 1, 2 and 3 only6. 2, 3 and 4 only

536

Question 378:What values of a, b and c are needed to balance the equation below?

aCa(OH)2 + bH3PO4 → Ca3(PO4)2 + cH2O

1. a = 3 b = 2 c = 62. a = 2 b = 2 c = 43. a = 3 b = 2 c = 14. a = 1 b = 2 c = 35. a = 4 b = 2 c = 66. a = 3 b = 2 c = 4

Question 379:What values of s, t and u are needed to balance the equation below?

sAgNO3 + tK3PO4 → 3Ag3PO4 + uKNO3

537

1. s = 9 t = 3 u = 92. s = 6 t = 3 u = 93. s = 9 t = 3 u = 64. s = 9 t = 6 u = 95. s = 3 t = 3 u = 96. s = 9 t = 3 u = 3

538

Question 380:Which of the following statements are true with regard to

displacement?

1. A less reactive halogen can displace a more reactive halogen.2. Chlorine cannot displace bromine or iodine from an aqueous

solution of its salts.3. Bromine can displace iodine because of the trend of reactivity.4. Fluorine can displace chlorine as it is higher up the group.5. Lithium can displace francium as it is higher up the group.

539

1. 3 only2. 5 only3. 1 and 2 only4. 3 and 4 only5. 2 , 3 and 5 only6. 3, 4 and 5 only

540

Question 381:What mass of magnesium oxide is produced when 75g of magnesium

is burned in excess oxygen?Relative Atomic Masses: Mg = 24, O = 16

541

1. 80g2. 100g3. 125g4. 145g5. 175g6. 225g

542

Question 382:Hydrogen can combine with hydroxide ions to produce water. Which

process is involved in this?

543

1. Hydration2. Oxidation3. Reduction4. Dehydration5. Evaporation6. Precipitation

544

Question 383:Which of the following statements about Ammonia are correct?

1. It has a formula of NH3.2. Nitrogen contributes 82% to its mass.3. It can be broken down again into nitrogen and hydrogen.4. It is covalently bonded.5. It is used to make fertilisers.

545

1. 1 and 2 only2. 1 and 4 only3. 1, 2 and 3 only4. 1, 2 and 5 only5. 3, 4 and 5 only6. 1, 2, 3, 4 and 5

546

Question 384:What colour will a universal indicator change to in a solution of milk

and lipase?

547

1. From green to orange.2. From red to green.3. From purple to green.4. From purple to orange.5. From yellow to purple.6. From purple to red.

548

Question 385:Vitamin C [C6H8O6] can be arti cially synthesised from glucose

[C6H12O6]. What type of reaction is this likely to be?

549

1. Dehydration2. Hydration3. Oxidation4. Reduction5. Displacement6. Evaporation

550

Question 386:Which of the following statements are true?

1. Cu64 will undergo oxidation faster than Cu65.2. Cu65 will undergo reduction faster than Cu64.3. Cu65 and Cu64have the same number of electrons.

551

1. 1 only2. 2 only3. 3 only4. 2 and 3 only5. 1 and 3 only6. 1, 2 and 3

552

Question 387:6g of Mg24 is added to a solution containing 30g of dissolved sulphuric

acid (H2SO4). Which of the following statements are true?Relative Atomic Masses: S = 32, Mg = 24, O = 16, H = 1

1. In this reaction, the magnesium is the limiting reagent2. In this reaction, sulphuric acid is the limiting reagent3. e mass of salt produced equals the original mass of sulphuric acid

553

1. 1 only2. 2 only3. 3 only4. 1 and 2 only5. 1 and 3 only6. 2 and 3 only

554

Question 388:In which of the following mixtures will a displacement reaction occur?

555

1. Cu + 2AgNO3

2. Cu + Fe(NO3)2

3. Ca + 2H2O4. Fe + Ca(OH)2

556

1. 1 only2. 2 only3. 3 only4. 4 only5. 1 and 2 only6. 1 and 3 only7. 1, 2 and 38. 1, 2, 3 and 4

557

Question 389:Which of the following statements is true about the following chain of

metals?Na → Ca → Mg → Al → Zn

Moving from left to right:

1. e reactivity of the metals increases.2. e likelihood of corrosion of the metals increases.3. More energy is required to separate these metals from their ores.4. e metals lose electrons more readily to form positive ions.

558

1. 1 and 2 only2. 1and 3 only3. 2 and 3 only4. 1 and 4 only5. 2, 3 and 4 only6. 1, 2, 3 and 47. None of the above

559

Question 390:In which of the following mixtures will a displacement reaction occur?

1. I2 + 2KBr2. Cl2 + 2NaBr3. Br2 + 2KI

560

1. 1 only2. 2 only3. 3 only4. 1 and 2 only5. 1 and 3 only6. 2 and 3 only7. 1, 2 and 3

561

Question 391:Which of the following statements about Al and Cu are true?

1. Al is used to build aircraft because it is lightweight and resistscorrosion.

2. Cu is used to build electrical wires because it is a good insulator.3. Both Al and Cu are good conductors of heat.4. Al is commonly alloyed with other metals to make coins.5. Al is resistant to corrosion because of a thin layer of aluminium

hydroxide on its surface.

562

1. 1 and 3 only2. 1 and 4 only3. 1, 3 and 5 only4. 1, 3, 4, 5 only5. 2, 4 and 5 only6. 2, 3, 4, 5 only

563

Question 392:21g of Li7 reacts completely with excess water. Given that the molar

gas volume is 24 dm3 under the conditions, what is the volume ofhydrogen produced?

564

1. 12 dm3

2. 24 dm3

3. 36 dm3

4. 48 dm3

5. 72 dm3

6. 120 dm3

565

Question 393:Which of the following statements regarding bonding are true?

1. NaCl has stronger ionic bonds than MgCl2.2. Transition metals are able to lose varying numbers of electrons to

form multiple stable positive ions.3. All covalently bonded structures have lower melting points than

ionically bonded compounds.4. All covalently bonded structures do not conduct electricity.

566

1. 1 only2. 2 only3. 3 only4. 4 only5. 1 and 2 only6. 2 and 3 only7. 3 and 4 only8. 1, 2 and 4 only

567

Question 394:Consider the following two equations:A.C + O2 → CO2 H = -394 kJ per moleB.CaCO3 → CaO + CO2 H = + 178 kJ per moleWhich of the following statements are true?

1. Reaction A is exothermic and Reaction B is endothermic.2. CO2 has less energy than C and O2.3. CaO is more stable than CaCO3.

568

1. 1 only2. 2 only3. 3 only4. 1 and 25. 1 and 36. 2 and 37. 1, 2 and 3

569

Question 395:Which of the following are true of regarding the oxides formed by Na,

Mg and Al?

1. All of the metals and their solid oxides conduct electricity.2. MgO has stronger bonds than Na2O.3. Metals are extracted from their molten ores by fractional distillation.

570

1. 1 only2. 2 only3. 3 only4. 1 and 2 only5. 2 and 3 only6. 1, 2 and 3

571

Question 396:Which of the following pairs have the same electronic con guration?

572

1. Li+ and Na+

2. Mg2+ and Ne3. Na2+ and Ne4. O2- and a Carbon atom

573

1. 1 only2. 1 and 2 only3. 1 and 3 only4. 2 and 3 only5. 2 and 4 only6. 1, 2, 3 and 4

574

Question 397:In relation to reactivity of elements in group 1 and 2, which of the

following statements is correct?

1. Reactivity decreases as you go down group 1.2. Reactivity increases as you go down group 2.3. Group 1 metals are generally less reactive than group 2 metals.

575

1. Only 12. Only 23. Only 34. 1 and 25. 2 and 36. 1 and 3

576

Question 398:What role do catalysts ful l in an endothermic reaction?

1. ey increase the temperature, causing the reaction to occur at afaster rate.

2. ey decrease the temperature, causing the reaction to occur at afaster rate.

3. ey reduce the energy of the reactants in order to trigger thereaction.

4. ey reduce the activation energy of the reaction.5. ey increase the activation energy of the reaction.

Question 399:Tritium H3 is an isotope of Hydrogen. Why is tritium commonly

referred to as ‘heavy hydrogen’?

1. Because H3 contains 3 protons making it heavier than H1 thatcontains 1 proton.

2. Because H3 contains 3 neutrons making it heavier than H1 thatcontains 1 neutron.

3. Because H3 contains 1 neutron and 2 protons making it heavier thanH1 that contains 1 neutron and 1 proton.

4. Because H3 contains 1 proton and 2 neutrons making it heavier thanH1 that contains 1 proton.

5. Because H3 contains 3 electrons making it heavier than H1 thatcontains 1 electron.

Question 400:In relation to redox reactions, which of the following statements are

correct?

1. Oxidation describes the loss of electrons.

577

2. Reduction increases the electron density of an ion, atom or molecule.3. Halogens are powerful reducing agents.

578

1. Only 12. Only 23. Only 34. 1 and 25. 2 and 36. 1 and 3

579

Question 401:Which of the following statements is correct?

1. At higher temperatures, gas molecules move at angles that causethem to collide with each other more frequently.

2. Gas molecules have lower energy after colliding with each other.3. At higher temperatures, gas molecules attract each other resulting in

more collisions.4. e average kinetic energy of gas molecules is the same for all gases

at the same temperature.5. e momentum of gas molecules decreases as pressure increases.

Question 402:Which of the following are exothermic reactions?

1. Burning Magnesium in pure oxygen2. e combustion of hydrogen3. Aerobic respiration4. Evaporation of water in the oceans5. Reaction between a strong acid and a strong base

580

1. 1, 2 and 42. 1, 2 and 53. 1, 3 and 54. 2, 3 and 45. 1, 2, 3 and 56. 1, 2, 3, 4 and 5

581

Question 403:Ethene reacts with oxygen to produce water and carbon dioxide.

Which elements are oxidised/reduced?

1. Carbon is reduced and oxygen is oxidised.2. Hydrogen is reduced and oxygen is oxidised.3. Carbon is oxidised and hydrogen is reduced.4. Hydrogen is oxidised and carbon is reduced.5. Carbon is oxidised and oxygen is reduced.6. None of the above.

Question 404:In the reaction between Zinc and Copper (II) sulphate which elements

act as oxidising + reducing agents?

1. Zinc is the reducing agent while sulphur is the oxidising agent.2. Zinc is the reducing agent while copper in CuSO4 is the oxidising

agent.3. Copper is the reducing agent while zinc is the oxidising agent.4. Oxygen is the reducing agent while copper in CuSO4 is the oxidising

agent.5. Sulphur is the reducing agent while oxygen is the oxidising agent.6. None of the above.

Question 405:Which of the following statements is true?

1. Acids are compounds that act as proton acceptors in aqueoussolution.

2. Acids only exist in a liquid state.3. Strong acids are partially ionised in a solution.4. Weak acids generally have a pH or 6 - 7.

582

5. e reaction between a weak and strong acid produces water andsalt.

Question 406:An unknown element, Z, has 3 isotopes: Z5, Z6 and Z8. Given that the

atomic mass of Z is 7, and the relative abundance of Z5 is 20%, which of thefollowing statements are correct?

1. Z5 and Z6 are present in the same abundance.2. Z8 is the most abundant of the isotopes.3. Z8 is more abundant than Z5 and Z6combined.

583

1. 1 only2. 2 only3. 3 only4. 1 and 2 only5. 2 and 3 only6. 1 and 3 only7. 1, 2 and 38. None of the statements are correct.

584

Question 407:Which of following best describes the products when an acid reacts

with a metal that is more reactive than hydrogen?

585

1. Salt and hydrogen2. Salt and ammonia3. Salt and water4. A weak acid and a weak base5. A strong acid and a strong base6. No reaction would occur.

586

Question 408:Choose the option which balances the following equation:

a FeSO4 + b K2Cr2O7 + c H2SO4 d (Fe)2(SO4)3 + e Cr2(SO4)3 + f K2SO4 + gH2O

Question 409:Which of the following statements is correct?

1. Matter consists of atoms that have a net electrical charge.2. Atoms and ions of the same element have different numbers of

protons and electrons but the same number of neutrons.3. Over 80% of an atom’s mass is provided by protons.4. Atoms of the same element that have different numbers of neutrons

react at signi cantly different rates.5. Protons in the nucleus of atoms repel each other as they are

positively charged.6. None of the above.

Question 410:Which of the following statements is correct?

1. e noble gasses are chemically inert and therefore useless to man.2. All the noble gasses have a full outer electron shell.3. e majority of noble gasses are brightly coloured.

587

4. e boiling point of the noble gasses decreases as you progress downthe group.

5. Neon is the most abundant noble gas.

Question 411:In relation to alkenes, which of the following statements is correct?

1. ey all contain double bonds.2. ey can all be reduced to alkanes.3. Aromatic compounds are also alkenes as they contain double bonds.

588

1. Only 12. Only 23. Only 34. 1 and 25. 2 and 36. 1 and 37. All of the above.8. None of the above.

589

Question 412:Chlorine is made up of two isotopes, Cl35 (atomic mass 34.969) and

Cl37 (atomic mass 36.966). Given that the atomic mass of chlorine is35.453, which of the following statements is correct?

1. Cl35 is about 3 times more abundant than Cl37.2. Cl35 is about 10 times more abundant than Cl37.3. Cl37 is about 3 times more abundant than Cl35.4. Cl37 is about 10 times more abundant than Cl35.5. Both isotopes are equally abundant.

Question 413:Which of the following statements regarding transition metals is

correct?

1. Transition metals form ions that have multiple colours.2. Transition metals usually form covalent bonds.3. Transition metals cannot be used as catalysts as they are too

reactive.4. Transition metals are poor conductors of electricity.5. Transition metals are frequently referred to as f-block elements.

Question 414:20 g of impure Na23 reacts completely with excess water to produce

8,000 cm3 of hydrogen gas under standard conditions. What is thepercentage purity of sodium?

[Under standard conditions 1 mole of gas occupies 24 dm3]

590

1. 88.0%2. 76.5%3. 66.0%4. 38.0%5. 15.3%

591

Question 415:An organic molecule contains 70.6% Carbon, 5.9% Hydrogen and 23.5%

Oxygen. It has a molecular mass of 136. What is its chemical formula?

592

1. C4H4O2. C5H4O3. C8H8O2

4. C10H8O2

5. C2H2O

593

Question 416:Choose the option which balances the following reaction:

aS + bHNO3 cH2SO4 + dNO2 + eH2O

Question 417:Which of the following statements is true?

1. Ethane and ethene can both dissolve in organic solvents.2. Ethane and ethene can both be hydrogenated in the presence of

Nickel.3. Breaking C=C requires double the energy needed to break C-C.

594

1. 1 only2. 2 only3. 3 only4. 1 and 2 only5. 2 and 3 only6. 1 and 3 only7. 1, 2 and 3

595

Question 418:Diamond, Graphite, Methane and Ammonia all exhibit covalent

bonding. Which row adequately describes the properties associated witheach?

596

1. 1 and 2 only2. 2 and 3 only3. 1 and 3 only4. 1 and 4 only5. 1, 2 and 36. 2, 3 and 47. 1,2 and 48. 1, 2, 3 and 4

597

Question 419:Which of the following statements about catalysts are true?

1. Catalysts reduce the energy required for a reaction to take place.2. Catalysts are used up in reactions.3. Catalysed reactions are almost always exothermic.

598

1. 1 only2. 2 only3. 1 and 24. 2 and 35. 1, 2 and

599

Question 420:What is the name of the molecule below?

600

1. But-1-ene2. But-2-ene3. Pent-3-ene4. Pent-1-ene5. Pent-2-ene6. Pentane7. Pentanoic acid

601

Question 421:Which of the following statements is correct regarding Group 1

elements? [Excluding Hydrogen]

1. e oxidation number of Group 1 elements usually decreases in mostreactions.

2. Reactivity decreases as you progress down Group 1.3. Group 1 elements do not react with water.4. All Group 1 elements react spontaneously with oxygen.5. All of the above.6. None of the above.

Question 422:Which of the following statements about electrolysis are correct?

1. e cathode attracts negatively charged ions.2. Atoms are reduced at the anode.3. Electrolysis can be used to separate mixtures.

602

1. Only 12. Only 23. Only 34. 1 and 25. 2 and 36. 1 and 37. 1, 2 and 38. None of the above.

603

Question 423:Which of the following is NOT an isomer of pentane?

604

1. CH3CH2CH2CH2CH3

2. CH3C(CH3)CH3CH3

3. CH3(CH2)3CH3

4. CH3C(CH3)2CH3

605

Question 424:Choose the option which balances the following reaction:Cu + HNO3 Cu(NO3)2 + NO + H2O

1. 8 Cu + 3 HNO3 8 Cu(NO3)2 + 4 NO + 2 H2O2. 3 Cu + 8 HNO3 2 Cu(NO3)2 + 3 NO + 4 H2O3. 5Cu + 7HNO3 5 Cu(NO3)2 + 4 NO + 8 H2O4. 6 Cu + 10 HNO3 6 Cu(NO3)2 + 3 NO + 7 H2O5. 3 Cu + 8 HNO3 3 Cu(NO3)2 + 2 NO + 4 H2O

Question 425:What of the following statements regarding alkenes is correct?

1. Alkenes are an inorganic homologous series.2. Alkenes always have three times as many hydrogen atoms as they do

carbon atoms.3. Bromine water changes from clear to brown in the presence of an

alkene.4. Alkenes are more reactive than alkanes because they are

unsaturated.5. Alkenes frequently take part in subtraction reactions.6. All of the above.

Question 426:Which of the following statements is correct regarding Group 17?

1. All Group 17 elements are electrophilic and therefore form negativelycharged ions.

2. All Group 17 elements are gasses a room temperature.3. e reaction between Sodium and Fluorine is less vigorous than

Sodium and Iodine.4. All Group 17 elements are non-coloured.

606

5. Some Group 17 elements are found naturally as unbonded atoms.6. All of the above.7. None of the above.

Question 427:Why does the electrolysis of NaCl solution (brine) require the strict

separation of the products of anode and cathode?

1. To prevent the preferential discharge of ions.2. In order to prevent spontaneous combustion.3. In order to prevent production of H2.4. In order to prevent the formation of HCl.5. In order to avoid CO poisoning.6. All of the above.

Question 428:In relation to the electrolysis of brine (NaCl), which of the following

statements are correct?

1. Electrolysis results in the production of hydrogen and chlorine gas.2. Electrolysis results in the production of sodium hydroxide.3. Hydrogen gas is released at the anode and chlorine gas is released at

the cathode.

607

1. Only 12. Only 23. Only 34. 1 and 25. 1 and 36. 2 and 37. All of the above

608

Question 429:Which of the following statements is correct?

1. Alkanes consist of multiple C-H bonds that are very weak.2. An alkane with 14 hydrogen atoms is called Heptane.3. All alkanes consist purely of hydrogen and carbon atoms.4. Alkanes burn in excess oxygen to produce carbon monoxide and

water.5. Bromine water is decolourised in the presence of an alkane.6. None of the above.

Question 430:Which of the following statements are correct?

1. All alcohols contain a hydroxyl functional group.2. Alcohols are highly soluble in water.3. Alcohols are sometimes used a biofuels.

609

1. Only 12. Only 23. Only 34. 1 and 25. 2 and 36. 1 and 37. 1, 2 and 3

610

Question 431:Which row of the table below is correct?

Question 432:How many grams of magnesium chloride are formed when 10 grams of

magnesium oxide are dissolved in excess hydrochloric acid? Relativeatomic masses: Mg = 24, O = 16, H = 1, Cl = 35.5

611

1. 10.002. 14.953. 20.004. 23.755. 47.556. More information needed

612

Question 433:Pentadecane has the molecular formula C15H32. Which of the

following statements is true?

1. Pentadecane has a lower boiling point than pentane.2. Pentadecane is more ammable than pentane.3. Pentadecane is more volatile than pentane.4. Pentadecane is more viscous than pentane.5. All of the above.6. None of the above.

Question 434:e rate of reaction is normally dependent upon:

1. e temperature.2. e concentration of reactants.3. e concentration of the catalyst.4. e surface area of the catalyst.

613

1. 1 and 22. 2 and 33. 2, 3 and 44. 1, 3 and 45. 1, 2 and 36. 1, 2, 3 and 4

614

Question 435:e equation below shows the complete combustion of a sample of

unknown hydrocarbon in excess oxygen.CaHb + O2 → cCO2 + dH2O

e product yielded 176 grams of CO2 and 108 grams of H2O. What isthe most likely formula of the unknown hydrocarbon? Relative atomicmasses: H = 1, C = 12, O = 16.

615

1. CH4

2. CH3

3. C2H6

4. C3H9

5. C2H4

6. C4H10

616

Question 436:What type of reaction must ethanol undergo in order to be converted

to ethylene oxide (C2H4O)?

617

1. Oxidation2. Reduction3. Dehydration4. Hydration5. Redox6. All of the above

618

Question 437:What values of a, b and c balance the equation below?

a Ba3N2 + 6H2O → b Ba(OH)2 + c NH3

Question 438:What values of a, b and c balance the equation below?

a FeS + 7O2 → b Fe2O3 + c SO2

Question 439:Magnesium consists of 3 isotopes: Mg23, Mg25, and Mg26

which arefound naturally in a ratio of 80:10:10.

Calculate the relative atomic mass of magnesium.

619

1. 23.32. 23.43. 23.54. 23.65. 24.66. 25.27. 25.5

620

Question 440:Consider the three reactions:

1. Cl2 + 2Br- → 2Cl- + Br2

2. Cu2+ + Mg→ Cu + Mg2+

3. Fe2O3 + 3CO→ 2Fe + 3CO2

Which of the following statements are correct?

1. Cl2 and Fe2O3 are reducing agents.2. CO and Cu2+ are oxidising agents.3. Br2 is a stronger oxidising agent than Cl2.4. Mg is a stronger reducing agent than Cu.5. All of the above.6. None of the above.

Question 441:Which row best describes the properties of NaCl?

Question 442:80g of Sodium hydroxide reacts with excess zinc nitrate to produce

zinc hydroxide. Calculate the mass of zinc hydroxide produced. Relative

621

atomic mass: N = 14, Zn = 65, O = 16, Na = 23.

622

1. 49g2. 95g3. 99g4. 100g5. 198g6. More information needed.

623

Question 443:Which of the following statements is correct?

1. e reaction between all Group 1 metals and water is exothermic.2. All Group 1 metals react with water to produce a metal oxide.3. All Group 1 metals react with water to produce elemental hydrogen.4. Sodium reacts more vigorously with water than Potassium.5. All of the above.6. None of the above.

Question 444:Which of the following statements is correct?

1. NaCl can be separated using sieves.2. CO2 can be separated using electrolysis.3. Dyes in a sample of ink can be separated using chromatography.4. Oil and water can be separated using fractional distillation.5. Methane and diesel can be separated using a separating funnel.6. None of the above.

Question 445:Which of the following statements about the reaction between

caesium and uoride are correct?

1. It is an exothermic reaction and therefore requires catalysts.2. It results in the formation of a salt.3. e addition of water will make the reaction safer.

624

1. Only 12. Only 23. Only 34. 1 and 25. 2 and 36. 1 and 37. All of the above.8. None of the above.

625

Question 446:Which of the following statements is generally true about stable

isotopes?

1. e nucleus contains an equal number of neutrons and protons.2. e nuclear charge is equal and opposite to the peripheral charge

due to the orbiting electrons.3. ey can all undergo radioactive decay into more stable isotopes.

626

1. Only 12. Only 23. Only 34. 1 and 25. 2 and 36. 1 and 37. All of the above.8. None of the above.

627

Question 447:Why do most salts have very high melting temperatures?

1. eir surface is able to radiate away a signi cant portion of heat totheir environment.

2. e ionic bonds holding them together are very strong.3. e covalent bonds holding them together are very strong.4. ey tend to form large macromolecules as each salt molecule bonds

with multiple other molecules.5. All of the above.

Question 448:A bottle of water contains 306ml of pure deionised water. How many

protons are in the bottle from the water? Avogadro Constant = 6 x 1023.

628

1. 1 x 1022

2. 1 x 1023

3. 1 x 1024

4. 1 x 1025

5. 1 x 1026

629

Question 449:On analysis, an organic substance is found to contain 41.4% Carbon,

55.2% Oxygen and 3.45% Hydrogen by mass. Which of the following couldbe the empirical formula of this substance?

630

1. C3O3H6

2. C3O3H12

3. C4O2H4

4. C4O4H4

5. C4O2H8

6. More information needed.

631

Question 450:A is a Group 2 element and B is a Group 17 element. Which row best

describes what happens when A reacts with B?

632

SECTION 2: PHYSICS

IF YOU HAVEN’T DONE PHYSICS at AS then you’ll have to ensure thatyou are con dent with commonly examined topics like Newtonianmechanics, electrical circuits and radioactive decay as you may not havecovered these at GCSE depending on the speci cation you did.

e rst step to improving in this section is to memorise by rote all theequations listed on the next page.

e majority of the physics questions involve a fair bit of maths – thismeans you need to be comfortable with converting between units and alsopowers of 10. Most questions require two step calculations. Considerthe example:

A metal ball is released from the roof a 20 metre building. Assumingair resistance equals is negligible; calculate the velocity at which the ballhits the ground. [g = 10ms-2]

1. 5 ms-1

2. 10 ms-1

3. 15 ms-1

4. 20 ms-1

5. 25 ms-1

When the ball hits the ground, all of its gravitational potential energyhas been converted to kinetic energy. us, Ep = Ek:

633

us,

Here, you were required to not only recall two equations but apply andrearrange them very quickly to get the answer; all in under 60 seconds.us, it is easy to understand why the physics questions are generallymuch harder than the biology and chemistry ones.

Note that if you were comfortable with basic Newtonian mechanics,you could have also solved this using a single suvat equation:

is is why you’re strongly advised to learn the ‘suvat’ equations onthe next page even if they’re technically not on the syllabus.

SI UnitsRemember that in order to get the correct answer you must always

work in SI units i.e. do your calculations in terms of metres (notcentimetres) and kilograms (not grams), etc.

Formulas you MUST know:

634

Equations of Motion:➢ s = ut + 0.5at2

➢ v= u + at➢ a = (v-u)/t➢ v2 = u2 + 2as

Equations relating to Force:➢ Force = mass x acceleration➢ Force = Momentum/Time➢ Pressure = Force / Area➢ Moment of a Force = Force x Distance➢ Work done = Force x Displacement

635

For objects in equilibrium:➢ Sum of Clockwise moments = Sum of Anti-clockwise moments➢ Sum of all resultant forces = 0

Equations relating to Energy:➢ Kinetic Energy = 0.5 mv2

➢ Δ in Gravitational Potential Energy = mgΔh➢ Energy Efficiency = (Useful energy/ Total energy) x 100%

Equations relating to Power:➢ Power = Work done / time➢ Power = Energy transferred / time➢ Power = Force x velocity

Electrical Equations:

636

➢ Q = It➢ V = IR➢ P = IV = I2R = V2/R➢ V = Potential difference (V, Volts)➢ R = Resistance (Ohms)➢ P = Power (W, Watts)➢ Q = Charge (C, Coulombs)➢ t= Time (s, seconds)

637

For Transformers: where:➢ V: Potential difference➢ n: Number of turns➢ p: Primary➢ s: Secondary

Other:➢ Weight = mass x g➢ Density = Mass / Volume➢ Momentum = Mass x Velocity➢ g = 9.81 ms-2 (unless otherwise stated)

638

PHYSICS QUESTIONS

Question 451:Which of the following statements are FALSE?

1. Electromagnetic waves cause things to heat up.2. X-rays and gamma rays can knock electrons out of their orbits.3. Loud sounds can make objects vibrate.4. Wave power can be used to generate electricity.5. Since waves carry energy away, the source of a wave loses energy.6. e amplitude of a wave determines its mass.

Question 452:A spacecraft is analysing a newly discovered exoplanet. A rock of

unknown mass falls on the planet from a height of 30 m. Given that g = 5.4ms-2 on the planet, calculate the speed of the rock when it hits the groundand the time it took to fall.

Question 453:

639

A canoe oating on the sea rises and falls 7 times in 49 seconds. ewaves pass it at a speed of 5 ms-1. How long are the waves?

640

1. 12 m2. 22 m3. 25 m4. 35 m5. 57 m6. 75 m

641

Question 454:Miss Orrell lifts her 37.5 kg bike for a distance of 1.3 m in 5 s. e

acceleration of free fall is 10 ms-2. What is the average power that shedevelops?

642

1. 9.8 W2. 12.9 W3. 57.9 W4. 79.5 W5. 97.5W6. 98.0 W

643

Question 455:A truck accelerates at 5.6 ms-2 from rest for 8 seconds. Calculate the

nal speed and the distance travelled in 8 seconds.

Question 456:Which of the following statements is true when a sky diver jumps out

of a plane?

1. e sky diver leaves the plane and will accelerate until the airresistance is greater than their weight.

2. e sky diver leaves the plane and will accelerate until the airresistance is less than their weight.

3. e sky diver leaves the plane and will accelerate until the airresistance equals their weight.

4. e sky diver leaves the plane and will accelerate until the airresistance equals their weight squared.

5. e sky diver will travel at a constant velocity after leaving the plane.

Question 457:A 100 g apple falls on Isaac’s head from a height of 20 m. Calculate the

apple’s momentum before the point of impact. Take g = 10 ms-2

644

1. 0.1 kgms-1

2. 0.2 kgms-1

3. 1 kgms-1

4. 2 kgms-1

5. 10 kgms-1

6. 20 kgms-1

645

Question 458:Which of the following do all electromagnetic waves all have in

common?

1. ey can travel through a vacuum.2. ey can be re ected.3. ey are the same length.4. ey have the same amount of energy.5. ey can be polarised.

646

1. 1, 2 and 3 only2. 1, 2, 3 and 4 only3. 4 and 5 only4. 3 and 4 only5. 1, 2 and 5 only6. 1 and 5 only

647

Question 459:A battery with an internal resistance of 0.8 Ω and e.m.f of 36 V is used

to power a drill with resistance 1 Ω. What is the current in the circuitwhen the drill is connected to the power supply?

648

1. 5 A2. 10 A3. 15 A4. 20 A5. 25 A6. 30 A

649

Question 460:Officer Bailey throws a 20 g dart at a speed of 100 ms-1. It strikes the

dartboard and is brought to rest in 10 milliseconds. Calculate the averageforce exerted on the dart by the dartboard.

650

1. 0.2 N2. 2 N3. 20 N4. 200 N5. 2,000 N6. 20,000 N

651

Question 461:Professor Huang lifts a 50 kg bag through a distance of 0.7 m in 3 s.

What average power does she develop to 3 signi cant gures? Take g =10ms-2

652

1. 112 W2. 113 W3. 114 W4. 115 W5. 116 W6. 117 W

653

Question 462:An electric scooter is travelling at a speed of 30 ms-1 and is kept going

against a 50 N frictional force by a driving force of 300 N in the direction ofmotion. Given that the engine runs at 200 V, calculate the current in thescooter.

654

1. 4.5 A2. 45 A3. 450 A4. 4,500 A5. 45,000 A6. More information needed.

655

Question 463:Which of the following statements about the physical de nition of

work are correct?

1. e unit of work is equivalent to Kgms-2.2. Work is de ned as a force causing displacement of the body upon

which it acts.

656

1. Only 12. Only 23. Only 34. 1 and 25. 2 and 36. 1 and 3

657

Question 464:Which of the following statements about kinetic energy are correct?

1. It is de ned as2. e unit of kinetic energy is equivalent to Pa x m3.3. Kinetic energy is equal to the amount of energy needed to decelerate

the body in question from its current speed.

658

1. Only 12. Only 23. Only 34. 1 and 25. 2 and 36. 1 and 37. 1, 2 and 3

659

Question 465:In relation to radiation, which of the following statements is FALSE?

1. Radiation is the emission of energy in the form of waves or particles.2. Radiation can be either ionising or non-ionising.3. Gamma radiation has very high energy.4. Alpha radiation is of higher energy than beta radiation.5. X-rays are an example of wave radiation.

Question 466:In relation to the physical de nition of half-life, which of the following

statements are correct?

1. In radioactive decay, the half-life is independent of atom type andisotope.

2. Half-life is de ned as the time required for exactly half of the entitiesto decay.

3. Half-life applies to situations of both exponential and non-exponential decay.

660

1. Only 12. Only 23. Only 34. 1 and 25. 2 and 36. 1 and 3

661

Question 467:In relation to nuclear fusion, which of the following statements is

FALSE?

1. Nuclear fusion is initiated by the absorption of neutrons.2. Nuclear fusion describes the fusion of hydrogen atoms to form

helium atoms.3. Nuclear fusion releases great amounts of energy.4. Nuclear fusion requires high activation temperatures.5. All of the statements above are false.

Question 468:In relation to nuclear ssion, which of the following statements is

correct?

1. Nuclear ssion is the basis of many nuclear weapons.2. Nuclear ssion is triggered by the shooting of neutrons at unstable

atoms.3. Nuclear ssion can trigger chain reactions.4. Nuclear ssion commonly results in the emission of ionising

radiation.5. All of the above.

Question 469:Two identical resistors (Ra and Rb) are connected in a series circuit.

Which of the following statements are true?

1. e current through both resistors is the same.2. e voltage through both resistors is the same.3. e voltage across the two resistors is given by Ohm’s Law.

662

1. Only 12. Only 23. Only 34. 1 and 25. 2 and 36. 1 and 37. 1, 2 and 38. None of the above.

663

Question 470:e Sun is 8 light-minutes away from the Earth. Estimate the

circumference of the Earth’s orbit around the Sun. Assume that the Earthis in a circular orbit around the Sun. Speed of light = 3 x 108 ms-1

664

1. 1024 m2. 1021 m3. 1018 m4. 1015 m5. 1012 m6. 109 m

665

Question 471:Which of the following statements about the physical de nition of

speed are true?

1. Speed is the same as velocity.2. e internationally standardised unit for speed is ms-2.3. Velocity = distance/time.

666

1. Only 12. Only 23. Only 34. 1 and 25. 2 and 36. 1 and 37. 1, 2 and 38. None of the above

667

Question 472:Which of the following statements best de nes Ohm’s Law?

1. e current through an insulator between two points is indirectlyproportional to the potential difference across the two points.

2. e current through an insulator between two points is directlyproportional to the potential difference across the two points.

3. e current through a conductor between two points is inverselyproportional to the potential difference across the two points.

4. e current through a conductor between two points is proportionalto the square of the potential difference across the two points.

5. e current through a conductor between two points is directlyproportional to the potential difference across the two points.

Question 473:Which of the following statements regarding Newton’s Second Law are

correct?

1. For objects at rest, Resultant Force must be 0 Newtons2. Force = Mass x Acceleration3. Force = Rate of change of Momentum

668

1. Only 12. Only 23. Only 34. 1 and 25. 2 and 36. 1 and 37. 1, 2 and 3

669

Question 474:Which of the following equations concerning electrical circuits are

correct?

670

1. Only 12. Only 23. Only 34. 1 and 25. 2 and 36. 1 and 37. 1, 2 and 38. None of the equations are correct.

671

Question 475:An elevator has a mass of 1,600 kg and is carrying passengers that have

a combined mass of 200 kg. A constant frictional force of 4,000 N retardsits motion upward. What force must the motor provide for the elevator tomove with an upward acceleration of 1 ms-2? Assume: g = 10 ms-2

672

1. 1,190 N2. 11,900 N3. 18,000 N4. 22,000 N5. 23,800 N

673

Question 476:A 1,000 kg car accelerates from rest at 5 ms-2 for 10 s. en, a braking

force is applied to bring it to rest within 20 seconds. What distance has thecar travelled?

674

1. 125 m2. 250 m3. 650 m4. 750 m5. 1,200 m6. More information needed

675

Question 477:An electric heater is connected to 120 V mains by a copper wire that

has a resistance of 8 ohms. What is the power of the heater?

676

1. 90 W2. 180 W3. 900 W4. 1800 W5. 9,000W6. 18,000 W7. More information needed

677

Question 478:In a particle accelerator electrons are accelerated through a potential

difference of 40 MV and emerge with an energy of 40MeV (1 MeV = 1.60 x10-13 J). Each pulse contains 5,000 electrons and constitutes a current of

250A. e current is zero between pulses. Assuming that the electronshave zero energy prior to being accelerated what is the power delivered bythe electron beam?

678

1. 1 kW2. 10 kW3. 100 kW4. 1000 kW5. 10000 kW6. More information needed

679

Question 479:Which of the following statements is true?

1. When an object is in equilibrium with its surroundings, there is noenergy transferred to or from the object and so its temperatureremains constant.

2. When an object is in equilibrium with its surroundings, it radiatesand absorbs energy at the same rate and so its temperature remainsconstant.

3. Radiation is faster than convection but slower than conduction.4. Radiation is faster than conduction but slower than convection.5. None of the above.

Question 480:A 6kg block is pulled from rest along a horizontal frictionless surface

by a constant horizontal force of 12 N. Calculate the speed of the blockafter it has moved 300 cm.

680

681

Question 481:A 100 V heater heats 1.5 litres of pure water from 10oC to 50oC in 50

minutes. Given that 1 kg of pure water requires 4,000 J to raise itstemperature by 1oC, calculate the resistance of the heater.

682

1. 12.5 ohms2. 25 ohms3. 125 ohms4. 250 ohms5. 500 ohms6. 850 ohms

683

Question 482:Which of the following statements are true?

1. Nuclear ssion is the basis of nuclear energy.2. Following ssion, the resulting atoms are a different element to the

original one.3. Nuclear ssion often results in the production of free neutrons and

photons.

684

1. Only 12. Only 23. Only 34. 1 and 25. 2 and 36. 1 and 37. 1, 2 and 38. None of the above

685

Question 483:Which of the following statements are true? Assume g = 10 ms-2.

1. Gravitational potential energy is de ned as Ep = m x g x Δ h.2. Gravitational potential energy is a measure of the work done against

gravity.3. A reservoir situated 1 km above ground level with 106 litres of water

has a potential energy of 1 Giga Joule.

686

1. Only 12. Only 23. Only 34. 1 and 25. 2 and 36. 1 and 37. 1, 2 and 38. None of the above

687

Question 484:Which of the following statements are correct in relation to Newton’s

3rd law?

1. For every action there is an equal and opposite reaction.2. According to Newton’s 3rd law, there are no isolated forces.3. Rockets cannot accelerate in deep space because there is nothing to

generate an equal and opposite force.

688

1. Only 12. Only 23. Only 34. 1 and 25. 2 and 36. 1 and 3

689

Question 485:Which of the following statements are correct?

1. Positively charged objects have gained electrons.2. Electrical charge in a circuit over a period of time can be calculated if

the voltage and resistance are known.3. Objects can be charged by friction.

690

1. Only 12. Only 23. Only 34. 1 and 25. 2 and 36. 1 and 37. 1, 2 and 3

691

Question 486:Which of the following statements is true?

1. e gravitational force between two objects is independent of theirmass.

2. Each planet in the solar system exerts a gravitational force on theEarth.

3. For satellites in a geostationary orbit, acceleration due to gravity isequal and opposite to the lift from engines.

4. Two objects that are dropped from the Eiffel tower will always landon the ground at the same time if they have the same mass.

5. All of the above.6. None of the above.

Question 487:Which of the following best de nes an electrical conductor?

1. Conductors are usually made from metals and they conductelectrical charge in multiple directions.

2. Conductors are usually made from non-metals and they conductelectrical charge in multiple directions.

3. Conductors are usually made from metals and they conductelectrical charge in one xed direction.

4. Conductors are usually made from non-metals and they conductelectrical charge in one xed direction.

5. Conductors allow the passage of electrical charge with zeroresistance because they contain freely mobile charged particles.

6. Conductors allow the passage of electrical charge with maximalresistance because they contain charged particles that are xed andstatic.

Question 488:

692

An 800 kg compact car delivers 20% of its power output to its wheels. Ifthe car has a mileage of 30 miles/gallon and travels at a speed of 60miles/hour, how much power is delivered to the wheels? 1 gallon of petrolcontains 9 x 10 8 J.

693

1. 10 kW2. 20 kW3. 40 kW4. 50 kW5. 100 kW

694

Question 489:Which of the following statements about beta radiation are true?

1. After a beta particle is emitted, the atomic mass number isunchanged.

2. Beta radiation can penetrate paper but not aluminium foil.3. A beta particle is emitted from the nucleus of the atom when an

electron changes into a neutron.

695

1. 1 only2. 2 only3. 1 and 34. 1 and 25. 2 and 36. 1, 2 and 3

696

Question 490:A car with a weight of 15,000 N is travelling at a speed of 15 ms-1 when

it crashes into a wall and is brought to rest in 10 milliseconds. Calculatethe average braking force exerted on the car by the wall. Take g = 10 ms-2

697

698

Question 491:Which of the following statements are correct?

1. Electrical insulators are usually metals e.g. copper.2. e ow of charge through electrical insulators is extremely low.3. Electrical insulators can be charged by rubbing them together.

699

1. Only 12. Only 23. Only 34. 1 and 25. 2 and 36. 1 and 37. 1, 2 and 3

700

e following information is needed for Questions 492 and 493:e graph below represents a car’s movement. At t=0 the car’s

displacement was 0 m.

Question 492:Which of the following statements are not true?

1. e car is reversing after t = 30.2. e car moves with constant acceleration from t = 0 to t = 10.3. e car moves with constant speed from t = 10 to t = 30.

701

1. 1 only2. 2 only3. 3 only4. 1 and 35. 1 and 26. 2 and 37. 1, 2 and 3

702

Question 493:Calculate the distance travelled by the car.

703

1. 200 m2. 300 m3. 350 m4. 400 m5. 500 m6. More information needed

704

Question 494:A 1,000 kg rocket is launched during a thunderstorm and reaches a

constant velocity 30 seconds after launch. Suddenly, a strong gust of windacts on it for 5 seconds with a force of 10,000 N in the direction ofmovement.

What is the resulting change in velocity?

705

1. 0.5 ms-1

2. 5 ms-1

3. 50 ms-1

4. 500 ms-1

5. 5000 ms-1

6. More information needed

706

Question 495:A 0.5 tonne crane lifts a 0.01 tonne wardrobe by 100 cm in 5,000

milliseconds.Calculate the average power developed by the crane. Take g = 10 ms-2.

707

1. 0.2 W2. 2 W3. 5 W4. 20 W5. 50 W6. More information needed

708

Question 496:A 20 V battery is connected to a circuit consisting of a 1 Ω and 2 Ω

resistor in parallel. Calculate the overall current of the circuit.

709

1. 6.67 A2. 8 A3. 10 A4. 12 A5. 20 A6. 30 A

710

Question 497:Which of the following statements is correct?

1. e speed of light changes when it enters water.2. e speed of light changes when it leaves water.3. e direction of light changes when it enters water.4. e direction of light changes when it leaves water.5. All of the above.6. None of the above.

Question 498:In a parallel circuit, a 60 V battery is connected to two branches.

Branch A contains 6 identical 5 Ω resistors and branch B contains 2identical 10 Ω resistors.

Calculate the current in branches A and B.

Question 499:Calculate the voltage of an electrical circuit that has a power output of

50,000,000,000 nW and a current of 0.000000004 GA.

711

1. 0.0125 GV2. 0.0125 MV3. 0.0125 kV4. 0.0125 V5. 0.0125 mV6. 0.0125 µV7. 0.0125 nV

712

Question 500:Which of the following statements about radioactive decay is correct?

1. Radioactive decay is highly predictable.2. An unstable element will continue to decay until it reaches a stable

nuclear con guration.3. All forms of radioactive decay release gamma rays.4. All forms of radioactive decay release X-rays.5. An atom’s nuclear charge is unchanged after it undergoes alpha

decay.6. None of the above.

Question 501:A circuit contains three identical resistors of unknown resistance

connected in series with a 15 V battery. e power output of the circuit is60 W.

Calculate the overall resistance of the circuit when two furtheridentical resistors are added to it.

713

1. 0.125 Ω2. 1.25 Ω3. 3.75 Ω4. 6.25 Ω5. 18.75 Ω6. More information needed.

714

Question 502:A 5,000 kg tractor’s engine uses 1 litre of fuel to move 0.1 km. 1 ml of

the fuel contains 20 kJ of energy.Calculate the engine’s efficiency. Take g = 10 ms-2

715

1. 2.5 %2. 25 %3. 38 %4. 50 %5. 75 %6. More information needed.

716

Question 503:Which of the following statements are correct?

1. Electromagnetic induction occurs when a wire moves relative to amagnet.

2. Electromagnetic induction occurs when a magnetic eld changes.3. An electrical current is generated when a coil rotates in a magnetic

eld.

717

1. Only 12. Only 23. Only 34. 1 and 25. 2 and 36. 1 and 37. 1, 2 and 3

718

Question 504:Which of the following statements are correct regarding parallel

circuits?

1. e current owing through a branch is dependent on the branch’sresistance.

2. e total current owing into the branches is equal to the totalcurrent owing out of the branches.

3. An ammeter will always give the same reading regardless of itslocation in the circuit.

719

1. Only 12. Only 23. Only 34. 1 and 25. 2 and 36. 1 and 37. All of the above

720

Question 505:Which of the following statements regarding series circuits are true?

1. e overall resistance of a circuit is given by the sum of all resistors inthe circuit.

2. Electrical current moves from the positive terminal to the negativeterminal.

3. Electrons move from the positive terminal to the negative terminal.

721

1. Only 12. Only 23. Only 34. 1 and 25. 2 and 36. 1 and 3

722

Question 506:e graphs below show current vs. voltage plots for 4 different

electrical components.

Which of the following graphs represents a resistor at constanttemperature, and which a lament lamp?

Question 507:Which of the following statements are true about vectors?

1. Vectors can be added or subtracted.2. All vector quantities have a de ned magnitude.3. All vector quantities have a de ned direction.4. Displacement is an example of a vector quantity.5. All of the above.6. None of the above.

Question 508:

723

e acceleration due to gravity on the Earth is six times greater thanthat on the moon. Dr Tyson records the weight of a rock as 250 N on themoon.

Calculate the rock’s density given that it has a volume of 250 cm³. TakegEarth = 10 ms-2

724

1. 0.2 kg/cm³2. 0.5 kg/cm³3. 0.6 kg/cm³4. 0.7 kg/cm³5. 0.8 kg/cm³6. More information needed.

725

Question 509:A radioactive element undergoes alpha decay. What is the atomic

mass and atomic number after 5 alpha particles have been released?

Question 510:A 20 A current passes through a circuit with resistance of 10 Ω. e

circuit is connected to a transformer that contains a primary coil with 5turns and a secondary coil with 10 turns. Calculate the potential differenceexiting the transformer.

726

1. 100 V2. 200 V3. 400 V4. 500 V5. 2,000 V6. 4,000 V7. 5,000 V

727

Question 511:A metal sphere of unknown mass is dropped from an altitude of 1 km

and reaches terminal velocity 300 m before it hits the ground. Given thatresistive forces do a total of 10 kJ of work for the last 100 m before the ballhits the ground, calculate the mass of the ball. Take g = 10ms-2.

728

1. 1 kg2. 2 kg3. 5 kg4. 10 kg5. 20 kg6. More information needed.

729

Question 512:Which of the following statements is true about the electromagnetic

spectrum?

1. e wavelength of ultraviolet waves is shorter than that of x-rays.2. For waves in the electromagnetic spectrum, wavelength is directly

proportional to frequency.3. Most electromagnetic waves can be stopped with a thin layer of

aluminium.4. Waves in the electromagnetic spectrum travel at the speed of sound.5. Humans are able to visualise the majority of the electromagnetic

spectrum.6. None of the above.

Question 513:In relation to the Doppler Effect, which of the following statements are

true?

1. If an object emitting a wave moves towards the sensor, thewavelength increases and frequency decreases.

2. An object that originally emitted a wave of a wavelength of 20 mmfollowed by a second reading delivering a wavelength of 15 mm ismoving towards the sensor.

3. e faster the object is moving away from the sensor, the greater theincrease in frequency.

730

1. Only 12. Only 23. Only 34. 1 and 25. 1 and 36. 2 and 37. 1, 2 and 38. None of the above statements are true.

731

Question 514:A 5 g bullet is travels at 1 km/s and hits a brick wall. It penetrates 50

cm before being brought to rest 100 ms after impact. Calculate the averagebraking force exerted by the wall on the bullet.

732

1. 50 N2. 500 N3. 5,000 N4. 50,000 N5. 500,000 N6. More information needed.

733

Question 515:Polonium (Po) is a highly radioactive element that has no known

stable isotope. Po210 undergoes radioactive decay to Pb206 and Y. Calculatethe number of protons in 10 moles of Y. [Avogadro’s Constant = 6 x 1023]

734

1. 02. 1.2 x 1024

3. 1.2 x 1025

4. 2.4 x 1024

5. 2.4 x 1025

6. More information needed

735

Question 516:Dr Sale measures the background radiation in a nuclear wasteland to

be 1,000 Bq. He then detects a spike of 16,000 Bq from a nuclear rod madeup of an unknown material. 300 days later, he visits and can no longerdetect a reading higher than 1,000 Bq from the rod, even though it hasn’tbeen disturbed.

What is the longest possible half-life of the nuclear rod?

736

1. 25 days2. 50 days3. 75 days4. 100 days5. 150 days6. More information needed

737

Question 517:A radioactive element undergoes a series of beta (β−) and gamma

decays. What are the number of protons and neutrons in the element afterthe emission of 5 beta particles and 2 gamma waves?

Question 518:Most symphony orchestras tune to ‘standard pitch’ ( frequency = 440

Hz). When they are tuning, sound directly from the orchestra reachesaudience members that are 500 m away in 1.5 seconds.

Estimate the wavelength of ‘standard pitch’.

738

1. 0.05 m2. 0.5 m3. 0.75 m4. 1.5 m5. 15 m6. More information needed

739

Question 519:A 1 kg cylindrical artillery shell with a radius of 50 mm is red at a

speed of 200 ms-1. It strikes an armour plated wall and is brought to rest in500 µs.

Calculate the average pressure exerted on the entire shell by the wall atthe time of impact.

740

1. 5 x 106 Pa2. 5 x 107 Pa3. 5 x 108 Pa4. 5 x 109 Pa5. 5 x 1010 Pa6. More information needed

741

Question 520:A 1,000 W display fountain launches 120 litres of water straight up

every minute. Given that the fountain is 10% efficient, calculate themaximum possible height that the stream of water could reach.

Assume that there is negligible air resistance and g = 10 ms-2.

742

1. 1 m2. 5 m3. 10 m4. 20 m5. 50m6. More information needed

743

Question 521In relation to transformers, which of the following is true?

1. Step up transformers increase the voltage leaving the transformer.2. In step down transformers, the number of turns in the primary coil is

smaller than in the secondary coil.3. For transformers that are 100% efficient: Ip Vp= Is Vs

744

1. Only 12. Only 23. Only 34. 1 and 25. 1 and 36. 2 and 37. 1, 2 and 38. None of the above.

745

Question 522:e half-life of Carbon-14 is 5,730 years. A bone is found that contains

6.25% of the amount of C14 that would be found in a modern one. How oldis the bone?

746

1. 11,460 years2. 17,190 years3. 22,920 years4. 28,650 years5. 34,380 years6. 40,110 years

747

Question 523:A wave has a velocity of 2,000 mm/s and a wavelength of 250 cm. What

is its frequency in MHz?

748

1. 8 x 10-3 MHz2. 8 x 10-4 MHz3. 8 x 10-5 MHz4. 8 x 10-6 MHz5. 8 x 10-7 MHz6. 8 x 10-8 MHz

749

Question 524:A radioactive element has a half-life of 25 days. After 350 days it has a

count rate of 50. What was its original count rate?

750

1. 102,4002. 162,2403. 204,8004. 409,6005. 819,2006. 1,638,4007. 3,276,800

751

Question 525:Which of the following units is NOT equivalent to a Volt (V)?

752

1. AΩ2. WA-1

3. Nms-1A-1

4. NmC5. JC-1

6. JA-1s-1

753

SECTION 2: MATHS

BMAT MATHS QUESTIONS ARE DESIGNED to be time draining- if yound yourself consistently not nishing, it might be worth leaving the

maths (and probably physics) questions until the very end.Good students sometimes have a habit of making easy questions

difficult; remember that the BMAT only tests GCSE level knowledge so youare not expected to know or use calculus or trigonometry in any part ofthe exam.

Formulas you MUST know:

Even good students who are studying maths at A2 can struggle withcertain BMAT maths topics because they’re usually glossed over at school.ese include:

Quadratic Formulae solutions for a quadratic equation in the form are given by:Remember that you can also use the discriminant to quickly see if a

quadratic equation has any solutions:Completing the Square

754

If a quadratic equation cannot be factorised easily and is in the formatthen you can rearrange it into the form

is looks more complicated than it is – remember that in the BMAT,you’re extremely unlikely to get quadratic equations where and theequation doesn’t have any easy factors. is gives you an easier equation:and is best understood with an example.

Consider:is equation cannot be factorised easily but note that:erefore, . Completing the square is an important skill – make sure

you’re comfortable with it.Difference between 2 SquaresIf you are asked to simplify expressions and nd that there are no

common factors but it involves square numbers – you might be able tofactorise by using the ‘difference between two squares’.

For example, can also be expressed as.

755

MATHS QUESTIONS

Question 526:Robert has a box of building blocks. e box contains 8 yellow blocks

and 12 red blocks. He picks three blocks from the box and stacks them uphigh. Calculate the probability that he stacks two red building blocks andone yellow building block, in any order.

756

757

Question 527:

Solve

758

1. 12.112. 13.493. 13.954. 14.25. 196. 265

759

Question 528:

Solve

760

1. 0.75 and -2. -0.75 and3. -5 and4. 5 and5. 12 only6. -12 only

761

Question 529:

Express as a single fraction.

762

1. 24

763

Question 530:e value of p is directly proportional to the cube root of q. When p =

12, q = 27. Find the value of q when p = 24.

764

1. 322. 643. 1244. 1285. 2166. 1728

765

Question 531:Write 722 as a product of its prime factors.

766

1. 26 x 34

2. 26 x 35

3. 24 x 34

4. 2 x 33

5. 26 x 36. 23 x 32

767

Question 532:

Calculate:

768

1. 0.00002022. 0.000200023. 0.000020024. 0.000000025. 0.0000020026. 0.000002002

769

Question 533:Given that y2 + ay + b = (y + 2)2 - 5, nd the values of a and b.

Question 534:

Express as a single fraction in its simplest form:

770

771

Question 535:A is inversely proportional to the square root of B. When A = 4, B = 25.Calculate the value of A when B = 16.

772

1. 0.82. 43. 54. 65. 106. 20

773

Question 536:S, T, U and V are points on the circumference of a circle, and O is the

centre of the circle.Given that angle SVU = 89°, calculate the size of the smaller angle SOU.

774

1. 89°2. 91°3. 102°4. 178°5. 182°6. 212°

775

Question 537:Open cylinder A has a surface area of 8π cm2 and a volume of 2π cm3.

Open cylinder B is an enlargement of A and has a surface area of 32π cm2.Calculate the volume of cylinder B.

776

1. 2π cm3

2. 8π cm3

3. 10π cm3

4. 14π cm3

5. 16π cm3

6. 32π cm3

777

Question 538:

Express in its simplest form.

778

779

Question 539:A bag contains 10 balls. 9 of those are white and 1 is black. What is the

probability that the black ball is drawn in the tenth and nal draw if thedrawn balls are not replaced?

780

1. 0

781

Question 540:Gambit has an ordinary deck of 52 cards. What is the probability of

Gambit drawing 2 Kings (without replacement)?

782

1. 0

1. None of the above

783

Question 541:I have two identical unfair dice, where the probability that the dice get

a 6 is twice as high as the probability of any other outcome, which are allequally likely. What is the probability that when I roll both dice the totalwill be 12?

784

1. 0

1. None of the above

785

Question 542:A roulette wheel consists of 36 numbered spots and 1 zero spot (i.e. 37

spots in total).What is the probability that the ball will stop in a spot either divisible

by 3 or 2?

786

1. 0

787

Question 543:I have a fair coin that I ip 4 times. What is the probability I get 2

heads and 2 tails?

788

1. None of the above

789

Question 544:Shivun rolls two fair dice. What is the probability that he gets a total of

5, 6 or 7?

790

1. None of the above

791

Question 545:Dr Savary has a bag that contains x red balls, y blue balls and z green

balls (and no others). He pulls out a ball, replaces it, and then pulls outanother. What is the probability that he picks one red ball and one greenball?

792

1. More information necessary

793

Question 546:Mr Kilbane has a bag that contains x red balls, y blue balls and z green

balls (and no others). He pulls out a ball, does NOT replace it, and thenpulls out another. What is the probability that he picks one red ball andone blue ball?

794

1. More information needed

795

Question 547:ere are two tennis players. e rst player wins the point with

probability p, and the second player wins the point with probability 1-p.e rules of tennis say that the rst player to score four points wins thegame, unless the score is 4-3. At this point the rst player to get two pointsahead wins.

What is the probability that the rst player wins in exactly 5 rounds?

796

1. 4p4(1-p)2. p4(1-p)3. 4p(1-p)4. 4p(1-p)4

5. 4p5(1-p)6. More information needed.

797

Question 548:

Solve the equation

798

1. —2. —3. —

799

Question 549:e volume of a sphere is V = πr3, and the surface area of a sphere is S =

4πr2. Express S in terms of V

800

1. S = (4π)2/3(3V)2/3

2. S = (8π)1/3(3V)2/3

3. S = (4π)1/3(9V)2/3

4. S = (4π)1/3(3V)2/3

5. S = (16π)1/3(9V)2/3

801

Question 550:Express the volume of a cube, V, in terms of its surface area, S.

802

1. V = (S/6)3/2

2. V = S3/2

3. V = (6/S)3/2

4. V = (S/6)1/2

5. V = (S/36)1/2

6. V = (S/36)3/2

803

Question 551:

Solve the equations and

804

1. No solutions possible.

805

Question 552:

Rearrange , to make x the subject.

806

807

Question 553:

Simplify

808

1. 9x20

2. 27x20

3. 87x20

4. 9x21

5. 27x21

6. 81x21

809

Question 554:

Simplify

810

1. 8x3

2. 8x

811

Question 555:What is the circumference of a circle with an area of 10π?

812

1. 10π2. 20π3. More information needed.

813

Question 556:

If , then calculate the value of (3.4).5

814

1. 192. 543. 1004. 1195. 132

815

Question 557:

If , calculate(2.3).2

816

1. 12. 23. 44. 8

817

Question 558:Solve x2 + 3x - 5 = 0

818

819

Question 559:

How many times do the curves and intersect?

820

1. 02. 13. 24. 35. 4

821

Question 560:Which of the following graphs do not intersect?

822

1. y = x2. y = x2

3. y = 1-x2

4. y = 2

823

1. 1 and 22. 2 and 33. 3 and 44. 1 and 35. 1 and 46. 2 and 4

824

Question 561:Calculate the product of 897,653 and 0.009764.

825

1. 87646.82. 8764.683. 876.4684. 87.64685. 8.764686. 0.876468

826

Question 562:

Solve for x:

827

828

Question 563:What is the area of an equilateral triangle with side length x.

829

830

Question 564:

Simplify

831

832

Question 565:

Solve the equation

833

834

Question 566:

Rearrange to make x the subject.

1. cannot be made the subject for this equation.

Question 567:

Rearrange to make y the subject.

835

836

Question 568:

Rearrange to make y the subject.

837

838

Question 569:e aspect ratio of my television screen is 4:3 and the diagonal is 50

inches. What is the area of my television screen?

839

1. 1,200 inches2

2. 1,000 inches2

3. 120 inches2

4. 100 inches2

5. More information needed.

840

Question 570:

Rearrange the equation to make x the subject.

841

842

Question 571:

Solve and

843

1. No solutions possible.

844

Question 572:

e two inequalities de ne a region on a plane. Which of the followingpoints is inside the region?

845

1. (2, 1)2. (2.5, 1)3. (1, 2)4. (3, 5)5. (1, 2.5)6. None of the above.

846

Question 573:

How many times do intersect?

847

1. 02. 13. 24. 35. 4

848

Question 574:

How many times do intersect?

849

1. 02. 13. 24. 35. 4

850

Question 575:A cube has unit length sides. What is the length of a line joining a

vertex to the midpoint of the opposite side?

851

852

Question 576:Solve for x, y, and z.

Question 577:Fully factorise: 3a3 – 30a2 + 75a

853

1. 3a(a – 3)3

2. a(3a – 5)2

3. 3a(a2 – 10a + 25)4. 3a(a – 5)2

5. 3a(a + 5)2

854

Question 578:Solve for x and y:

Question 579:

Evaluate:

855

1. 0

856

Question 580:All license plates are 6 characters long. e rst 3 characters consist of

letters and the next 3 characters of numbers. How many unique licenseplates are possible?

857

1. 676,0002. 6,760,0003. 67,600,0004. 1,757,6005. 17,576,0006. 175,760,000

858

Question 581:

How many solutions are there for:

859

1. 02. 13. 24. 3

860

1. In nite solutions.

861

Question 582:

Evaluate:

862

863

Question 583:Bryan earned a total of £ 1,240 last week from renting out three ats.

From this, he had to pay 10% of the rent from the 1-bedroom at forrepairs, 20% of the rent from the 2-bedroom at for repairs, and 30% fromthe 3-bedroom at for repairs. e 3-bedroom at costs twice as much asthe 1-bedroom at. Given that the total repair bill was £ 276 calculate therent for each apartment.

Question 584:

Evaluate:

864

1. 02. 253. 324. 495. 566. 200

865

Question 585:What is the area of a regular hexagon with side length 1?

866

1. 62. More information needed

867

Question 586:Dexter moves into a new rectangular room that is 19 metres longer

than it is wide, and its total area is 780 square metres. What are the room’sdimensions?

868

1. Width = 20 m; Length = -39 m2. Width = 20 m; Length = 39 m3. Width = 39 m; Length = 20 m4. Width = -39 m; Length = 20 m5. Width = -20 m; Length = 39 m

869

Question 587:Tom uses 34 meters of fencing to enclose his rectangular lot. He

measured the diagonals to 13 metres long. What is the length and width ofthe lot?

870

1. 3 m by 4 m2. 5 m by 12 m3. 6 m by 12 m4. 8 m by 15 m5. 9 m by 15 m6. 10 m by 10 m

871

Question 588:

Solve

872

1. 12. 1.53. 34. 3.55. 4.56. None of the above

873

Question 589:

Calculate:

874

1. 0.0332. 0.00333. 0.000334. 0.0000335. 0.0000033

875

Question 590:Calculate the area of the triangle shown to the right:

876

1. 32. 6

877

Question 591:

Rearrange to make x the subject.

878

879

Question 592:When 5 is subtracted from 5x the result is half the sum of 2 and 6x.

What is the value of x?

880

1. 02. 13. 24. 35. 46. 6

881

Question 593:

Estimate

882

1. 02. 13. 24. 35. 46. 5

883

Question 594:At a Pizza Parlour, you can order single, double or triple cheese in the

crust. You also have the option to include ham, olives, pepperoni, bellpepper, meat balls, tomato slices, and pineapples. How many differenttypes of pizza are available at the Pizza Parlour?

884

1. 102. 963. 1924. 3845. 7686. None of the above

885

Question 595:

Solve the simultaneous equations, for x, y > 0

886

887

Question 596:Which of the following statements is FALSE?

1. Congruent objects always have the same dimensions and shape.2. Congruent objects can be mirror images of each other.3. Congruent objects do not always have the same angles.4. Congruent objects can be rotations of each other.5. Two triangles are congruent if they have two sides and one angle of

the same magnitude.

Question 597:Solve the inequality x2 ≥ 6 – x

888

1. x ≤ -3 and x ≤ 22. x ≤ -3 and x ≥ 23. x ≥ -3 and x ≤ 24. x ≥ -3 and x ≥ 25. x ≥ 2 only6. x ≥ -3 only

889

Question 598:e hypotenuse of an equilateral right-angled triangle is x cm. What is

the area of the triangle in terms of x?

890

891

Question 599:

Mr Heard derives a formula: Q= . He doubles the values of X and Y,halves the value of A and triples the value of B. What happens to value ofQ?

892

1. Decreases by2. Increases by3. Decreases by4. Increases by5. Increases by6. Decreases by

893

Question 600:Consider the graphs y = x2 - 2x + 3, and y = x2 - 6x - 10. Which of the

following is true?

1. Both equations intersect the x-axis.2. Neither equation intersects the x-axis.3. e rst equation does not intersect the x-axis; the second equation

intersects the x-axis.4. e rst equation intersects the x-axis; the second equation does not

intersect the x-axis.

894

SECTION 3

e BasicsIn section 3, you have to write a one A4 page essay on one of three

essay titles. Whilst different questions will inevitably demand differinglevels of comprehension and knowledge, it is important to realise that oneof the major skills being tested is actually your ability to construct a logicaland coherent argument- and to convey it to the lay-reader.

Section 3 of the BMAT is frequently neglected by lots of students, whochoose to spend their time on sections 1 & 2 instead. However, it has thehighest returns per hour of work out of all three sections so is well worthputting time into.

e aim of section 3 is not to write as much as you can. Rather, theexaminer is looking for you to make interesting and well supported points,and tie everything neatly together for a strong conclusion. Make sureyou’re writing critically and concisely; not rambling on. Irrelevantmaterial can actually lower your score. You only get one side of A4 foryour BMAT essay, so make it count!

Essay StructureMost BMAT essays consist of 3 parts:

1) Explain what a quote or a statement means.2) Argue for or against the statement.3) Ask you “to what extent” you agree with the statement.

Number 1 should be the smallest portion of the essay (no more than 4lines) and be used to provide a smooth segue into the rather more

895

demanding “argue for/against” part of the question. is main partrequires a rm grasp of the concept being discussed and the ability tostrengthen and support the argument with a wide variety of examplesfrom multiple elds. is section should give a balanced approach to thequestion, exploring at least two distinct ideas. Supporting evidenceshould be provided throughout the essay, with examples referred to whenpossible.

e nal part effectively asks for your personal opinion and is a chancefor you to shine- be brave and make an innovative yet �rmly groundedconclusion for an exquisite mark. e conclusion should bring togetherall sides of the argument, in order to reach a clear and concise answer tothe question. ere should be an obvious logical structure to the essay,which re ects careful planning and preparation.

PARAGRAPHS

Paragraphs are an important formatting tool which show that youhave thought through your arguments and are able to structure your ideasclearly. A new paragraph should be used every time a new idea isintroduced. ere is no single correct way to arrange paragraphs, but it’simportant that each paragraph ows smoothly from the last. A slick,interconnected essay shows that you have the ability to communicate andorganise your ideas effectively.

Given that you only have a limit of one A4 page to write in – youshouldn’t have more than 5 paragraphs (use indents to showparagraphs – don’t leave empty lines!). In general, 2 of these 5 will be takenup by the introduction and conclusion respectively.

Remember- the emphasis should remain on quality and not quantity.An essay with fewer paragraphs, but with well-developed ideas, is muchmore effective than a number of short, unsubstantial paragraphs that failto fully grasp the question at hand.

896

Approaching the EssaySection 3 can be broken down into 3 components; selecting your essay

title, planning and writing it.

Most students think that the “writing” component is most important.is is simply not true.

e vast majority of problems are caused by a lack of planningand essay selection- usually because students just want to get writing asthey are worried about nishing on time. irty minutes is long enough tobe able to plan your essay well and still have time to write it so don’t feelpressured to immediately start writing.

Step 1: SelectingSelecting your essay is crucial- make sure you’re comfortable with the

topic and ensure you understand the actual question- it sounds silly butabout 25% of essays that we mark score poorly because they don’t actuallyanswer the question!

Take two minutes to read all the questions. Whilst one essay mightoriginally seem the easiest, if you haven’t thought through it you mightquickly nd yourself running out of ideas. Likewise, a seemingly difficultessay might actually offer you a good opportunity to make interestingpoints.

Use this time to carefully select which question you will answer bygauging how accessible and comfortable you are with it given your

897

background knowledge. Section 3, however, is not a test of knowledge butrather a test of how well you are able to argue.

It’s surprisingly easy to change a question into something similar, butwith a different meaning. us, you may end up answering a completelydifferent essay title. Once you’ve decided which question you’re going todo, read it very carefully through a few times to make sure you fullyunderstand it. Answer all aspects of the question. Keep reading it as youanswer to ensure you stay on track!

Step 2: Planning

WHY SHOULD I PLAN MY ESSAY?

ere are multiple reasons you should plan your essay for the rst 5-10minutes of section 3:

As you don’t have much space to write, make the most of it by writinga very well organised essay.It allows you to get all your thoughts ready before you put pen topaper.You’ll write faster once you have a plan.You run the risk of missing the point of the essay or only answeringpart of it if you don’t plan adequately.

HOW MUCH TIME SHOULD I PLAN FOR?

ere is no set period of time that should be dedicated to planning,and everyone will dedicate a different length of time to the planningprocess. You should spend as long planning your essay as you require, butit is essential that you leave enough time to write the essay. As a roughguide, it is worth spending about 5-10 minutes to plan and theremaining time on writing the essay. However, this is not a strict rule, and

898

you are advised to tailor your time management to suit your individualstyle.

HOW SHOULD I GO ABOUT THE PLANNING PROCESS?

ere are a variety of methods that can be employed in order to planessays (e.g. bullet-points, mind-maps etc). If you don’t already know whatworks best, it’s a good idea to experiment with different methods.

Generally, the rst step is to gather ideas relevant to the question,which will form the basic arguments around which the essay is to be built.You can then begin to structure your essay, including the way that pointswill be linked. At this stage it is worth considering the balance of yourargument, and con rming that you have considered arguments from bothsides of the debate. Once this general structure has been established, it isuseful to consider any examples or real world information that may help tosupport your arguments. Finally, you can begin to assess the plan as awhole, and establish what your conclusion will be based on yourarguments.

Step 3: WritingIntroduction

WHY ARE INTRODUCTIONS IMPORTANT?

An introduction provides tutors with their rst opportunity toexamine your work. e introduction is where rst impressions areformed, and these can be extremely important in producing a convincingargument. A well-constructed introduction shows that you have reallythought about the question, and can indicate the logical ow of argumentsthat is to come.

WHAT SHOULD AN INTRODUCTION DO?

899

A good introduction should brie�y explain the statement or quoteand give any relevant background information in a concise manner.However, don’t fall into the trap of just repeating the statement in adifferent way. e introduction is the rst opportunity to suggest ananswer to the question posed- the main body is effectively yourjusti cation for this answer.

Main Body

HOW DO I GO ABOUT MAKING A CONVINCING POINT?

Each idea that you propose should be supported and justi ed, in orderto build a convincing overall argument. A point can be solidi ed through abasic Point Evidence Evaluation process. By following this process, youcan be assured each sentence within a paragraph builds upon the last, andthat all the ideas presented are well solidi ed.

HOW DO I ACHIEVE A LOGICAL FLOW BETWEEN IDEAS?

One of the most effective ways of displaying a good understanding ofthe question is to keep a logical ow throughout your essay. is meanslinking points effectively between paragraphs, and creating a congruenttrain of thought for the examiner as the argument develops. A good way togenerate this ow of ideas is to provide ongoing comparisons ofarguments, and discussing whether points support or dispute one another.

SHOULD I USE EXAMPLES?

In short – yes! Examples can help boost the validity of arguments, andcan help display high quality writing skills. Examples can add a lot ofweight to your argument and make an essay much more relevant to thereader. When using examples, you should ensure that they are relevant to

900

the point being made, as they will not help to support an argument if theyare not.

Some questions will provide more opportunities to include examplesthan others so don’t worry if you aren’t able to use as many examples asyou would have liked. ere is no set rule about how many examplesshould be included!

Conclusione conclusion provides an opportunity to emphasise the overall

sentiment of your essay which readers can then take away. It shouldsummarise what has been discussed during the main body and give ade nitive answer to the question.

Some students use the conclusion to introduce a new idea thathasn’t been discussed. is can be an interesting addition to an essay,and can help make you stand out. However, it is by no means, a necessity.In fact, a well-organised, ‘standard’ conclusion is likely to be more effectivethan an adventurous but poorly executed one.

Medical Ethicsere is normally a medical ethics questions in most years so it’s well

worth knowing the basics. Whilst there are huge ethical textbooksavailable– you only need to be familiar with the basic principles for thepurposes of the BMAT. ese principles can be applied to all casesregardless what the social/ethnic background the healthcare professionalor patient is from. In addition to being helpful in the BMAT, you’ll need toknow them for the interview stages anyway so they’re well worth learningnow. e principles are:

Bene�cence: e wellbeing of the patient should be the doctor’s rstpriority. In medicine this means that one must act in the patient’s best

901

interests to ensure the best outcome is achieved for them i.e. ‘Do Good’.Non-Male�cence: is is the principle of avoiding harm to the patient

(i.e. Do no harm). ere can be a danger that in a willingness to treat,doctors can sometimes cause more harm to the patient than good. iscan especially be the case with major interventions, such as chemotherapyor surgery. Where a course of action has both potential harms andpotential bene ts, non-male cence must be balanced against bene cence.

Autonomy: The patient has the right to determine their ownhealth care. This therefore requires the doctor to be a good

communicator, so that the patient is sufficiently informed tomake their own decisions. ‘Informed consent’ is thus a vital

precursor to any treatment. A doctor must respect a patient’srefusal for treatment even if they think it is not the correct

choice. Note that patients cannot demand treatment – onlyrefuse it, e.g. an alcoholic patient can refuse rehabilitation but

cannot demand a liver transplant.

ere are many situations where the application of autonomy can bequite complex, for example:➢ Treating Children: Consent is required from the parents, although theautonomy of the child is taken into account increasingly as they get older.➢ Treating adults without the capacity to make important decisions.e rst challenge with this is in assessing whether or not a patient hasthe capacity to make the decisions. Just because a patient has a mentalillness does not necessarily mean that they lack the capacity to makedecisions about their health care. Where patients do lack capacity, thepower to make decisions is transferred to the next of kin (or Legal Power ofAttorney, if one has been set up).

902

Justice: is principle deals with the fair distribution and allocation ofhealthcare resources for the population.

Consent: is is an extension of Autonomy- patients must agree to aprocedure or intervention. For consent to be valid, it must be voluntaryinformed consent. is means that the patient must have sufficientmental capacity to make the decision and must be presented with all therelevant information (bene ts, side effects and the likely complications) ina way they can understand.

Con�dentiality: Patients expect that the information they reveal todoctors will be kept private- this is a key component in maintaining thetrust between patients and doctors. You must ensure that patient detailsare kept con dential. Con dentiality can be broken if you suspect that apatient is a risk to themselves or to others e.g. Terrorism, suicides.

When answering a question on medical ethics, you need to ensure thatyou show an appreciation for the fact that there are often two sides to theargument. Where appropriate, you should outline both points of view andhow they pertain to the main principles of medical ethics and then cometo a reasoned judgement.

Common Mistakes

IGNORING THE OTHER SIDE OF THE ARGUMENT

Although you’re normally required to support one side of the debate, itis important to consider arguments against your judgement in order toget the higher marks. A good way to do this is to propose an argument thatmight be used against you, and then to argue why it doesn’t hold true orseem relevant. You may use the format: “some may say that…but this doesn’tseem to be important because…” in order to dispel opposition arguments,whilst still displaying that you have considered them. For example, “somemay say that fox hunting shouldn’t be banned because it is a tradition.

903

However, witch hunting was also once a tradition – we must move on with thetimes”.

ANSWERING THE TOPIC/ANSWERING ONLY PART OF THEQUESTION

One of the most common mistakes is to only answer a part of thequestion whilst ignoring the rest of it as it’s inaccessible. According to theofficial mark scheme, in order to get a score of 3 or more, you mustwrite “…an answer that addresses ALL aspects of the question”. isshould be your minimum standard- anything else that you write shouldthen point you towards achieving 4/5.

LONG INTRODUCTIONS

Some students can start rambling and make introductions too longand unfocussed. Although background information about the topic can beuseful, it is normally not necessary. Instead, the emphasis should beplaced on responding to the question. Some students also justrephrase the question rather than actually explaining it. e examinerknows what the question is, and repeating it in the introduction is simplya waste of space in an essay where you are limited to just one A4 side.

NOT INCLUDING A CONCLUSION

An essay that lacks a conclusion is incomplete and can signal that theanswer has not been considered carefully or that your organisation skillsare lacking. e conclusion should be a distinct paragraph in its ownright and not just a couple of rushed lines at the end of the essay.

SITTING ON THE FENCE

904

Students sometimes don’t reach a clear conclusion. You need toensure that you give a decisive answer to the question and clearlyexplain how you’ve reached this judgement. Essays that do not come to aclear conclusion generally have a smaller impact and score lower.

EXCEEDING THE ONE PAGE LIMIT

e page limit is there for a reason – don’t exceed it under anycircumstances as any material over the limit won’t be marked and it willappear that you haven’t read the instructions.

NOT USING ALL THE AVAILABLE SPACE

Remember that you only have one A4 side to write on so ensure youmake the maximum use of the space available to you. Don’t leave lines toshow paragraphs – instead, you should use indents. Similarly, you shouldalso use the top-most line in the response sheet and avoid crossing entiresentences out.

905

Marking your EssaysPracticing section 3 can be tricky because most students don’t know

how to mark their essay. However, if you have a willing friend/familymember, it is fairly easy to mark your own work. You can use the diagrambelow to get an idea of your score – keep in mind that this is just a veryrough guide – examiners will look at several other factors when decidingon your overall score.

906

ANNOTATED ESSAYS

Example Essay 1:“A doctor should never disclose medical information about his

patients”What does this statement mean? Argue to the contrary using

examples to strengthen your response.To what extent do you agree with this statement?e statement suggests that one of a doctor’s most vital qualities is

maintaining con�dentiality of a patient’s medical record. is involves alldoctors with various specialities in different work places such as the clinics.

Disclosing medical informations regarding to their patients by doctors isconsidered as an unacceptable act within the medical society. For Example,by informing unrelated people about the patient might result in theindividual’s most embarrased health situation to be exposed. For example,suffering pain from their private parts and this may disgust other. issituation would inevitably upset the patient as their health privacy has beenbreached by others without consent leading to a sence of distrust towardsdoctors.

However, disclosing such matters to certain suitable people such as familyand relatives may be crucial. For example, if the patient is the head of thefamily or the guardian to the children. As these individuals are in charge ofleading and taking are of the family, they need to be able to performmundane task (such as providing good support to the children) at theiroptimum. Also, to ensure that the members realise that the patient should not

907

over exert him or herself despite their health conditions. Also, a suddencollapse will reduce shock when the family is to rush the patient back to thehospital knowing that the illness is related to the situation.

Overall, a patient’s con�dentially should not be disclosed without consentor any importance by all means. is is to respect their health privacy and toavoid any inconvenience within the medical society.

EXAMINER’S COMMENTS:

Introduction: e student appears to have an understanding of thetopic but frequently makes statements that don’t add much to theargument e.g. the second sentence of the rst paragraph. e introductionwould be better used to set up the counter arguments that will form thebulk of the main body.

Main Body: e rst paragraph actually supports the statement and istherefore not actually answering the question. e example doesn’t reallyadd much either. e key issue that needed to be discussed was a doctor’sduty to the patient – not about “disgusting others”. e last sentence of the

rst paragraph is good and starts to address the question but it comes fartoo late.

e second paragraph is better but misses the key points of the essaythat were necessary to discuss i.e. when can patient con dentiality bebroken? Examples would include suspected terrorism, noti able diseasesand criminal activity, suicides etc.

Conclusion: e conclusion doesn’t really address the counter-arguments for breaking con dentiality or give a balanced answer. It alsocontains confusing terminology e.g. one discloses con dentialinformation, not ‘con dentiality’. e sentence concerning “inconveniencewithin the medical community” is also somewhat ambiguous.

Language: e grammar hinders the points that the student is tryingto make throughout the essay e.g. “suffering pain from their private parts

908

and this may disgust other”. ere are also frequent spelling mistakes like“embarrased” and “sence” that reduce the uency signi cantly.

Score: D2Example Essay 2“A doctor should never disclose medical information about his

patients”What does this statement mean? Argue to the contrary using

examples to strengthen your response.To what extent do you agree with this statement?is statement is one of the duties set out by the General Medical Council

for doctors to comply with, which is to respect patient’s autonomy. It meansthat a doctor cannot share patient’s medical information to other partiesunless the patients, themselves, have granted permissions to do so.

e ethical principle of respecting patients’ autonomy cannot be appliedin all cases, as some cases require doctors to disclose medical informationabout patients. First, when it involves a criminal act that has been comittedby a patient, a doctor has to report to any appropriate authority, such as thepolice. is is because the patient may potentially cause even more harm toothers and as doctors, they have to prevent that from happening. An exampleof a case would be if a patient has suffered from a gunshot wound and he hadtold his doctor that he had gotten it when he murdered someone.

Next, another incident when a doctor has no choice but to disclosepatients’ medical information is when it may affect the health of society andcould potentially cause an epidemic. Such patients might have an infectiousdisease and do not wish to let other people know about it. For instance, therehas been many cases in West Africa where people who have Ebola are afraidto let their neighbours or friends �nd out because they do not want to bestigmatised and ostrilised from the society. However, these patients couldspread the disease and so a doctor must not withold the information. Last ofall, if a patient is underage, then he/ she is still not competent enough to make

909

her own decision. erefore, any medical information must be shared withhis/ her legal guardian.

Respecting patient’s autonomy by never disclosing their information isalso important because patients have the right to chose who gets to knowabout it. It is his own body. He is the only person who knows the consequencesof sharing this sensitive information. In conclusion, I believe that neverdisclosing patients’ medical information cannot be complied in everyincident. Respecting their autonomy is important but we have to treat eachcase separately.

EXAMINER’S COMMENTS:

Introduction: The introduction is well written but could be improvedby making it explicitly clear that con dentiality can be breached in certaincircumstances. is would then set up the main body nicely as the studentwould then be able to go straight into giving examples.

Main Body: e rst sentence of the second paragraph is well writtenbut should have gone in the introduction. ere is good breadth ofargument with the important points being covered like a doctor’s duty toprevent harm to others, ‘public good’ and the issue of ‘capacity’. However,there is unnecessary padding that doesn’t add much e.g. there was no needto expand on your example of infectious diseases. e extra space fromavoiding this would have allowed the student to write about the fact thatalthough con dentiality must sometimes be breached, a doctor hascertain professional duties. For example, informing the patient both beforeand after and explaining why they have disclosed what they have to tryand mitigate any damage to the doctor patient relationship. In this waypublic trust in medical professionals would be maintained.

Conclusion: e conclusion concisely summarises the arguments putforth in the main body and offers a nice resolution by saying that each caseis different.

910

Language: Whilst it is clear that the student understands the question– there is some confusion as to the difference between “autonomy” and“con dentiality” (It’s important to know the basics of medical ethics asthey’ll be helpful for the interview stage as well).Furthermore, there areminor spelling mistakes like “comitted” and “withold”. In the conclusion,they also assume that patients are only male – “it is his own body” vs. “it istheir own body”.

Score: B3.5Example Essay 3“A doctor should never disclose medical information about his

patients”What does this statement mean? Argue to the contrary using

examples to strengthen your response.To what extent do you agree with this statement?Con�dentiality is a basic patient right. e patient provides information

to the doctor not to be unnecessarily shared with others without theirknowledge or permission. On this basis, the statement argues that a doctorshould never reveal the medical data, such as results from tests orprescriptions given.

However, it can be argued that there are many circumstances whereby itis necesary to breach patient con�dentiality and disclose medicalinformation. More speci�cally, if the patient poses a threat to the publichealth, their medical situation should be disclosed immediately so thatactions can be taken to prevent the spread. For instance, under the PublicHealth Act 1988, if a patient is suspected of communicable diseases such astuberculosis, the doctor is required to inform the local health authoritiesimmediately so that they can make precautions to protect the other citizens.In addition, a doctor should also disclose medical information if the patienthas broken the law. For instance, the doctor should reveal medical data to the

911

detectives and other relevant professionals if they request for it, to enablethem to come to a conclusion of the case more quickly and accurately.

However, I agree with this statement to a large extent. After all, thepatient should have the right over what happens to his medical documentsand information. Revealing information about the patient unnecessarily willtake this basic right away, and it is extremely unfair for the patient.Furthermore, this unprofessional decision may undermine the con�dencebetween the patient and doctor. e patient may be less willing to reveal vitalpersonal information to the doctor in the future, in fear that he might releasethis information as well. is would be extremely detrimental to thediagnoses and treatment for the patient or the doctor might not be able togain sufficient information to make a more informed decision.

In conclusion, a doctor should never disclose medical information abouthis patients unless there are other external circumstances that oblige thedoctor to do so. Breaking this con�dentiality will cause the patient-doctorrelationship to collapse, compromising the trust between them. However, insome cases, the decision to disclose is not that clear-cut; if a patient hadsexually-transmitted infections, should the doctor disclose this information tohis spouse? Such situations have to be decided on a case-by-case basis.

EXAMINER’S COMMENTS:

Introduction: is is a bold introduction that catches one’s attentionand gets straight to the point. e student however does make a rathergeneralised statement - “con dentiality is a basic patient right”. Apedantic examiner could easily challenge this and thus, it’s important tobe careful with your wording.

Main Body: ere is a good level of breadth and depth of argumenthere. However, there are again some generalising statements that areincorrect e.g. doctors don’t need to break con dentiality if a patient hasbroken ANY law – just serious ones e.g. committed or intend to commit

912

murder/terrorism etc. ere is however, excellent discussion of theconsequences of breaking con dentiality and a good level of detail (e.g.Public health act 1988).

Conclusion: An excellent conclusion that not only summarises themain arguments from both sides but also builds upon these to offer asolution as to when to break con dentiality by treating it on a case-by-casebasis.

Language: ere is only one spelling mistake (“necessry”) andalthough the somewhat general phrases stop this from being a perfect A5,it is still an excellent essay that displays good insight. Spend time makingsure that you write exactly what you mean, instead of being loose withyour words and conveying an incorrect message.

Score: A4.5Example Essay 4“Medicine is a science; not an art.”Explain what this statement means. Argue to the contrary that

medicine is in fact an art using examples to illustrate your answer. To whatextent, if any, is medicine a science?

I will explain the following statement “Medicine is a science, not an art”and Argue. to that medicine is in fact an art with supports and to what extentis medicine a science.

I will talk about medicine and art in my opinion why, I think medicine isin fact an art and argue with the above statement. I think medicine is an artbecause of the human body its like a piece of artwork with creations and it isfascinating even more than a artwork of Picccasso or Ven Gough’s painting.It’s like the artries and vessels ressemble the brushstroke of a painting and theheart is the meaning of the painting. To study about that and become aDoctor is like studying art to become and artist. Both Medicine and Artdepend on passion if you don’t have the passion you will not enjoy saving livesand will not create beautiful paintings. Medicine and Art are the most

913

fascinating majors. ey are completely different but at the sane timecompletely the same. Emotionally the same.

Now I will talk about the what extent is medicine a science. It depends onwhat course you are choosing in medicine For e.g. Biomedical it all counts asscience it includes chemistry and maths which makes you a Biochemist orPhisyology, these are some interesting courses. Medicine is science because itdepends on knowledge the years to study to become a doctor and save liveswhich is not an easy opprotunity to get.

In my essay I wrote about why I think that medicine is in face not allabout sciene but about art too and argued with the above essay. I thinkMedicine is as fascinating and breathtaking as Art is with all the colours andbueatiful creations made my artists and saved by doctors.

EXAMINER’S COMMENTS:

Introduction: Although it is sometimes useful to outline what you aregoing to discuss when writing academic essays – you simply do not haveenough space to do this in the BMAT. e introduction should be a clearand concise explanation of the statement, which isn’t really done in thisessay. Repeating the statement in the essay should also be avoided.

Main Body: e student doesn’t really have a good grasp of what thequestion is asking and as a result, the argument is off topic and slightlyincoherent. e question requires the student to discuss things likemedicine is a science because doctors put into practice medical principlesthat have an empirical factual basis. Whilst the third paragraph does thisto a certain extent – the message is diluted somewhat because of a lack offocus. Medicine is also an ‘art’ because doctors also need to be able tocommunicate well with patients, to interpret clinical signs etc. e studentseems have interpreted the question to literally mean “medicine is art” vs.“medicine is an art”.

914

Conclusion: When writing a conclusion it is good practice to justmake your point, as opposed to telling the reader you are making it. us,meta-writing is again not necessary in the conclusion. e nal sentence,although interesting is again, off-topic.

Language: ere are frequent spelling mistakes e.g. “sane time”,“ressemble” and “bueatiful”. e phrasing and grammatical errors are moreserious and signi�cantly affect the essay’s �uency e.g. “talk about the whatextent”.

Score: D1Example Essay 5“Medicine is a science; not an art.”Explain what this statement means. Argue to the contrary that

medicine is in fact an art using examples to illustrate your answer. To whatextent, if any, is medicine a science?

Medicine. Arguably one of the most advancing �elds in today’s society. Asa result, many of us have often thought about what medicine actuallyencompasses. e statement, “medicine is a science; not an art.” is one that isconstantly the subject of debated today. It questions to what extent thatmedicine may be considered an art, and to what a science. I feel that thestatement does not belove that medicine is an art form but instead, is welland truly a science, and is well within the de�nition of one.

An art form is usually something which is viewed as being expressive andemotional, as well as also being delicate. One can argue that the notion ofcaring for patients can be viewed as the non-scienti�c aspect of medicine,and therefore could be considered art as it requires one to be emotional andexpressive. In order for a patient’s rehabilitation process to be complete andsuccesfull, the care directed at the patient should be tailored for them as therecovery of the psychological side of the human body is just as important asthe phsiological aspect. In order to be truly effective, the doctor should be ableto be empathetic and try and understand the patient’s pain when comforting

915

them. is aspect of medicine does not involve the science of the human bodyor the knowledge of the intricate metabolic reactors which allow the humanbody to function so effectively.

However, another side of the debate could be that medicine is very mucha science. is is due to the fact that medicine involves the analysis of ahuman body, for example, when diagnosing a patient or maybeunderstanding the effects a drug could bring to speci�c situations in thehuman body, something that is viewed with the utmost importance whenadministering a drug. e fact that in order to succesfully become a medicalpractitioner, one has to be aware of the physiology of the human body andalso the immense and thorough knowledge of the anatomy of the humanbody, is the reason that medicine is associated with mainly being a science.is stems back to the days when we used to learn Biology and Chemistryback in school, and because Medicine is largely based on those two coresubjects, medicine, as a result, is widely regarded as solely being a science.

EXAMINER’S COMMENTS:

Introduction: e opening is catchy although unnecessarily long-winded. us, whilst the rst sentence grabs the attention, the rest of itdoes little to keep it the next few sentences are very wordy and don’tactually say very much. is is a prime example of just rephrasing thestatement rather than explaining it.

Main Body: ere are a good range of examples in the secondparagraph- especially those about the rehabilitation process and empathy.Whilst the pro-science arguments are also well made, they are a bit one-dimensional. It would be better to discuss how trials are done to ensuresafety rather than just concentrating on the human anatomy andphysiology. e last sentence also doesn’t really add anything and ifanything detracts from the nal paragraph.

916

Conclusion: e essay is badly let down by a lack of a conclusion. istitle required a critical analysis of the strengths of the two sides of thearguments in order to produce a well-constructed conclusion thatanswered the question.

Language: ere are some very elegant turns of phrase thatsometimes result in a loss of focus. is doesn’t affect the essay’sreadability (and therefore the language score) but indirectly affects thestrength of argument quite substantially. Nevertheless, as all parts of thequestion are answered to a sufficient level, it still scores a solid 3.

Score: A3Example Essay 6“Medicine is a science; not an art.”Explain what this statement means. Argue to the contrary that

medicine is in fact an art using examples to illustrate your answer. To whatextent, if any, is medicine a science?

is statement argues that medicine is more deeply rooted with the factsand set observations associated with scienti�c principles, and that no aspectof medicine is in itself open to interpretation; or art.

However many of the facets of Medicine could very well be regarded asart. e manual dexterity and presision required by surgeons; particularly thevisually aesthetic �nish reconstructive or plastic surgery aims towards has adeep basis in artistic ability. Medicine as a career has a hugely importantsocial aspect too; healthcare proffessionals are expected to be involved withcommunication and, when it comes to patients, even dealing with emotionshas an interpretative aspect, and as such could be viewed as art. ere is notone set approach to these situations; rather the outcome is reliant on adoctor’s own personal judgement and choices as to the best course of action.eoretical medicine, too, in terms of research may �nd success in new,recently discovered techniques; the development of which requires thinking

917

‘outside of the box’. e clinical aspect of diagnostic medicine too is subject tounique approached, particularly when introducing extremely complex cases.

Although to the same extent, medicine of course has a heavy involvementwith biological structures, chemical processes and even principles of physics-generally all empirical; based on evidence and set in stone. In many casesthere is a clear distinction between the right interpretation and thus course ofaction And the wrong one; for example when prescribing- for many patients(such as when allergies are involved) there are a whole host of drugs andcourses of action that are unnacceptable. erefore, the understanding of thehuman body and it’s inner workings that is so crucial for appropriate andsuccessful medicine could well be argued to be science. Yet it’s application; it’suses by doctors and other healthcare proffessionals is less impirical, moreopen to interpretation, and therefore more so an art.

I feel that despite the fact medicine involves understanding andknowledge of the physiology and anatomy of the human body, it also involvesthe integral of caring for others, which is de�nitely more of an art form than ascience, showing us that despite all the debates, medicine manages tocombine art and science together resulting in the formation of a wondrousprofession.

EXAMINER’S COMMENTS:

Introduction: An efficient introduction that explains the statementwell. It could be improved by setting up the counter-argument.

Main Body: is is a good effort at a tricky essay – there is goodbreadth of argument with lots of examples like surgical precision andcommunication. ere is also good consideration of the counter-arguments for why medicine is a science with good depth of argument e.g.drug prescriptions.

Conclusion: A strong conclusion that addresses the question well andsummarises arguments from both sides concisely. It uses unnecessarily

918

romantic language “wondrous profession” but nevertheless is an effectiveclosing paragraph.

Language: Although there are a noticeable number of spellingmistakes (“proffessionals” and “impirical”), they do not detract from theessay’s �ow.

Score: B4.5Example Essay 7“e primary duty of a doctor is to prolong life as much as

possible”What does this statement mean? Argue to the contrary, that the

primary duty of a doctor is not to prolong life. To what extent do you agreewith this statement?

As a doctor, the main aim is patient beni�scence always. is involvesperforming all possible tests, treatments and any procedure that would resultin preserving the health of the patient and so increasing their life-span.

It is however in not all cases that patient bene�scence would apply to pro-longing life but rather allowing the patient to die, as it is to their bene�t, orposes more advantages to them and their families as well as NHS. Cases likethis include terminally ill cancer patients whom the doctors have foundpalliative care to be the best option for them rather than continuouschemotherapy which will discomfort them further, and hence violate theprinciple of beni�scence. As much as a doctor is obligated to look after thehealth of individuals, it is the individuals themselves that decide if to followadvices of the doctor, for example, antiretroviral patients who do not taketheir medication which would prolong their lives cannot be forced to takethem, but the doctor will surely have done their part in being a carer and anadvicer and therefore if the patients die before the expected time, it wouldhave been the patient’s autonomous decicion. Considering the increasingcosts of treatments and healthcare nowadays, it is most essential that doctorsuse available sources wisely. If a patient requires treatment that cannot be

919

afforded by the region or nation they are in, doctor’s role in that situation is toprovide as much comfort and pain relief treatment possible within the meansof his surroundings, and therefore cannot be made accountable to prolongingthat patient’s life by a treatment which is inaccessible.

In conclusion I agree that a Doctor must do the best he can for hispatients however in cases of limitations a doctor’s responsibility is to providerelief in that moment.

EXAMINER’S COMMENTS:

Introduction: is is a classic case of answering a topic rather thanthe question. is title requires an analysis of the differences betweenquality of life vs. quantity of life. Instead, the student starts to rambleabout bene cence which although related, is only part of the essay.

e introduction also contains broad sweeping generalisations likeperforming ‘all possible tests, treatment and any procedures’. In thecurrent economic climate, it is not feasible or necessary to do this. If thisstudent were to be interviewed, these apparently minor points could easilygive the impression that the student doesn’t know much about the NHS orhasn’t learned much from their work experience.

Main Body: Whilst there are good and relevant examples usedthroughout the main body, they are not applied correctly to form anyserious argument. e essay would bene t from a clearer discussion of thebalance between simply prolonging life and improving a patient’s qualityof life even if this means taking medication that could hasten death e.g.Morphine in the ‘doctrine of double effect’. e section on patientautonomy again is not as relevant for this essay nor is the discussion aboutpatients being in charge of their own health.

Conclusion: e conclusion discusses the limitations of a doctor’sresponsibility which again isn’t necessary for this essay- reinforcing theoverall impression that the student does not understand the key issues.

920

Language: e lack of a structure reduces the ow of the essay. isisn’t helped by the spelling mistakes “advicer” and “decicion” which alongwith the rambling nature of the main body seriously reduce the essay’sreadability.

Score: D1Example Essay 8“e primary duty of a doctor is to prolong life as much as

possible”What does this statement mean? Argue to the contrary, that the

primary duty of a doctor is not to prolong life. To what extent do you agreewith this statement?

e most important responsibility of doctor is to cure diseases and extenda life of patients at his most ability acquired. e statement also states thatdoctors should try their best to prolong life of the sufferers as the �rstprinciple to consider. However, some might argue with this.

It is true to say that doctors are responsible for improving the conditionsof diseases and alleviate the symptoms. is does not necessarily mean thatis a major factor to tackle with each disease. Preventive care should beintroduced at an early stage, and therefore the primary duty of medicalprofessions, especially doctors should adopt this principle to be their mainconcern. For example, doctors should be aware of other health conditions ofhis or her patients that might be developed in the future regarding to patient’slifestyle or eating habit. To only address the present problems, relating toparticular disease is not enough. Hence, prolonging life of patients is not theprimary concern of doctors but to improve quality of life of the sufferers. Bypreventing the possible disease and acknowledging the patients are a propermost important role of a doctor.

Some people might argue with the statement as there have been a verycontroversial issue raised in recent years, euthanasia. Nowadays it isevidently seen that some patients in Switzerland and some other countries

921

have their right to urge a doctor to help end their lives peacefully. Doctor mayput an emphasis on methods or alternative ways to help prolong life ofpatient. eir prime concern is also �nding the best bene�cial treatment inorder to �ght with the disease unless there is a possible way. erefore, puttingpatients at ease by ending their lives is also a primary concern to a doctor insome countries.

To some extent, I agree and support this statement as doctors have todelegate roles as healer to those who are in pain. Although it is illegal in someregions of the world to allow doctors taking life of a patient with their consent,it does mean that this method apart from prolonging life is one of the mainduty for doctors to be well aware.

EXAMINER’S COMMENTS:

Introduction: This is a concise introduction that effectively justrephrases the statement rather than developing on it much –

effectively not advancing an argument and thus wastingvaluable space. A discussion of what a doctor’s primary duty

actually is would have been more appropriate here and wouldhave set up the main body far better.

Main Body: is is a rather confusing and muddled account. esecond paragraph is difficult to follow and strays from the real topic athand. e student correctly identi es that a doctor’s job is to improvequality of life and not duration. However, this isn’t expressed with anydegree of clarity or any examples. e third paragraph regardingeuthanasia is better but again it is difficult to assess what point thestudent is trying to make.

Conclusion: Whilst it is perfectly �ne to present a one-sidedargument, you need to at least consider the counter-

922

arguments. Thus, it was necessary here to consider that bothquality of life and are important. In addition, the euthanasiapart is somewhat misguided – not all doctors who don’t put

duration of life as their top priority are in favour of euthanasia.

Language: ere are frequent grammatical errors e.g. “as there havebeen a very” which reduce the essay’s uency and. e sentence phrasingalso impacts the essay’s overall readability to lead to a poor languagescore.

Score: C2Example Essay 9“e primary duty of a doctor is to prolong life as much as

possible”What does this statement mean? Argue to the contrary, that the

primary duty of a doctor is not to prolong life. To what extent do you agreewith this statement?

A doctors’ job is to cure disease through medical treatment to extend thelife of the patient as much as possible. However, there is a certain limit towhich doctors can go in order to prolong life as often quality of life is equally(if not more) important as quantity of life.

Doctors provide treatments to patients to help them overcome theirdisease so that they can live longer. is is also what the patients want. Forexample, for patients with kidney diseases, doctors will suggest them to havedialysis in order to remove the toxic substance in their body, which will killthem. rough dialysis, the patient’s life will be extended and as this is thepatient’s will, a doctor’s primary duty is prolong life. People take preventativemethod, such as endoscopy of large intestine for symptoms of cancer, to avoidlate discovery of disease, which will lead to a high chance of death. ereforeas a doctor has the skills to help people to extend their, they should do asmuch as they can to ful�l the patient’s wish, which is prolonging life.

923

However, doctors should also respect the patient’s autonomy if a patientdoesn’t want a treatment, even the treatment is effective, doctors should notcarry the treatment out, as everyone have the right to control their own life,even doctors want to help patient, doctors should not over-ride the will of thepatients.

Although one of a doctor’s duties is to prolong life – this shouldn’t be atthe expense of quality of life. A doctors primary duty is to offer the bestpossible medical advice and minimise suffering. Although the impact of this isusually to prolong life, in some cases, it may result in maximising quality oflife.

EXAMINER’S COMMENTS:

Introduction: An excellent introduction that sets the scene verynicely for the main body and immediately conveys to the examiner thatthe student understands the essay of the essay.

Main Body: ere are good points made throughout e.g. dialysis andthe inclusion of preventative treatment shows a good insight intomedicine. However, the student is not able to put together a substantialenough argument against prolonging life as they spend too much spacediscussing the reasons for prolonging life (which are less important). isis a perfect example of an unbalanced essay. In general, there is limitedfocus on the question and as a result, weak depth of argument.

Conclusion: A satisfactory conclusion that expresses the sentimentsof the conclusion nicely and addresses the question well.

Language: e introduction and conclusion rescue this essay – asthey convey a high degree of understanding in only a couple of sentences.e essay itself reads well and there are no obvious errors that reduce its

uency.Score: A3.5Example Essay 10

924

Animal euthanasia should be made illegal.Explain what this statement means. Argue to the contrary that animal

euthanasia should remain legal. To what extent do you agree with thestatement?

‘Animal euthanasia should be made illegal’. In other words this meansaided death. For example choosing someone to help end your life. ere aremany on going debates that euthanasia should remane legal, and one ofthese reazons could be because it could be seen as ‘inhumane’ to watch otherssuffer. If someone can see an animal obviously dying it may not be seen as justto let them die (when treatment/cure is not possible. However the averageperson may not be 100% sure that the animal is sufering e.g if they arewatching a bird therefore if they committed euthanasia they may not becompletely certain it was not murder. erefore in the case of animals (withthe exception of humans) should only be done by animal experts, in that �eld,with the aim to avoid murder.

Another reason for as to why euthanasia should remain legal could be inthe example of elderly humans with dementia and Parkinsons disease. eseelderly people cannot remember much and can do very little for themselves,they mayaby �nd communication extremely difficult. In this case a questioncould be raised – is life worth living if you cannot do or think for yourselfanymore- maybe the purpose in life was to have fun – in which case they maywant nothing more than to end their life, however due to their circumstancesthey cannot end their life without help. However on the contrare people maybe brainwashed into euthanasia- for a number of reasons- one including thatthe person could be very wealthy and someone may want that money.

Overall I think euthanasia should remain legal however everybody shouldmake a signed statement regarding their own personal views when they arestill young, �t and healthy to make their own decisions. is could be updatedevery �ve to ten years.

925

EXAMINER’S COMMENTS:

Introduction: In general, there is no need to repeat the questionword-for-word as this just wastes space. e introduction set off alarmbells as the student is discussing human euthanasia instead of animaleuthanasia.

Main Body: It is difficult to separate the introduction from the mainbody of the essay. e second paragraph is difficult to follow and makesense of. In particular, the part about ‘animal experts’ committingeuthanasia is especially confusing as it is unclear if they student is arguingfor or against animal euthanasia. e third paragraph about dementia andParkinson’s is not relevant to this essay. is means that the student hasmissed the point of the essay.

Conclusion: e conclusion does little to address the statement.Conclusions should use the arguments made throughout the essay toprovide a succinct summary e.g. “in light of the human duty to end animalsuffering I believe animal euthanasia should be legal. Although this has thepotential to be abused, the animals’ quality of life is more important.”

Language: Avoid double negatives and colloquialisms such asbrainwashed. ere are a large number of mistakes in the essay rangingfrom spelling (“contrare” and “mayaby”) to colloquial language(“brainwashed”). However, the biggest difficulty derives from the phrasingthat aversely impacts the readability of the essay leading to a poorlanguage score.

Score: C1Example Essay 11Animal euthanasia should be made illegal.Explain what this statement means. Argue to the contrary that animal

euthanasia should remain legal. To what extent do you agree with thestatement?

926

e statement refers to the ethical dilemma of euthanasia. Euthanasia,synonymous of mercy killing, is the ending of someone’s life because of aparticular situation in which this living creature’s future will be painful,sometimes short or it will ultimately be better to end it soon. e �rstargument for this statement is the fact that we as humans are generally kindhearted and benevolent: we see pain as a thing to be eradicated if notsuppressed, and hence euthanasia could be seen as merciful, given thisnature of ours. However, the natural argument against this is the undeniablefact that in the eyes of many, euthanasia is glori�ed murder, which goesagainst most people’s morals since killing is seen as a negative thing in mostsocieties nowadays. another argument in favour of this process could be thefact that killing is not only condemned but also morally wrong since itinvolves us “removing” an otherwise healthy living being. e counter-argument illustrated by this statement is the fact that most people would lookupon this as a grateful act of mercy, where although the consequences aretaken into account, it is, morally, for some, the best thing to do, particularly ifan animal, a source of fondness for some, is involved. e last argument toput forth is that animals are intelligent creatures capable of feeling pain likeus and should hence receive the same mercy as is sometimes shown tohumans. Naturally, people would say that in some cases this would bemurder rather than mercy killing, eg the killing of old horses for glue ratherthan because of old age. I conclude this should remain legal since animalscannot voice their own opinion and hence give more weight to a decision.

EXAMINER’S COMMENTS:

Introduction: is is a good albeit somewhat long introduction. estudent has de ned euthanasia well and then established that there is anethical dilemma surrounding it. However, the very long sentences make itneedlessly difficult to follow.

927

Main Body: Whilst the writing style is excellent, there is a limitedamount of content here. e student presents arguments for both sidessimultaneously which sometimes makes it difficult to follow. is is mademore confusing by the lack of paragraphs which means that the essaydoesn’t ow as well.

ere also some long and rambling sentences which detract from theclarity of argument. e essay would bene t from a more focussedapproach in which the student gets to the point. e point about killinghorses for glue is also not as relevant to euthanasia outside a slipperyslope argument (which isn’t expanded upon).

Conclusion: e conclusion does not really build upon any of thearguments from the main body. is gives the impression that it wasrushed with little planning.

Language: ere are no obvious spelling errors but colloquialisms like‘nowadays’ should be avoided. Overall, the student clearly understands thetopic – but the essay is let down by a limited focus on the question andpoor structure due to very long sentences and a lack of paragraphs.

Score: A3Example Essay 12Animal euthanasia should be made illegal.Explain what this statement means. Argue to the contrary that animal

euthanasia should remain legal. To what extent do you agree with thestatement?

is statement means that the purposeful act of killing animals, carriedout by veterinary practitioners should be made against the law. Currently,human euthanasia is illegal and the introduction of legal euthanasia bringswith it many potentially harmful implications such as the ‘putting down’ ofhealthy animals.

An ethical pillar of medicine is non-male�cence. By making animaleuthanasia illegal we uphold this pillar and avoiding causing harm to

928

potentially healthy animals. If animal euthanasia were to be legalised thensome people may think it justi�ed to slaughter animal for less than noblepurposes.

However, there are many cases in which euthanasia may be the best wayof progression in the medical treatment of animals. For example: if an injuredor terminally ill animal has no chance of recovery and is suffering theneuthanasia may be the most kind and compassionate thing to do e.g. if ahorse has broken its leg and will never be able to walk again. In addition, ifthe quality of life of an animal is very low e.g. they have no home, are starvingand there is nowhere for them to live, then euthanasia may also be the mostcompassionate course of action. is may be especially the case where anarea is overpopulated with stray cats and dogs.

Another case where euthanasia seems the most bene�cial course ofaction is if an animal has become infected with a disease that could spread toother animals and humans potentially causing widespread and signi�cantharm e.g. if a dog becomes infected with rabies or a cow becomes infectedwith foot and mouth disease.

To conclude, I disagree with the statement as I think animal euthanasiaremain legal. It should however, only be carried out by a veterinaryprofessional and only when the animal is undergoing signi�cant suffering.

EXAMINER’S COMMENTS:

Introduction: A concise and focussed introduction that answers therst part of the question well and sets up the main body nicely.

Main Body: e student presents a sophisticated argument thataddresses all the aspects of the question and uses good examples to backup their points. e arguments are well thought out and naturally followon from each other. It would be better to argue why euthanasia shouldremain legal before giving reasons for it to be made illegal. is wouldhelp improve the ow.

929

Conclusion: A succinct and well-supported conclusion that tiestogether the major arguments in the main body. It also introduces a newidea –only vets should be allowed to perform euthanasia. is is a goodpoint but should have been developed somewhat more.

Language: e student clearly understands the essay and putstogether a strong essay. ere are no glaring spelling or grammaticalerrors.

Score: A4.5

930

Summary

General Advice✓ Always answer the question clearly – this is the key thing examiner lookfor in an essay.✓ Analyse each argument made, justifying or dismissing with logicalreasoning.✓ Keep an eye on the time/space available – an incomplete essay may betaken as a sign of a candidate with poor organisational skills.✓ Use pre-existing knowledge when possible – examples and real worlddata can be a great way to strengthen an argument- but don’t make upstatistics!✓ Present ideas in a neat, logical fashion (easier for an examiner toabsorb).✓ Complete some practice papers in advance, in order to best establishyour personal approach to the paper (particularly timings, how you planetc.).

Attempt to answer a question that you don’t fully understand, orignore part of a question.

931

Rush or attempt to use too many arguments – it is much better tohave fewer, more substantial points.Attempt to be too clever, or present false knowledge to support anargument – a tutor may call out incorrect facts etc.Panic if you don’t know the answer the examiner wants – there is noright answer, the essay is not a test of knowledge but a chance todisplay reasoning skill.Leave an essay un nished – if time/space is short, wrap up the essayearly in order to provide a conclusive response to the question.

932

MOCK PAPER A

Section 1Question 1:A square sheet of paper is 20cms long. How many times must it be

folded in half before it covers an area of 12.5cm2?

933

1. 32. 43. 54. 65. 7

934

Question 2:Mountain climbing is viewed by some as an extreme sport, while for

others it is simply an exhilarating pastime that offers the ultimatechallenge of strength, endurance, and sacri ce. It can be highly dangerous,even fatal, especially when the climber is out of his or her depth, or simplygets overwhelmed by weather, terrain, ice, or other dangers of themountain. Inexperience, poor planning, and inadequate equipment can allcontribute to injury or death, so knowing what to do right matters.

Despite all the negatives, when done right, mountain climbing is anexciting, exhilarating, and rewarding experience. is article is anoverview beginner’s guide and outlines the initial basics to learn; in reality,each step is deserving of an article in its own right, and entire tomes havebeen written on climbing mountains, so you’re advised to spend a gooddeal of your beginner’s learning immersed in reading widely. is basicoverview will give you an idea of what is involved in a climb.

Which statement best summarises this paragraph?

1. Mountain climbing is an extreme sport fraught with dangers.2. Without extensive experience embarking on a mountain climb is

fatal.3. A comprehensive literature search is the key to enjoying mountain

climbing.4. Mountain climbing is difficult and is a skill that matures with age if

pursued.5. e terrain is the biggest unknown when climbing a mountain and

therefore presents the biggest danger.

Question 3:50% of an isolated population contract a new strain of resistant

Malaria. Only 20% are symptomatic of which 10% are female. Whatpercentage of the total population do symptomatic males represent?

935

A) 1%B) 9%C) 10%D) 80%

936

Question 4:John is a UK citizen yet is looking to buy a holiday home in the South

of France. He is purchasing his new home through an agency: unlike anormal estate agent as they offer monthly discount sales of up to 30%. As aFrench company, the agency sells in Euros. John decides to hold off on hispurchase until the sale, in the interest of saving money. What is the majorassumption made in doing this?A) e house he likes will not be bought in the meantimeB) e agency will not be declared bankruptC) e value of the pound will fall more than 30%D) e value of the pound will fall less than 30%E) e value of the euro may increase by up to 35% in the coming weeks

Question 5:In childcare professions, by law, there must be an adult to child ratio of

no more than 1:4. Child minders are hired on a salary of £8.50 an hour.What is the maximum number of children that can be continuallysupervised for a period of 24 hours on a budget of £1,000?

937

1. 12. 83. 124. 165. 468

938

Question 6:A table of admission prices for the local cinema is shown below:

How much would a group of 3 adults, 5 children, a concession and 4students save by visiting at an off-peak time rather than a peak time?

939

1. £11.502. £13.503. £15.504. £17.505. £18.50

940

Question 7:

Which frame comes next in the following sequence?

Question 8:Flow mediated dilation is a method used to assess vascular function

within the body. It essentially adopts the use of an ultrasound scan tomeasure the percentage increase in the width of an artery before and afterocclusion with a blood pressure cuff. Ultrasound scans are taken by onesonographer, and the average lumen diameter is then measured by ananalyst. What is a potential aw in the methodology of this technique?A) Results will not be comparable within an individual if different arteriesstart at different diametersB) Results will not be comparable between individuals if they havedifferent baseline arterial diametersC) Ultrasound is an outdated technique with no use in modern medicineD) is methodology is subject to human errorE) is methodology is not repeatable

Question 9:If it takes 20 minutes to board an aeroplane, 15 minutes to disembark

and the ight lasts two and a half hours. In the event of a delay it is notuncommon to add 20 minutes to the ight time. Megan is catching the

941

ight in question as she needs to attend a meeting at 5pm. e location ofthe meeting is 15 minutes from the airport without traffic; 25 minuteswith. Which of the following statements is valid considering thisinformation?

1. If Megan wants to be on time for her meeting, given all possibilitiesdescribed, the latest she can begin boarding at the departure airportis 1.30pm.

2. If Megan starts boarding at 1.40pm she will certainly be late.3. If Megan aims to start boarding at 1.10pm she will arrive in time

whether the plane is delayed or not.4. If Megan wishes to be on time she doesn’t have to worry about the

plane being delayed as she can make up the time during thetransport time from the arrival airport to the meeting.

5. None of the above are correct.

942

Question 10:A cask of whiskey holds a total volume of 500L. Every two and a half

minutes half of the total volume is collected and discarded. How manyminutes will it take for the entire cask to be emptied?

943

A) 80B) 160C) 200D) 240E) ∞

944

Question 11:

What can be concluded from the graph above?A) Having a larger waist size causes an increase in BMIB) Having a larger BMI causes an increase in waist sizeC) Waist size is reciprocal to BMID) No conclusions can be drawn from this graphE) None of the above are correct

Question 12:

Which frame comes next?

945

946

Question 13:Car A has a fuel tank capacity of 30 gallons and achieves 40mpg. Car B

on the other hand has a fuel tank capacity of 50 gallons but only achieves30mpg. Both cars drive until they run out of fuel. If car A starts with a fulltank of petrol and travels 200 miles further than car B; how full was car B’sfuel tank?

947

A) 1/5B) 1/4C) 1/3D) 1/2E) 2/3

948

The information below relates to questions 14 and 15:

e art of change ringing adopts the use of 6 bells, numbered 1 to 6 inorder of weight (1 being the lightest). Initially the bells are rung in thisorder: 1, 2, 3, 4, 5, 6 however the aim is to ultimately ring all the possibleorders of a 6 number sequence. e rules for doing this are very simple:each bell can only move a maximum of one place in the sequence everytime it rings.

Question 14:What is the total possible number of permutations of 6 bells?

949

A) 160B) 220C) 660D) 720E) 1160

950

Question 15:Based on the information provided which of the following could be a

possible series of bell sequences?

951

1. 1 2 3 4 5 2 1 4 3 5 2 3 1 5 4

2. 1 2 3 4 5

2 1 4 3 52 4 1 5 3

1. 1 2 3 4 5 4 2 1 3 5 4 2 3 1 5

2. 1 2 3 4 5 1 4 3 2 5 1 2 3 4 5

3. 1 2 3 4 5 4 1 3 2 5 5 3 1 2 4

952

Question 16:e keypad to a safe comprises the digits 1 - 9. e code itself can be of

indeterminate length. e code is therefore set by choosing a referencenumber so that when a code is entered the average of all the numbersentered must equal the chosen reference number.

Which of the following is true?A) If the reference number was set greater than 9, the safe would be lockedforeverA) is safe is extremely insecure as if random digits were pressed for longenough it would average out at the correct reference numberB) More than one number is always required to achieve the referencenumberC) All of the above are trueD) None of the above are true

Question 17:e use of antibiotics is one of the major paradoxes in modern

medicine. Antibiotics themselves provide a selection pressure to drive theevolution of antibiotic resistant strains of bacteria. is is largely due tothe rapid growth rate of bacterial colonies and asexual cell division. Assuch a widespread initiative is in place to limit the prescription ofantibiotics.

Which of the following is a fair assumption?A) Antibiotic resistance is impossible to avoid as it is driven by evolutionB) If bacteria reproduced at a slower rate antibiotic resistance would notbe such an issueC) Medicine always creates more problems than it solvesD) In the past antibiotics were used frivolouslyE) All of the above could be possible

The information below relates to questions 18 – 22:

953

e Spaghetti Bolognese recipe below serves 10 people and eachportion contains 300 kcal.

954

➢ 1kg mince➢ 220g pancetta, diced➢ 30g crushed garlic➢ 1kg tinned tomatoes➢ 300g diced onions➢ 300g sliced mushrooms➢ 200g grated cheese

955

Question 18:What quantity of cheese is required to prepare a meal for 350 people?

956

A) 0.7kgB) 7kgC) 70kgD) 700kgE) 7000k

957

Question 19:If 12 portions represent 120% of an individuals recommended calori c

intake, what is that individuals recommended calori c intake?

958

A) 2,600kcalB) 2,800kcalC) 3,000kcalD) 3,200kcalE) 3,400kcal

959

Question 20:e recommended ratio of pasta to Bolognese is 4:1. If cooking for 30

people how much pasta should be used?

960

A) 30.3kgB) 36.6kgC) 42.9kgD) 49.2kgE) 55.5kg

961

Question 21:What is the ratio of onions to the rest of the ingredients if garlic and

pancetta are ignored?

962

A) 1/2.05B) 1/3.9C) 1/6.7D) 1/9.3

963

E) 1/10

964

Question 22:It takes 4 minutes to prepare the ingredients per portion, and a further

8 minutes per portion to cook. Simon has ample preparation space but islimited to cooking 8 portions at a time. What is the shortest period of timeit would take him to turn all the ingredients into a meal for 25 people,assuming he didn’t start cooking until all the ingredients were prepared?

965

A) 3 hoursB) 3 hours 40C) 4 hoursD) 4 hours 40E) 5 hours

966

Question 23:A company sells custom design t-shirts. A breakdown of their costs is

shown below:

Customers with an ever before printed design must also pay asurcharge of £50 to cover the cost of building a jig. What is the total costfor an order of unique stag do t-shirts; 50 in colour, 200 in black and white?

967

A) £650B) £700C) £750D) £800E) £850

968

Question 24:e Scouts is a movement for young people rst established by Lord

Baden Powell. As the founder he was the rst chief scout of theassociation. Since his initial appointment there have been a number ofnotable chief scouts including Peter Duncan and Bear Grylls. Some of the

rst camping trips conducted by Lord Powell’s scout troop were on BrownSea Island.

Now the Scout movement is a worldwide global phenomenon givingchildren from all backgrounds the opportunity not only to embark uponadventure but also to engage in the understanding and teaching of foreignculture. Traditionally religion formed the back bone of the scoutingmovement which was re ected in the scouts promise: “I promise to do myduty to god and to the queen”.

Which of the following applies to the scout movement?A) Scouts work for the QueenB) e scout network is aimed at adventurous individualsC) Chief scout is appointed by the queenD) You have to be religious to be a scoutE) None of the above

Question 25:

Which frame comes next in the sequence above?

969

Question 26:e use of human cadavers in the teaching of anatomy is hotly

debated. Whilst many argue that it is an invaluable teaching resource,demonstrating far more than a text book can. Others describe how it is anoutdated method which puts unfair stress on an already bereaved family.One of the biggest pros for using human tissue in anatomical teaching isthe variation that it displays. Whilst textbooks demonstrate a standardmodel averaged over many 100s of specimens, many argue that it is thevariation between cadavers that really reinforces anatomical knowledge.

e opposition argues that it is a cruel process that damages thegrieving process of the effected family. For the use of the cadaver oftenoccupies a period of up to 12 months. As such the relative in question isreturned to the bereaved family for burial around the time it would beexpected that they were recovering as described in the grieving model.

Does the article support or reject the use of cadavers in anatomicalteaching?

970

A) Supports the useB) Rejects the useC) ImpartialD) Can’t tellE) None of the above

971

Question 27:A ferry is carrying its full capacity. At the time of departure (7am) the

travel time to the nearest hour is announced as 13 hours. What is thelatest that the ferry could arrive at its destination?

972

A) 08.00B) 20.00C) 20.29D) 20.30E) 20.50

973

Question 28:A game is played using a circle of 55 stepping stones. A die is rolled

showing the numbers 1 -6. e number on the die tells you how manysteps you may take during your go. e only rule is that during your go youmust take your steps in the routine two steps forward, 1 step back.

What is the minimum number or rolls required to win?

974

A) 28B) 55C) 110D) 165E) 200

975

Question 29:On a race track there are 3 cars recording average lap times of 40

seconds, 60 seconds and 70 seconds. ey all started simultaneously 4minutes ago. How much longer will the race need to continue for them toall cross the start line again at the same time?

976

1. 23.33 hours2. 46.67 hours3. 60.00 hours4. 83.33 hours5. 106.67 hours

977

Question 30:A class of 60 2nd year medical students are conducting an experiment

to measure the velocity of nerve conduction along their radial arteries.is work builds on a previous result obtained demonstrating the effectsof how right handed men have faster nerve conduction velocities thangender matched left handed individuals. 60% of the class are female ofwhich 3% were unable to take part due to underlying heart conditions. 2 ofthe male members of the class were also unable to take part. On averagethe female cohort had faster nerve conduction velocities than men in theirdominant arm.

Right handed women have the fastest nerve conduction velocities.

978

A) TrueB) FalseC) Can’t tell

979

Question 31:Mark is making a double tetrahedron dice by joining two square based

pyramids together at their bases. Each square based pyramid is 5cm wideand 8cm tall. What are of card would have been required to produce thenets for the whole die?

980

A) 150cm2

B) 180 cm2

C) 210 cm2

D) 240 cm2

E) 270 cm2

981

Question 32:A serial dilution is performed by lining up 10 wells and lling each one

with 9ml of distilled water. 1 ml of a concentrated solvent is then added tothe rst well and mixed. 1 ml of this new solution is drawn from the rstwell and added to the second and mixed. e process is repeated until all10 wells have been used.

If the solvent starts off at concentration x, what will its nalconcentration be after 10 wells of serial dilution?

982

1. x/109

2. x/1010

3. x/1011

4. x/1012

5. x/1013

983

Question 33:A student decides to measure the volume of all the blood in his body.

He does this by injecting a known quantity of substrate into his arm,waiting a period of 20 minutes, then drawing a blood sample andmeasuring the concentration of the substrate in his blood.

What assumption has he made here?A) e substrate is only soluble in bloodB) e substrate is not bioavailableC) e substrate is not excretedD) e substrate is not degradedE) All of the aboveF)

Question 34:Jason is ordering a buffet for a party. e buffet company can provide a

basic spread at £10 a head. However more luxurious items carry asurcharge. Jason is particularly interested in cup-cakes and shell sh. Withthese items included the buffet company provides a new quote of £10 ahead. In addition to simply ordering the food Jason must also purchasecutlery and plates. Plates come in packs of 20 for £8 whilst cutlery is soldin bundles of 60 sets for £10.

With a budget of £2,300 (to the nearest 10 people) what is themaximum number of people Jason can provide food on a plate for?

984

A) 180B) 190C) 209D) 210E) 220

985

Question 35:What were once methods of hunting have now become popular sports.

Examples include archery, the javelin throw, the discuss through and eventhrowing a boomerang. Why such dangerous hobbies have begun to thriveis now being looked into by social scientists.

One such explanation is that it is because they are dangerous we ndthem appealing in the rst place. Others argue that it is a throwback to ourancestral heritage, where as a hunter gatherer being a pro cient hunterwas something to show off and aunt. Whilst this may be the case it is wellobserved that many nd the chase of a hunt exciting if not controversial.

Sports like archery provide excitement analogous to that of the chaseduring a hunter gatherer hunt.

986

A) TrueB) FalseC) Can’t tell

987

END OF SECTIONSection 2Question 1:A crocodile’s tail weighs 30kg. Its head weighs as much as the tail and

one half of the body and legs. e body and legs together weigh as much asthe tail and head combined.

What is the total weight of the crocodile?

988

A) 220kgB) 240kgC) 260kgD) 280kgE) 300kg

989

Question 2:A body is travelling at x ms-1 with y J of kinetic energy. After a period of

retardation the kinetic energy of the body is 1/16y. Assuming that themass of the body has remained constant what is its new velocity?

990

A) 1/196xB) 1/16xC) 1/8xD) 1/4xE) 4x

991

Question 3:Which of the following cannot be classi ed as an organ?

992

1. Blood2. Bone3. Larynx4. Pituitary Gland5. Prostate6. Skeletal Muscle7. Skin

993

A) 1 and 6B) 2 and 3C) 5 and 7D) 1 and 5E) 1,4, 5 and 6

994

Question 4:An increase in aerobic respiratory rate could be associated with which

of the following physiological changes?

995

1. A larger percentage of water vapour in expired air2. Increased expired CO23. Increased inspired O24. Perspiration5. Vasodilation

996

A) 3 onlyB) 1 and 2 onlyC) 1, 2 and 3 onlyD) 2, 3 and 5E) All of the above

997

Question 5:e nephron is to the kidney, as the ? is to striated muscle:

998

A) Actin lamentB) ArteryC) Myo brilD) SarcomereE) Vein

999

Question 6:A diabetic patient’s glucagon and insulin levels are measured over 4

hours. During this time the patient is given two large boluses of glucose. Agraphical representation of this is shown below.

At which times would you expect the patients’ blood glucose to begreatest?A) 05:00 and 12:00B) 07:00 and 14.00C) 08:00 and 15:00D) 10:00 and 13:00E) 06:00, 10:00 and 16:00

Question 7:In addition to the A, B or O classi cation, blood groups can also be

distinguished by the presence of Rhesus antigen (Rh). Care must be takenin blood transfusion as once blood types are mixed a Rh -ve individual willmount an immune response against Rh +ve blood. is is particular wellexempli ed in haemolytic disease of the newborn – where a Rh-ve mothercarries a Rh+ve foetus.

Applying what is written here and your knowledge of the humanimmune system, explain why the mother’s rst child would be relativelysafe and unaffected, yet further offspring would be at high risk.A) e rst pregnancy is always such a shock to the body it compromisesthe immune system

1000

B) Antibodies take longer than 9 months to produce and mature to anactive stateC) First born children are immunologically privilegedD) ere is a high risk of haemorrhage to both mother and child duringbirthE) Plasma T cells require time to multiply to lethal levels

Question 8:e pH of a solution has the greatest effect on which type of

interaction?

1001

A) Van der WaalsB) Induced dipoleC) Ionic bondingD) Metallic interactionE) Hydrogen bonding

1002

Question 9:At present a large effort is being made to produce tailored patient care.

One of the ultimate goals of this is to be able to grow personal, geneticallyidentical organs for those with end stage organ failure. is process will

rst require the harbouring of what cell type?

1003

A) Cells from the organ that is failingB) Haematopoietic stem cellsC) Embryonic stem cellsD) Adult stem cellsE) All of the above

1004

Question 10:When comparing different isotopes of the same element, which of the

following may change?

1005

1. Atomic number2. Mass number3. Number of electrons4. Chemical reactivity

1006

A) 1 onlyB) 1 and 2 onlyA) 3 onlyB) 2 and 3 onlyC) All of the above

1007

Question 11:From which of the following elemental groups are you most likely to

nd a catalyst?

1008

A) Alkali MetalsB) d-block elementsC) Alkaline Earth MetalsD) Noble GasesE) Halogens

1009

Question 12:1.338kg of francium are mixed in a reaction vessel with an excess of

distilled water. What volume will the hydrogen produced occupy at roomtemperature and pressure?

1010

A) 20.4dm3

A) 36dm3

B) 40.8dm3

C) 60.12dm3

D) 72dm3

1011

Question 13:e composition of a compound is Carbon 30%, Hydrogen 40%,

Fluorine 20%, and Chlorine 10%.What is the empirical formula of this compound?

1012

A) CH2FClB) C3H2F2ClA) C3H4FClB) C3H4F2ClC) C4H4F2Cl

1013

Question 14:What is the actual molecular formula of the compound in question 13

if the Mr is 340.5?

1014

A) C3H4F2ClB) C6H8F4Cl2

A) C9H12F6Cl3

B) C12H16F8Cl4

C) C15H20F10Cl5

1015

Question 15:1.2 x 1010 kg of sugar is dissolved in 4 x 1012L of distilled water. What is

the concentration?

1016

A) 3 x 10-2 g/dLB) 3 x10-1 g/dLC) 3 x 101 g/dLD) 3 x 102 g/dLE) 3 x 103 g/dL

1017

Question 16:Which of the following is not essential for the progression of an

exothermic chemical reaction?A) Presence of a catalystB) Increase in entropyC) Achieving activation energyD) Attaining an electron con guration more closely resembling that of anoble gasE) None of the above

Question 17:What is a common use of cationic surfactants?

1018

A) ShampooB) LubricantA) CosmeticsB) DetergentsC) All of the above

1019

Question 18:Which of the following is a unit equivalent to the Volt?

1020

A) A.Ω-1

A) J.C-1

B) W.s-1

C) C.sD) W.C.Ω

1021

Question 19:Complete the sentence below:A voltmeter is connected in _____ and therefore has _____ resistance;

whereas an ammeter is connected in _____ and has _____ resistance.A) Parallel, zero, parallel, in niteB) Parallel, zero, series, in niteC) Parallel, in nite, series, zeroD) Series, zero, parallel, in niteE) Series, in nite, parallel, zero

Question 20:A body “A” of mass 12kg travelling at 15m/s undergoes inelastic

collision with a xed, stationary object “B” of mass 20kg over a period of0.5 seconds.

After the collision body A has a new velocity of 3m/s. What force musthave been dissipated during the collision?

1022

A) 288NA) 298NB) 308NC) 318ND) 328N

1023

Question 21:What process is illustrated here:

1024

A) ermal decompositionB) Alpha decayA) Beta decayB) Gamma decay

1025

Question 22:A radio dish is broadcasting messages into deep space on a 20 Hz

radio frequency of wavelength 3km. With every hour how much furtherdoes the signal travel into deep space?

1026

A) 200,000 kmB) 216,000 kmC) 232,000 kmD) 248,000 kmE) 264,000 km

1027

Question 23:

A formula: is given. Would you expect this formula to calculate:

1028

A) A lengthB) An areaC) A volumeD) A volume of rotationE) A geometric average

1029

Question 24:

Evaluate the following:

1030

A) 2.09979 x 106

B) 2.09979 x 107

C) 2.09979 x 108

D) 2.09979 x 109

A) 2.09979 x 1010

1031

Question 25:Calculate a – b

A) 0o

B) 5o

C) 10o

D) 15o

E) 20o

Question 26:Jack has a bag with a complete set of snooker balls (15 red, 1 yellow, 1

green, 1 brown, 1 blue , 1 pink and 1 black ball) within it. Blindfolded Jackdraws two balls from the bag.

What is the probability that he draws a blue and a black ball in anyorder?

1032

A) 2/41A) 2/210B) 1/210C) 1/105D) 2/441

1033

Question 27:An experiment is repeated using an identical methodology and upon

further review it is proven to demonstrate identical scienti c practice. Ifthe result obtained is different to the rst, this would be due to:

1034

A) Calibration BiasB) Systematic BiasC) Random ChanceD) Serial dilutionE) Inaccuracies in the methodology

1035

END OF SECTIONSection 3

1) Doctors should be wearing white coats as it helps produce a placeboeffect making the treatment more effective.

Explain what is meant by this statement. Argue to the contrary. Towhat extend do you agree with the statement? What points can you seethat contradict this statement?2) “Medicine is a science of uncertainty and an art of probability.”

William OslerExplain what this statement means. Argue to the contrary. To what

extent do you agree with the statement?3) “e New England Journal of Medicine reports that 9 out of 10doctors agree that 1 out of 10 doctors is an idiot.”

Jay LenoWhat do you understand by this statement? Explain why the

assumption above may be inaccurate and argue to the contrary.4) “My father was a research scientist in tropical medicine, so I alwaysassumed I would be a scientist, too. I felt that medicine was too vagueand inexact, so I chose physics.”

Stephen HawkingExplain what this statement means. Argue to the contrary. To what

extent do you agree with the statement?END OF PAPER

1036

MOCK PAPER B

Section 1Question 1:

Which frame should come next?

Question 2:Josh is painting the outside walls of his house. e paint he has chosen

is sold only in 10L tins. Each tin costs £4.99. Assuming a litre of paintcovers an area of 5m2, and the total surface area of Josh’s outside walls is1050m2; what is the total cost of the paint required if Josh wants to apply 3coats?

1037

A) £104.79B) £209.58C) £314.37D) £419.16E) £523.95

1038

Question 3:e stars of the night sky have remained unchanged for many

hundreds of years, which allows sailors to navigate using the North Starstill to this day. However this only applies within the northern hemisphereas the populations of the southern hemisphere are subject to analternative night sky.

An asterism can be used to locate the North Star, it comes by manynames including the plough, the saucepan and the big dipper. Whilst theNorth Star’s position remains xed in the sky (allowing it to reliably alwayspoint north) the rest of the stars traverse around the North Star in asingular motion. In a very long time, the North Star will one day movefrom its location due to the movement of the Earth.

Which of the following is NOT an assumption made in this argument?A) e Earth is rotating on its axisB) Sailors still have need to navigate using the starsC) An analogous southern star is used to navigate in the SouthernhemisphereD) e plough is not the only method of locating the North StarE) None of the above

Question 4:John wishes to deposit a cheque. e bank’s opening times are 9am

until 5pm Monday to Friday, 10am until 4pm on Saturdays and the bank isclosed on Sundays. It takes on average 42 bank hours for the money from acheque to become available.

If John needs the money by 8pm Tuesday, what is the latest he cancash the cheque?

1039

A) 5pm the Saturday beforeB) 5pm the Friday beforeC) 1pm the ursday beforeD) 1pm the Wednesday beforeE) 9am the Tuesday before

1040

Question 5:How many different diamonds are there in the image shown below?

A) 25B) 32C) 48D) 58E) 63

Question 6:In 4 years time I will be one third the age that my brother will be next

year. In 20 years time he will be double my age. How old am I?

1041

A) 4B) 9C) 15D) 17E) 23

1042

Question 7:Aneurysmal disease has been proven to induce systemic in ammatory

effects, reaching far beyond the site of the aneurysm. e in ammatorymediator responsible for these processes remains unknown, however theeffects of systemic in ammation have been well categorised and observedexperimentally in pig models.

In particular this in ammation induces an aberration of endothelialfunction within the inner most layer of blood vessel walls. eendothelium not only represents the lining of blood vessels but also acts asa transducer converting the haemodynamic forces of blood into abiological response. An example of this is the NO pathway, which uses theshear stress induced by increased blood ow to drive the formation of NO.NO diffuses from the endothelium into the smooth muscle surroundingblood vessels to promote vasodilation and therefore acts to reduce blood

ow.Failure of this process induces high risk of vascular damage and

therefore cardiovascular diseases such as thrombosis and atherosclerosis.What is a valid implication from the text above?

A) Aneurysmal disease does not affect the NO pathwayB) Aneurysms directly increase the likelihood of cardiovascular diseaseC) Aneurysms are the opposite of transducersD) Observations of this kind should be made in humans to see if theresults can be replicatedE) Aneurysms induce high blood ow

Question 8:A traffic surveyor is stood at a T-junction between a main road and a

side street; he is only interested in traffic leaving the side street. He logs theclass of vehicle, the colour and the direction of travel once on the mainroad.

1043

During an 8 hour period he observes a total of 346 vehicles includingbikes. Of which 200 were travelling west whilst the rest travelled east. eover whelming majority of vehicles seen were cars at 90% with bikes, vansand articulated lorries together comprising the remaining 10%. Red wasthe most common colour observed whilst green was the least. Black andwhite vehicles were seen in equal quantities.

Which of the following is an accurate inference based on his survey?A) Global sales are highest for those vehicles which are coloured redB) Cars are the most popular vehicle on all roadsC) Green vehicle sales are down in the area that the surveyor was basedD) e daily average rate of traffic out of a T junction in Britain is 346vehicles over 8 hoursE) To the east of the junction is a dead end

Question 9:

Question 10:A television is delivered in a box that has volume 60% larger than that

of the television. e television is 150cm x 100cm x 10m. How muchsurplus volume is there?

1044

A) 0.09m2

A) 0.9 m2

B) 9 m2

C) 90 m2

D) 900 m2

1045

Question 11:Matthew and David are deciding where they would like to go camping

Friday to Sunday. Upon completing their research they discover thefollowing:➢ Whitmore Bay charges £5.50 a night and does not require a booking.e site provides showers, washing up facilities and easy access to a beach➢ Port Eynon charges £5 a night and a booking is compulsory. Howeverthe site does not provide showers but does have 240V sockets free ofcharge➢ Jackson Bay charges £7 a night and is billed as a luxury site withcompulsory booking, private showers, toilets, mobile phone chargingfacilities and kitchens.

David present the following suggestion:As Port Eynon is the farthest distance to travel the bene t of its cheap

nightly rate is negated by the cost of petrol. Instead he recommends theyvisit Jackson Bay as it is the shortest distance to travel and will thereforebe the cheapest.

Which of the following best illustrates a aw in this argument?A) Whitmore bay may be only a few miles further which means the totalcost would be less than visiting Jackson BayB) With kitchen facilities available they will be tempted to buy more foodincreasing the costC) e campsite may be fully bookedD) ere may be a booking fee driving the cost up above that of the othercampsitesE) All of the above

Question 12:e manufacture of any new pharmaceutical is not permitted without

scrupulous testing and analysis. is has led to the widespread, andcontroversial use of animal models in science.

1046

Whilst it is possible to test cyto-toxicity on simple cell cultures, to trulypredict the effect of a drug within a physiological system it must be trialledin a whole organism. With animals cheap to maintain, readily available,rapidly reproducing and not subject to the same strict ethical laws theyhave become an invaluable component of modern scienti c practice.

Which of the following best illustrates the main conclusion of thisargument?A) New pharmaceuticals cannot be approved without animalexperimentationB) Cell culture experiments are unhelpfulC) Modern medicine would not have achieved its current standardwithout animal experimentationD) Logistically animals are easier to keep than humans for mandatoryexperimentsE) All of the above

e information below relates to questions 13 – 17:

e graph above shows item pricing from a wholesaler. e wholesaleris happy to deliver for a cost of £35 to companies or £5 to individuals. Anyorder over the cost of £100 qualify for free delivery. Items are de ned as

1047

how they come to the wholesaler therefore 1 item = 2 rugby balls or 1football or 5 tennis balls.

Question 13:What is the total cost to an individual purchasing 12 rugby balls and

120 tennis balls?

1048

A) £174B) £179C) £208D) £534E) £588

1049

Question 14:A private gym wishes to purchase 10 of everything, how short are they

of the free delivery boundary?

1050

A) £5.00B) £5.01C) £10.00D) £10.01E) ey are already over the boundary

1051

Question 15:What is the most number of balls that can be bought by an individual

with £1,000 pounds.Who is sitting in the front right seat?

1052

1. 2002. 2503. 5004. 1,0005. 1,250

1053

Question 16:e wholesaler sells all his products for a pro t of 120%. If he sells

£1,320 worth of goods at his prices, what did he actually spend onacquiring them himself ?

1054

A) £400B) £600C) £800D) £1,100E) £1,120

1055

Question 17:If the wholesaler pays 25% tax on the amount over £12,000 pounds;

how much tax does he pay when receiving an order of 2000 of each item?

1056

A) £2,000B) £3,000C) £4,000D) £5,000E) £6,000

1057

Question 18:

Which frame best ts the algorithm observed in the sequence ofdiagrams above?

Question 19:A train driver runs a service between Cardiff and Merthyr. On average

a one way trip takes 40 minutes to drive but he requires 5 minutes tounload passengers and a further 5 minutes to pick up new ones. As thecrow ies the distance between Cardiff and Merthyr is 22 miles.

Assuming he works an 8 hour shift with two 20 minute breaks, andwhen he arrives to work the rst train is already loaded with passengershow far does he travel?

1058

A) 132B) 143C) 154D) 176E) 198

1059

Question 20:e massive volume of traffic that travels down the M4 corridor

regularly leads to congestion at times of commute morning and evening. Acase is being made by local councils in congestion areas to introduce relieflanes thus widening the motorway in an attempt to relieve the congestion.is would involve introducing either a new 2 or 4 lanes to the motorwayon average costing 1 million pound per lane per 10 miles.

Many conservationist groups are concerned as this will involve thedestruction of large areas of countryside either side of the motorway. eyargue that the side of a motorway is a unique habitat with many rarespecies residing there.

e local councils argue that with many hundreds if not thousands ofcars siding idle on the motorway pumping pollutants out into thesurrounding areas, it is better for the wildlife if the congestion is eased andtraffic can ow through. e councils have also remarked that ifcongestion is eased there would be less money needed to repair the roadsfrom car incidents with could in theory be given to the conservationistgroups as a grant.

Which of the following is assumed in this passage?A) Wildlife living on the side of the motorway cannot be re-homedB) Congestion causes car incidentsC) Relief lanes have been proven to improve traffic jamsD) A and BE) B and CF) All of the aboveG) None of the above

Question 21:Apples and oranges are sold in packs of 5 for the price of £1 and £1.25

respectively. Alternatively apples can be purchased individually for 30pand oranges can be purchased individually for 50p.

1060

Helen is making a fruit salad, she remarks that order would have costher an extra £6.25 if she had purchased the fruit individually. Which of thefollowing could have been her order?A) 15 apples 10 orangesB) 15 apples 15 orangesC) 25 apples 10 orangesD) 25 apples, 15 orangesE) 30 apples, 30 oranges

Question 22:Janet is conducting an experiment to assess the sensitivity of a

bacterial culture to a range of antibiotics. She grows the bacteria so theycover an entire petri dish and then pipettes a single drop of antibiotic atdifferent locations. A schematic of her results are shown below whereblack represents growth of bacteria.

Which of the following best describes Janet’s results?A) is strain of bacteria is susceptible to all antibiotics usedB) is strain of bacteria is susceptible to none of the antibiotics usedC) E was the most effective antibioticD) C was the most ineffectiveE) D is the most likely to be used in further testing

Question 23:

1061

Laura is blowing up balloons for a birthday party. e average volumeof a balloon is 300cm3 and Laura’s maximum forced expiratory rate in asingle breathe is 4.5l/min. What is the fastest Laura could in ate 25balloons assuming it takes her 0.5 secs to breathe in per balloon in andsomebody else ties the balloons for her?

1062

1. 112.5 seconds2. 122.5 seconds3. 132.5 seconds4. 142.5 seconds5. 152.5 seconds

1063

Question 24:George reasons that A is equal to B which is not equal to C. In which

case C is equal to D which is equal to E.Which of the following, if true, would most weaken George’s argument?

1064

A) A does not equal DB) B is equal to EC) A and B are not not equalD) C is equal to 0E) None of the above

1065

Question 25:In a single day how many times do the hour, minute and second hands

of analogue clock all point to the same number?

1066

A) 12B) 24C) 36D) 48E) 72

1067

Question 26:

A game of noughts and crosses is illustrated below. James is noughtsand it is his turn to play; he is thinking about the outcome of the game andtrying to devise a winning strategy. If James plays perfectly can he win?

A) Cannot winB) Possible to winC) Certain to win

Question 27:

Question 28:

1068

e statement “e human race is not dependent on electricity”assumes what?A) We have no other energy resourceB) Electricity is cheapC) Electrical appliances dominate our livesD) Electricity is now the accepted energy source and is therefore the onlyone availableE) All of the above

Question 29:Wine is sold in cases of 6 bottles. A bottle of wine holds 70cl of uid

whereas a wine glass holds 175ml. Cases of wine are currently on offer £42a case but by one get one free.

If Elin is hosting a 3 course dinner party for 27 of her friends, and shewould like to provide everyone with a glass of wine per course, how muchwill the wine cost her?

1069

1. £422. £843. £1264. £1685. £210

1070

Question 30:Hannah buys a television series in boxset. It contains a full 7 series

with each series comprising 12 episodes. Rounded to the nearest 10 eachepisode lasts 40 minutes.

What is the shortest amount of time it could possible take to watch allthe episodes back to back?

1071

A) 49 hoursB) 51 hoursC) 53 hoursD) 56 hoursE) 60 hours

1072

Question 31:Many are familiar with the story that aided in the discovery of “germ”.

Semmelweis worked in a hospital where maternal death rates duringlabour were astronomically high. He noticed that medical students oftenwent straight from dissection of cadavers to the maternity wards. As anexperiment Semmelweis split the student cohort in half. Half did theirmaternity rotation instead before dissection whereas the other halfmaintained their traditional routine. In the new routine, maternity wardbefore dissection, Semmelweis recorded an enormous reduction inmaternal deaths and thus the concept of the pathogen was born.

What is best exempli ed in this passage?A) Science is a process of trial and errorB) Great discoveries come from pattern recognitionC) Provision of healthcare is closely associated with technologicaladvancementsD) Experiments always require a controlE) All of the above

Question 32:Jack sits at a table opposite a stranger. e stranger says here I have 3

precious jewels: a diamond, a sapphire and an emerald. He tells Jack that ifhe makes a truthful statement Jack will get one of the stones, if he lies hewill get nothing.

What must Jack say to ensure he gets the sapphire?

1. Tell the stranger his name2. Tell the stranger he must give him the sapphire3. Tell the stranger he wants the emerald4. Tell the stranger he does not want the emerald or the diamond5. Tell the stranger he will not give him the emerald or the diamond

Question 33:

1073

Simon invests 100 pounds in a saver account that awards compoundinterest on a 6 monthly basis at 50%. Simon’s current account awardscompound interest on a yearly basis at 90%.

After 2 years will Simon’s investment in the saver account yield moremoney than it would have in the current account?

1074

A) YesB) NoC) Can’t tell

1075

Question 34:My mobile phone has a 4 number pin code using the values 1 – 9. To

determine this I use a standard algorithm of multiplying the rst twonumbers, subtracting the third and then dividing by the fourth. I changethe code by changing the answer to this algorithm – I call this the key.What is the largest possible key?

1076

A) 42B) 55C) 70D) 80E) 81

1077

Question 35:

Which frame comes next in the sequence above?

END OF SECTIONSection 2Question 1:GLUT2 is an essential, ATP independent, mediator in the liver’s uptake

of plasma glucose. is is an example of:

1078

1. Active transport2. Diffusion3. Exocytosis4. Facilitated Diffusion5. Osmosis

1079

Question 2:e molecular weight of glucose is 180 g/mol. 5.76Kg of glucose is split

evenly between two cell cultures under anaerobic conditions. One cellculture is taken from human cardiac muscle, whilst the other is a yeastculture. What will be the difference (in moles) between the amount of CO2

produced between the two cultures.

1080

A) 0 molB) 4 molC) 8 molD) 12 molE) 16 mol

1081

Question 3:Which of the following cell types will have the greatest ux along

endocytotic pathways?

1082

A) Kidney cellsB) Liver cellsC) Nerve cellsD) Red blood cellsE) White blood cell

1083

Question 4:Compared to the Krebs cycle, the Calvin cycle demonstrates which of

the following differences?

1084

A) CO2 as a substrate rather than a productB) Photon dependentC) Utilisation of different electron transportersD) Net loss of ATPE) All of the above

1085

Question 5:

Pepsin and trypsin are both digestive enzymes. Pepsin acts in thestomach whereas trypsin is secreted by the pancreas. Which graph below(trypsin in black and pepsin in grey) would most accurately demonstratetheir relative activity against pH?

Question 6:MRSA is an example of:

1086

A) Natural selectionB) Genetic engineeringC) Sexual reproductionD) LamarckismE) Co-dominance

1087

Question 7:What is the electron con guration of magnesium in MgCl2?

1088

A) 2,8B) 2,8,2C) 2,8,4D) 2,8,8E) None of the above

1089

Question 8:A calcium sample is run in a mass spectrometer. It is later discovered

that the sample was contaminated with the most abundant isotope ofchromium. A section of the trace is shown below. What was the actualabundance of the most common calcium isotope?

1090

A) 1/9B) 6/17C) 1/2D) 11/19E) 17/19

1091

Question 9:A warehouse receives 15 tonnes of arsenic in bulk. Assuming that the

sample is at least 80% pure, what is the minimum amount, in moles, ofarsenic that they have obtained?

1092

A) 1.6 x 105

B) 2 x 105

C) 1.6 x 106

D) 2 x 106

E) 1.6 x 107

1093

Question 10:A sample of silicon is run in a mass spectrometer. e resultant trace

shows m/z peaks at 26 and 30 with relative abundance 60% and 30%respectively. What other isotope of silicon must have been in the sample togive an average atomic mass of 28?

1094

A) 28B) 30C) 32D) 34E) 36

1095

Question 11:72.9g of pure magnesium ribbon is mixed in a reaction vessel with the

equivalent of 54g of steam. e ensuing reaction produces 72dm3 ofhydrogen. Which of the following statements is true?

1096

A) is is a complete reactionB) is is a partial reactionC) ere is an excess of steamD) ere is an excess of magnesiumE) Magnesium hydroxide is a product

1097

Question 12:Which species is the reducing agent in: 3Cu2+ + 3S2- + 8H+ + 8NO3

-

3Cu2+ + 3SO42- + 8NO + 4H2O

1098

A) Cu2+

B) S2-

C) H+

D) NO3-

E) H2O

1099

Question 13:Which of the following is not true of alkanes?

A) CnH2n+2B) SaturatedC) ReactiveD) Produce only CO2 and water when burnt in an excess of oxygenE) None of the above

Question 14:A rubber balloon is in ated and rubbed against a sample of animal fur

for a period of 15 seconds. At the end of this process the balloon iscarrying a charge of -5 coulombs. What magnitude of current must havebeen induced during the process of rubbing the balloon against the animalfur; and in which direction was it owing?

1100

A) 0.33A into the balloonB) 0.33A into the furC) 0.33A in no net directionD) 75A into the balloonE) 75A into the fur

1101

Question 15:Which of the following is a unit equivalent to the Amp?

1102

A) V.ΩB) (W.V)/sC) C.ΩD) (J.s-1)/VE) C.s

1103

Question 16:e output of a step down transformer is measured at 24V and 10A.

Given that the transformer is 80% efficient what must the initial powerinput have been?

1104

A) 240WB) 260WC) 280WD) 300WE) 320W

1105

Question 17:An electric winch system hoists a mass of 20kg 30 metres into the air

over a period of 20 seconds. What is the power output of the winchassuming the system is 100% efficient?

1106

A) 100WB) 200WC) 300WD) 400WE) 500W

1107

Question 18:6 x 1010 atoms of a radioactive substance remain, the activity of the

substance is quanti ed as 3.6 x 109. What is the decay constant of thismaterial?

1108

A) 0.00006B) 0.0006C) 0.006D) 0.06E) 0.6

1109

Question 19:An 80W lament bulb draws 0.5A of household electricity. What is the

efficiency of the bulb?

1110

A) 25%B) 33%C) 50%D) 66%E) 75%

1111

Question 20 :

Rearrange the following equation in terms of t:

A)

B)

C)

D)

E) Question 21:An investment of £500 is made in a compound interest account. At the

end of 3 years the balance reads £1687.50. What is the interest rate?

1112

A) 20%B) 35%C) 50%D) 65%E) 80%

1113

Question 22:What is the equation of the line of best t for the scatter graph below?

1114

A) y = 0.2x + 0.35B) y = 0.2x – 0.35C) y = 0.4x + 0.35D) y = 0.4x – 0.35E) y = 0.6x + 0.35

1115

Question 23 :

Simplify:

1116

A)

B)

C)

D)

A)

1117

Question 24:Which of the following is a suitable descriptive statistic for non-

normally distributed data?

1118

A) MeanB) Normal rangeC) Con dence intervalD) Interquartile rangeE) Mode

1119

Question 25:Which best describes the purpose of statistics?

A) Evaluate acceptable scienti c practiceB) Reduce the ability of others to criticise the dataC) To quickly analyse dataD) Calculate values representative of the population from a subset sampleE) To allow for universal comparison of scienti c methods

Question 26:A rotating disc has two wells, in which bacteria are cultured. e rst

well is 10 cm from the centre whereas the second well is 20 cm from thecentre. If the inner well completes a revolution in 1 second, how muchfaster is the outer well travelling?

1120

A) 0.314m/sB) 0.628m/sC) 0.942m/sD) 1.256m/sE) 1.590m/s

1121

Question 27:

Which is the equivalent function to: ?

1122

A)

B)

C)

D)

A)

1123

END OF SECTIONSection 3

1) “Progress is made by trial and failure; the failures are generally ahundred times more numerous than the successes; yet they are usuallyleft unchronicled.”

Williams RamseyExplain what this statement means. Argue to the contrary. To what

extent do you agree with the statement?2) “He who studies medicine without books sails an uncharted sea, buthe who studies medicine without patients does not go to sea at all.”

William OslerExplain what this statement means. Argue to the contrary. To what

extent do you agree with the statement?1) ‘“Medicine is the restoration of discordant elements; sickness is thediscord of the elements infused into the living body”

Leonardo da VinciExplain what this statement means. Argue to the contrary. To what

extent do you think this simpli cation holds true within modernmedicine?2) “Modern medicine is a negation of health. It isn’t organized to servehuman health, but only itself, as an institution. It makes more peoplesick than it heals.”

Ivan IllichWhat does this statement mean? Argue to the contrary, that the

primary duty of a doctor is not to prolong life. To what extent do you agreewith this statement?

END OF PAPER

1124

MOCK PAPER C

Section 1Question 1:Adam, Beth and Charlie are going on holiday together. A single room

costs £60 per night, a double room costs £105 per night and a four personroom costs £215 per night. It is possible to opt out from the cleaningservice and to pay £12 less each night per room.

What is the minimum amount the three friends could pay for theirholiday for a three night stay at the hotel?

1125

1. £1222. £1443. £2034. £4235. £432

1126

Question 2:I have two 96ml glasses of squash. e rst is comprised of squash and

water. e second is comprised of water and squash. I take 48ml from therst glass and add it to glass two. I then take 72ml from glass two and add

it to glass one. How much squash is now in each glass?

1. 16ml squash in glass one and 72ml squash in glass two2. 40ml squash in glass one and 32ml squash in glass two3. 48ml squash in glass one and 32ml squash in glass two4. 48ml squash in glass one and 40ml squash in glass two5. 80ml squash in glass one and 40ml squash in glass two

Question 3:It may amount to millions of pounds each year of taxpayers’ money;

however it is strongly advisable for the HPV vaccination in schools toremain. e vaccine, given to teenage girls, has the potential tosigni cantly reduce cervical cancer deaths and furthermore, the vaccineswill decrease the requirement for biopsies and invasive procedures relatedto the follow-up tests. Extensive clinical trials and continued monitoringsuggest that both Gardasil and Cervarix are safe and tolerated well byrecipients. Moreover, studies demonstrate that a large majority of teenagegirls and their parents are in support of the vaccine.

Which of the following is the conclusion of the above argument?

1. HPV vaccines are safe and well tolerated2. It is strongly advisable for the HPV vaccination in schools to remain3. e HPV vaccine amounts to millions of pounds each year of

taxpayers’ money4. e vaccine has the potential to signi cantly reduce cervical cancer

deaths5. Vaccinations are vital to disease prevention across the population

1127

Question 4:Anna cycles to school, which takes 30 minutes. James takes the bus,

which leaves from the same place as Anna, but 6 minutes later and gets toschool at the same time as Anna. It takes the bus 12 minutes to get to thepost office, which is 3km away. e speed of the bus is the speed of thebike. One day Anna leaves 4 minutes late. How far does she get before sheis overtaken by the bus?

1128

1. 1.5km2. 2km3. 3km4. 4km5. 6km

1129

Question 5:e set two maths teacher is trying to work out who needs to be

moved up to set one and who to award a certi cate at the end of term. estudents must ful l certain criteria:

Who would move up a set and who would receive a certi cate?

1. Bahara would move up a set and receive a certi cate2. Bahara and Lucy would move up a set and Bahara would receive a

certi cate3. Bahara, Terry and Lucy would move up a set and Bahara and Shiv

would receive a certi cate4. Lucy would move up a set and Bahara would receive a certi cate5. Lucy would move up a set and Bahara and Terry would receive a

certi cate

Question 6:18 years ago A was 25 years younger than B is now. In 21 years time, A

will be 28 years older than B was 14 years ago. How old is A now if A is B?

1130

1. 272. 283. 354. 425. 46

1131

Question 7:e time now is 10.45am. I am preparing a meal for 16 guests who will

arrive tomorrow for afternoon tea. I want to make 3 scones for each guest,which can be baked in batches of 6. Each batch takes 35 minutes toprepare and 25 minutes to cook in the oven and I can start the next batchwhile the previous batch is in the oven. I also want to make 2 cupcakes foreach guest, which can be baked in batches of 8. It takes 15 minutes toprepare the mixture for each batch and 20 minutes to cook them in theoven. I will also make 3 cucumber sandwiches for each guest. 6 cucumbersandwiches take 5 minutes to prepare.

What will the time be when I nish making all the food for tomorrow?

1132

1. 4:35pm2. 5.55pm3. 6:00pm4. 6:05pm5. 7:20pm

1133

Question 8:

What is the difference between the height of the smallest and tallestpyramids?

1134

1. 1m2. 5m3. 4m4. 6m5. 8m

1135

Question 9:e wage of Employees at Star Bakery is calculated as: £210 + (Age x

1.2) – 0.8 (100 - % attendance)Jessica is 35 and her attendance is 96% – Samira is 65 and her

attendance is 89%What is the difference between their wages?

1136

1. £30.42. £60.53. £248.84. £263.25. £279.2

1137

Question 10:It is important that research universities demonstrate convincing

support of teaching. Undergraduates comprise an overwhelmingproportion of all students and universities should make an effort to caterto the requirements of the majority of their student body. After all, many ofthese students may choose to pursue a path involving research and astrong education would provide students with skills equipped towards acareer in research.

What is the conclusion of the above argument?

1. Undergraduates comprise an overwhelming proportion of allstudents

2. A strong education would provide a strong foundation and skillsequipped towards a career in research

3. Research universities should strongly support teaching4. Institutions should provide undergraduates with a high quality

learning experience5. Research has a greater impact than teaching and limited universities

funds should mainly be invested in research

Question 11:American football has reached a level of violence that puts its players

at too high a level of risk. It has been suggested that the NFL, thegoverning body for American football should get rid of the iconic helmets.e hard plastic helmets all have to meet minimum impact-resistancestandards intended to enhance safety, however in reality they gave playersa false sense of security that only resulted in harder collisions. Someplayers now suffer from early onset dementia, mood swings anddepression. e proposal to ban helmets for good should be supported.Moreover, it would prevent costly legal settlements involving the NFL andex-players suffering from head trauma.

1138

What is the conclusion of the above argument?

1. Sports players should not be exposed to unnecessary danger2. Helmets give players a false sense of security3. Players can suffer from early onset dementia, mood swings and

depression4. e proposal to ban helmets should be supported5. American football is too violent and puts its players at risk

Question 12:At the nal stop (stop 6), 10 people get off the tube. At the previous

stop (stop 5) of passengers got off. At stop 4, of passengers got off. At stop3, of passengers got off and at stops 1 and 2, of passengers got off.

How many passengers got on at the rst stop?

1139

1. 102. 363. 904. 1085. 3600

1140

Question 13:Everyone likes English. Some students born in spring like Maths and

some like Biology. All students born in winter like Music and some like Art.Of those born in autumn, no one likes Biology and everyone likes Art.

Which of the following is true?

1. Some students born in spring like both Biology and Maths2. Students born in spring, winter and autumn all like Art3. No one born in winter or autumn likes Biology4. No one who likes Biology also likes Art5. Some students born in winter like 3 subjects

Question 14:Until the twentieth century, the whole purpose of art was to create

beautiful, awless works. Artists attained a level of skill and craft that tookdecades to perfect and could not be mirrored by those who had not takengreat pains to master it. e serenity and beauty produced frommovements such as impressionism has however culminated in repulsiveand horri c displays of rotting carcasses designed to provoke anemotional response rather than admiration. ese works cannot bedescribed as beautiful by either the public or art critics. While these worksmay be engaging on an intellectual or academic level, they no longerconstitute art.

Which of the following is an assumption of the above argument?

1. Beauty is a de ning property of art2. All modern art is ugly3. Twenty rst century artists do not study for decades4. e impressionist movement created beautiful works of art5. Some modern art provokes an emotional response

Question 15:

1141

e cost of sunglasses is reduced over the bank holiday weekend. OnSaturday, the price of the sunglasses on Friday is reduced by 10%. OnSunday the price of the sunglasses on Saturday is reduced by 10%. OnMonday, the price of the sunglasses on Sunday is reduced by a further 10%.What percentage of the price on Friday is the price of the sunglasses onMonday?

1142

1. 55.12%2. 59.10%3. 63.80%4. 70.34%5. 72.9%

1143

Question 16:Putting the digit 7 on the right hand side of a two digit number causes

the number to increase by 565. What is the value of the two digit number?

1144

1. 272. 523. 624. 665. 627

1145

Question 17:When folded, which box can be made from the net shown below?

.Question 18:e grid is comprised of 49 squares. e shape’s area is 588cm2. What

is its perimeter in cm?

1146

1. 262. 493. 844. 1265. 182

1147

Questions 19-21 refer to the following information:

Question 19:A child weighs 35kg and is 120cm tall. What is the BMI of the child to

the nearest 2 d.p?

1148

1. 0.00242. 24.283. 24.314. 42.015. 42.33

1149

Question 20:What is the BMR of a 32 year old woman weighing 80kg and

measuring 1.7m in height?

1150

1. 643.7 kcal2. 1537 kcal3. 1541.5 kcal4. 1707.5 kcal5. 2707.5 kcal

1151

Question 21:What is the recommended intake of a 45 year old man weighing 80kg

and measuring 1.7m in height who does little to no exercise each week?

1152

1. 1642.5 kcal2. 1771.8 kcal3. 1851 kcal4. 1971 kcal5. 2712.5 kcal

1153

Questions 22 and 23 relate to the following passage:e achievement levels of teenagers could be higher if school started

later. Teenagers are getting too little sleep because they attend schoolsthat start at 8:30am or earlier. A low level of sleep disrupts the body’scircadian rhythms and can contribute to health problems such as obesityand depression. Some doctors are now urging schools to start later inorder for teenagers to get sufficient sleep; ideally 8.5 to 9.5 hours eachnight. During adolescence, the hormone melatonin is releasedcomparatively later in the day and the secretion levels climb at night.Consequently, teenagers can have trouble getting to sleep earlier in thenight before sufficient melatonin is present. A school in America testedthis idea by starting one hour later and the percentage of GCSEs at gradesA*-C increased by 16%. Schools in the UK should follow by example andshift start times later.

Question 22:Which of the following is a aw in the above argument?

1. Slippery slope2. Hasty generalisation3. It confuses correlation with cause4. Circular argument5. Schools should not prioritise academic achievement

Question 23:Which of the following, if true, would most strengthen the above

argument?

1. Teenagers who are more alert will disrupt the class more2. Getting more sleep at night is proven to increase activity levels3. American schools and British schools teach the same curriculum

1154

4. e school in America did not alter any other aspects of the schoolduring the trial for example curriculum, teachers, and number ofstudents in each class

5. Teachers get tired towards the end of the school day and are lesseffective

Question 24:e UK energy market is highly competitive. In an effort to attract

more business and increase revenue, the company EnergyFirst hasdivested signi cant funds into its publicity sector. Last month, theydoubled their advertising expenditures, becoming the energy company toinvest the greatest proportion of investment into advertising. As a result itis expected that EnergyFirst will expand its customer base at a rateexceeding its competitors in the ensuing months. Other energy companiesare likely to follow by example.

Which of the following, if true, is most likely to weaken the aboveargument?

1. Other companies invest more money into good customer service2. Research into the energy industry demonstrates a low correlation

between advertising investment and new customers3. e UK energy market is not highly competitive4. EnergyFirst currently has the smallest customer base5. Customers are heavily in uenced by visual advertising

Question 25:e consumption of large quantities of red meat is suggested to have

negative health rami cations. Carnitine is a compound present in redmeat and a link has been discovered between carnitine and thedevelopment of atherosclerosis, involving the hardening and narrowing ofarteries. Intestinal bacteria convert carnitine to trimethylamine-N-oxide,

1155

which has properties that are damaging to the heart. Moreover, red meatconsumption has been associated with a reduced life expectancy. It maybe that charring meat generates toxins that elevate the chance ofdeveloping stomach cancer. If people want to be healthy, a vegetarian dietis preferable to a diet including meat. Vegetarians often have lowercholesterol and blood pressure and a reduced risk of heart disease.

Which of the following is an assumption of the above argument?

1. Diet is essential to health and we should all want to be healthy2. Vegetarians do the same amount of exercise as meat eaters3. Meat has no health bene ts4. People who eat red meat die earlier5. Red meat is the best source of iron

Question 26:Auckland is 11 hours ahead of London. Calgary is 7 hours behind

London. Boston is 5 hours behind London. e ight from Auckland toLondon is 22 hours, but the plane must stop for 2 hours in Hong Kong. e

ight from London to Calgary is 8 hours 30 minutes. e ight fromCalgary to Boston is 6 hours 30 minutes. Sam leaves Auckland at 10am forLondon. On arrival to London, he waits 3 hours then gets the plane toCalgary. Once in Calgary, he waits 1.5 hours and gets the plane to Boston.What time is it when Sam arrives in Boston?

1156

1. 13:30pm2. 22.30pm3. 01:00am4. 01:30am5. 03:30am

1157

Question 27:Light A ashes every 18 seconds, light B ashes every 33 seconds and

light C ashes every 27 seconds. e three lights all ashed at the sametime 5 minutes ago. How long will it be until they next all ashsimultaneously?

1158

1. 33 seconds2. 294 seconds3. 300 seconds4. 333 seconds5. 594 seconds

1159

Question 28:ere are 30 students in a class. ey must all play at least 1

instrument, but no more than 3 instruments. 70% play the piano, 40% playthe violin, 20% play the guitar and 10% play the saxophone. Which of thefollowing statements must be true?

1. 3 or more students play piano and violin2. 12 students or less play the piano and the violin3. 9 or more students do not play piano or violin

1160

1. 1 only2. 1 and 23. 2 only4. 2 and 35. 3 only

1161

Question 29:Neil, Simon and Lucy are playing a game to see who can role the

highest number with two dice. ey start with £50 each. e losers have tohalf their money and give it to the winner of each game. If it is a draw, thetwo winners share the loser’s money. If all three tie, then they keep theirmoney. Neil wins game 1, Simon and Lucy win game 2 and 5 and Lucy winsgames 3 and 4. How much money does Lucy gain?

1162

1. £15.632. £753. £75.164. £78.135. £128.1

1163

Question 30:Drivers in the age group 17-19 comprise 1.5% of all drivers; however

12% of all collisions involve young drivers in this age category. e RACFoundation wants a graduated licensing system with a 1 year probationaryperiod with restrictions on what new drivers can do on roads. Additionally,driving instructors need to emphasise the dangers of driving too fast anddriving tests should be designed to make new drivers more focused onnoticing potential hazards. ese changes are essential and could stop4,500 injuries on an annual basis.

What is the assumption of the above argument?

1. Young drivers are more likely to have more passengers than other agegroups

2. Young drivers spend more hours driving than older drivers3. Young drivers are responsible for the collisions4. e cars that young people drive are unsafe5. Most young drivers involved in accidents are male

Question 31:e mean weight of 6 apples is 180g. e lightest apple weighs 167g.

What is the highest possible weight of the heaviest apple?

1164

1. 193g2. 225g3. 235g4. 245g5. 255g

1165

Question 32:Which option represents the mirror image of the image shown?

Question 33:e farmer had 184 sheep to sell. He intended to sell each for £112.

However, he sold less than one quarter on day 1. As a result he reduced theprice by . On the second day he sold twice as many and made £3528 morethan on day 1, leaving him with less than of original. How many sheep didhe sell altogether?

1166

1. 422. 843. 984. 1125. 126

1167

Question 34:ere is no empirical evidence that human activities directly result in

global warming and this is used as a reason against decreasing carbonemissions. However, many scientists believe that human activity is highlylikely to cause global warming since higher levels of greenhouse gasescause the atmosphere to thicken, retaining heat. It therefore seemssensible that we should not wait for proof considering the catastrophiceffects of climate change, regardless of subsequent ndings. Similarly, if atree branch had a signi cant chance of falling on you, it would be sensibleto move away immediately.

What is the main conclusion?

1. Many scientists believe that human activity is highly likely to causeglobal warming

2. We should not wait for proof of climate change3. If a tree branch had a signi cant chance of falling on you, it would be

sensible to move away immediately4. e effects of climate change are catastrophic5. ere is no empirical evidence that human activities directly result in

global warming so we should not reduce carbon emissions

Question 35:e average of 8 numbers is y. If 13 and 31 are added, the mean of the

10 numbers is also y. What is y?

1168

1. 112. 223. 254. 275. 44

1169

END OF SECTION

1170

Section 2Question 1:Which of the following are correct regarding polymers?

1. Sucrose is formed by the condensation of hundreds ofmonosaccharides

2. Lactose is found in milk and is formed by condensation of twoglucose molecules

3. Glucose has two isomers4. Glycogen, starch and cellulose are all polysaccharides formed by

condensation of multiple glucose molecules5. People with lactose intolerance lack lactase and can experience

diarrhoea after drinking milk

1171

1. 1 only2. 1, 2 and 33. 1 and 3 only4. 3, 4 and 55. 4 and 5 only

1172

Question 2:Which of the following statements regarding enzymes are correct?

1. Enzymes are denatured at high temperatures or extreme pH values2. Amylase is produced in the salivary glands only and converts starch

to sugars3. Lipases catalyse the breakdown of oils and fats into glycerol and fatty

acids. is takes place in the small intestine4. Bile is stored in the pancreas and travels down the bile duct to

neutralise stomach acid5. Isomerase can be used to convert glucose into fructose for use in

slimming products

1173

1. 1 and 3 only2. 1, 3 and 4 only3. 1, 3 and 54. 2 and 4 only5. 3 and 5 only

1174

Question 3:Which of the following describes the role of the colon?

1. Food is combined with bile and digestive enzymes2. Storage of faeces3. Reabsorption of water4. Faeces leave the alimentary canal5. Any digested food is absorbed into the lymph and blood

Question 4:Which of the following are true?

1. A nerve impulse is transmitted along the nerve axon as an electricalimpulse and across the synapse by diffusion of chemicalneurotransmitters

2. drugs that block synaptic transmission can cause complete paralysis3. the fatty sheath around the axon slows the speed at which nerve

impulses are transmitted4. the peripheral nervous system includes the brain and spinal cord5. a re ex arc bypasses the brain and enables a fast, autonomic

response

1175

1. 1 and 22. 1, 2 and 33. 1, 2 and 54. 2, 4 and 55. 3, 4 and 5

1176

Question 5:Which of the following statements are true regarding the transition

elements?

1. Iron (II) compounds are light green2. Transition elements are neither malleable nor ductile3. Transition metal carbonates may undergo thermal decomposition4. Transition metal hydroxides are soluble in water5. When Cu2+ ions are mixed with sodium hydroxide solution, a blue

precipitate is formed

1177

1. 1 and 22. 1 and 33. 1, 3 and 54. 3 and 55. 5 only

1178

Question 6:What is the value of C when the equation is balanced?

5 PhCH3 + A KMnO4 + 9 H2SO4 = 5 PhCOOH + B K2SO4 + CMnSO4 + 14 H2O

1179

1. 32. 43. 54. 75. 9

1180

Question 7:Tongue-rolling is controlled by the dominant allele T, while non-

rolling is controlled by the recessive allele, t.Red-green colour blindness is controlled by a sex-linked gene on the X

chromosome. Normal colour vision is controlled by dominant allele B,while red-green colour blindness is controlled by the recessive allele, b.

e mother of a family is colour blind and heterozygous for tongue-rolling, while the father has normal colour vision and is a non-roller.

Which of the following statements are correct?

1. More males than females in a population are red-green colour blind2. 50% of children will be non-rollers3. All the male children will be colour-blind

1181

1. 1 and 2 only2. 1, 2 and 33. 2 only4. 2 and 3 only5. 3 only

1182

Question 8:

Make y the subject of the formula:

1183

1184

Question 9:What is the mass in grams of calcium chloride, CaCl2 in 25cm³ of a

solution with a concentration of 0.1 mol l-1? (Ar of Ca is 40 and Ar of Cl is35)

1185

1. 0.28g2. 0.46g3. 0.48g4. 0.72g5. 1.28g

1186

Question 10:

Consider the equations: A: y = 3x and B: y = – 7. At what values of x dothe two equations intersect?

1187

1. x=2 and x=92. x=3 and x=63. x=6 and x=274. x=65. x=18

1188

Question 11:Which of the following statements regarding the circulatory system

are correct?

1. e pulmonary artery carries oxygenated blood from the rightventricle to the lungs

2. e aorta has a high content of elastic tissue and carries oxygenatedblood from the left ventricle around the body

3. e mitral valve is between the pulmonary vein and the left atrium4. e vena cava carries deoxygenated blood from the body to the right

atrium

1189

1. 1 and 32. 1 and 23. 2 only4. 2 and 45. 3 only

1190

Question 12:A compound with a molar mass of 120 gmol-1 contains 12 g of carbon,

2 g of hydrogen and 16 g oxygen. What is the molecular formula of thecompound?

1191

1. CH2O2. C2H4O2

3. C4H2O4. C4H8O4

5. C8H16O8

1192

Question 13:Rupert plays one game of tennis and one game of squash.

e probability that he will win the tennis game is

e probability that he will win the squash game isWhat is the probability that he will win one game only?

1193

1194

Question 14:

What is the median of the following numbers: ; 0. ; ; 0.25; 0.75 ; ;

1195

1. 0.2. 0.75

1196

Question 15:16.4g of nitrobenzene is produced from 13g of benzene in excess nitric

acid: C6H6 + HNO3 -> C6H5NO2 + H2OWhat is the percentage yield of nitrobenzene (C6H5NO2)?

1197

1. 65%2. 67%3. 72%4. 78%5. 80%

1198

Question 16:Which of the following points regarding electromagnetic waves are

correct?

1. Radiowaves have the longest wavelength and the lowest frequency2. Infrared has a shorter wavelength than visible light and is used in

optical bre communication, heater and night vision equipment3. All of the waves from gamma to radio waves travel at the speed of

light (about 300,000,000 m/s)4. Infrared radiation is used to sterilise food and to kill cancer cells5. Darker skins absorb more UV light, so less ultraviolet radiation

reaches the deeper tissues

1199

1. 1 and 22. 1 and 33. 1, 3 and 54. 2 and 35. 2 and 4

1200

Question 17:Two carriages of a train collide and then start moving together in the

same direction. Carriage 1 has mass 12,000 kg and moves at 5ms-1 beforethe collision. Carriage 2 has mass 8,000 kg and is stationary before thecollision. What is the velocity of the two carriages after the collision?

1201

1. 2 ms-1

2. 3 ms-1

3. 4 ms-1

4. 4.5 ms-1

5. 5 ms-1

1202

Question 18:Which of the following statements are true?

1. Control rods are used to absorb electrons in a nuclear reactor tocontrol the chain reaction

2. Nuclear fusion is commonly used as an energy source3. An alpha particle is comprised of two protons and two neutrons and

is the same as a helium nucleus4. When C undergoes beta decay, an electron and N are produced5. Beta particles are less ionising than gamma rays and more ionising

than alpha particles

1203

1. 1 and 22. 1 and 33. 3 and 44. 3, 4 and 55. None of the statements are true

1204

Question 19:

Simplify fully:

1205

1206

Question 20:Which of the following are true?

1. Lightning, as well as nitrogen- xing bacteria, converts nitrogen gasto nitrate compounds

2. Decomposers return nitrogen to the soil as ammonia3. e shells of marine animals contain calcium carbonate, which is

derived from dietary carbon4. Nitrogen is used to make the amino acids found in proteins

1207

1. 1 only2. 1 and 23. 2 and 34. 2, 3 and 45. ey are all true

1208

Question 21:

Write in the form: p

1209

1210

Question 22:Which of the following statements are false?

1. Simple molecules do not conduct electricity because there are nofree electrons and there is no overall charge

2. e carbon and silicon atoms in silica are arranged in a giant latticestructure and it has a very high melting point

3. Ionic compounds do not conduct electricity when dissolved in wateror when melted because the ions are too far apart

4. Alloys are harder than pure metals

1211

1. 1 and 22. 1, 2 and 43. 1, 2, 3 and 44. 2 and 45. 3 only

1212

Question 23:e graph below shows a circle with radius 5 and centre (0,0).

What are the values of x when the line y=3x-5 meets the circle?

1213

1. x=0 or x=32. x=0 or x=3.53. x=1 or x=3.54. x=1.5 or x=-35. x=1.5 or x=-2

1214

Question 24:Which if the following statements regarding heat transfer are correct?

1. Heat energy is transferred from hotter to colder places by conductionbecause particles in liquid and gases move more quickly whenheated

2. Liquid and gas particles in hot areas are less dense than in cold areas3. Radiation does not need particles to travel4. Dull surfaces are good at absorbing and poor at re ecting infrared

radiation, whereas shiny surfaces are poor at absorbing, but good atre ecting infrared radiation

1215

1. 1 and 22. 1, 2 and 43. 2 and 34. 2 and 45. 4 only

1216

Question 25:e following points refer to the halogens:

1. Iodine is a grey solid and can be used to sterilise wounds. It forms apurple vapour when warmed

2. e melting and boiling points increase as you go up the group3. Fluorine is very dangerous and reacts instantly with iron wool,

whereas iodine has to be strongly heated as well as the iron wool fora reaction to occur and the reaction is slow

4. When bromine is added to sodium chloride, the bromine displaceschlorine from sodium chloride

5. e hydrogen atom and chlorine atom in hydrogen chloride arejoined by a covalent bond

Which of the above statements are false?

1217

1. 1, 3 and 52. 1, 2 and 33. 2 and 44. 3 only5. 3, 4 and 5

1218

Question 26:Consider the triangle below where BE=4cm, EC=2cm and AC=9cm.

What is the length of side DE?

1219

1. 4cm2. 5.5cm3. 6cm4. 7.5cm5. 8cm

1220

Question 27:A ball is projected vertically upwards with an initial speed of 40 ms-1.

What is the maximum height reached? (Take gravity to be 10 ms-2 andassume negligible air resistance).

1221

1. 25m2. 45m3. 60m4. 75m5. 80m

1222

END OF SECTIONSection 31) ‘e NHS should not treat obese patients’Explain what this statement means. Argue to the contrary, that we

should not all become vegetarian. To what extent do you agree with thisstatement?

2) ‘We should all become vegetarian’Explain what this statement means. Argue to the contrary, that we

should not all become vegetarian. To what extent do you agree with thisstatement?

3) ‘Certain vaccines should be mandatory’Explain what this statement means. Argue to the contrary, that we

should not all become vegetarian. To what extent do you agree with thisstatement?

4) ‘Compassion is the most important quality of a healthcareprofessional’

Explain what this statement means. Argue to the contrary, that weshould not all become vegetarian. To what extent do you agree with thisstatement?

END OF PAPER

1223

ANSWERS

1224

Answer Key

1225

1226

1227

1228

1229

MOCK PAPERS ANSWER KEY

1230

1231

1232

WORKED ANSWERS

Question 1: AWhilst B, C and D may be true, they are not completely stated, A is

clearly stated and so is the correct answer.Question 2: Ce main argument of the rst paragraph is to propose the point that

it is more society that controls gender behaviour not genetics. A and D donot indicate either as they only allude to the end result of genderbehaviour and so are incorrect. Hormonal effects are not mentioned in the

rst paragraph and so B is incorrect. C would undermine the argumentthat society predominately controls gender, and so is correct.

Question 3: AB, C and D are not stated and so are incorrect. A is directly stated and

so is correct.Question 4: AB and D are contraindicated by the statement and so are incorrect. C

could be true but implies children always like the same thing as theirsame-gendered parent irrelevant of how they are treated as a child, whichis contrary to the statement and so is not correct. A is correct as is theoverall message.

Question 5: CD may help prevent problems with sexual identity but does not

prevent stereotyping and so is incorrect. A is not stated, and B is implied

1233

but not stated and so are incorrect. C is the end message of how to preventgender stereotyping and so is correct.

Question 6: DA, B and C may be true but are not mentioned in the statement and so

are incorrect. e statement implies that children born with differentexternal organs to those that their sex chromosomes would match may

nd it difficult to accept this difference and be uncomfortable.Question 7: De text states that ‘ose who regularly took 30-minute naps were

more than twice as likely to remember simple words such as those of newtoys.’ Which means those who napped were twice as likely to rememberteddy’s name than the 5% who did not, 5% x 2 = 10%, which would be twiceas likely, ruling out A and B. But being ‘more than twice’ the only possibleanswer is D.

Question 8: Ae answer is to work out 10% (the percentage of napping toddlers

more likely to suffer night disturbances) of 75% (the percentage of toddlerswho regularly nap). Hence 10 % of 75% is 7.5%.

Question 9: AB, C and D may be true but there is nothing in the text to support

them. A is suggested, as the passage states ‘non-napping counterparts,who also had higher incidences of memory impairment, behaviouralproblems and learning difficulties’. If the impaired memory were thecause, as opposed to the result, of irregular sleeping then it would offer analternative reason why those who nap less remember less.

Question 10: BA and C are possible implications but not stated and so are incorrect.

It is said that parents cite napping having ‘the bene ts of their childhaving a regular routine’ so hence B is more correct than D as it refers tothe bene t to the toddlers’ rather than the parents.

1234

Question 11: DB, if true would counteract the conclusion, as it would imply that, the

study is skewed. e same is true of C, which if true would implyunreliable results as the toddler sample are all the same age within a year,but not within a few weeks. A, if true, would not provide any additionalsupport to the conclusion and so is incorrect. D if true would provide themost support for the conclusion as it proposes using groups with a higherincidence of napping in comparison to those with a lower incidence.

Question 12: CAlthough it can be argued that A, B, D and E are true they are not the

best answer to demonstrate a aw in Tom’s father’s argument. C is the bestbecause it accounts for other factors determining success for theGeography A-level exam such as aptitude for the subject.

Question 13: DA is never stated and is incorrect. B and C are referred to being ‘many

people’s’ beliefs, and are cited as others’ opinions not an argumentsupported by evidence in the passage, and so are not valid conclusions. Itis implied that the NHS may have to reduce its services in the future, someof which could be fertility treatments hence D is the most correct answer.

Question 14: AC does not severely affect the strength of the argument, as it is only

relevant to the length of the time taken for the effects of the argument tocome into place.

D is incorrect, as people breaking speed limits already would notnegate the argument that speed limits should be removed, but could evenbe seen as supporting it. ese people may count as the ‘dangerous drivers’who would be ultimately weeded out of the population.

B may affect part of the argument’s logic (as it undermines the ideathat dangerous drivers are born to dangerous drivers), but the nalconclusion that dangerous drivers will end up killing only themselves still

1235

stands, and so the ultimate population of only safe drivers may beobtained. e fact that one dead dangerous driver could have produced asafe one does not necessarily challenge the main point of this argument.

A if true would most weaken the argument as it states that fast driveris more likely to harm others and not the driver itself, which would negatethe whole argument.

Question 15: DWhilst is it stated that the Government assesses risk it is not described

as an obligation, hence A is incorrect. e overall conclusion of thestatement is that on balance the Government was justi ed in notspending money on ooding preparation, as it was unlikely to occur, so C,B and E are incorrect and D is correct.

Question 16: AC is incorrect and D is a possible course of action rather than a

conclusion. B and E are possible inferences but not the conclusion of thestatement. e overall conclusion of the statement is that the way thatchildren interact has changed to the solitary act of playing computergames.

Question 17: Be passage does state that in this case the £473 million could have

been put to better use, however, there is no mention that no drug shouldever be stockpiled for a similar possible pandemic. e passage discussesthe lack of evidence behind Tami u and therefore is stating that in asituation where there is a lack of evidence, there may not be justi cationfor stockpiling millions of pounds worth of the drug. Stockpiling in thecase of drugs with high effectiveness is not discussed so we should notassume this is a generic argument against preparation for any pandemicand stockpiling of any drug.

Question 18: B

1236

e passage discusses the fact that unhealthy eating is associated withother aspects of an unhealthy lifestyle so the argument that tackling onlythe unhealthy eating aspect does not logically follow. e other statementsare all possible reasons why the solution given may not be optimal, but arenot directly referred to in the passage.

Question 19: Ais is a tricky question in which A, B, C and D are all true. However,

the question asks for the conclusion of the passage, which is bestrepresented by A.

B is a premise that gives justi cation for why the elderly should takecare of themselves and C provides a justi cation for why they may not.

D is implied in the text but statement A is explicitly stated.E is incorrect as the passage implies that people should spend the

money that they have in old age, not stop saving altogether.Question 20: Be passage states stem cell research is an area where there are

possible high nancial and personal gains, however there is no mention ofthese being the main driving factors in either this area of research orothers.

Although rivalry between groups may be a reason driving publishing,this is not mentioned in the passage.

e image discrepancies were in only one paper but the passageimplies the protocol and replication problems were in both papers.

Question 21: AD actually weakens the argument, and is therefore not a conclusion. C

is simply a fact stated to introduce the argument, and is not a conclusion.B is a reason given in the passage to support the main conclusion. If weaccept B as being true, it helps support the statement in A. E is notdiscussed in the passage. A is the main conclusion of this passage

Question 22: C

1237

e passage describes improved safety features and better brakes incars, and concludes that this means the road limit could be increased to80mph without causing more road fatalities. However, if C is not true, thisconclusion no longer follows on from this reasoning. At no point is itstated that C is true, so C is therefore the assumption in the passage. estatements in B and D are not required to be true for the argument’sconclusion to lead on from its reasoning. A is a statement which isstrengthened by this passage, and is not an assumption from the passage.E is not relevant to the conclusion or mentioned in the passage.

Question 23: CAnswers A and D are both reasons given to explain ngerprints under

the theory of evolution, and contribute towards the notion given in C, thatthey do not offer support to intelligent design. us, A and D are reasonsgiven in the passage, and C is the main conclusion. B is simply a fact statedto introduce the passage, whilst E actually contradicts somethingmentioned in the argument (namely that Intelligent Design is religious-based, and scienti cally discredited). Neither of these options areconclusions.

Question 24: AAnswers C, D and E obviously present ways in which the conclusions

drawn from the study could be wrong, without any mistakes being madeby those carrying out the study, and thus are potential reasons. B is also apotential reason, because those with a low alcohol consumption couldhave many other risk factors for cancer, and end up with a higher overallrisk. If the study does not take account of these, it could produceerroneous conclusions. A cannot be a valid reason because the passagestates that it is ‘proven’ that alcohol increases the risk of cancer. us, wemust accept this as true, so A is not a potential reason.

Question 25: B

1238

e passage states that the average speed including time spent stoodstill at stations was 115mph. us, A is incorrect, as the stopping pointshave already been included in the calculations of journey time. Similarly,the passage states that the train completes its journey at Kings cross, so Dis incorrect. C is not correct because we have been given the total length ofthe journey. Whether it took the most direct route is irrelevant. E iscompletely irrelevant and does not affect the answer. B is an assumption,because we have only been given the scheduled time of departure. If thetrain was delayed in leaving, it would not have left at 3:30, and so wouldhave arrived after 5:30.

Question 26: Ae argument discusses healthcare spending in England and Scotland,

and whether this means the population in Scotland will be healthier. Itsays nothing about whether this system is fair, and does not mention theexpenditure in Wales. us, C and D are incorrect. Similarly, the argumentmakes no reference to whether healthcare spending should be increased,so B is incorrect. E is true but not the main message of the passage. epassage does suggest that the higher healthcare expenditure per person inScotland does not necessarily mean that the Scottish population will behealthier, so A is a conclusion from this passage.

Question 27: DC is an incorrect statement, as the passage says that Polio hasn’t been

eradicated yet. A and B are reasons given to support the conclusion, whichis that given in D. E, meanwhile, is an opinion given in the passage, and isnot relevant to the passage’s conclusion.

Question 28: Ais passage provides various positive points of the Y chromosome,

before describing how all of this means it is a fantastic tool for geneticanalysis. us, the conclusion is clearly that given in A. e statement in Bis a further point given to provide evidence of its utility, as stated in the

1239

passage. us B is not a conclusion in itself, but further evidence tosupport the main conclusion, given in A. C is also a reason given tosupport the conclusion in A, whilst D is simply a fact stated to introducethe passage. As for E, there is no mention of Genghis Khan’s children (onlyhis descendants).

Question 29: AAnswers C and E are not valid assumptions because the argument has

stated that a patient must be treated with antibiotics for a bacterialinfection to clear. B is not a aw, because this does not affect whether theantibiotics would clear the infection if it were bacterial. D is an irrelevantstatement, and also disagrees with a stated phrase in the passage (thatantibiotics are required to clear a bacterial infection). A is a valid aw,because the passage does not say that antibiotics are sufficient orguaranteed to clear a bacterial infection, simply that they are necessary.us, it is possible that the infection is bacterial but the antibiotics failedto clear it.

Question 30: BA, C and D, if accepted as true, all contribute towards supporting the

statement given in B, which is a valid conclusion given in this passage.us, A, C and D are all reasons given to support the main conclusion,which is the statement given in B. E is not a valid conclusion, as thepassage makes no reference to action that should be taken relating tosmoking, it simply discusses its position as the main risk factor for lungcancer.

Question 31: AD is only given as a method, with no mention of its effectiveness. We

do not know if C is true because it is not stated. B is not discussed in thepassage. Whilst statement E is true, it is supporting evidence for theconclusion, not the conclusion itself.

Question 32: C & E

1240

Whilst A and B may be true, cost is not mentioned as a deterringfactor and we are only concerned with use in the UK, so they are irrelevant.Whether cannabis was the only class C drug is not important to theargument so D is not correct. C and E are the correct answers because thestatement concerns the use of cannabis in the UK, directly stating use willdecrease from people knowing it has been upgraded to a more dangerouscategory and from fearing longer prison sentences from higher-classdrugs.

Question 33: BWhilst A and C may be true, they are not part of the argument. D is a

possible, but cannot be logically proposed from the information above. Ewould be a aw if the argument were ‘all levels of sports teams reducebullying’ but the passage explicitly states ‘well-performing’ teams. Hence Bis correct as it undermines the whole argument, reversing the cause andeffect.

Question 34: BOptions A, C and D do not directly weaken the argument as if any 16

year olds were buying/drinking alcohol (whether the minority or majority)– police would still be spending time catching them. e suggested bene tto reduce police time spent catching underage drinkers would be negatedif B were true, hence it is the correct answer.

Question 35: DA is an interpretation of the last sentence and doesn’t accurately

summarise the argument in the passage. B is untrue as there is nomention of if the government can afford to give grants or not. C and E areincorrect as the passage only talks about small businesses. D is correct asit best summarises the change in government policy regarding smallbusinesses.

Question 36: A

1241

e statement discusses a case that was reported, but aims to arguethat there may be important errors occurring everyday in medicine that gounreported. Option A if true, would signi cantly weaken this argument aswould negate it being a possibility. B, C, D and E may be true, but they donot negate the argument – if doctors are trained, accidents like the abovemay still occur. Operations that are successful do not affect those that arenot, nor do unavoidable errors have any relation to avoidable ones. atthe patient may have died without these errors similarly does not meanthat errors, when they do occur, should not be considered errors.

Question 37: Ae main point of the statement is to highlight that although there are

numerous safety precautions in place to protect patients, when theweaknesses in these precautions align big errors can occur. So A is correct.While E, C, B and D may well be true, they are not the overall conclusionof the statement.

Question 38: Bough not the rst to be cited, the original error is cited as being the

incorrect copying of the sidedness of the kidney to be removed, hence B isthe correct option. e other options represent errors that in the ‘Swisscheese model’ would have not been allowed to occur if the original hadnot taken place.

Question 39: DIn this instance the ‘tip of the iceberg’ refers to the number of medical

errors reported, implying there may be a signi cantly larger proportionthat go unreported, hence the correct option is D, and not B.

Question 40: Ae description given about the consultant’s performance versus

emotional arousal, is described as initially increasing then eventuallydecreasing over time, which is best represented by graph A.

Question 41: B

1242

e consultant says that the ‘public perception is that medicalknowledge increases steadily over time’ which is best represented by graphB. e consultant says the regarding the acquisition of medical knowledge,‘many doctors [reach] their peak in the middle of their careers’, which isbest described by the graph D.

Question 42: BObesity is not mentioned in the passage, so E is incorrect. ere is no

mention exercise speci cally as it relates to old age, so A and D are alsowrong. e diseases associated with lack of exercise are not speci callystated to cause early death, only that they are associated with olderpeople, so C is also incorrect. e passage does, however, argue that lack ofexercise is associated with illness, and so exercise would be linked to alack of illness, or good health, so B is correct.

Question 43: Ee preference of women to have their babies at hospital versus home

is not commented upon so B is incorrect. F is never inferred, only thatmidwives are capable of assisting in normal births and assessing whenwomen need to be transferred to be to hospital, so it is wrong. A and D arepossible inferences at certain points but not conclusions of the statement.C is never implied, only that normal home births are no more risky thanthose in hospital. e overall conclusion of the statement is that the homebirths should be encouraged where possible as they are not more risky inthe cases of normal births, and hospital births are an unsustainable cost incash-strapped NHS.

Question 44: BWhile A, C and D would, if true, make the practicalities of increasing

home births more difficult they would not weaken the argument as Bwould. Where the statement’s whole argument rests on home births beingas safe as hospital B, if true, would negate this.

Question 45: D

1243

e statement says ‘With the increase in availability of healthresources we now, too often, use services such as a full medical team for aprocess that women have been completing single-handedly for thousandsof years.’ us implying D, ‘excessive availability of health resources’ is thecause of ‘medicalisation of childbirth’.

Question 46: E1 and 3 identify weaknesses in the argument. If campaigns are what

help keep deaths by re low, they can be seen as ‘necessary’, and theirnecessity may be proven by the promisingly low re-related mortalities. Ifthere are more people with hernias than in res, more people can possiblydie from hernias, but this does not mean the res are less dangerous to the( fewer) individuals involved in them. 2 is irrelevant, as the argument isabout how dangerous res are in their entirety, not in relation to theirconstituent parts. erefore E, ‘1 and 3 only’, is correct.

Question 47: BSince ‘some footballers’ that like Maths are not necessarily the same

‘some’ who like History we can exclude A and D. Equally, while C may ormay not be true, we are not given any information about rugby players’preference for History, so it is incorrect. We know that all basketballplayers like English and Chemistry, and that none of them like History, butas we do not know about a third subject they may like E is incorrect. Weknow all of the rugby players like English and Geography and some ofthem Chemistry, hence there must be a section of rugby players that likeall three subjects so B is correct.

Question 48: De passage discusses the problems surrounding controlling drugs,

and focuses on the rapid manufacture of new ‘legal highs’: it is thereforeimplied that this is the current major problem. e passage also suggeststhat as the authorities cannot keep up with drugs manufacture, the legalityof drugs doesn’t re ect their risks.

1244

1 is incorrect as the passage says health professionals feel legality isless relevant now, but it doesn’t say that it is not still important. 3 isincorrect as the last sentence says a potential problem of legal highs is thatthe risks are not as clear, which contradicts the statement that the publicare not concerned about any risks.

Question 49: Be passage is discussing how banning those with the mentioned

medical conditions from mountain climbing are essential to ensuringsafety. It does not claim that this is sufficient to ensure safety, simply that itis necessary. us C is irrelevant, as risks from other activities do not affectthe risk from mountain climbing. D is also irrelevant, because theargument discusses how it is essential to ensure safety of people onWilderTravel holidays, so those using other companies are irrelevant. A isan irrelevant statement because the passage is discussing what should bedone to ensure safety, not whether this is the morally correct course ofaction. us, a discussion of whether people should choose to accept therisks is not relevant. However, B is a aw, because the guidelines onlymention those with severe allergies, so thinking those with less severeallergies are in danger is a false assumption that has been made by thedirectors.

Question 50: De hospital director’s comments make it abundantly clear that the

most important aspect of the new candidate is good surgery skills,because the hospital’s surgery success record requires improvement. If weaccept his reasoning as being true, then it is clear that the candidate whois most pro cient at surgery should be hired, and patient interactionshould not be the deciding factor. us, Candidate 3 should be hired, assuggested by D.

Question 51: A

1245

Answers B and D are irrelevant to the argument’s conclusion, since theargument only talks about how medical complications could be avoided ifwinter tyres were tted. Whether this is possible (as in B) or whether thereare other options (as in D) are irrelevant to this conclusion. C is not anassumption because the passage states that delays cause manycomplications, which could be avoided with quicker treatment. However,the argument does not state that winter tyres would allow ambulances toreach patients more quickly, so A is an assumption.

Question 52: Be passage discusses how anti-vaccine campaigns cause deaths by

spreading misinformation and reducing vaccination rates. It claims thattherefore in order to protect people, we should block the campaigners fromspreading such misinformation freely. us it is made clear that this actionshould be taken because the campaigners cause deaths, not simply becausethey are spreading misinformation. us, B is the principle embodied inthe passage, and C is incorrect. A actually demonstrates an oppositeprinciple, whilst D is a somewhat irrelevant statement, as the passagemakes no reference to whether we should promote successful publichealth programmes.

Question 53: De passage states that the tumour has established its own blood

supply (it says this was shown during the testing), and that a blood supplyis necessary for the tumour to grow beyond a few centimetres. us A andB are not assumptions. C is not an assumption, as it actually disagreeswith something the passage has implied. e passage has actually saidthat action must be taken, implying that something can be done to stopthe tumour. However, at no point has it been said that a blood supply issufficient for a tumour to grow larger than a few centimetres. If this is nottrue, then the argument’s conclusion that we should expect the tumour togrow larger than a few centimetres, and that action must be taken, no

1246

longer readily follows on from its reasoning. It is possible the tumour willstill fail to grow larger than a few centimetres. us, D is an assumption inthe passage, and a aw in its reasoning.

Question 54: AD is incorrect, as the passage has stated the runners are people

running to raise money for the GNAA. B and C, meanwhile, are incorrectas the passage is only talking about whether the GNAA will be able to get anew helicopter. us, references to whether it wishes to, or whether this isthe best use of money, are irrelevant. A, however, is an assumption on thepart of the passage’s writer. e passage says that the GNAA will be able toget a helicopter if £500,000 is raised, but this does not mean that it won’tbe able to if the £500,000 is not raised by the runners. It could well be thatthey secure funding from elsewhere, or that prices drop. e money beingsufficient to get a new helicopter does not mean it is necessary to get one.

Question 55: CB and D somewhat strengthen this argument, suggesting that more

people going on courses leads to better growth, and that people who havegone on these courses are more attractive to employers. A does not reallyaffect the strength of the argument, as the current rate of growth does notaffect whether government subsidies would lead to increased growth. C,however, weakens the argument signi cantly by suggesting that peoplewould not be more likely to attend the courses if the government were tosubsidise them, as the cost has little effect on the numbers of peopleattending.

Question 56: DB is simply a fact stated in the passage. It does not draw upon any

other reasons given in the passage, so it is not a conclusion. C is not aconclusion because it does not follow on from the passage’s reasoning. epassage discusses what should be done if Pluto is to be classi ed as aplanet, it does not make any mention of whether this should happen. A

1247

and D are both valid conclusions from the passage. However, on closerexamination we can see that if we accept A as being true, it gives us goodreason to believe the statement in D. us, D is the main conclusion in thepassage, whilst A is an intermediate conclusion, which goes on to supportthis main conclusion.

Question 57: CA, B and D would all affect whether the calculation of the Glasgow

train’s arrival time is correct, but none are assumptions because all ofthese things have been stated in the passage. However, the passage has notstated that the trains will travel at the same speed, and if this is not true,then the conclusion that the Glasgow train will arrive at 8:30pm is nolonger valid. us, C is an assumption.

Question 58: AC can actually be seen to be probably untrue, as the passage mentions

a need to escape immune responses, suggesting that the immune systemcan tackle these cells. E is true but not representative of the mainargument made in the passage. B and D are not de�nitely true. e passagementions several essential steps that must occur, but this does not meanthat they are sufficient for carcinogenesis to occur, or guaranteed to allowit. Equally, the passage makes no reference to multiple mechanisms bywhich carcinogenesis can occur. It could be there is only one pattern inwhich these steps can occur. A, however, can be reliably concluded,because the passage does mention several steps that are essential forcarcinogenesis to occur.

Question 59: DAnswers A and C are stated in the passage (the passage states

‘deservedly known’), so these can be reliably concluded. B can also beconcluded, as it is stated that in over 50% of cancers, a loss of functionalP53 is identi ed. D however, cannot be concluded, as the passage simplystates that any cell that has a mutation in P53 is at risk of developing

1248

dangerous mutations. us, it cannot be concluded that a given cell willdevelop such a mutation.

Question 60: DD is not an assumption because Sam’s calculations are based on the

cost per 1000 miles, not on a given amount of fuel being used up. us, hehas not assumed anything about whether the fuel usage is the same foreach car. All of the others are assumptions, which have not beenconsidered. Each of these will affect the total saving he will make if theyare not true. For example, if the Diesel car costs £100 more than the Petrolcar, the total saving will be £1700, not £1800 as calculated.

Question 61: De passage discusses how alcohol is more dangerous than cannabis,

and states that this highlights the gross inconsistencies in UK drugs policy.us, D is the main conclusion of the passage, whilst A is a reason given tosupport this conclusion. e passage simply highlights that the policy isgrossly inconsistent, and does not mention whether it should be changed,or how (whether alcohol should be banned or cannabis allowed).

us, B and C are not valid conclusions from this passage. e factalcohol is freely advertised only mentioned brie y in the passage to addstrength to the argument that alcohol is more accessible than cannabis,but no judgment is made on whether this should not be so, so E is also nota valid conclusion from this passage.

Question 62: Be passage discusses how if rst aid supplies were available, many

accidents could be avoided. B correctly points out that this is a aw – rstaid supplies may help treat accidents and reduce the prevalence of injuriesand deaths, but there is no reason why rst aid supplies should reduce theincidence of accidents. Answers C and D are irrelevant, since the argumentis talking about how rst aid supplies could reduce accidents, not injuriesor deaths. us, discussing cases in which they could not treat the injuries,

1249

or whether they need other components to do so is irrelevant. Equally A isirrelevant, as the argument is simply talking about what could happen if

rst aid supplies were stocked in homes, and makes no reference towhether this is nancially viable.

Question 63: CAnswers A and D are not aws because the passage does not conclude

the things mentioned in these. No mention is made to the safety of thedrug, and the argument only states that it is thought the compound maybe of use in combating cancer. No premature conclusions are drawn, onlysuggestions are made. B is not a aw because we can see that theexperiments may produce misleading results if the wrong solutions areused, suggesting that DNA replication is inhibited even if it is not. C,however, is a valid aw because the argument erroneously concludes thatthe wrong solutions must have been used when it says the experiments donot re�ect what is actually happening. is clearly indicates a convictionthat the wrong solutions were used, which does not follow on from theexperiments being old.

Question 64: Be passage has not said anything about who scored the winning goal,

so A is not an assumption. C is also incorrect, because the passage statesthat South Shields won the game. B correctly identi es that whilst beatingSouth Shields was sufficient to win the league, it was not necessary. IfRotherham wins their other 2 games, they will still win the league, so Bdemonstrates an assumption in the passage. D is not relevant, as it doesnot affect the erroneous nature of the claim that Rotherham will not winthe league having lost the match to South Shields.

Question 65: BC and D actually strengthen or reinforce the CEO’s reasoning, with C

suggesting as time progresses Middlesbrough will have more and morepeople compared to Warrington, whilst D suggests that the market share

1250

in Warrington may not be as high as suggested, adding further reasons tobuild in Middlesbrough. A somewhat weakens the CEO’s argument, but itis not a aw in the reasoning, because the CEO is simply talking about howMiddlesbrough will bring them within the range of more people, so themarket share comment is a counterargument, not a aw in his reasoning.B, however, is a valid aw in this argument. Just because Warrington’spopulation is falling, and Middlesbrough’s is rising, does not necessarilymean that Middlesbrough’s will be higher.

Question 66: D1 and 2 are assumptions. e information given does not necessarily

lead on to the conclusion that these extinction events will continuewithout further conservation efforts. Equally, there is nothing in thepassage that says conservation efforts cannot be stepped up withoutincreased funding. However, 3 is not an assumption, because the passagestates that global warming has caused changed weather patterns, whichhave caused destruction of many habitats, which have led to manyextinction events. us, it is given that global warming has indirectlycaused these extinctions, and so the answer is D.

Question 67: Ee argument is suggesting that in Austria, the rail service’s high

passenger numbers and approval ratings are accounted for by the fact thatroad travel is difficult in much of Austria. It then concludes that the publicsubsidies have no effect. We can see that 1 instantly weakens thisargument by providing evidence to the contrary, (in France, difficult roadtravel is not prevalent and so cannot account for the high passengernumbers/approval ratings the country possesses). 3 also weakens thisconclusion by suggesting multiple factors affect the situation. is makesthe conclusion based on the evidence from Austria less strong. us, theanswer is E. 2 actually strengthens the argument that the public subsidies

1251

do not cause high passenger numbers/approval ratings, as Italy has highsubsidies but low passenger numbers/approval ratings.

Question 68: CA is incorrect, in 2011 24% of men and 26% of women were obese (one

should not confuse this with the rates of combined obese and overweight).B is also incorrect, as what it states is true for adults; however, the guresfor children aged 2-15 have changed little over the past year. D is notstated or implied by the passage.

C is implied in the last two sentences of the article, and so the correctanswer.

Question 69: DWhilst the text suggests that smokers tend to have high-risk life-styles,

the text does not claim that all smokers drink excessively. Some smokersmight, but others might not. is differentiation is important to make.

Question 70: DBe careful of using your own knowledge here! Whilst A and B may be

true, they are not the main message of the passage. C may be true but isnot discussed in the passage. E is speculative, as the passage does not sayif the transplant would be a ‘good alternative’. D is correct as it echoes themain message of the passage.

Question 71: FSmoking and Diabetes are risk factors for vascular disease (not a

cause). Vascular disease does not always lead to infarction. e passagedoes not give sufficient detail about necrotic tissue to conclude C or D.

Question 72: BA is irrelevant to the argument’s conclusion. Meanwhile E does

nothing to alter the conclusion, as the fact that schools receive similarfunds does not affect the fact that more funding could provide betterresources, and thus improve educational attainment. C actually weakensthe argument; by implying that banning the richer from using the state

1252

school system would not raise many funds, as most do not use it anyway. Ddoes not strengthen the conclusion as stating that a gap exists does not doanything to suggest that more funding will help close it. B clearly supportsthe conclusion that more funding, and better resources, would help closethe gap in educational attainment.

Question 73: AD and E are irrelevant to the argument’s conclusion. C is actually

contradicting the argument. B is stated in the passage, so is not anassumption of the passage. A describes an assumption: the increase ofDVDs does not, necessarily, cause the loss of cinema customers.

Question 74: Ce question refers to aeroplanes being the fastest form of transport,

and states that this means that travelling by air will allow John to arrive assoon as possible. C correctly points out that the argument has neglected totake into account other delays induced by travelling by aeroplane. Costand legality are irrelevant to the question, so B and E are incorrect.Meanwhile, D actually reinforces the argument, and A refers to futurepossible developments that will not affect John’s current journey.

Question 75: De argument states that people should not seek to prevent spiders

from entering their homes. It does not say anything about whether peopleshould like spiders being in their home, so A is incorrect. e argumentalso makes no allusion to the notion of people preventing ies fromentering their homes, so B is incorrect. e argument also does notmention or implies that any efforts should be made to encourage spidersto enter homes, or that they should be cultivated, so C and E are alsoincorrect.

Question 76: AA correctly identi es an assumption in the argument. At no point is it

stated that bacterial infections in hospitals are resulting in deaths. B, C, D

1253

and E are all valid points but they do not affect the notion that pressure formore antibiotic research would save lives. erefore, none of thesestatements affect the conclusion of the argument and as such they are notassumptions in this context.

Question 77: Be passage does not state that John disregards arguments because of

the gender of the speaker, so D is incorrect. A and C are also wrong, asJohn states he nds women with armpit hair necessarily unattractive, so adifferent face or the knowledge of concealed hair would not make him ndthe female in question more appealing to his aesthetic. John does not stateKatherine wants other women to stop shaving, so E is incorrect.

B is the correct answer, as Katherine was simply speaking aboutsocietal norms, and at no point is it said she was trying to convince John to

nd her, with armpit hair, attractive.Question 78: DA is irrelevant to the argument, which says nothing about what will

happen to Medicine in the future. e argument is describing how Sunitais incorrect, and how better medicine is not responsible for a high deathrate from infectious disease in third world countries, and how bettermedicine will actually decrease this rate. C is a direct contradiction to thisconclusion, so is incorrect. E is a fact stated in the argument to explainsome of its reasoning, and is not a conclusion, therefore E is incorrect.

Both B and D are valid conclusions from the argument. However, B isnot the main conclusion, because the fact that ‘Better medicine is notresponsible for a high death rate from infectious disease in third worldcountries’ actually supports the statement in D, ‘Better medicine will leadto a decrease in the death rate from infectious disease in third worldcountries’. erefore, B is an example of an intermediate conclusion in thisargument, which contributes to supporting the main conclusion, which isthat given in D.

1254

Question 79: Ae statement in A, that housing prices will be higher if demand for

housing is higher, is not stated in this argument. However, it is implied tobe true, and if it is not true, then the argument’s conclusion is not validfrom the reasoning given. erefore A correctly identi es an assumptionin the argument. e other statements do not affect how the reasons givenin the argument lead to the conclusion of the argument, and are thereforenot assumptions in the argument.

Question 80: BA and E are both contradictory to the argument, which concludes that

because of the new research, Jellicoe motors should hire a candidate withgood team-working skills. C refers to an irrelevant scenario, as theargument is referring to only one candidate being hired, and at no pointdoes it state or imply that several should be hired.

B correctly identi es the conclusion of the argument that Jellicoemotors should hire a new candidate with good team-working skills inorder to boost their productivity and pro ts. D meanwhile exaggerates theconsequences of not following this course of action. e argument doesnot make any reference to the notion that Jellicoe motors will struggle tobe pro table if they do not hire a candidate with good team-working skills.

Question 81: ED is in direct contradiction to the argument, so is not the main

conclusion. Meanwhile, B is a reason stated in the argument to explainsome of the situations described. It is not a conclusion, as it does notfollow on from the reasons given in the argument.

A and E are both valid conclusions from the argument. However, onlyE is the main conclusion. is is because both A goes on to support thestatement in E. If bacterial resistance to current antibiotics could result inthousands of deaths, this supports the notion that the UK government

1255

must provide incentives for pharmaceutical rms to research newantibiotics if it does not wish to risk thousands of deaths.

Meanwhile, C appears to be another intermediate conclusion in theargument that also supports the main conclusion. However, on closeinspection this is not the case. C refers to the UK government directlyinvesting in new antibiotic research, whilst the argument refers to thegovernment providing incentives for pharmaceutical rms to do so.erefore, C is not a valid conclusion from the argument.

Question 82: BE is completely irrelevant because the question is referring to an

unsustainable solution if the UN’s development targets are met, so thelikelihood of them being met is irrelevant. C is irrelevant because they donot affect the fact that the situation would be unsustainable if everybodyused the amount of water used by those in developed countries, as statedin the question. A is also irrelevant, as the passage does not mention priceas a factor to be considered within the argument.

Meanwhile, D would actually strengthen the argument’s conclusion.erefore, the answer is B. B correctly identi es that if those in

developed countries use less water, it may be possible for everyone to usethe same amount as these people and still be in a sustainable situation.

Question 83: Cere is no mention of treatment, so A is incorrect. A need to travel

abroad for the post is not stated either, so B is incorrect. e need for acool head is stated explicitly, but not necessarily that this be a leader, so Dis also wrong. Other qualities are irrelevant to the argument, so E is alsoincorrect. C would only be relevant if there was indeed a link between ‘aspeci c phobia’ and ‘a general tendency to panic’. us, C highlights the

aw: if a fear of ying does not necessitate a general disposition of panic,the argument for not hiring this employee crumbles.

Question 84: C

1256

e passage does not suggest there are no more university places, nordoes it make a distinction between the qualities of different universities, soA is incorrect and D is irrelevant. e argument does not deny the fact thatpeople can be successful without a university education, so B is alsowrong. C is correct, as the passage speci cally states ‘many moregraduates’, but not all, are equipped with better skills and better earningpotential. is suggests not all degrees produce these skill-sets in theirgraduates, and so not all university places will create high-earningemployees.

Question 85: DB is unrelated to the argument, as other contributing factors would

not negate the damaging potential of TV. Watching sport on televisionwould not be akin to actually playing sport, so A is also incorrect. epossibility of eye damage is stated as caused by TV, so C is incorrect.However, if people watch television and partake in sport, which thepassage seems to imply cannot happen, they may not suffer the negativeeffects of obesity and social exclusion. For example, they may play sportduring the day and watch television in the evening, thus experiencing thebene ts of exercise and also enjoying the sedentary activity. erefore,various potential threats supposedly posed by watching excessivetelevision are undermined, and D is correct.

Question 86: CD directly counters the above argument, and so is incorrect. ough A,

B and D are all suggested or stated by the passage, they each act asevidence for the main conclusion, C, describing the ‘multiple reasons tolegalise cannabis’.

Question 87: CC is not an assumption as it has been explicitly stated in the question

that the salary is xed, and therefore it will not change. e rest of thestatements are all assumptions that Mohan has made. At no point has it

1257

been stated that any of the other statements are true, but they are allrequired to be true for Mohan’s reasoning to be correct. erefore, they areall assumptions Mohan has made.

Question 88: Ae answer is not B because, although the Holocaust was a tragedy,

this is not explicitly stated in the passage. It cannot be C or E, as these arealso not directly stated above. D provides an intermediary conclusion thatleads to the main conclusion of A: we should not let terrible things happenagain, and through teaching we can achieve this, so therefore ‘we shouldteach about the Holocaust in schools’.

Question 89: CDVDs are irrelevant – though one could access disturbing material

through a DVD, this does not mean the material to be seen on TV is lessdisturbing. e argument also is not concerned with adults, and thesuggestion is that violence in any quantity may have a detrimental effect,even if a show is not entirely made up of it. A, B and D are thus not thecorrect answers. C contradicts the argument, as it suggests there is no linkbetween witnessing and re-enacting what one has witnessed. Childrenmay watch the scenes of rape and recognise the horror of the action, andso be sworn off ever committing that crime.

Question 90: CA is irrelevant, as the passage states it could teach children, not that it

necessarily would. B and C are also irrelevant, as the entertainmentquality of the show or the likeability of its protagonist would notundermine the logic of the argument. C is the correct answer, as it showshow the question uses one model of success and projects it onto all othermodels, which is illogical: just because Frank succeeds without morality,does not mean all others must reject morality to succeed.

Question 91: A

1258

B, C, D and E are all irrelevant to Freddy’s argument that he cannot saya sexist thing because he is a feminist. e woman’s discomfort, Neil’sfeminist stance, the appropriateness of making comments about men, orlewd comments in general do not affect his claim. e presumed linkbetween the two (inability to say something sexist, and feminist self-description) is the aw in Freddy’s argument: someone may believe inequal rights for the genders, and still say a sexist thing.

Question 92: AAt no point is it stated or implied that car companies should prioritise

pro ts over the environment, so C) is incorrect. Neither is it stated that thepublic do not care about helping the environment, so E) is incorrect.

B) is a reason given in the argument, whilst D) is impossible if weaccept the argument’s reasons as true, so neither of these are conclusions.

Question 93: De easiest thing to do is draw the relative positions. We know

Harrington is north of Westside and Pilbury. We know that Twotown isbetween Pilbury and Westside. Crewville is south of Twotown, Westsideand Harrington but we do not know but its location relative to Pilbury.

Question 94: BBy making a grid and lling in the relevant information the days Dr

James works can be deduced:

1259

➢ No one works Sunday.➢ All work Saturday.➢ Dr Evans works Mondays and Fridays.➢ Dr Luca cannot work Monday or ursday.➢ So, Dr James works Monday.➢ And, Dr Evans and Dr James must work ursday.➢ Dr Evans cannot work 4 days consecutively so he cannot workWednesday.➢ Which means Dr James and Luca must work Wednesday.➢ (mentioned earlier in the question) Dr Evans only works 4 days, socannot work Tuesday.➢ Which means Dr James and Luca work Tuesday.➢ Dr James cannot work 5 days consecutively so cannot work Friday.➢ Which means Dr Luca must work Friday.

Question 95: EWorking algebraically, using the call out rate as C, and rate per mile as

M.So, C + 4m = 11C + 5m = 13Hence; (C + 5m) – (C + 4m) = £13 - £11M = £2Substituting this back into C + 4m = 11C + (4 x 2) = 11Hence, C = £3us a ride of 9 mile will cost £3 + (9 x £2) = £21.

1260

Question 96: EUse the information to create a Venn diagram.We don’t know the exact position of both Trolls and Elves, so A and D

are true. Goblins are mythical but not magical, so C is true. Gnomes areneither so B is true. But E is not true.

Question 97: De best method may be work backwards from 7pm. e packing (15

minutes) of all 100 tiles must have started by 6:45pm, hence the cooling(20 minutes) of the last 50 tiles started by 6:25pm, and the heating (45minutes) by 5:40pm. e rst 50 heating (45 minutes) must have started by4:35pm, and cooling (20 minutes) by 5:20pm. e decoration (50 minutes)of the second 50 can occur anytime during 4:35pm- 5:40pm as this is whenthe rst 50 are heating and cooling in the kiln, and so does not add time.e rst 50 take 50 minutes to decorate and so must be started by 3:45pm.

Question 98: ESpeed = distance/time. Hence for the faster, pain impulse the speed is

1m/ 0.001 seconds. Hence the speed of the pain impulse is 1000 metres persecond. e normal touch impulse is half this speed and so is 500 metresper second.

Question 99: EUsing the months of the year, Melissa could be born in March or May,

Jack in June or July and Alina in April or August. With the information thatMelissa and Jack’s birthdays are 3 months apart the only possiblecombination is March and June. Hence Alina must be born in August,which means it is another 7 months until Melissa’s birthday in March.

Question 100: APC Bryan cannot work with PC Adams because they have already

worked together for 7 days in a row, so C is incorrect. B is incorrectbecause if PC Dirk worked with PC Bryan that would leave PC Adams with

1261

PC Carter who does not want to work with him. PC Carter can work withPC Bryan.

Question 101: CPaying for my next 5 appointments will cost £50 per appointment

before accounting for the 10% reduction, hence the cost counting thededuction is £45 per appointment. So the total for 4 appointments = 5 x£45 = £225 for the hair. en add £15 for the rst manicure and £10 x 2 forthe subsequent manicures using the same bottle of polish bringing anoverall total of £260.

Question 102: DElena is married to Alex or David, but we are told that Bertha is

married to David and so Alex must be married to Elena. Hence David,Bertha, Elena and Alex are the four adults. Bertha and David’s child isGemma. So Charlie and Frankie must be Alex and Elena’s two children.Leaving only options A or D as possibilities. Only Frankie and Gemma aregirls so Charlie must be a boy.

Question 103: CUsing, x (minutes) as the, unknown amount of time, the second

student took to examine, we can plot the time taken with the informationprovided thus:

1st student2nd student 3rd student1st examination: 4x1 2x 12xBreak: 8 minutes1st examination:x1 x 1 xHence the total time taken, 45minutes (14:30-15:15)Is represented by, 4x + 2x + 2x + x+ x+ x+ 1+1+ 8+1+145 minutes = 11x (minutes) + 12 minutes33 minutes = 11x (minutes)Hence, x = 3 minutes, so the amount of time the second student took

the rst time, 2x, is 6 minutes.

1262

Question 104: E & FTo work out the amount of change is the sum £5 - (2 x £1.65), which =

£3.30. Logically we can then work out that the 3 coins in the change thatare the same must be 1p as no other 3 coin combination can yield £1.70when made up with 5 more coins. us we know that 3 of the coins are 1p,1p & 1p. We can then deduce that there must also have been 2p and 5pcoins in the change as £1.70 is divisible by ten. e only way then to makeup the remaining £1.60 in 3 different coins is to have £1, 50p and 10p,Hence the change in coins is 1p, 1p, 1p, 2p, 5p, 10p, 50p and £1. So the twocoins not given in change are £2 and 20p.

Question 105: DIf we express the speed of each train as W ms-1. en the relative speed

of the two trains is 2W ms-1.Using Speed=distance/time: 2W = (140 + 140)/ 14.us, 2W = 20, and W = 10. us, the speed of each train is 10 ms-1.To convert from metres to kilometres, divide by 1,000. To convert from

seconds to hours, divide by 3,600.erefore, the conversion factor is to divide by 1,000/3,600 = 10/36 =

5/18us, to convert from ms-1 to kmph, multiply by 18/5. erefore, the

nal speed of the train is 18/5 x 10 = 36km/hr.Question 106: CTaking the day to be 24 hours long, this means the rst tap lls 1/6 of

the pool in an hour, the second 1/48, the third and the fourth .Taking 288 as the lowest common denominator, this gives: + + + which

= full in one hour. Hence the pool will be full in 4 hours.e pool lls by approximately every 15 minutes.us, in 4 Hours 15:us, in 4 Hours 30:us, in 4 Hours 45:

1263

Question 107: BEvery day up until day 28 the ant gains a net distance of 1cm, so at the

end of day 27 the ant is at 27cm height and therefore only 1cm below thetop. On day 28 the 3cm the ant climbs in the day is enough to take it to thetop of the ditch and so it is able to climb out.

Question 108: A

To solve this question three different sums are needed to usethe information given to deduce the costs of the various items.

With the information that 30 oranges cost £12, £12/30 = 40pper orange with the 20% discount, hence oranges must cost

50p at full price. With the information that 5 sausages and 10oranges cost £8.50, we know that the oranges at a 10%

discount account for 10 x 45p = £4.50 so 5 undiscountedsausages cost £4 so each full price sausage is £4/5 = 80p. Finally

we know that 10 sausages and 10 apples cost £9, at 10%discount the sausages cost 72p each thus accounting for 10 x72p = £7.20 of the £9, hence the 10 apples at a 10% discountmust cost £1.80, so each apple costs 18p at 10% discount. So

an apple is 20p full price. Now to add up the �nal total: 2oranges + 13 sausages + 2 apples = (2 x 50p) + (13 x 72p) + (12 x

18p) = £12.52.

Question 109: CIf we take the number of haircuts per year to be x, the information we

have can be shown:

1264

As we know that changing to either membership option would costthe same for the year, we can express the cost for the year, y as;

VIP: y = £125 + 50xExecutive VIP: y = £200 + 45xerefore: £125 + 50x = £200 + 45xSimpli ed 5x = £75, therefore the number of haircuts a year, x is 15.Substituting in x, we can therefore work out:

Hence the amount saved by buying membership is £25.Question 110: BAll thieves are criminals. So the circle must be fully inside the square,

we are told judges cannot be criminals so the star must be completelyseparate from the other two.

Question 111: CWe are told that March and May have the same last number, which

must be either 3 or 13. Taking the information from the question that oneof the factors is related to the letters of the month names, we can interpretthat 13 represents the M which starts both March and May. erefore weknow the rule is that the last number is the position of the starting letter.Knowing that there is another factor about the letters of the month thatcontrols the code we can work out that one of the number may code forthe number of letters. Which in March would be 5, which is the second

1265

letter, so we have the rule of the 2nd number. Finally through observationwe may note that the rst number codes for the months’ relative positionin the year. Hence the code of April will be 4, ( for its position), 5 ( for thenumber of letters in the name) and 1 for the position of the starting letter‘A’) and so 451 is the code.

Question 112: DIf b is the number of years older than 5, and a the number of A*s, the

money given to the children can be expressed:£5 + £3b + £10aHence for Josie £5 + (£3 x 11) + (£10 x 9) = £128We know that Carson receives £44 less yearly, and his b value is 13, so

his amount can be expressed:£5 + (£3 x 13) + (£10a) = £84Simpli ed: £44 + £10a = £84I.e. £10a = £40,So Carson’s ‘a’ value, i.e. his number of A*s is 4, so the difference

between Josie and Carson is 5.Question 113: B & CUsing the information to make a diagram:

Hence A is incorrect. D and E may be true but we do not have enoughinformation to say for sure. B is correct as we know peaches are moreexpensive than oranges but not about their price relative to pears. Equallywe know C to be true as grapes are more expensive than apples so theymust be more expensive than pears.

Question 114: B

1266

It’s easy to assume all the cuts should be in the vertical plane as a cakeis usually sliced, however there is a way to achieve this with fewer cuts.Only three cutting motions are needed. Start by cutting in thehorizontal plane through the centre of the cake to divide the top halffrom the bottom half. en slice in the vertical plane into quarters to give8 equally sized pieces with just three cuts.

Question 115: B & DAfter the changes have been made, at 12 PM (GMT +1):

Russell thinks it is 11 AM

Tom thinks it is 12 PM

Mark thinks it is 1 PM

us, in current GMT+1 time zone, Mark will arrive an hour early at 11AM, Russell an hour late at 1 PM and Tom on time at 12 PM. ere istherefore a two hour difference between the rst and last arrival. Foroptions E and F, be careful: the time zone listed is NOT GMT +1 thateveryone else is working in. 1PM in GMT +3 = 11am GMT +1 (the summertime zone just entered) so that is Mark’s actual arrival time; 12pm GMT +0is the old time zone that Russell didn’t change out of so that is Russell’scorrect arrival time.

Question 116: DUsing Bella’s statements, as she must contradicted herself with her two

statements, as one of them must be true, we know that it was de nitelyeither Charlotte or Edward. Looking to the other statements, e.g. Darcy’swe know that it was either Charlotte or Bella, as only one of the twostatements saying it was both of them can have been a lie. Hence it musthave been Charlotte.

Question 117: F

1267

e only way to measure 0.1 litres or 100ml, is to ll the 300ml beaker,pour into the half litre/ 500 ml beaker, ll the 300ml again and pour(200ml) into the 500ml, which will make it full, leaving 100ml left in the300ml beaker. e process requires 600ml of solution to ll the 300mlbeaker twice.

Question 118: DIf you know how many houses there are on the street it is possible to

work out the average, which then you can round up and down and to ndthe sequence of number, e.g. if you know there are 6 houses in the street870/ 6 = 145. Which is not a house number because they are even so goingup and down one even number consequentially one discovers that thenumbers are 146, 144, 148, 150, 142 and 140. But it is not possible todetermine Francis’ house number without knowing its relative position i.e.highest, 3rd highest, lowest etc.

Question 119: DExpressed through time:

Question 120: B & DBlood loss can be described as 0.2 L/min.For the man:8 litres – 40% (3.2 L) = 4.8 L When he collapses, taking 16 minutes (3.2

/ 0.2 = 16)For the woman:

1268

7 litres – 40% (2.8L) = 4.2: when she collapses, taking 14 minutes (2.8 /0.2 = 14)

Hence the woman collapses 2 minutes before the man so B is correct,and A is incorrect. e total blood loss is 3.2L + 2.8L which = 6L so C isincorrect. e man’s blood loss is 3.2L when he collapses so E is incorrect.e woman has a remaining blood volume of 4.2L when she collapses so Dis correct. Blood loss is 0.2 L/min, which equates to 5 minutes per litre,which is 10 minutes per 2 litres not 12 L, so F is incorrect.

Question 121: BWork out the times taken by each girl – (distance/pace) x 60 (converts

to minutes) + lag time to startJenny: (13/8) x 60 = 97.5 minutesHelen: (13/10) x 60 + 15 = 93 minutesRachel (13/11) x 60 + 25 = 95.9 minutesQuestion 122: CWork through each statement and the true gures.

1. Overlap of pain and u-like symptoms must be at least 4% (56+48-100). 4% of 150: 0.04 x 150=6

2. 30% high blood pressure and 20% diabetes, so max percentage withboth must be 20%. 20% of 150: 0.2*150 = 30

3. Total number of patients – patients with u-like symptoms –patients with high blood pressure. Assume different populations toget max number without either. 150 – (0.56 x 150) – (0.3 x 150) = 21

4. is is an obvious trap that you might fall into if you added up thepercentages and noted that the total was >100%. However, this isn’t aproblem as patients can discussed two problems.

Question 123: Cis is easiest to work out if you give all products an original price, I

have used £100. You can then work out the higher price, and the

1269

subsequent sale price, and thus the discount from the original £100 price.As the price increases and decreases are in percentages, they will be thesame for all items regardless of the price so it does not matter what theinitial gure you start with is.

Marked up price: 100 x 1.15 = £115Sale price: 115 x 0.75 = £86.25Percentage reduction from initial price is 100 – 86.25 = 13.75%Question 124: De recipe states 2 eggs makes 12 pancakes, therefore each egg makes

6 pancakes, so the number Steve must make should be a multiple of 6 toensure he uses a whole egg.

Steve requires a minimum of 15 x 3 = 45 pancakes. To ensure use ofwhole eggs, this should be increased to 48 pancakes.

e original recipe is for 12 pancakes, therefore to make 48 pancakes,require 4x recipe (48/12).

erefore quantities: 8 eggs, 400g plain our and 1200 ml milk.Question 125: BWork through the question backwards.In 6 litres of diluted bleach, there are 4.8 litres of water and 1.2 litres of

partially diluted bleach.In the 1.2 litres of partially diluted bleach, there is 9 parts water to one

part original warehouse bleach. Remember that a ratio of 1:9 means 1/10bleach and 9/10 water. erefore working through, there is 120ml ofwarehouse bleach needed.

Question 126: CWe know that Charles is born in 2002, therefore in 2010 he must be 8.

ere are 3 years between Charles and Adam, and Charles is the middlegrandchild. As Bertie is older than Adam, Adam must be younger thanCharles so Adam must be 5 in 2010. In 2010, if Adam is 5, Bertie must be 10(states he is double the age of Adam).

1270

e question asks for ages in 2015: Adam = 10, Bertie = 15, Charles = 13Question 127: BMake the statements into algebraic equations and then solve them as

you would simultaneous equations. Let a denote the at xed rate for hire,and b the price per half hour.

Cost = a + b(time in mins/30)Peter: a + 6b (6 half hours) = 14.50 (equation 1)Kevin: 2a + 18b = 41, or this can be simpli ed to give cost per kayak, a +

9b = 20.5 (equation 2)If you subtract equation 1 from equation 2:3b = 6, therefore b = 2Substitute b into either equation to calculate a, using equation 1, a +

12 = 14.50, therefore a = 2.50Finally use these values to work out the cost for 2 hours:2.50 ( at fee) + 4 x 2 (4half hours x cost/half hour) = £10.50Question 128: EIt is most helpful to write out all the numbers from 0 – 9 in digital

format to most easily see which light elements are used for each number.You can then cross out any numbers which don’t use all the lights from thedigit 7.

Go through the digits methodically and you can cross out: 1, 2, 4, 5,and 6. ese numbers don’t contain all three bars from the digit 7.

Question 129: BIn this question it is worth remembering it will take more people a

shorter amount of time.

1271

Work out how many man hours it takes to build the house. Days xhours x builders

12 x 7 x 4 = 336 hoursWork out how many hours it will take the 7man workforce: 336/7 = 48

hoursConvert to 8 hour days: 48/8 = 6 daysQuestion 130: D

By far the easiest way to do these type of questions is to draw a Venndiagram (use question marks if you are unsure about the exact position):

Now, it is a case of going through each statement:

1. Incorrect - Acacia may be fabacaea. Acacia are not astragalus, butdoes not logically follow that they therefore can’t be fabacaea.

2. Incorrect – astragalus and gummifer are not necessarily separatewithin fabacaea.

3. Incorrect – the statement is not reversible so the fact that allastragalus and gummifer are fabacaea does not mean all facacaea aregummifer and/or astragalus. E.g. Fabacaea could be acacia.

4. Correct

1272

5. Incorrect – Whilst some acacia could be gummifer, there is nocertainty that they are.

Question 131: DArea of a trapezium = (a+b)/2 x hArea of cushion = (50+30)/2 x 50 = 2000cm2

Since each width of fabric is 1m wide, both sides of one cushion can tinto one width. e required length is therefore 75cm x 4 = 3m with a costof 3 x £10 = £30.

Cost of seamstress = £25 x 4 = £100Total cost is £130Question 132: Cere are 30 days in September, so Lisa will buy 30 coffees.In Milk, every 10th coffee is free, so Lisa will pay for 27 coffees at 2.40 =

£64.80In Beans, Lisa gets 20 points each day and needs 220 points to get a

free coffee, which is 11 days, with 5 points left over. erefore, in 30 daysshe will get 2 free coffees. e cost for 28 coffees at 2.15 is £60.20

Beans is cheaper, and the difference is £64.80 - £60.20 = £4.60.Question 133: CWork backwards and take note of how often each bus comes.Must get off 220 bus at 10.57 latest. Can get 10.40 bus therefore (arrive

at 10.54).Latest can get on 283 bus is 10.15 as to make the 220 bus connection.

283 comes every 10mins (question doesn’t state at what points past thehour), so Paula should be at the bus stop at 10.06 to ensure a bus comes by10.15 at the latest. If the bus comes every 10mins, even if a bus comes at10.05 which Paula will miss, the next bus will come at 10.15 and thereforeshe will still be on time.

erefore Paula must leave at 10.01Question 134: B

1273

You are working out the time taken to reach the same distance (D).Make sure to take into account changing speeds of train A, and that train Bleaves 20 minutes earlier.

Make sure you keep the answers consistent in the time units you areusing, the worked answer is all in minutes (hence the need to multiply by60).

Train A: time for rst

So the distance where it equals B isYou need to use D-20 to account for the fact you have already

calculated the time at the slower speed for the rst 20km

Train B:Make the equations equal each other as they describe the same time

and distance, and solve.

Simpli es to soTrain B will take 60 minutes to travel 90 km and train A will take 40

minutes (but as it leaves 20 minutes later, this will be point at which it

1274

passes).Question 135: CWork out the annual cost of local gym: 12 x 15 = £180Upfront cost + class costs of university gym must therefore be >£180.Subtract upfront cost to nd number of classes: 180 – 35 = £145Divide by cost per class (£3) to nd number of classes: 145/3 = 48 1/348 1/3 classes would make the two gyms the same price, so for the

local gym to be cheaper, you would need to attend 49 classes.Question 136: CA is de nitely true, since the question states that all herbal drugs are

not medicines. B is also de nitely true as all antibiotics are medicineswhich are all drugs. C is de nitely false, because all antibiotics aremedicine, yet no herbal drugs are medicines. D is true as all antibiotics aremedicines.

Question 137: CAnswer A cannot be reliably concluded, because from the information

given a non-“Fast” train could stop at Newark, but not at Northallerton orDurham. We have no information on whether all trains stopping atNewark also stop at Northallerton.

Answer B is not correct because 8 is the average number of trains thatstop at Northallerton. It is possible that on some days more than 16 trainsrun, and more than 8 will thus stop at Northallerton. Answer D is incorrectbecause it is mentioned that all trains stopping at Northallerton also stopat Durham, giving a total 6 stops as a minimum for a train stopping atNorthallerton (the others being the 4 stops which all trains stop at).

Answer E is incorrect for a similar reason to A. We have noinformation on whether all trains stopping at Newark also stop atNorthallerton, so cannot determine that they must also stop at Durham.

Answer C is correct because “Fast” trains make less than 5 stops. Sinceall trains already stop at 4 stops (Peterborough, York, Darlington and

1275

Newcastle), they cannot then stop at Durham, as this would give 5 stops.Question 138: DFrom the information we are given, we can compose the following

image of how these towns are located (not to scale, but shows thedirection of each town with respect to the others):

From this “map”, we can see that all statements apart from D are true.Statement D is de nitely not true, since Blueville is south west of Haston itcannot be East of Haston.

Question 139: CWe are told that in order to form a government, a party (or coalition)

must have over 50% of seats. us, they must have at least 50% of the totalseats plus 1, which is 301 seats.

We are told that we are looking for the minimum number of seats thegreens can have in order to form a coalition with red and orange. us, weare seeking for Red and Orange to have the maximum number of seatspossible, under the criteria given.

us we can calculate as follows:- No party has over 45% of seats, so the maximum that the Red party canhave is 45%, which is 270 seats.- No party except for red and blue has won more than 4% of seats. We aretold that the green party won the 4th highest number of seats, so it is

1276

possible that the Orange party won the 3rd highest.- us, the maximum number of seats the orange party can have won is 4%of the total, which is 24 seats.- us, the maximum possible combined total of the Red and Orangeparty’s seats won is 294.

us, in order to achieve a total of 301 seats in a Red-Orange-Greencoalition, the Green party have to have won at least 7 seats.

Question 140: EExpressing the amount each child receives:

Question 141: De total amount of money received;£100, = M + M + D + M + 2D + M + 3D + M + 4DSimpli ed, thus is:£100 = 5M + 10DQuestion 142: Ce two youngest are expressed as M and M + D. Simpli ed as 2M + D.e three oldest are expressed as M + 2D, M + 3D and M + 4D,

Simpli ed as 3M + 9DHence 7 times the two youngest together is expressed 7(2M + D), so

altogether the Answer is 7(2M + D) = 3M + 9D.Question 143: ATo work this out, simplify the two equations:7(2M + D) = 3M + 9D14M + 7D = 3M + 9D11M = 2D

1277

M =Question 144: A

Substitute M into the equation

Question 145: C & Ee easiest way to work this out is using a table. With the information

we know:

Ellen made carrot cake and it was not last. It now cannot be 1st or 3rdas these places are taken so it must be second:

Aleena’s was better than the tiramisu, so she can’t have come last,therefore Aleena must have placed rst

1278

And the girl who made the Victoria sponge was better than Veronica:

Question 146: De information given can be expressed to show the results that the

teams must have had to make their points total.

e results so far total 3 wins, 6 losses and 7 draws. Since, the numberof draws must be even, there must have been another draw. So we knowone of the Eire Lions results is a draw.

e difference between wins (3) and losses (6) is 3. us, there must beanother 3 wins to account for this difference. So the Eire Lions results

1279

must be 3 wins and 1 draw. us, they scored 3 x 3 + 1 = 10.Question 147: DRemember to consider the gender of each person. en draw a quick

diagram to show the given information you can see that only D is correct.Question 148: BAfter the rst round; he knocks off 8 bottles to leave 8 left on the shelf.

He then puts back 4 bottles. ere are therefore 12 left on the shelf. Afterthe second round, he has hit 3 bottles and damages 6 bottles in total, andan additional 2 at the end. He then puts up 2 new bottles to leave 12 – 8 + 2= 6 bottles left on the shelf. After the nal round, John knocks off 3 bottlesfrom the shelf to leave 3 bottles standing.

Question 149: DBased on the information we have we can plot the travel times below.

Change over times are in a smaller font.

Hence on the St Mark’s line, St Mark’s to Archite takes 4 x 2.5 minutes =10 minutes.

1280

Question 150: AGoing from stop to stop on the Straightly line end Buft to Straightly

would take 14 minutes, but we are told earlier on there is an express trainthat goes end to end and only takes 6.

Question 151: Be quickest route from Baxton to Pop Lane is via Marven and

Cambridge Square, which takes 5 + 2 + 2.5 + 0.5 + 2 = 12 minutes. Baxton toPop Lane via Barts would take 4 + 1 + 6 + 3 + 2 = 16 minutes, which islonger so E is incorrect. Other options include times failing to takeaccount of, or incorrectly adding changeover times, and so are incorrect.

Question 152: CFrom Cambridge Square:

➢ Catensey is (2.5 x 3 =) 7.5 minutes away.➢ Woopole, is ( 4 + 3 + 1 +2 + 2 =) 12 minutes.➢ Buft is (1 + 1 + 2 + 2 =) 6 minutes.➢ Westland is (4 + 2 + 2.5 + 2.5 =) 11 minutes.

Question 153: BWith the new delay information we can plot the travel times as before,

adjusted for the delays. Plus a 5 minute delay on the platforms whenwaiting on any platform for a train.

1281

e quickest way from Westland to Marven now uses the non-delayedreliable rectangle line. Four stops on the rectangle line take 6 mins each so24 minutes in total on the train. Add to this the additional 5 minutesplatform waiting time to give a total journey time of 29 minutes.

Question 154: C

Baxton to Archite via Barts using only the Rectangle line takes (5 + 6+6+ 6 +6=) 29 minutes.Baxton to Woopole on the Rectangle line, then Oval to Archite viaKeyton takes (5 + 6 + 1 + 5 + 8 + 8 + 8 =) 41 minutesBaxton to Archite on the Oval line only takes (5 + (8 x 4) = ) 37minutesBaxton to Woopole on the Oval line, then Rectangle to Archite viaBarts takes (5 + 8 + 1 + 5 + 6 + 6 + 6 =) 37 minutesAs the bus takes 27-31 minutes, it is not possible to tell from betweenthe options which will be slower/quicker so option C is the rightanswer.

1282

Question 155: D

Remember the 5-minute platform wait. We are not told that the StMark’s express train from end to end is no longer running so we mustassume that it is, which takes 5 minutes (plus the wait at St Mark’s to go toCatensey).

en, there is a 5 minute wait at Catensey to Archite, and a 2 + 5minute changeover at Archite onto the Rectangle line which then takes 6minutes to West Quays. 5 + 5 + 5 + 5 + 2 + 5 + 6 = 33 minutes. Via Lime Stthe journey takes 5 + 5 + 5+ 2 + 5 + 6+ 6 = 29 minutes.

Question 156: DFrom the information:

“Simon’s horse wore number 1.”“...the horse that wore 3, which was wearing red...”“the horse wearing blue wore number 4.”

We can plot the information below:

1283

In addition: “e horse wearing green; Celia’s, came second”Which means Celia’s horse must have worn number two because it

cannot have worn number 1 because that is Simon’s horse. Also it cannothave worn number three or four because they wore red and bluerespectively. So we can plot this further deduction:

We also know that

“Arthur’s horse beat Simon’s horse”“Celia’s horse beat the horse that wore number 1.” i.e. Simon’s

We know Celia’s horse came second, and that both Celia’s and Arthur’shorses beat Simon’s. is means that Simon’s horse must have come last.So;

And knowing that:

1284

“Only one horse wore the same number as the position it nished in.”

e horses wearing numbers 3 and 4 must have placed 1st and 3rd

respectively. Hence:

\“Lila’s horse wasn’t painted yellow nor blue”So Lila’s must have been red, and Simon’s yellow. Leaving the only

option for Arthur’s to be blue. So we now know:

Question 157: CYear 1 – 40 x 1.2 = 48Year 2 – 48 x 1.2= 57.6Year 3 – 57.6 x 1.1= 63.36Year 4 – 63.36 x 1.1 = 69.696.Question 158: CTo minimise the total cost to the company, they want the wage bills for

each site to be less than £200,000. Working this out involves some trial anderror; you can speed this up by splitting employees who earn similaramounts between the sites e.g. Nicola and John as they are the top twoearners.

1285

Nicola + Daniel + Luke = £ 198,500 and John + Emma + Victoria = £199,150

Question 159: CRemember that pick up and drop off stops may be the same stop,

therefore the minimum number of stops the bus had to make was 7. iswould take 7 x 1.5 = 10.5 minutes.

erefore the total journey time = 24 + 10.5 = 34.5 minutes.Question 160: Ae best method here is to work backwards. We know the potatoes

have to be served immediately, so they should be nished roasting at 4pm,so they should start roasting 50 minutes prior to that, at 3:10. We alsoknow they have to be roasted immediately after boiling, so they should beprepared by 3:05, in order to boil in time. She should therefore startpreparing them no later than 2:47, though she could prepare them earlier.

e chicken needs to be cooked by 3:55 to give it time to stand, so itshould begin roasting at 2:40, and Sally should begin to prepare it no laterthan 2:25.

You can construct a rough timeline:

We can see from this timeline that from 2:40 onwards, there will be nolong enough period of time in which there is a free space in the cooker forthe vegetables to be boiled. ey therefore must be nished cooking at3:05. e latest time prior to this that Sally has time to prepare them (5minutes) is at 2:40, between preparing the chicken and the potatoes. She

1286

should therefore begin preparing the vegetables at 2:42, then begin boilingat 2:47, so they can be nished cooking by 2:55, in time for the potatoes toboil at 3:05.

Chicken: 2:25Potatoes: 2:47Vegetables: 2:42Question 161: Ce quickest way to do this is via trial and error. However, for the sake

of completion: let each child’s age be denoted by the letter of their name,and form an equation for their total age:

e age of each child can be written in terms of Paul’s age.

P = 2J, therefore

Now substitute in to get in terms of P only:

1287

us:

Simplify to give:

thus,Substitute P = 24 into the equations for the other children to get: J = 12,

A = 18, R = 26Question 162: Ae total number of buttons is 71 + 86 + 83 = 240. e total number of

suitable buttons is 22 + 8 = 30. us, she will have to remove a maximum of210 buttons in order to guarantee picking a suitable button on the nextattempt.

Question 163: Eis question requires you to calculate the adjusted score for Ben for

each segment. If Ben has a 50% chance of hitting the segment he is aimingfor, we can assume he hits each adjacent segment 25% of the time. us:

E.g. if he aims at segment 1: He will score

1288

Now it is a simple case of trying the given options to see whichsegment gives the highest score. In this case, it is segment 19:

Question 164: Ce total cost is £8.75, and Victoria uses a £5.00 note, leaving a total

cost of £3.65 to be paid using change.Up to 20p can be paid using 1p and 2p pieces, so she could use 20 1p

coins to make up this amount.Up to 50p can be paid using 5p and 10p pieces, so she could use 10 5p

pieces to make up this amount. is gives a total of 30 coins, and a totalpayment of £0.70.

Up to £1.00 can be paid using 20p pieces and 50p pieces. us, shecould use up to 5 20p pieces, giving a total of 35 coins used, and a totalpayment of £1.70.

e smallest denomination of coin that can now be used is a £1.00coin. Hence Victoria must use 2 £1.00 coins, giving a total of 37 coins, anda total payment of £3.70. However, we know that the total cost to pay inchange was £3.65, and that Victoria paid the exact amount, receiving nochange. us, we must take away coins to the value of 5p, removing thesmallest number of coins possible. is is achieved by taking away 1 5ppiece, giving a grand total of 36 coins.

Question 165: Be time could be 21:25, if rst 2 digits were reversed by the glass of

water (21 would be reversed to give 15). A cannot be the answer, becausethis would involve altering the last 2 digits, and we can see that 25 on adigital clock, when reversed simply gives 25 (the 2 on the left becomes a 5on the right, and the 5 on the right becomes a 2 on the left). C cannot bethe answer, as this involves reversing the middle 2 digits. As with the righttwo digits, the middle 2 digits of 2:5 would simply reverse to give itself, 2:5.

1289

D could be the time if the 2nd and 4th digits were reversed, as they wouldboth become 2’s. However, the question says that 2 adjacent digits arereversed, meaning that the 2nd and 4th digits cannot be reversed asrequired here. E is not possible as it would require all four numbers to bereversed.

us, the answer is B.Question 166: BWe can see from the question that Lorkdon is a democracy and

therefore cannot have been invaded by a democracy because of the treaty(we are assuming this treaty is upheld, as said in the question). us,Nordic (which has invaded Lorkdon) must be a dictatorship. Now, we cansee that Worsid has been invaded by a dictatorship, and has invaded adictatorship. e question states that no dictatorship has undergone bothof these events. us, we know that Worsid cannot be a dictatorship. Wealso know from the question that each of these countries is either adictatorship or a democracy. us, Worsid must be a democracy.

Question 167: Ce total price of all of these items would usually be £17. However,

with the DVD offer, the customer saves £1, giving a total cost of £16. us,the customer will need to receive £34 in change.

Question 168: BTo answer this, we simply calculate how much total room in the pan

will be taken up by the food for each guest:- 2 rashers of bacon, giving a total of 14% of the available space.- 4 sausages, taking up a total of 12% of the available space.- 1 egg takes up 12% of the available space.

Adding these gures together, we see that each guest’s food takes up atotal of 38% of the available space.

us, Ryan can only cook for 2 guests at once, since 38% multiplied by3 is 114%, and we cannot use up more than 100% of the available space in

1290

the pan.Question 169: CTo calculate this, let the total number of employees be termed “Y”.We can see that £60 is the total cost for providing cakes for 40% of “Y”.We know that £2 is required for each cake. us, we can work out that

30 must be 40% of Y.0.4Y = 60/20.4Y = 30Y = 75us, we can calculate that the total number of employees must be 75.Question 170: Ee normal waiting time for treatment is 3 weeks. However, the higher

demand in Bob’s local district mean this waiting time is extended by 50%,giving a total of 4.5 weeks.

en, we must consider the delay induced because Bob is a lower riskcase, which extends the waiting time by another 20%. 20% of 4.5 is 0.9, sothere is a delay of another 0.9 weeks for treatment.

us, Bob can expect to wait 5.4 weeks for specialist treatment on histumour.

Question 171: DIn the class of 30, 40% drink alcohol at least once a month, which is 12.

Of these, 75% drink alcohol once a week, which is 9. Of these, 1 in 3 smokemarijuana, which is 3.

In the class of 30, 60% drink alcohol less than once a month, which is18. Of these, 1 in 3 smoke marijuana, which is 6.

erefore the total number of students who smoke marijuana is 3+6,which is 9.

Question 172: Ce sequence can either be thought of as doubling the previous

number then adding 2, or adding 1 then doubling. Double 46 is 92, plus 2 is

1291

94.Question 173: BIf the mode of 5 numbers of 3, it must feature at least two threes. If the

median is 8, we know that the 3rd largest number is an 8. Hence we knowthat the 3 smallest numbers are 3, 3, and 8. Because the mean is 7, weknow that the 5 numbers must add up to 35. e three smallest numbersadd up to 14. Hence the two largest must add up to 21.

Question 174: Ee biggest difference in the weight of potatoes will be if the bag with

only 5 potatoes in weighs the maximum, 1100g, and the bag with 10potatoes weighs the minimum, 900g. If there are 5 equally heavy potatoesin a bag weighing 1100g, each weighs 220g. If there are 10 equally heavypotatoes in a 900g bag, each weighs 90g. e difference between these is130g.

Question 175: Dere are 60 teams, and 4 teams in each group, so there are 15 groups.

In each group, if each team plays each other once, there will be 6 matchesin each group, making a total of 90 matches in the group stage. ere arethen 16 teams in the knockout stages, so 8 matches in the rst roundknockout, then 4, then 2, then 1 nal match when only two teams are left.Hence there are 105 matches altogether (90 + 8 + 4 + 2 + 1 = 105).

Question 176: AWe know the husband’s PIN number must be divisible by 8 because it

has been multiplied by 2 3 times and had a multiple of 8 added to it. elargest 4 digit number which is divisible by 8 is 9992. Minus 200 is 9792.Divide by 2 is 4896. Hence the largest the husband’s last 4 card digits canbe is 4896. Minus 200 is 4696. Divide by 2 is 2348. Hence the largest my last4 card digits can be is 2348. Minus 200 is 2148. Divide by 2 is 1074. Hencethe largest my PIN number can be is 1074.

Question 177: C

1292

If the rst invitation is sent as early as possible, it will be sent on the50th birthday. It will be accepted after 2 reminders and hence conductedat 50 years 11 months. e time between each screening will be 3 years 11months. Hence, the second screening will be at 54 years 10 months. ethird screening will be at 58 years 9 months. Hence, the fourth screeningwill be at 62 years 8 months.

Question 178: AEllie has worked for the company for more than ve but less than six

whole years. At the end of each whole year she receives a pay rise inthousands equal to the number of years of her tenure. erefore at the endof the rst year the raise is £1,000, then at the end of the second year it is£2,000 and so on to year 5. us the total amount of her pay comprised bythe pay rises is £15,000, so the basic pay before accounting for these riseswas £40,000 - £15,000 = £25,000.

Question 179: Be trains come into the station together every 40 minutes, as the

lowest common multiple of 2, 5 and 8 is 40. Hence, if the last time trainscame together was 15 minutes ago, the next time will be in 25 minutes.

Question 180: CIf you smoke, your risk of getting Disease X is 1 in 24. If you drink

alcohol, your risk of getting Disease X is 1 in 6. Each tablet of the drughalves your risk. erefore a drinker taking 1 tablet means their risk is 1 in12, and taking 2 tablets means their risk is 1 in 24, the same as someonewho smokes.

Question 181: A

ere are 10 red and 8 green balls. Clearly the most likely combinationinvolves these colours only. Since there are more red balls than green, theprobability of red-red is greater than green-green. However, there are two

1293

possible ways to draw a combination, either the red rst followed by greenor green rst followed by red. e probability of red-red = .

e probability of red and green = . erefore the combination of redand green is more likely.

Question 182: Be least likely combination of balls to draw is blue and yellow. You are

much more likely to draw a green ball than either a blue or yellow onebecause there are many more in the bag. Since the draw is taken withoutreplacement, yellow and yellow is impossible because there is only oneyellow ball.

Question 183: FSince there is only 1 blue and 1 yellow ball, it is possible to take 18 balls

which are red or green. You would need to take 19 of the 20 balls to becertain of getting either the blue ball or the yellow ball.

Question 184: Ce smallest number of parties required would theoretically be 3 –

Namely Labour, the Liberal Democrats and UKIP, giving a total of 355seats. However, the Liberal Democrats will not form a coalition with UKIP,so this will not be possible. us, there are 2 options:➢ Labour can form a coalition with the Greens and UKIP, which is notcontradictory to anything mentioned in the question. is would give atotal of 325 seats, and would thus need the next 2 largest parties (eScottish National Party and Plaid Cymru) in order to get more than 350seats, meaning 5 parties would need to be involved.➢ Alternatively, Labour can form a coalition with the Liberal Democratsand the Green Party. is would give a total of 340 seats. Only one moreparty (e.g. the Scottish National Party) would be required to exceed 350seats, giving a grand total of 4 parties.

1294

us, the smallest number of parties needed to form a coalition wouldbe 4.

Question 185: E360 appointments are attended and only 90% of those booked are

attended, meaning there were originally 400 appointments booked in and40 have been missed. 1 in 2 of the booked appointments were for malepatients, so 200 appointments were for male patients. Male patients arethree times as likely to miss booked appointments, so of the 40 that weremissed, 30 were missed by men. Given that of 200 booked appointments,30 were missed, this means 170 were attended.

Question 186: BIf every one of 60 students studies 3 subjects, this is 180 subject

choices altogether. 60 of these are Maths, because everyone takes Maths.60% of 60 is 36, so 36 are Biology. 50% of 60 is 30, so 30 are Economics and30 are Chemistry. 60+36+30+30=156, so there are 24 subject choices leftwhich must be Physics.

Question 187: BIf 100,000 people are diagnosed with chlamydia and 0.6 partners are

informed each, this is 60,000 people, of which 80% (so 48,000) have tests.12,000 of the partners who are informed, as well as 240,000 who are not(300,000 – 60,000) do not have tests. is makes 252,000 who are nottested. We can assume that half of these people would have tested positivefor chlamydia, which is 126,000. So the answer is 126,000.

Question 188: CTiles can be added at either end of the 3 lines of 2 tiles horizontally or

at either end of the 2 lines of 2 tiles vertically. is is a total of 10, but intwo cases these positions are the same (at the bottom of the left handvertical line and the top of the right hand vertical line). So the answer is 10– 2 = 8.

Question 189: C

1295

Harry needs a total of 4000ml + 1200ml = 5200ml of squash. He has1040ml of concentrated squash, which is a fth of the total dilute squashhe needs. So he will need 4 parts water to every 1 part concentratedsquash, therefore the resulting liquid is 1/5 squash and 4/5 water.

Question 190: Cere are 24 different possible arrangements (4 x 3 x 2 x 1), which

means that there are 23 other possible arrangements than Alex, Beth,Cathy, Daniel.

Question 191: FA is incorrect because the distance travelled is only 10 miles. B is

incorrect because the distance travelled is 19 miles. C is incorrect becauseno town is visited twice. D is incorrect because Hondale and Baleford areboth visited twice. E is incorrect because no town is visited twice.erefore F is the correct answer.

Question 192: CGeorgia is shorter than her Mum and Dad, and each of her siblings is

at least as tall as Mum (and we know Mum is shorter than Dad becauseEllie is between the two), so we know Georgia is the shortest. We knowthat Ellie, Tom and Dad are all taller than Mum, so Mum is secondshortest. Ellie is shorter than Dad and Tom is taller than Dad, so we canwork out that Ellie must be third shortest.

Question 193: ADanielle must be sat next to Caitlin. Bella must be sat next to the

teaching assistant. Hence these two pairs must sit in different rows. Onepair must be sat at the front with Ashley, and the other must be sat at theback with Emily. Since the teaching assistant has to sit on the left, thismust mean that Bella is sat in the middle seat and either Ashley or Emily(depending on which row they are in) is sat in the right hand seat.However, Bella cannot sit next to Emily, so this means Bella and the

1296

teaching assistant must be in the front row. So Ashley must be sat in thefront right seat.

Question 194: Ce dishwasher is run 2+p times a week, where p is the number of

people in the house. Let the number of people in the house when the son isnot home be s, and when the son is home it is s+1. In 30 weeks when theson is home, she would buy 6 packs of dishwasher tablets. In 30 weekswhen the son is not home, she would buy 5 packs of dishwasher tablets. So1.2 times as many packs of dishwasher tablets are bought when he ishome. So 2+s+1 is 1.2 time 2+s.

i.e.erefore 0.2s = 0.6s = 3When her son is home, there are s + 1 = 4 people in the house.Question 195: ANo remaining days in the year obey the rule. e next date that does is

01/01/2015 (integers are 0, 1, 2, 5). is is 6 days later than the speci eddate.

Question 196: BIf each town is due North, South, East or West of at least 2 other towns

and we know that one is east and one is north of a third, then they must bearranged in a square. So Yellowtown is 4 miles east of Bluetown to make asquare, which means it must be 5 miles north of Redtown. So Redtown is 5miles south of Yellowtown.

Question 197: BJenna pours 4/5 of 250 ml into each glass, which is 200 ml. Since she

has 1500 ml of wine, she pours 100 ml into the last glass, which is 2/5 ofthe 250 ml full capacity.

1297

Question 198: Ee maximum number of girls in Miss Ellis’s class with brown eyes and

brown hair is 10, because the two thirds of the girls with brown eyes couldalso all have brown hair. e minimum number is 0 because it could bethat all the boys, and the third of the girls without brown eyes, all hadbrown hair, which would be 2/3 of the class.

Question 199: EA negative “score” results from any combination of throws which

includes a 1 but from no other combination. Given that a negative scorehas a 0.75 probability, a positive or zero score has a 0.25 probability.erefore throwing two numbers that are not 1 twice in a row has aprobability of 0.25. Hence, the probability of throwing a non-1 number oneach throw is √0.25 = 0.5. So the probability of throwing a 1 on anindividual throw is 1 – 0.5 = 0.5.

Question 200: CWe can work out from the information given the adult at rate and the

charge per stop. Let the charge per stop be s and the at rate be f.erefore: 15s + f = 1.70

8s + f = 1.14We can hence work out that: 7s = 0.56, so s = 0.08. Hence, f = 0.50Megan is an adult so she pays this rate. For 30 stops, the rate will be

0.08 x 30 + 0.50 = 2.90.Question 201: BWe found in the previous question that the at rate for adults is £0.50

and the rate per stop is £0.08. We know that the child rate is half the atrate and a quarter of the “per stop” rate, so the child at rate is £0.25 andthe rate per stop is 2p. So for 25 stops, Alice pays:

0.02 x 25 + 0.25 = 0.75Question 202: C

1298

We should rst work out how many stops James can travel. For £2, hecan afford to travel as many stops as £1.50 will take him once the at rateis taken into account. e per stop rate is 8p per stop, so he can travel 18stops, so he will need to go to the 18th stop from town. So he will need towalk past 7 stops to get to the stop he can afford to travel from.

Question 203: De picture will need a 12 inch by 16 inch mount, which will cost £8. It

will need a 13 inch by 17 inch frame, which will cost £26. So the cost ofmounting and framing the picture will be £8 + £26 = £34.

Question 204: CMounting and framing an 8 by 8 inch painting will cost £5 for the

mount and £22 for the frame, which is £27. Mounting and framing a 10 by10 inch painting will cost £6 for the mount and £26 for the frame, which is£32. e difference is £32 - £27 = £5.

Question 205: BWe found in the last question that mounting and framing a 10 by 10

inch painting will cost £6 for the mount and £26 for the frame, which is£32 total. We can calculate that each additional inch of mount and framefor a square painting costs £2.50; £2 for the frame and £0.50 for the mount.So an 11 inch painting will cost £34.50 to frame and mount, a 12 inch £37,a 13 inch £39.50, a 14 inch £42. e biggest painting that can be mountedand framed for £40 is a 13 inch painting.

Question 206: DRecognise that the pattern is “consonants move forward by two

consonants; vowel stay the same”. is allows coding of the word MAGICALto PAJIFAN to RALIHAQ.

1299

Question 207: CIf f donates the at rate, and k denotes the rate per km, we can form

simultaneous equations:f + 5k = £6 AND f + 3k = £4.20Subtract equation two from equation one:( f + 5k) - ( f + 3k) = £6 - £4.20us, 2k = £1.80 and k = £0.90erefore, f + (5 x 0.90) = £6So, f + £4.50 = £6. us, f = £1.507k will be £1.50 + 7 x £0.90 = £7.80Question 208: Ce increase from 2001/2 to 2011/12 was 1,019 to 11,736, which equals

a linear increase of 10,717 admissions.So, in 20 years, we would expect to see an increase by 10,717 x 2 =

21,434. Add this to the number in 2011 to give 33,170 admissions.Question 209: AAs the question uses percentages, it does not matter what gure you

use. To make calculations easier, use an initial price of £100. When on sale,the dress is 20% off, so using a normal price of £100, the dress would be£80. When the dresses are 20% off, the shop is making a 25% pro t.erefore: £80 = 1.25 x purchase price.

erefore, the purchase price is: . us, the normal pro t is £100 - £64= £36. I.e. when a dress sells for £100, the shop makes £36 or 36% pro t.

Question 210: C

1300

1. Incorrect. ere must be 6 general committee clinical students, plusthe treasurer, and 2 sabbatical roles, none of whom can bepreclinical, so there must be a maximum of 11 preclinical students.

2. Correct. ere must two general for each year plus welfare and socialofficers, totalling to 6.

3. Incorrect. e committee is made up of 20 students, 2 roles aresabbatical, so there are 18 studying students, and therefore there canbe 3 from each year.

4. Correct. ere are 18 studying students on the committee, and theremust be 6 general committee members from pre-clinical, plus welfareand social, therefore there must be a minimum of 8 pre-clinicalstudents, so there must be 10 clinical students.

5. Incorrect. You need to count up the number of speci c roles on thecommittee, which is 5, and there must be 2 students from each year,which is 12. is leaves 3 more positions, which the question doesn’tstate can’t be rst years. erefore there could be up to 5 rst years.

6. Incorrect. ere must be at least 2 general committee members fromeach year. However, the worked answer to 5 shows there are 15general committee members which are split across the 6 years, andso there must be an uneven distribution.

Question 211: BRemember 2012 was a leap year. Work through each month, adding

the correct number of days, to work out what day each 13th would be on.If a month was 28 days, the 13th would be the same day each month,

therefore to work this out quickly, you only need to count on the numberof days over 28. For example, in a month with 31 days, the 13th will be 3weekdays (31-28) later.

us if 13th January is a Friday, 13th February is a Monday, (Februaryhas 29 days in 2012), 13th March is a Tuesday and 13th April is a Friday.

1301

Question 212: Eere are 18 sheep in total. e question states there are 8 male sheep,

which means there are 10 female sheep before some die. 5 female sheepdie, so there are 5 female sheep alive to give birth to lambs. Each delivers 2lambs, making 10 lambs in total. ere are 4 male sheep and 5 mothers sothe total is 10 + 4 + 5 = 19 sheep.

Question 213: DWe can see from the fact that all the possible answers end “AME” that

the letters “AME” must be translated to the last 3 letters of the coded word,“JVN”, under the code. J is the 10th letter of the alphabet so it is 9 letters onfrom A (V is the 21st letter of the alphabet and M is the 13th, and N is the14th letter of the alphabet and E is the 5th, therefore these pairs are also 9letters apart). erefore P is the code for the letter 9 letters before it in thealphabet. P is the 16th letter of the alphabet, therefore it is the code for the7th letter of the alphabet, G. erefore from these solutions the onlypossibility for the original word is GAME.

Question 214: CLet x be the number of people who get on the bus at the station.It is easiest to work backwards. After the 4th stop, there are 5 people

on the bus. At the 4th stop, half the people who were on the bus got off(and therefore half stayed on) and 2 people got on. erefore, 5 is equal to2 plus half the number of people who were on the bus after the 3rd stop. Sohalf the number of people who were on the bus after the 3rd stop must be3. erefore, after the 3rd stop, there must have been 6 people on the bus.

We can then say that 6 is equal to 2 plus half the number of people whowere on the bus after the 2nd stop. erefore there were 8 people on thebus after the 2nd stop.

We can then say that 8 is equal to 2 plus half the number of people whowere on the bus after the 1st stop. erefore there were 12 people on thebus after the 1st stop.

1302

We can then say that 12 is equal to 2 plus half the number of peoplewho got on the bus at the station. erefore the number of people who goton the bus at the station is 20.

Question 215: BWe know from the question that I have purchased small cans of blue

and white paint, and that blue paint accounted for 50% of the total cost.Since a can of blue paint is 4 X the price of a can of white paint, we know Imust have purchased 4 cans of white paint for each can of blue paint.

Each can of small paint covers a total of 10m2, and I have painted atotal of 100m2, in doing so using up all the paint. erefore, I must havepurchased 10 cans of paint. erefore, I must have purchased 2 cans ofblue paint and 8 cans of white paint. So I must have painted 20m2 of wallspace blue.

Question 216: Ee cost for x cakes under this offer can be expressed as: x(42-x2)Following this formula, we can see that 2 cakes would cost 76p, 3 cakes

would cost 99p, and 4 cakes would cost 104p. As the number of cakesincreases beyond 4, we see that the overall price actually drops, as 5 cakeswould cost 85p and 6 cakes would cost 36p. is con rms Isobel’sprediction that the offer is a bad deal for the baker, as it ends up cheaperfor the customer to purchase more cakes. It is clear that 6 cakes is thesmallest number for which the price will be under 40p, and the price willcontinue to drop as more cakes are purchased.

Question 217: CAdding up the percentages of students in University A who do

“Science” subjects gives:23.50 + 6.25 + 30.25 = 60%.60% of 800 students is 480, so 480 students in University A do “Science”

subjects.

1303

Adding up the percentages of students in University B who do “Sciencesubjects” gives:

13.25 + 14.75 + 7.00 = 35%. 35% of 1200 students is 420, so 420 studentsin University B do “Science” subjects. erefore:

480 – 420 = 6060 more students in University A than University B take a “Science”

subject.Question 218: CLet the number of miles Sonia is travelling be x. Because she is

crossing 1 international border, travelling by Traveleasy Coaches will costSonia: £(5 + 0.5x)

Travelling by Europremier coaches will cost Sonia: £(15 + 0.1x).Because we know the cost is the same for both companies, the number

of miles she is travelling can be found by setting these two expressionsequal to each other: 5 + 0.5x = 15 + 0.1x.

is equation can be rearranged to give: 0.4x = 10erefore: x = 10/0.4 = 25Question 219: ETo nd out whether many of these statements are true it is necessary

to work out the departure and arrival times, and journey time, for eachgirl.

Lauren departs at 2:30pm and arrives at 4pm, therefore her journeytakes 1.5 hours

Chloe departs at 1:30pm and her journey takes 1 hour longer than 1.5hours (Lauren’s journey), therefore her journey takes 2.5 hours and shearrives at 4pm

Amy arrives at 4:15pm and her journey takes 2 times 1.5 hours(Lauren’s journey), therefore her journey takes 3 hours and she departs at1:15pm.

1304

Looking at each statement, the only one which is de nitely true is E:Amy departs at 1:15pm and Chloe departs at 1:30pm therefore Amydeparted before Chloe.

D may be true, but nothing in the question shows it is de�nitely true,so it can be safely ignored.

Question 220: BFirst consider how many items of clothing she can take by weight. e

weight allowance is 20kg. Take off 2kg for the weight of the empty suitcase,then take off another 3kg (3 X 1000g) for the books she wishes to take.erefore she can t 15kg of clothes in her suitcase. To nd out how manyitems of clothing this is, we can divide 15kg=15000g by 400g: 15000/400 =150/4 = 37.5

So she can pack up to 37 items of clothing by weight.Now consider the volume of clothes she can t in. e total volume of

the suitcase is:50cm x 50cm x 20cm = 50000cm3

e volume of each book is: 0.2m x 0.1m x 0.05m = 1000cm3

So the volume of space available for clothes is: 50000 – (3 x 1000) =47000cm3

To nd out how many items of clothing she can t in this space, wecan divide 47000 by 1500:

47000/1500 = 470/15 = 31 1/3So she can pack up to 31 items of clothing by volume.Although she can t 37 items by weight, they will not t in the volume

of the suitcase, so the maximum number of items of clothing she can packis 31.

Question 221: DWe can work out the Answer by considering each option:Bed Shop A: £120 + £70 = £190Bed Shop B: £90 + £90 = £180

1305

Bed Shop C: £140 + (1/2 x £60) = £170Bed Shop D: (2/3) x (£140+£100) = (2/3) x (£240) = £160Bed Shop E: £175erefore the cheapest is Bed Shop D.Question 222: Ce numbers of socks of each colour is irrelevant, so long as there is

more than one of each (which there is). ere are only 4 colours of socks,so if Joseph takes 5 socks, it is guaranteed that at least 2 of them will be thesame colour.

Question 223: DPaper comes in packs of 500, and with each pack 20 magazines can be

printed. Each pack costs £3.Card comes in packs of 60, and with each pack 60 magazines can be

printed. Each pack costs £3 x 2 = £6.Each ink cartridge prints 130 sheets, which is 130/26 = 5 magazines.

Each cartridge costs £5.e lowest common multiple of 20, 60 and 5 is 60, so it is possible to

work out the total cost for printing 60 magazines. Printing 60 magazineswill require 3 packs of paper at £3, 1 pack of card at £6 and 12 inkcartridges at £5. So the total cost of printing 60 magazines is: (3 x 3) + 6 +(12 x 5) = £75.

e total budget is £300.£300/£75 = 4So we can print 4x60 magazines in this budget, which is 240

magazines.Question 224: E

We can express the information we have as:

1306

So the six additional lengths make up 1/20 of Rebecca’s intendeddistance. So the number of lengths she intended to complete was: 20 x 6 =120.

Question 225: BSammy has a choice of 3 avours for the rst sweet that he eats. Each

of the other sweets he eats cannot be the same avour as the sweet he hasjust eaten. So he has a choice of 2 avours for each of these four sweets. Sothe total number of ways that he can make his choices is:

3 × 2 × 2 × 2 × 2 = 48Question 226: CSuppose that today Gill is x years old. It follows that Granny is 15x

years old. In 4 years’ time, Gill will be (x+4) years old and Granny will be15x+4 years old. We know that in 4 years’ time, Granny’s age is equal toGill’s age squared, so: 15x + 4 = (x + 4)2

Expanding and rearranging, we get: x2 – 7x + 12 = 0We can factorise this to get: (x - 3)(x - 4)So x is either 3 or 4. Gill’s age today is either 3 or 4 so Granny is either

45 or 60. We know Granny’s age is an even number, so she must be 60 andhence Gill must be 4. So the difference in their ages is 56 years.

Question 227: CIf Pierre is telling the truth, everyone else is not telling the truth. But,

also in this case, what Qadr said is not true, and hence Ratna is telling thetruth. So we have a contradiction. So we deduce that Pierre is not tellingthe truth. erefore, Qadr is telling the truth, and so Ratna is not tellingthe truth. So Sven is also telling the truth, and hence Tanya is not tellingthe truth. So Qadr and Sven are telling the truth and the other three arenot telling the truth.

Question 228: DAngus walks for 20 minutes at 3 mph and runs for 20 minutes at 6

mph. 20 minutes is one-third of an hour. So the number of miles that

1307

Angus covers is: 3 × 13 + 6 × 13 = 6Bruce covers the same distance. So Bruce walks 12 × 3 miles at 3 mph

which takes him 30 minutes and runs the same distance at 6 mph whichtakes him 15 minutes. So altogether it takes Bruce 45 minutes to nish thecourse.

Question 229: BAlthough you could do this quickly by forming simultaneous

equations, it is even quicker to note that 72 x 4 = 288. Since Species 24601each have 4 legs; it leaves a single member of species 8472 to account forthe other 2 legs.

Question 230: ENone of the options can be concluded for certain. We are not told

whether any chicken dishes are spicy, only that they are all creamy. Whilstall vegetable dishes are spicy, some non-vegetable dishes could also bespicy. ere is no information on whether dishes can be both creamy andspicy, nor on which, if any, dishes contain tomatoes. Remember, if you’rereally stuck, draw a Venn diagram for these types of questions.

Question 231: CAt 10mph, we can express the time it takes Lucy to get home as: 60 x

8/10 = 48Since Simon sets off 20 minutes later, his time taken to get home, in

order to arrive at the same time, must be:48 – 20 = 28erefore his cycling speed must be: 48/28 x 10 = 17mphQuestion 232: Ae total pro t from the rst transaction can be expressed as: 2000 x 8

= 16,000pe total pro t from the second transaction is: 1000 x 6 = 6,000perefore the total pro t is 22,000p or £220 before charges. ere are

four transactions at a cost of £20 each, therefore the overall pro t is: £220

1308

– (20 x 4) = £140Question 233: CFor the total score to be odd, there must be either three odd or one

odd and two even scores obtained. Since the solitary odd score could beeither the rst, second or third throw there are four possible outcomesthat result in an odd total score. Additionally, there are the same numberof possibilities giving an even score (either all three even or two odd andone even scores obtained), and the chance of throwing odd or even withany given dart is equal. erefore, there is an equal probability of threedarts totalling to an odd score as to an even score, and so the chance of anodd score is ½.

Question 234 Cis is a compound interest question. £5,000 must be increased by 5%,

and then the answer needs to be increased by 5% for four more iterations.After one year: £5,000 x 1.05 = £5,250

Increasing sequentially gives 5512, 5788, 6077 and 6381 after ve years.erefore the answer is £6,381.

Question 235: DIf in 5 years’ time the sum of their ages is 62, the sum of their ages

today will be: 62 – (5 x 2) = 52erefore if they were the same age they would both be 26, but with a

12 year age gap they are 20 and 32 today. Michael is the older brother, so 2years ago he would have been aged 30.

Question 236: ATearing out every page which is a multiple of 3 removes 166 pages. All

multiples of 6 are multiples of 3, so no more pages are torn out with thatinstruction. Finally, half of the remaining pages are removed, whichequates to an additional 167 pages. erefore 333 pages are removed intotal. e total surface area of these pages is 15 x 30 x 333 = 149,850 cm2 =14.9m2. At 110 gm2, 14.9 m2 weighs 14.9 x 110 = 1,650g (1,648g unrounded)

1309

Question 237: De cost of fertiliser is 80p/kg = 8p/100g. At 200g the incremental

increase in yield is 65 pence/m. At each additional 100g it will be reducedby 30%, therefore at 300g/m it is 45.5p, at 400g/m it is 31.8p, at 500g/m it is22.3p, at 600g/m it is 15.6p, at 700g/m it is 10.9p, and at 800g it is 7.6p. Soat 800g the gain in yield is less than the cost of the fertiliser to produce thegain, and so it is no longer cost effective to fertilise more.

Question 238: DStatements A, C and E are all de nitely true. Meanwhile, statement B

may be not true but is not de nitely untrue, as this depends on thenumber of cats and rabbit owned.

Only statement D is de nitely untrue. e type of animal requiring themost food is a dog, and as can be seen from the tables, Furry Friendsactually sells the most expensive dog food, not the cheapest.

Question 239: Ce largest decrease in bank balance occurs between January 1st and

February 1st, totalling £171, re ecting the amount spent during the monthof January, £1171. However, because there is a pay rise beginning on March10th, we need to consider that from April onwards, the bank balance willhave increased by £1100, not £1000. is means that the same decrease inbank balance re ects £100 more spending if it occurs after March. ismeans that 2 months now have seen more spending than February.Between March 1st and April 1st, the bank balance has decreased by £139.With the salary increase, the salary is now £1100, so the total spending forthe month of March is £1239. is is greater than the total spending duringthe month of January.

Similarly, the month of April has also seen more spending than Januaryonce the pay rise is considered, a total of £1225 of spending. However, thisis still less than the month of March.

Question 240: C

1310

If Amy gets a taxi, she can set off 100 minutes before 1700, which is1520.

If Amy gets a train, she must get the 1500 train as the later train arrivesafter 1700, so she must set off at 1500.

Since Northtown airport is 30 minutes from Northtown station, thereis no way Amy can get the ight and still arrive at Northtown station by1700. erefore Amy should get a taxi and should leave at 1520.

Question 241: CWe can decompose the elements of the multiplication grid into their

prime factors, thus:

bc = 7 x 17, so one of b and c must be 7 and the other must be 17. bmust be 17 because bd is a multiple of 17 and not of 7, and c must be 7because ac is a multiple of 7 and not of 17. ac is 168, so a must be 168divided by 7, which is 24. ad is 720 so d must be 720 divided by 24, which is30. Hence the answer is 30.

Alternatively approach the question by eliminating all answers whichare not factors of both 720 and 510.

Question 242: E48% of the students are girls, which is 720 students. Hence 80 is 1/9 of

the girls, so 1/9 of boys are mixed race. e remaining 780 students areboys, so 87 boys are mixed race to the nearest person. ere is a shortcutto this question. Notice that 80 girls are mixed race, and the proportion isthe same for boys. As there are more boys than girls we know the answer isgreater than 80. Option E 90 is the only option for which this holds true.

Question 243: D

1311

Don’t be fooled – this is surprisingly easy. We can see that betweenMonday and ursday, Christine has worked a total of 30 hours. We canalso calculate how long her shift on Friday was supposed to be. She is ableto make up the hours by working 3 extra hours next week, and 5 hours onSunday. us, the Friday shift must have been planned to be 8 hours long.Adding this to the other 30 hours, we see that Christine was supposed towork 38 hours this week.

Question 244: C130°. Each hour is 1/12 of a complete turn, equalling 30°. e smaller

angle between 8 and 12 on the clock face is 4 gaps, therefore 120°. Inaddition, there is 1/3 of the distance between 3 and 4 still to turn, so anadditional 10° must be added on to account for that.

Question 245: Be total price of all of these items would usually be £17. However,

with the DVD offer, the customer saves £1, giving a total cost of £16. us,the customer will need to receive £34 in change.

Question 246: E

1. Incorrect. UCL study found eating more portions of fruit andvegetables was bene cial.

2. Incorrect. is is a possible reason but has yet to be fullyinvestigated.

3. Incorrect. Fruit and vegetables are more protective againstcardiovascular disease, and were shown to have little effect on cancerrates.

4. Incorrect. Inconclusive – people who ate more vegetables generallyhad a lower mortality but unknown if this is due to eating morevegetables or other associated factors.

5. Correct. Although this has previously been the case, this study didnot nd so. ‘they recorded no additional decline for people who ateover 5 portions’.

1312

6. Incorrect. e 5% decline per portion was only up to 5 portions andno additional reduction in mortality for 7 than 5 portions.

7. Incorrect. Study only looks at cancers in general and states need tolook into speci c cancers.

Question 247: CDeaths in meta-analysis = 56423/800000 = 0.07 or 7%1% lower in UCL study so 6%6% of 65,000 = 65000 x 0.06 = 3,900Question 248: B

1. Eating more fruit and vegetables doesn’t particularly lower overallrisk but need research into speci c cancer risk.

2. e UCL research alone found that increasing the number of fruitand vegetable portions had a bene cial effect, even though thiswasn’t the overall conclusion when combined with results from themeta-analysis.

3. e results were not exactly the same but showed similar overalltrends.

4. Although this may be true, there is no mention of this in the passage.5. Fruit and vegetables are protective against cardiovascular disease,

but not exclusively. ey also reduce the rates of death from allcauses.

6. e UCL study is in England only and the meta-analysis acombination of studies from around the world.

7. Suggested by the UCL research, but not the meta-analysis, so not anoverall conclusion of the article.

Question 249: ERemember that you don’t need to calculate exact values for

question 249 – 251. us, you should round numbers frequently to make

1313

this more manageable. Work out percentage of beer and wineconsumption and then the actual value using the total alcoholconsumption gure:

Belarus: 17.3 + 5.2 = 22.5%;0.225 x 17.5 = 3.94Lithuania: Missing gure 100 – 7.8 – 34.1 – 11.6 = 46.546.5 + 7.8 = 54.3%0.543 x 15.4 = 8.36France: 18.8 + 56.4 = 75.2%0.752 x 12.2 = 9.17Ireland: 48.1 + 26.1 = 74.20.742 x 11.9 = 8.83Andorra: missing gure 100 – 34.6 – 20.1 = 45.334.6 + 45.3 = 79.9%0.799 x 13.8 = 11.0Question 250: DRussia:2010 – Total = 11.5+3.6 = 15.1. Spirits = 0.51 x 15.1 = 7.72020 – Total = 14.5. Spirits = 0.51 x 14.5 = 7.4Difference = 0.3 LBelarus:2010 – Total = 14.4 + 3.2 = 17.6. Spirits = 0.466 x 17.6 = 8.22020 – Total = 17.1. Spirits = 0.466 x 17.1 = 8.0Difference = 0.2 LLithuania:2010 – Total = 15.4. Spirits = 0.341 x 15.4 = 5.32020 – Total = 16.2. Spirits = 0.341 x 16.2 = 5.5Difference = 0.2 LGrenada:

1314

2010 – Total = 12.5. Spirits % = 100 – 29.3 – 4.3 – 0.2 = 66.2%. Spirits =0.662 x 12.5 = 8.3

2020 – Total = 10.4. Spirits = 0.662 x 10.4 = 6.8Difference = 1.5 LIreland:2010 – Total = 11.9. Spirits = 0.187 x 11.9 = 2.22020 – Total = 10.9. Spirits = 0.187 x 10.9 = 2Difference = 0.2 LQuestion 251: CWork out 4.9 as a percentage of total beer consumption in Czech

Republic and search other rows for similar percentage.4.9/13 = 0.38, approx. 38% which is very similar to percentage

consumption in Russia (37.6).Question 252: BWe can add up the total incidence of the 6 cancers in men, which is

94,000. en we can add up the total incidence in women, which is101,000. As a percentage of 10 million, this is 0.94% of men and 1.01% ofwomen. erefore the difference is 0.07%.

Question 253: CGiven there are 1.15 times as many men as women, the incidence of

each cancer amongst men needs to be greater than 1.15 times theincidence amongst women in order for a man to be more likely to developit. e incidence is at least 1.15 higher in men for 3 cancers (prostate, lungand bladder).

Question 254: DIf 10% of cancer patients are in Sydney, there are 10,300

prostate/bladder/breast cancer patients and 9,200 lung/bowel/uteruscancer patients in Sydney. Hence the total number of hospital visits is10,300 + 18,400, which is 28,700.

Question 255: A

1315

e proportion of men with bladder cancer is 2/3 and women 1/3.Question 256: DFirst we work out the size of each standard drink. 50 standard drinks

of vodka is equivalent to 1250ml, so one drink is 25ml or 0.025 litres. 11.4standard drinks of beer is 10 pints of 5700ml, so one standard drink is500ml or 0.5 litres. 3 standard drinks of cocktail is 750ml so one is 250ml or0.25 litres. 3.75 standard drinks of wine is 750ml, so one is 200ml or 0.2litres.

We can then work out the number of units in each drink. Vodka has0.025 x 40 = 1 unit, Beer has 0.5 x 3 = 1.5 units, Cocktail has 0.25 x 8 = 2units and Wine has 0.2 x 12.5 = 2.5 units. Since the drink with the mostunits is wine, the answer is D.

Question 257: BWe found in the last question that vodka has 1 unit, beer has 1.5,

cocktail has 2 and wine has 2.5. Hence in the week, Hannah drinks 23.5units and Mark drinks 29 units. Hence Hannah exceeds the recommendedamount by 9.5 units and Mark by 9 units.

Question 258: DWe found that vodka has 1 unit, beer has 1.5, cocktail has 2 and wine

has 2.5. Hence it is possible to make 5 combinations of drinks that are 4units: 4 vodkas, 2 cocktails, 2 vodkas and a cocktail, 1 vodka and 2 beers,or a wine and a beer.

Question 259: De total number of males in Greentown is 12,890. Adding up the rest

of the age categories, we can see that 10,140 of these are in the older agecategories. Hence there are 2750 males under 20.

Question 260: CGiven that in the rst question we found the number of males under

20 is 2,750, we can then add up the totals in the age categories (apart from40-59) in order to nd that 15,000 of the residents of Greentown are in

1316

other age categories. Hence 9,320 of the population are aged 40-59. Weknow that 4,130 of these are male, therefore 5,190 must be female.

Question 261: Ce age group with the highest ratio of males:females is 20-39, with

approximately 1.9 males per females (approximately 3800:2000). As a ratioof females to males, this is 1:1.9.

Question 262: Cere are 4 instances where the line for Newcastle is at from one

month to the next per year, hence in 2008-2012 (5 years) there are 20occasions when the average temperature is the same from month tomonth. During 2007, there are 2 occasions, and during 2013 there are 3.

Question 263: Ae average temperature is lower than the previous month in London

for all months from August to December, which is 5 months. However, inAugust and November in Newcastle, the average temperature remains thesame as the previous month. Hence there are only 3 months where theaverage temperature is lower in both cities. Hence from 2007 to 2012, thereare 18 months where the average temperature is lower than the previousmonth. During 2013, the only included month where the temperature islower in both cities than the previous month is September. Hence thereare 19 months in total when the temperature is lower in both cities thanthe previous month.

Question 264: BFirstly, work out the difference between average temperatures for each

month (2, 3, 1, 2, 1, 3, 3, 2, 2, 5, 1, 0). en sum them to give 25. Divide bythe number of months (12) to give 21/12, which is 2ºC to the nearest 0.5ºC.

Question 265: Dere is not enough information to tell which month the highest sales

are in. We know it increases up to a point and decreases after it, but as we

1317

don’t know by how much we cannot project where the maximum sales willbe.

Question 266: BGiven that by observation, Q2 and Q3 both account for 1/3 of the sales

and Q4 accounts for 1/4, this leaves that Q1 accounts for 1/12 of sales.1/12 of £354,720 is £29,560.

Question 267: AQuarter 2 accounts for 1/3 of the sales, which is £60,000 in sales

revenue. If a tub of ice cream is sold for £2 and costs the manufacturer£1.50, this means pro t is 1/4 of sales revenue. Hence £15,000 pro t ismade during Q2. Hence the answer is A.

Question 268: DA. and B – Incorrect. Both could be true but neither is de�nitely true as

it is dependent on the relative number of families with each number ofchildren, which is not given in the question. erefore we cannot know forcertain whether these statements are true.

C – Incorrect. C is de nitely untrue as half of the families spend £400 amonth on food, which totals £4800 a year.

D – Correct. is option is true as 1/6 of families with 1 child and 1/6of families with 3 children spent £100 a month on food.

E – Incorrect. is option is de nitely untrue as the averageexpenditure for families with 2 children is actually £400 a month.

Question 269: B2210 out of 2500 lled in responses, meaning that 290 did not. 290 as a

percentage of 2500 is roughly 12% (11.6%) of the school that did notrespond.

Question 270: Ce percentage of students that saw bullying and reported it was 35%,

so 65% of those who saw it did not which is equivalent to 725 students. Of

1318

this 725, 146 which roughly equals 20%, gave the reason that they did notthink it was important.

Question 271: BOf the students who told a teacher, 286 did not witness any action. Of

those who did notice action, i.e. 110, only 40% noticed any direct actionwith the bully involved. 40% of 110 is 44, so the correct answer is B.

Question 272: D“427 cited fears of being found out” which means about 59% out of the

725 students that did not tell about the bullying, cited that it was becausethey worried about others nding out.

Question 273: FNorth-east: 56 per 100,000 on average. is means that there must be a

higher proportion of women than this and a lower proportion of men,such that the average is 56/100,000

We must make the reasonable assumption that there are the samenumber of men and women in the population as the question asks us toapproximate.

erefore there are 18.6/50,000 men and 37.3/50,000 womenis scales to 74.4/100,000 women which is roughly 74/100,000.Question 274: C8 million children – question tells to approximate to 4 million girls and

4 million boys.Girls: 20% eat 5 portions fruit and vegetables a day. 20% of 4 million: 4

x 0.2 = 0.8 millionBoys: 16% eat 5 portions of fruit and vegetables a day. 16% of 4 million:

4 x 0.16 = 0.64 millionNumber of more girls: 800,000 – 640,000 = 160,000.Question 275: B

1. Incorrect. Women: 13619+10144+6569 = 30332. Men: 16818 + 9726 +7669 + 6311 = 40524

1319

2. Correct. Flu + pneumonia, lung cancer and chronic lower respiratorydiseases = 15361 + 13619 + 14927 = 43907

3. Incorrect. More common cause of death but no informationsurrounding prevalence.

4. Incorrect. Colon cancer ranking 8 for both.

Question 276: Ae government has claimed a 20% reduction, so we are looking for an

assessment criterion which has reduced 20% from 2013 to 2014. We cansee that only “Number of people waiting for over 4 hours in A&E” hasreduced by 20%, so this must be the criterion the government has used todescribe “waiting times in A&E”. us, the answer is A.

Question 277: BRovers must have played 10 games overall as they played each other’s

team twice. ey lost 9 games scoring no points and so must have won 1game, which scores 3 points.

Question 278: ATo have nished between City and United, Athletic must have got

between 23 and 25 points. Hence they must have got 24 points because noteam got the same number of points as another. Athletic won 7 gameswhich is 21 points, so they must have also got 3 points from drawing 3games. is accounts for all 10 games they played, so they did not lose anygames.

Question 279: CUnited won 8 games and drew 1, which is 25 points. Rangers drew 2

games and won none, which is 2 points. erefore the difference in pointsis 23.

Question 280: CType 1 departments reached the new target of 95% at least three times

since it was introduced. All the other statements are correct.Question 281: C

1320

Total attendances in Q1 08-9: 5.0 millionTotal attendances in Q1 04-5: 4.5 millione difference = 0.5 million0.5/5 x 100 = 10% increaseQuestion 282: Cere are 16 quarters in total since the new target came into effect.4/16 = 0.25, so the target has been hit 25% of the time i.e. missed 75%

of the time.Question 283: CRanjna must leave Singapore by 20:00 to get to Bali by 22:00. e latest

ight she can therefore get is the 19:00. us, she must arrive in Singaporeby 17:00 (accounting 2 hours for the stopover). e ight from Manchesterto Singapore takes 14 hours. Manchester is 8 hours behind Singapore soshe must leave Manchester 22 hours before 17:00 on Wednesday i.e. by19:00 on Wednesday. us, the latest ight she can get is the 18:00 onWednesday.

Question 284: De 08:00 ight will arrive at Singapore for 22:00 on Monday (GMT) or

06:00 Tuesday Singapore time. She then needs a 2 hour stopover, soearliest connecting ight she can get is 08:30 on Tuesday. e ight landsin Bali at 10:30. She then spends 1 hour and 45 minutes getting to herdestination – arriving at 12:15 Tuesday.

Question 285: C

1. Incorrect. e graph is about level, and certainly not the steepestgradient post 2007.

2. Incorrect. Although there has been a general decline, there are someblips of increased smoking.

3. Correct.4. Incorrect. e smoking rate in men decreased from 51% in 1974 to

21% in 2010. us, it decreased by more than a half.

1321

5. Incorrect. e percentage difference between men and womensmokers has been minimal in the 21st century.

Question 286: DFor this type of question you will have to use trial and error after

you’ve analysed the data pattern to nd the correct answer. e quickestway to do this is to examine outliers to try and match them to data in thetable e.g. the left-most point is an outlier for the X-axis but average for they-axis. Also look for any duplicated results in the table and if they arepresent on the graph, e.g. Hannah and Alice weigh 68 kg but this can’t befound on the graph.

Question 287: Cis is pretty straightforward; the point is at approximately 172-174

cm in height and 164 -166 cm in arm span. Matthew is the only studentwho ts these dimensions.

Question 288: Cis is straightforward – just label the diagram using the information

in the text and it becomes obvious that C is the correct answer.Question 289: CSince we do not know whether they went to university or not, we must

add the number of women with children who work and those who went touniversity, 2, to the number of women with children who work but did notgo to university, 1 (2 + 1 = 3).

Question 290: CTo work this out we must add up all the numbers within the rectangle,

4 + 6 + 1 + 2 + 11 + 12 + 7 + 15 = 58Question 291: ECalculate the number of men + women who have children and work

i.e. 11 + 5 + 2 = 18Question 292: C

1322

To solve this we must work out the total number of people who hadchildren i.e. 3 + 6 + 5 + 11 + 1 + 2 =28. en we work out the total number ofpeople who went to university, but that do not also have children so thatthese are not counted twice: 13 + 12 = 25. en we add these two numberstogether, 28 + 25 = 53 and subtract the number of people who fell into bothcategories i.e. 53 - (5 + 11 + 2) = 35

Question 293: CTo work this out we must add up all the numbers outside the rectangle

that also fall within both the circle and the square, which is 5.Question 294: D & Eis question asks for identi cation of the blank space, which is the

space within the triangle, the rectangle and the square i.e. indicatingworking women who went to university but did not have children. isalso reveals non-working men who did not have children and did not go touniversity.

Question 295: Ce normal price of these items would be £18.50 (£8 + £7 + £3.50).

However, with the 50% discount on meat products, the price in the sale forthese items will be £9.25. us, Alfred would receive £10.75 of change froma £20 note.

Question 296: Ce number of games played and points scored is a red herring in this

question. e important data is ‘Goals For’ and ‘Goals Against’. As this is ade ned league and the teams have only played each other, the ‘Goals For’column must equal the ‘Goals Against’ column.

Total Goals For = 16 + 11 + 8 + 7 + 8 + 4 = 54Total Goals Against = 2 + Wilmslow + 7 + 9 + 12 + 14 = 44 + WilmslowFor both columns to be equal, Wilmslow must have a total of 54 – 44 =

10 Goals Against.Question 297: C

1323

Working with the table it is possible to work out that the BMIs of Julieand Lydia must be 21 and 23, and hence their weights 100 and 115 lbs.us Emma’s weight is 120 lbs, and her BMI must be 22, making her heightequivalent to 160 cm.

Question 298: CWorking through the results, starting with the highest and lowest

values, it is possible to plot all values and decipher which point is marked.Question 299: Dis is a question of estimation. e average production across the

year is at least 7 million barrels per day. Multiplying this by 365 givesaround 2,550 million barrels per year. All other options require less than 7million barrels daily production to be produced, and it is clear there is atleast 7 million barrels per day. erefore the answer is 2,700 million.

Alternatively we can estimate using 30 days per month, andmultiplying the amount of barrels produced per day in each month by 30(this is more accurate but more time consuming).6+7+7+7.5+7.5+7+7.5+8+8.5+8.5+8+9 = 91.5, multiplying by 30 gives justover 2,700 million barrels.

Question 300: CUse both graphs. For July, multiply the oil price by the amount sold in

the month, and multiply by the number of days in the month. us, July =7.5 million barrels x $75 per barrel x 31 days = $17,400 million = $17.4billion

Question 301: ADNA consists of 4 bases: adenine, guanine, thymine and cysteine. e

sugar backbone consists of deoxyribose, hence the name DNA. DNA isfound in the cytoplasm of prokaryotes.

1324

Question 302: FMitochondria are responsible for energy production by ATP synthesis.

Animal cells do not have a cell wall, only a cell membrane. eendoplasmic reticulum is important in protein synthesis, as this is wherethe proteins are assembled.

Question 303: FIf you aren’t studying A-level biology, this question may stretch you.

However, it is possible to reach an answer by process of elimination.Mitochondria are the ‘powerhouse’ of the cell in aerobic respiration,responsible for cell energy production rather than DNA replication orprotein synthesis. As energy producers they are required in muscle cells inlarge numbers, and in sperm cells to drive the tail responsible formovement. ey are enveloped by a double membrane, possibly becausethey started out as independent prokaryotes engulfed by eukaryotic cells.

Question 304: Ae majority of bacteria are commensals and don’t lead to disease.Question 305: CBacteria carry genetic information on plasmids and not in nuclei like

animal cells. ey don’t need meiosis for replication, as they do not requiregametes. Bacterial genomes consist of DNA, just like animal cells.

Question 306: CActive transport requires a transport protein and ATP, as work is being

done against an electrochemical gradient. Unlike diffusion, the relativeconcentrations of the materials being transported aren’t important.

Question 307: DMeiosis produces haploid gametes. is allows for fusion of 2 gametes

to reach a full diploid set of chromosomes again in the zygote.Question 308: BMendelian inheritance separates traits into dominant or recessive. It

applies to all sexually reproducing organisms. Don’t get confused by

1325

statement C – the offspring of 2 heterozygotes has a 25% chance ofexpressing a recessive trait, but it will be homozygous recessive.

Question 309: AHormones are released into the bloodstream and act on receptors in

different organs in order to cause relatively slow changes to the body’sphysiology. Hormones frequently interact with the nervous system, e.g.Adrenaline and Insulin, however, they don’t directly cause muscles tocontract. Almost all hormones are synthesised.

Question 310: DNeuronal signalling can happen via direct electrical stimulation of

nerves or via chemical stimulation of synapses which produces a currentthat travels along the nerves. Electrical synapses are very rare inmammals, the majority of mammalian synapses are chemical.

Question 311: DRemember that pH changes cause changes in electrical charge on

proteins (= polypeptides) that could interfere with protein – proteininteractions. Whilst the other statements are all correct to a certainextent, they are the downstream effects of what would happen if enzymes(which are also proteins) didn’t work.

Question 312: Ae bacterial cell wall is made up of murein and protects the

bacterium from the external environment, in particular from osmoticstresses, and is important in most bacteria.

Question 313: CSexual reproduction relies on formation of gametes during meiosis.

Mitosis doesn’t produce genetically distinct cells. Mitosis is, however, thebasis for tissue growth.

Question 314: AA mutation is a permanent change in the nucleotide sequence of DNA.

Whilst mutations may lead to changes in organelles and chromosomes, or

1326

even be harmful, they are strictly de ned as permanent changes to theDNA or RNA sequence.

Question 315: EMutations are fairly common, but in the vast majority of cases do not

have any impact on phenotype due to the redundancy of the genome.Sometimes they can confer selective advantages and allow organisms tosurvive better (i.e. evolve by natural selection), or they can lead to cancersas cells start dividing uncontrollably.

Question 316: DAntibodies represent a pivotal molecule of the immune system. ey

provide very pointed and selective targeting of pathogens and toxinswithout causing damage to the body’s own cells.

Question 317: AKidneys are not involved in digestion, but do lter the blood of waste

products. Glucose is found in high concentrations in the urine of diabetics,who cannot absorb it without working insulin.

Question 318: DHormones are slower acting than nerves and act for a longer time.

Hormones also act in a more general way. Adrenaline is also a hormonereleased into the body causing the ght-or- ight response. Although it isquick acting, it still lasts for a longer time than a nervous response, as youcan still feel its effects for a time after the response, e.g. shaking hands.

Question 319: DHomeostasis is about minimising changes to the internal environment

by modulating both input and output.Question 320: Bere is less energy and biomass each time you move up a trophic

level. Only 10% of consumed energy is transferred to the next trophic level,so only one tenth of the previous biomass can be sustained in the nexttrophic level up.

1327

Question 321: AIn asexual reproduction, there is no fusion of gametes as the single

parent cell divides. ere is therefore no mixing of chromosomes and, as aresult, no genetic variation.

Question 322: Ee image is rst formed on the retina which conveys it to the brain

via a sensory nerve. e brain then sends an impulse to the muscle via amotor neuron.

Question 323: DBlood from the kidney returns to the heart via the renal (kidney-

related) vein, which drains into the inferior vena cava. e blood thenpasses through the pulmonary vasculature (veins carry blood to the heart,arteries away from the heart) before going into the aorta and eventuallythe hepatic (liver-related) artery.

Question 324: FClones are genetically identical by de nition, and a large number of

them could conceivably reduce the gene pool of a population. In adult cellcloning, the genetic material of an egg is replaced with the geneticmaterial of an adult cell. Cloning is possible for all DNA based life forms,including plants and other types of animals.

Question 325: FGene varieties cause intraspecies variation, e.g. different eye colours. If

mutations confer a selective advantage, those individuals with themutation will survive to reproduce and grow in numbers. Geneticvariation is caused by mixing of parent genomes and mutations. Specieswith similar characteristics often do have similar genes.

Question 326: FAlleles are different versions of the same gene. If you are a homozygous

for a trait, you have two identical alleles for that particular gene, and if youare heterozygous you have two different alleles of that gene. Recessive

1328

traits only appear in the phenotype when there are no dominant alleles forthat trait, i.e. two recessive alleles are carried.

Question 327: DRemember that red blood cells don’t have a nucleus and therefore have

no DNA. In meiosis, a diploid cell divides in such a way so as to producefour haploid cells. Any type of cell division will require energy.

Question 328: Ce hypothalamus detects too little water in the blood, so the pituary

gland releases ADH. e kidney maintains the blood water level, andallows less water to be lost in the urine until the blood water level returnsto normal.

Question 329: EVenous blood has a higher level of carbon dioxide and lower oxygen.

Carbon dioxide forms carbonic acid in aqueous solution, thus making thepH of venous blood slightly more acidic than arterial blood. is leavesonly E and F as possibilities, but releasing pH levels cannot uctuatesigni cantly gives pH 7.4.

Question 330: Ee cytoplasm is 80% water, but also contains, among other things,

electrolytes and proteins. e cytoplasm doesn’t contain everything, e.g.DNA is found in the nucleus.

Question 331: CATP is produced in mitochondria in aerobic respiration and in the

cytoplasm during anaerobic respiration only.Question 332: Ce cell membrane allows both active transport and passive transport

by diffusion of certain ions and molecules, and is found in eukaryotes andprokaryotes like bacteria. It is a phospholipid bilayer.

Question 333: A

1329

1 and 2 only: 223 PAIRS = 446 chromosomes; meiosis produces 4daughter cells with half of the original number of chromosomes each,while mitosis produces two daughter cells with the original number ofchromosomes each.

Question 334: EIf Bob is homozygous dominant (RR) the probability of having a child

with red hair is 0%. However, if Bob is heterozygous (Rr), there is a 50%chance of having a child with red hair, since Mary must be homozygousrecessive (rr) to have red hair. As we do not know Bob’s genotype, bothpossibilities must be considered.

Question 335: AIf an offspring is born with red hair, it con rms Bob is heterozygous

(Rr). He cannot have a red-haired child if he is homozygous dominant(RR), and would himself have red hair were he homozygous recessive (rr).

Question 336: AMonohybrid cross rr and Rr results in 50% Rr and 50% rr offspring. 50%

of offspring will have black hair, but they will be heterozygous for the hairallele.

Question 337: CWhen the chest walls expand, the intra-thoracic pressure decreases.

is causes the atmospheric pressure outside the chest to be greater thanpressure inside the chest, resulting in a ow of air into the chest.

Question 338: AProducers are found at the bottom of food chains and always have the

largest biomass.Question 339: FAll the statements are true; the carbon and nitrogen cycles are

examinable in Section 2, so make sure you understand them! eatmosphere is 79% inert N2 gas, which must be ‘ xed’ to useable forms by

1330

high-energy lightning strikes or by bacterial mediation. Humans alsomanually x nitrogen for fertilisers with the Haber process.

Question 340: HNone of the above statements are correct. Mutations can be silent,

cause a loss of function, or even a gain in function, depending on the exactlocation in the gene and the base affected. Mutations only cause a changein protein structure if the amino acids expressed by the gene affected arechanged. is is normally due to a shift in reading frame. Whilst cancerarises as a result of a series of mutations, very few mutations actually leadto cancer.

Question 341: CRemember that heart rate is controlled via the autonomic nervous

system, which isn’t a part of the central nervous system.Question 342: HNone of the above are correct. ere is no voluntary input to the heart

in the form of a neuronal connection. Parasympathetic neurones slow theheart and sympathetic nervous input accelerates heart rate.

Question 343: BIf lipase is not working, fat from the diet will not be broken down, and

will build up in the stool. Lactase, for instance, is responsible for breakingdown lactose, and its malfunctioning causes lactose-intolerance.

Question 344: FOxygenated blood ows from the lungs to the heart via the pulmonary

vein. e pulmonary artery carries deoxygenated blood from the heart tothe lungs. Animals like sh have single circulatory systems. Deoxygenatedblood is found in the superior vena cava, returning to the heart from thebody. Veins in the arms and hands frequently don’t have valves.

Question 345: EEnzymatic digestion takes place throughout the GI tract, including in

the mouth (e.g. amylase), stomach (e.g. pepsin), and small intestine (e.g.

1331

trypsin). e large intestine is primarily responsible for water absorption,whilst the rectum acts as a temporary store for faecal matter (i.e. digestionhas nished by the rectum).

Question 346: Bis is an example of the monosynaptic stretch re ex; these re exes

are performed at the spinal level and therefore don’t involve the brain.Question 347: AStatement 2 describes diffusion, as CO2 is moving with the

concentration gradient. Statement 3 describes active transport, as aminoacids are moving against the concentration gradient.

Question 348: I3 is the correct equation for animals, and 4 is correct for plants.Question 349: Ce mitochondria are only the site for aerobic respiration, as

anaerobic respiration occurs in the cytoplasm. Aerobic respirationproduces more ATP per substrate than anaerobic respiration, andtherefore is also more efficient. e chemical equation for glucose beingrespired aerobically is: C6H12O6 + 6O2 6CO2 + 6H2O. us, the molar ratiois 1:6 (i.e. each mole glucose produces 6 moles of CO2).

Question 350: Be nucleus contains the DNA and chromosomes of the cell. e

cytoplasm contains enzymes, salts and amino acids in addition to water.e plasma membrane is a bilayer. Lastly, the cell wall is indeedresponsible for protecting vs. increased osmotic pressures.

Question 351: DWhen a medium is hypertonic relative to the cell cytoplasm, it is more

concentrated than the cytoplasm, and when it is hypotonic, it is lessconcentrated. So, when a medium is hypotonic relative to the cellcytoplasm, the cell will gain water through osmosis. When the medium isisotonic, there will be no net movement of water across the cell

1332

membrane. Lastly, when the medium is hypertonic relative to the cellcytoplasm, the cell will lose water by osmosis.

Question 352: AStem cells have the ability to differentiate and produce other kinds of

cells. However, they also have the ability to generate cells of their own kindand stem cells are able to maintain their undifferentiated state. e twotypes of stem cells are embryonic stem cells and adult stem cells. e adultstem cells are present in both children and adults.

Question 353: BAll of the following statements are examples of natural selection,

except for the breeding of horses. Breeding and animal husbandry arenotable methods of arti cial selection, which are brought about byhumans.

Question 354: CEnzymes create a stable environment to stabilise the transition state.

Enzymes do not distort substrates. Enzymes generally have little effect ontemperature directly. Lastly, they are able to provide alternative pathwaysfor reactions to occur.

Question 355: CA negative feedback system seeks to minimise changes in a system by

modulating the response in accordance with the error that’s generated.Salivating before a meal is an example of a feed-forward system (i.e.salivating is an anticipatory response). rowing a dart does not involveany feedback (during the action). pH and blood pressure are bothimportant homeostatic variables that are controlled via powerful negativefeedback mechanisms, e.g. massive haemorrhage leads to compensatorytachycardia.

Question 356: AOne of the major functions of white blood cells is to defend the body

against bacterial and fungal infections. ey can kill pathogens by

1333

engul ng them and also use antibodies to help them recognise pathogens.Antibodies are produced by white blood cells.

Question 357: Be CV system does indeed transport nutrients and hormones. It also

increases blood ow to exercising muscles (via differential vasodilatation)and also helps with thermoregulation (e.g. vasoconstriction in response tocold). e respiratory system is responsible for oxygenating blood.

Question 358: CAdrenaline always increases heart rate and is almost always released

during sympathetic responses. It travels primarily in the blood and affectsmultiple organ systems. It is also a potent vasoconstrictor.

Question 359: BProtein synthesis occurs in the cytoplasm. Proteins are usually coded

by several amino acids. Red blood cells lack a nucleus and, therefore, theDNA to create new proteins. Protein synthesis is a key part of mitosis, as itallows the parent cell to grow prior to division.

Question 360: FRemember that most enzymes work better in neutral environments

(amylase works even better at slightly alkaline pH). us, adding sodiumbicarbonate will increase the pH and hence increase the rate of activity.Adding carbohydrate will have no effect, as the enzyme is alreadysaturated. Adding amylase will increase the amount of carbohydrate thatcan be converted per unit time. Increasing the temperature to 100o C willdenature the enzyme and reduce the rate.

Question 361: ETaking the normal allele to be C and the diseased allele to be c, one can

model the scenario with the following Punnett square:

1334

e gender of the children is irrelevant as the inheritance is autosomalrecessive, but we see that all children produced would inherit at least onediseased allele.

Question 362: FAll of the organs listed have endocrine functions. e thyroid produces

thyroid hormone. e ovary produces oestrogen. e pancreas secretesglucagon and insulin. e adrenal gland secretes adrenaline. e testesproduce testosterone.

Question 363: AInsulin works to decrease blood glucose levels. Glucagon causes blood

glucose levels to increase; glycogen is a carbohydrate. Adrenaline works toincrease heart rate.

Question 364: Ae left side of the heart contains oxygenated blood from the lungs

which will be pumped to the body. e right side of the heart containsdeoxygenated blood from the body to be pumped to the lungs.

Question 365: ASince Individual 1 is homozygous normal, and individual 5 is

heterozygous and affected, the disease must be dominant. Since malesonly have one X-chromosome, they cannot be carriers for X-linkedconditions. If Nafram syndrome was X-linked, then parents 5 and 6 wouldproduce sons who always have no disease and daughters that always do.As this is not the case shown in individuals 7-10, the disease must beautosomal dominant.

Question 366: C

1335

We know that the inheritance of Nafram syndrome is autosomaldominant, so using N to mean a diseased allele and n to mean a normalallele, 5, 7 and 8 must be Nn because they have an unaffected parent. 2 isalso Nn, as if it was NN all its progeny would be Nn and so affected by thedisease, which is not the case, as 3 and 4 are unaffected.

Question 367: ASince 6 is disease free, his genotype must be nn. us, neither of 6’s

parents could be NN, as otherwise 6 would have at least one diseasedallele.

Question 368: AUrine passes from the kidney into the ureter and is then stored in the

bladder. It is nally released through the urethra.Question 369: FDeoxygenated blood from the body ows through the inferior vena

cava to the right atrium where it ows to the right ventricle to be pumpedvia the pulmonary artery to the lungs where it is oxygenated. It thenreturns to the heart via the pulmonary vein into the left atrium into theleft ventricle where it is pumped to the body via the aorta.

Question 370: EDuring inspiration, the pressure in the lungs decreases as the

diaphragm contracts, increasing the volume of the lungs. e intercostalmuscles contract in inspiration, lifting the rib cage.

Question 371: DWhilst A, B, C and E are true of the DNA code, they do not represent

the property described, which is that more than one combination ofcodons can encode the same amino acid, e.g. Serine is coded by thesequences: TCT, TCC, TCA, TCG.

Question 372: Be degenerate nature of the code can help to reduce the deleterious

effects of point mutations. e several 3-nucleotide combinations that

1336

code for each amino acid are usually similar such that a point mutation,i.e. a substitution of one nucleotide for another, can still result in the sameamino acid as the one coded for by the original sequence.

e degenerate nature of the code does little to protect againstdeletions/insertions/duplications, which will cause the bases to be read inincorrect triplets, i.e. result in a frame shift.

Question 373: De hypothalamus is the site of central thermoreceptors. A decrease in

environmental temperature decreases sweat secretion and causescutaneous vasoconstriction to minimise heat loss from the blood.

Question 374: Ae movement of carbon dioxide in the lungs and neurotransmitters

in a synapse are both examples of diffusion. Glucose reabsorption is anactive process, as it requires work to be done against a concentrationgradient.

Question 375: FSome enzymes contain other molecules besides protein, e.g. metal

ions. Enzymes can increase rates of reaction that may result in heatgain/loss, depending on if the reaction is exothermic or endothermic. eyare prone to variations in pH and are highly speci c to their individualsubstrate.

Question 376: DDifferent isotopes are differentiated by the number of neutrons in the

core. is gives them different molecular weights and different chemicalproperties with regards to stability. e number of protons de nes eachelement, and the number of electrons its charge.

Question 377: EA displacement reaction occurs when a more reactive element

displaces a less reactive element in its compound. Reaction 4 will nothappen as lead is less reactive than sodium

1337

Question 378: Aere needs to be 3Ca, 12H, 14O and 2P on each side. Only option A

satis es this.Question 379: ATo balance the equation there needs to be 9Ag, 9N, 9O3, 9K, 3P on each

side. Only option A satis es this.Question 380: DA more reactive halogen can displace a less reactive halogen. us,

chlorine can displace bromine and iodine from an aqueous solution of itssalts, and uorine can replace chlorine. e trend is the opposite for alkalimetals, where reactivity increases down the group as electrons are furtherfrom the core and easier to lose.

Question 381: C2Mg + O2 = 2MgOso, 2 x 24 = 48 and 2 x (24 + 16) = 80so, 48 g of magnesium produces 80g of magnesium oxideso 1g of magnesium produces 1g x 80g/48g = 1.666g oxideso 75g x 1.666 = 125gQuestion 382: BH2 + 2OH- 2H2O + e-

us, the hydrogen loses electrons i.e. is oxidised.Question 383: FAmmonia is 1 nitrogen and 3 hydrogen atoms bonded covalently. N =

14g and H = 1g per mole, so percentage of N in NH3 = 14g/17g = 82%. It canbe produced from N2 through xation or the industrial Haber process foruse in fertiliser, and may break down to its components.

Question 384: AMilk is weakly acidic, pH 6.5-7.0, and contains fat. is is broken down

by lipase to form fatty acids - turning the solution slightly more acidic.Question 385: C

1338

Glucose loses four hydrogen atoms; one de nition of an oxidationreaction is a reaction in which there is loss of hydrogen.

Question 386: CIsotopes have the same number of protons and electrons, but a

different number of neutrons. e number of neutrons has no impact onthe rate of reactions.

Question 387: EMg + H2SO4 → MgSO4 + H2

Number of moles of Mg = moles.1 mole of Mg reacts with 1 mole H2SO4 to produce 1 mole of

magnesium sulphate. erefore, 0.25 moles H2SO4 will react to produce0.25 moles of MgSO4.

Mr of H2SO4 = 2 + 32 + 64 = 98g per molee mass of H2SO4 used = 0.25 moles x 98g per mole = 24.5g.Since 30g of H2SO4 is present, H2SO4 is in excess and the magnesium is

the limiting reagent.Mr of MgSO4 = 24 + 32 + 64 = 120g per molee mass of MgSO4 produced = 0.25 moles x 120g per mole = 30g which

is the same mass as that of sulphuric acid in the original reaction.Question 388: FReactivity series of metals:Cu is more reactive than Ag and will displace it.Ca is more reactive than H and will displace it.2 and 4 are incorrect because Fe is higher in the reactivity series than

Cu and Fe is lower in the reactivity series than Ca, so no displacement willoccur.

Question 389: GMoving left to right is the equivalent of moving down the metal

reactivity series (i.e. Na is most reactive and Zn is least reactive). erefore,moving from left to right, the reactivity of the metals decreases, likelihood

1339

of corrosion decreases, less energy is required to separate metals fromtheir ores and metals lose electrons less readily to form positive ions.

Question 390: FHalogens become less reactive as you progress down group 17. us in

order of increasing reactivity from left to right: I→ Br→ Cl. erefore, I willnot displace Br, Cl will displace Br and Br will displace I.

Question 391: AWires are made out of copper because it is a good conductor of

electricity. Copper is also used in coins (not aluminium). Aluminium isresistant to corrosion but because of a layer of aluminium oxide (nothydroxide).

Question 392: C2Li + 2H2O → 2LiOH + H2

erefore, 2 moles of Li react to produce 1 mole of H2 gas (24 dm3).e number of moles of Li = moles.us, 1.5 moles of H2 gas are produced = 36 dm3.Question 393: BMgCl2 contains stronger bonds than NaCl because Mg ions have a 2+

charge, thus having a stronger electrostatic pull for negative chloride ions.e smaller atomic radius also means that the nucleus has less distancebetween it and incoming electrons. Transition metals are able to formmultiple stable ions e.g. Fe2+ and Fe3+.

Covalently bonded structures do tend to have lower MPs than ionicallybonded, but the giant covalent structures (diamond and graphite forexample) have very high melting points. Graphite is an example of acovalently bonded structure which conducts electricity.

Question 394: DEnergy is released from reaction A, as shown by a negative enthalpy.

e reaction is therefore exothermic. Since energy is released, the productCO2 has less energy than the reactants did. erefore, CO2 is more stable.

1340

Reaction B has a positive enthalpy, which means energy must be put intothe reaction for it to occur i.e. it’s an endothermic reaction. at meansthat the products (CaO and CO2) have more energy and are less stablethan the reactants (CaCO3).

Question 395: BSolid oxides are unable to conduct electricity because the ions are

immobile. Metals are extracted from their molten ores by electrolysis.Fractional distillation is used to separate miscible liquids with similarboiling points. Mg2+ ions have a greater positive charge and a smaller ionicradius than Na+ ions, and therefore have stronger bonds.

Question 396: ELi+ (2) and Na+ (2, 8)Mg2+ (2, 8) and Ne (2, 8)Na2+ (2, 7) and Ne (2, 8)O2- (2, 4) and a Carbon atom (2, 4)Question 397: BReactivity of both group 1 and 2 increases as you go down the groups

because the valence electrons that react are further away from thepositively charged nucleus (which means the electrostatic attractionbetween them is weaker). Group 1 metals are usually more reactivebecause they only need to donate one electron, whilst group 2 metals mustdonate two electrons.

Question 398: Dis is a straightforward question that tests basic understanding of

kinetics. Catalysts help overcome energy barriers by reducing theactivation energy necessary for a reaction.

Question 399: DH1 contains 1 proton and no neutrons. Isotopes have the same

numbers of protons, but different numbers of neutrons. us, H3 containstwo more neutrons than H1.

1341

Question 400: DOxidation is the loss of electrons and reduction is the gain of electrons

(therefore increasing electron density). Halogens tend to act as electronrecipients in reactions and are therefore good oxidising agents.

Question 401: Dese statements all come from the Kinetic eory of Gases, an

idealised model of gases that allows for the derivation of the ideal gas law.e angle at which gas molecules move is not related to temperature;movement is random. Gas molecules lose no energy when they collidewith each other, collisions are assumed elastic. e average kinetic energyof gas molecules is the same for all gases at the same temperature as theyare assumed to be point masses. Momentum = mass x velocity. erefore,the momentum of gas molecules increases with pressure as a greater forceis exerted on each molecule.

Question 402: EAn exothermic reaction is de ned as a chemical reaction that releases

energy. us, aerobic respiration producing life energy, the burning ofmagnesium, and the reacting of acids/bases are almost always exothermicprocesses. Similarly, the combustion of most things (including hydrogen)is exothermic. Evaporation of water is a physical process in which nochemical reaction is taking place.

Question 403: E2 C3H6 + 9 O2 6 H2O + 6 CO2

Assign the oxidation numbers for each element:For C3H6: C = -2; H = +1For O2: O = 0For H2O: H = +1; O = -2For CO2: C = +4; O = -2Look for the changes in the oxidation numbers:H remained at +1

1342

C changed from -2 to +4. us, it was oxidizedO changed from 0 to -2. us, it was reduced.Question 404: Be equation for the reaction is: Zn + CuSO4 ZnSO4 + CuAssign oxidation numbers for each element:For Zn: Zn = 0For CuSO4: Cu = +2; S = +6; O = -2For ZnSO4: Zn = +2; S = +6; O = -2For Cu: Cu = 0With these oxidation numbers, we can see that Zn was oxidized and

Cu in CuSO4 was reduced. us, Zn acted as the reducing agent and Cu inCuSO4 is the oxidizing agent.

Question 405: BAcids are proton donors which only exist in aqueous solution, which is

a liquid state. Strong acids are fully ionised in solution and the reactionbetween an acid and a base salt + water.

e pH of weak acids is usually between 4 and 6.Question 406: GLet x be the relative abundance of Z6 and y the relative abundance of

Z8.e average atomic mass takes the abundances of all 3 isotopes into

account.us, (Abundance of Z5)(Mass Z5) + (Abundance of Z6)(Mass Z6) +

(Abundance of Z8)(Mass Z8) = 7erefore: (5 x 0.2) + 6x + 8y = 7So: 6x + 8y = 6Divide by two to give: 3x + 4y = 3e abundances of all isotopes = 100% = 1is gives: 0.2 + x + y = 1Solve the two equations simultaneously:

1343

y = 0.8 – x3x + 4(0.8 – x) = 33x + 3.2 – 4x = 3erefore, x = 0.2y = 0.8 - 0.2 = 0.6us, the overall abundances are Z5 = 20%, Z6 = 20% and Z8 = 60%.

erefore, all the statements are correct.Question 407: AIf a metal is more reactive than hydrogen, a displacement reaction will

occur resulting in the formation of a salt with the metal cation andhydrogen.

Question 408: B6 FeSO4 + K2Cr2O7 + 7 H2SO4 3 (Fe)2(SO4)3 + Cr2(SO4)3 + K2SO4 + 7 H2OIn order to save time, you have to quickly eliminate options (rather

than try every combination out).e quickest way is to do this is algebraically:For Potassium:2b = 2e = 2ferefore, b = f.Option F does not ful l b = e = f.For Iron:a = 2dOptions C, D and E don’t ful l a = 2d.For Hydrogen:2c = 2gerefore, c = g.Option A does not ful l c = g.is leaves option B as the answer.Question 409: E

1344

Atoms are electrically neutral. Ions have different numbers ofelectrons when compared to atoms of the same element. Protons providejust under 50% of an atom’s mass, the other 50% is provided by neutrons.Isotopes don’t exhibit signi cantly different kinetics. Protons do indeedrepel each other in the nucleus (which is one reason why neutrons areneeded: to reduce the electrical charge density).

Question 410: Be noble gasses are extremely useful, e.g. helium in blimps, neon

signs, argon in bulbs. ey are colourless and odourless and have novalence electrons. As with the rest of the periodic table, boiling pointincreases as you progress down the group (because of increased Van derWaals forces). Helium is the most abundant noble gas (and indeed the 2nd

most abundant element in the universe).Question 411: DAlkenes can be hydrogenated (i.e. reduced) to alkanes. Aromatic

compounds are commonly written as cyclic alkenes, but their propertiesdiffer from those of alkenes. erefore alkenes and aromatic compoundsdo not belong to the same chemical class.

Question 412: Ae average atomic mass takes the abundances of both isotopes into

account:(Abundance of Cl35)(Mass Cl35) + (Abundance of Cl37)(Mass Cl37) =

35.45334.969(Abundance of Cl35) + 36.966(Abundance of Cl37) = 35.453e abundances of both isotopes = 100% = 1I.e. abundance of Cl35 + abundance of Cl37 = 1erefore: x + y = 1 which can be rearranged to give: y = 1-xerefore: x + (1 – x) = 1.34.969x + 36.966(1-x) = 35.453x = 0.758

1345

1 - x = 0.242erefore, Cl35is 3 times more abundant than Cl37.Note that you could approximate the values here to arrive at the

solution even quicker, e.g. 34.969 35, 36.966 37 and 35.453 35.5Question 413: ATransition metals form multiple stable ions which may have many

different colours (e.g. green Fe2+ and brown Fe3+). ey usually form ionicbonds and are commonly used as catalysts (e.g. iron in the Haber process,Nickel in alkene hydrogenation). ey are excellent conductors ofelectricity and are known as the d-block elements.

Question 414: B2Na + 2H2O 2NaOH + H2

8000 cm3 = 8 dm3 = ⅓ moles of H2

2 moles of Na react completely to form 1 mole of H2.erefore, ⅔ moles of Na must have reacted to produce ⅓ moles of

Hydrogen. ⅔ x 23g per mole = 15.3g.% Purity of sample = x 100 = 76.5%Question 415: C

Assume total mass of molecule is 100g. erefore, it contains 70.6gcarbon, 5.9g hydrogen and 23.5g oxygen. Now, calculate the number ofmoles of each element using

1346

erefore, the molar ratios give an empirical formula of C6H6O1.5 =C4H4O.

Molar mass of the empirical formula = (4 x 12) + (4 x 1) + 16 = 68.Molar mass of chemical formula = 136. erefore, the chemical

formula = C8H8O2.Question 416: BS + 6 HNO3 H2SO4 + 6 NO2 + 2 H2OIn order to save time, you have to quickly eliminate options (rather

than try every combination out).e quickest way to do this is algebraically:For Hydrogen:b = 2c + 2eOptions A, C, D, E and F don’t ful l b = 2c + 2e.is leaves options B as the only possible answer.Note how quickly we were able to get the correct answer here by

choosing an element that appears in 3 molecules (as opposed to Sulphuror Nitrogen which only appear in 2).

Question 417: AAlkenes undergo addition reactions, such as that with hydrogen, when

catalysed by nickel, whilst alkanes do not as they are already fullysaturated. e C=C bond is stronger than the C-C bond, but it is notexactly twice as strong, so will not require twice the energy to break it.Both molecules are organic and will dissolve in organic solvents.

Question 418: FDiamond is unable to conduct electricity because all the electrons are

involved in covalent bonds. Graphite is insoluble in water + organicsolvents. Graphite is also able to conduct electricity because there are freeelectrons that are not involved in covalent bonds.

1347

Methane and Ammonia both have low melting points. Methane is nota polar molecule, so cannot conduct electricity or dissolve in water.Ammonia is polar and will dissolve in water. It can conduct electricity inaqueous form, but not as a gas.

Question 419: ACatalysts increase the rate of reaction by providing an alternative

reaction path with a lower activation energy, which means that less energyis required and so costs are reduced. e point of equilibrium, the natureof the products, and the overall energy change are unaffected by catalysts.

Question 420: Ee 5 carbon atoms in this hydrocarbon make it a “pent” stem. e

C=C bond makes it an alkene, and the location of this bond is the 2ndposition, making the molecule pent-2-ene.

Question 421: FGroup 1 elements form positively charged ions in most reactions and

therefore lose electrons. us, the oxidation number must increase. eirreactivity increases as the valence electrons are further away from thepositively charged nucleus down group. ey all react vigorously withwater, but only the latter half of Group 1 elements react spontaneouslywith oxygen.

Question 422: He cathode attracts positively charged ions. e cathode reduces ions

and the anode oxidises ions. Electrolysis can be used to separatecompounds but not mixtures (i.e. substances that are not chemicallyjoined).

Question 423: BPentane, C5H12, has a total of 3 isomers. A, C and D are correctly

con gured. However, the 4th Carbon atom in option B has more than 4bonds which wouldn’t be possible. If you’re stuck on this – draw them out!

Question 424: E

1348

3 Cu + 8 HNO3 3 Cu(NO3)2 + 2 NO + 4 H2OIn order to save time, you have to quickly eliminate options (rather

than try every combination out).e quickest way to do this is algebraically, by rst assigning

coefficients to the equation:aCu + bHNO3 cCu(NO3)2 + dNO + eH2OFor Nitrogen: b = 2c + d.In this case, only option E satis es b = 2c + d.Note that using copper wouldn’t be as useful, as all the options satisfy

a = c.Question 425: DAlkenes are an organic series and have twice as many hydrogen atoms

as carbon atoms. Bromine water is decolourised in their presence and theytake part in addition reactions. Alkenes are more reactive than alkanesbecause they contain a C=C bond.

Question 426: AGroup 17 elements are missing one valence electron, so form negative

ions. Bromine is a liquid at room temperature, and is also coloured brown.Reactivity decreases as you progress down Group 17, so uorine reactsmore vigorously than iodine. All Group 17 elements are found bound toeach other, e.g. F2 and Cl2.

Question 427: DCO poisoning and spontaneous combustion do not occur in the

electrolysis of brine. e products of cathode and anode in the electrolysisof brine are Cl2 and H2. If these two gases react with each other they canform HCl, which is extremely corrosive.

Question 428: De hydrogen produced is positively charged and therefore needs to be

reduced by the addition of an electron before being released. is happensat the cathode. e chlorine produced is negatively charged and therefore

1349

needs to lose electrons. is happens at the anode. NaOH is formed in thisprocess.

Question 429: CAlkanes are made of chains of singly bonded carbon and hydrogen

atoms. C-H bonds are very strong and confer alkanes a great deal ofstability. An alkane with 14 hydrogen atoms is called Hexane, as it has 6carbon atoms. Alkanes burn in excess oxygen to produce carbon dioxideand water. Bromine water is decolourised in the presence of alkenes.

Question 430: GYou’ve probably got a lot of experience of organic chemistry by now, so

this should be fairly straightforward. Alcohols by de nition contain an R-OH functional group and because of this polar group are highly soluble inwater. Ethanol is a common biofuel.

Question 431: EAlkanes are saturated (and therefore non-reducible), have the general

formula CnH2n+2 and have no effect on Bromine solution. Alkenes areunsaturated (and therefore reducible), have the general formula CnH2n andturn bromine water colourless because they can undergo an additionreaction with bromine.

Question 432: De balanced equation for the reaction between magnesium oxide and

hydrochloric acid is:MgO + 2HCl → MgCl2 +H2

e relative molecular mass of MgO is 24 + 16 = 40g per mole.erefore 10g of MgO represents 10/40 = 0.25 moles.As the ratio of MgO to MgCl2 is 1:1, we know that the amount of MgCl2

produced will also be 0.25 moles. One mole of MgCl2 has a molecular massof 24 + (2 x 35.5) = 95g per mole.

erefore the reaction will produce 0.25 x 95 = 23.75g of MgCl2.Question 433: D

1350

Moving up the alkane series, as size and mass of the moleculeincreases (and thus the Van der Waals forces increase), the boiling pointand viscosity increase and the ammability and volatility decrease.erefore pentadecane will be more viscous than pentane.

Question 434: FAll of the factors mentioned will affect the rate of a reaction. e

temperature affects the movement rate of particles, which if moving fasterin higher temperatures will collide more often, thus increasing the rate ofreaction. Collision rate is also increased with a higher concentration ofreactants, and with a higher concentration of a catalyst or one with largersurface area, which will provide more active sites, thus increasing the rateof reaction.

Question 435: Ce total atomic mass of the end product is C[12 + (2 x 16)] + D[(2x1)

+16] = 44C + 18DWe know that 176 = 44C. erefore C = 4, and that 108 = 18D so D = 6.us, the equation becomes: CaHb + O2 → 4CO2 + 6H2O.is gives a ratio of 4C to 12H, which is a ratio of 1:3 carbon to

hydrogen. is means the unknown hydrocarbon must be a multiple ofthis ratio. By balancing the equation we can see that the unknownhydrocarbon must be ethane, C2H6: 2C2H6 + 7O2 → 4CO2 + 6H2O.

Question 436: AC2H5OH C2H4O. us, ethanol has lost two hydrogen atoms, i.e. has

been oxidised. Note that although another substrate may be reduced(therefore making it a redox reaction), ethanol has only been oxidised.

Question 437: Bis is fairly straightforward but you can save time by doing it

algebraically:For Barium: 3a = bFor Nitrogen: 2a = c

1351

Let a = 1, thus, b = 3 and c = 2Question 438: Eere are 14 oxygen atoms on the left side. us: 3b + 2c = 14.Note also that for Sulphur: a = c, and for Iron: a = 2b.is sets up an easy trio of simultaneous equations:Substitute a into the rst equation to give: 1.5a + 2a = 14. us: a =

14/3.5 = 4.erefore, a = c = 4 and b = 2Question 439: Ce average atomic mass takes the abundances of all isotopes into

account:Mass = (Abundance of Mg23)(Mass Mg23) + (Abundance of Mg25)(Mass

Mg25) + (Abundance of Mg26)(Mass Mg26)

Question 440: DCl2 and Fe2O3 are reduced in their reactions and are therefore

oxidising agents. Similarly, CO and Cu2+ are oxidised in their reactions andare therefore reducing agents. Cl is a stronger oxidising agent than Br as itis higher up in the reactivity series, and will displace negative Br ions fromits compounds to form the oxidised Br2. Mg is a stronger reducing agentthan Cu, as it is higher up in the reactivity series. us, Mg would displacea positive copper ion from its compound to form copper atoms. ereforeMg reduces Cu.

Question 441: CNaCl is an ionic compound and therefore has a high melting point. It is

highly soluble in water but only conducts electricity in solution/as a liquid.Question 442: C

1352

e equation for the reaction is: 2NaOH + Zn(NO3)2 → 2NaNO3 +Zn(OH)2

erefore, the molar ratio between NaOH and Zn(OH)2 is 2:1.Molecular Mass of NaOH = 23 + 16 + 1 = 40Molecular Mass of Zn(OH)2= 65 + 17 x 2 = 99us, the number of moles of NaOH that react = 80/40 = 2 moles.erefore, 1 mole of Zn(OH)2 is produced. Mass = 99g per mole x 1

mole = 99gQuestion 443: AMetal + Water → Hydroxide + Hydrogen gas; the reaction is always

exothermic. Reactivity increases down the group, so potassium reactsmore vigorously with water than sodium.

Question 444: CElectrolysis separates NaCl into sodium and chloride ions but not CO2

(which is a covalently bound gas). Sieves cannot separate ionically boundcompounds like NaCl. Dyes are miscible liquids and can be separated bychromatography. Oil and water are immiscible liquids, so a separatingfunnel is necessary to separate the mixtures. Methane and diesel areseparated from each other during fractional distillation, as they havedifferent boiling points.

Question 445: Be reaction between water and caesium can cause spontaneous

combustion and therefore doesn’t make the reaction safer. e reactionbetween caesium and uoride is highly exothermic and does not require acatalyst. e reaction produces CsF which is a salt.

Question 446: Be nucleus of larger elements contain more neutrons than protons to

reduce the charge density, e.g. Br80 contains 35 protons but 45 neutrons.Stable isotopes very rarely undergo radioactive decay.

Question 447: B

1353

e vast majority of salts contain ionic bonds that require a signi cantamount of heat energy to break.

Question 448: E306ml of water is 306g, which is the equivalent of 306g/18g per mole of

H2O = 17 moles. 17 times Avogadro’s constant gives the number ofmolecules present, which is 1.02 x 1025. ere are 10 protons and 10electrons in each water molecule. Hence there are 1.02 x 1026 protons.

Question 449: De number of moles of each element = Mass/Molar Mass. Let the %

represent the mass in grams: Hydrogen: 3.45g/1g per mole = 3.45 molesOxygen: 55.2g/16g per mole = 3.45 molesCarbon: 41.4g/12g per mole = 3.45 molesus, the molar ratio is 1:1:1. e only option that satis es this is

option D.Question 450: CGroup 17 elements are non-metals, whilst group 2 elements are

metals. us, the Group 17 element must gain electrons when it reactswith the Group 2 element, i.e. B is reduced. e easy way to calculate theformula is to swap the valences of both elements: A is +2 and B is -1. us,the compound is AB2.

Question 451: Fat the amplitude of a wave determines its mass is false. Waves are

not objects and do not have mass.Question 452: AWe know that displacement s = 30 m, initial speed u = 0 ms-1,

acceleration a = 5.4 ms-2, nal speed v = ?, time t = ?And that v2 = u2 + 2asv2 = 0 + 2 x 5.4 x 30v2 = 324 so v = 18 ms-1

and s = ut + 1/2 at2 so 30 = 1/2 x 5.4 x t2

1354

t2 = 30/2.7 so t = 3.3 sQuestion 453: De wavelength is given by: velocity v = λf and frequency f = 1/T so v =

λ/T giving wavelength λ = vTe period T = 49 s/7 so λ = 5 ms-1 x 7 s = 35 mQuestion 454: Eis is a straightforward question as you only have to put the numbers

into the equation (made harder by the numbers being hard to work with).

Question 455: Gv = u + atv = 0 + 5.6 x 8 = 44.8 ms-1

AndQuestion 456: Ce sky diver leaves the plane and will accelerate until the air

resistance equals their weight – this is their terminal velocity. e skydiver will accelerate under the force of gravity. If the air resistance forceexceeded the force of gravity the sky diver would accelerate away from theground, and if it was less than the force of gravity they would continue toaccelerate toward the ground.

Question 457: Ds = 20 m, u = 0 ms-1, a = 10 ms-2

and v2 = u2 + 2asv2 = 0 + 2 x 10 x 20

1355

v2 = 400; v = 20 ms-1

Momentum = Mass x velocity = 20 x 0.1 = 2 kgms-1

Question 458: EElectromagnetic waves have varying wavelengths and frequencies and

their energy is proportional to their frequency.Question 459: De total resistance = R + r = 0.8 + 1 = 1.8 Ω

andQuestion 460: CUse Newton’s second law and remember to work in SI units:

So

Question 461: FIn this case, the work being done is moving the bag 0.7 m

i.e.

= 117 W to 3 signi cant guresQuestion 462: B

1356

Firstly, use P = Fv to calculate the power [Ignore the frictional force aswe are not concerned with the resultant force here].

So P = 300 x 30 = 9000 Wen, use P = IV to calculate the current.I = P/V = 9000/200 = 45 AQuestion 463: CWork is de ned as W = F x s. Work can also be de ned as work = force

x distance moved in the direction of force. Work is measured in joules and1 Joule = 1 Newton x 1 Metre, and 1 Newton = 1 Kg x ms-2 [F = ma].

us, 1 Joule = Kgm2s-2

Question 464: GJoules are the unit of energy (and also Work = Force x Distance). us,

1 Joule = 1 N x 1 m.Pa is the unit of Pressure (= Force/Area). us, Pa = N x m-2. So J = Nm-2

x m3 = Pa x m3. Newton’s third law describes that every action produces anequal and opposite reaction. For this reason, the energy required todecelerate a body is equal to the amount of energy it possess duringmovement, i.e. its kinetic energy, which is de ned as in statement 1.

Question 465: DAlpha radiation is of the lower energy, as it represents the movement

of a fairly large particle consisting of 2 neutrons and 2 protons. Betaradiation consists of high-energy, high-speed electrons or positrons.

Question 466: Ee half-life does depend on atom type and isotope, as these

parameters signi cantly impact on the physical properties of the atom ingeneral, so statement 1 is false. Statement 2 is the correct de nition ofhalf-life. Statement 3 is also correct: half-life in exponential decay willalways have the same duration, independent of the quantity of the matterin question; in non-exponential decay, half-life is dependent on thequantity of matter in question.

1357

Question 467: AIn contrast to nuclear ssion, where neutrons are shot at unstable

atoms, nuclear fusion is based on the high speed, high-temperaturecollision of molecules, most commonly hydrogen, to form a new, stableatom while releasing energy.

Question 468: ENuclear ssion releases a signi cant amount of energy, which is the

basis of many nuclear weapons. Shooting neutrons at unstable atomsdestabilises the nuclei which in turn leads to a chain reaction and ssion.Nuclear ssion can lead to the release of ionizing gamma radiation.

Question 469: Ge total resistance of the circuit would be twice the resistance of one

resistor and proportional to the voltage, as given by Ohm’s Law. Since it isa series circuit, the same current ows through each resistor and sincethey are identical the potential difference across each resistor will be thesame.

Question 470: Ee distance between Earth and Sun = Time x Speed = 60 x 8 seconds x

3 x 108 ms-1 = 480 x 3 x 108 mApproximately = 1500 x 108 = 1.5 x 1011 m.e circumference of Earth’s orbit around the sun is given by 2πr = 2 x

3 x 1.5 x 1011

= 9 x 1011 = 1012 mQuestion 471: HSpeed is a scalar quantity whilst velocity is a vector describing both

magnitude and direction. Speed describes the distance a moving objectcovers over time (i.e. speed = distance/time), whereas velocity describesthe rate of change of the displacement of an object (i.e. velocity =displacement/time). e internationally standardised unit for speed ismeters per second (ms-1), while ms-2 is the unit of acceleration.

1358

Question 472: EOhm’s Law only applies to conductors and can be mathematically

expressed as. e easiest way to do this is to write down the equations forstatements c, d and e. C: ; D: E: . us, statement E is correct.

Question 473: G

Any object at rest is not accelerating and therefore has no resultantforce. Strictly speaking, Newton’s second law is actually: Force = rate ofchange of momentum, which can be mathematically manipulated to givestatement 2:

Question 474: D

Statement 3 is incorrect, as. Statement 1 substitutes and statement 2substitutes.

Question 475: EWeight of elevator + people = mg = 10 x (1600 + 200) = 18,000 NApplying Newton’s second law of motion on the car gives:us, the resultant force is given by:FM = Motor Force – [Frictional Force + Weight]FM = M – 4,000 – 18,000Use Newton’s second law to give: FM = M – 22,000 N = maus, M – 22,000 N = 1,800a

1359

Since the lift must accelerate at 1ms-2: M = 1,800 kg x 1 ms-2 + 22,000 NM = 23,800 NQuestion 476: DTotal Distance = Distance during acceleration phase + Distance during

braking phase

Distance during acceleration phase is given by:

And use to calculate the deceleration:

Distance during the deceleration phase is given by:

Question 477: GIt is not possible to calculate the power of the heater as we don’t know

the current that ows through it or its internal resistance. e 8 ohmsrefers to the external copper wire and not the heater. Whilst it’s importantthat you know how to use equations like P = IV, it’s more important thatyou know when you can’t use them!

Question 478: F

1360

is question has a lot of numbers but not any information on time,which is necessary to calculate power. You cannot calculate power byusing P= IV as you don’t know how many electrons are acceleratedthrough the potential difference per unit time. us, more information isrequired to calculate the power.

Question 479: BWhen an object is in equilibrium with its surroundings, it radiates and

absorbs energy at the same rate and so its temperature remains constanti.e. there is no net energy transfer. Radiation is slower than conduction andconvection.

Question 480: A

e work done by the force is given by:

Since the surface is frictionless,.

us,Question 481: C

1361

Question 482: Ge large amount of energy released during atomic ssion is the basis

underlying nuclear power plants. Splitting an atom into two or more partswill by de nition produce molecules of different sizes than the originalatom; therefore it produces two new atoms. e free neutrons andphotons produced by the splitting of atoms form the basis of the energyrelease.

Question 483: DGravitational potential energy is just an extension of the equation

work done = force x distance ( force is the weight of the object, mg, anddistance is the height, h). e reservoir in statement 3 would have apotential energy of 1010 Joules i.e. 10 Giga Joules (Ep= 106 kg x 10 N x 103

m).Question 484: DStatement 1 is the common formulation of Newton’s third law.

Statement 2 presents a consequence of the application of Newton’s thirdlaw.

Statement 3 is false: rockets can still accelerate because the productsof burning fuel are ejected in the opposite direction from which the rocketneeds to accelerate.

Question 485: E

Positively charged objects have lost electrons.Objects can become charged by friction as electrons are transferred

from one object to the other.Question 486: BEach body of mass exerts a gravitational force on another body with

mass. is is true for all planets as well.

1362

Gravitational force is dependent on the mass of both objects. Satellitesstay in orbit due to centripetal force that acts tangentially to gravity (notbecause of the thrust from their engines). Two objects will only land at thesame time if they also have the same shape or they are in a vaccum (asotherwise air resistance would result in different terminal velocities).

Question 487: AMetals conduct electrical charge easily and provide little resistance to

the ow of electrons. Charge can also ow in several directions. However,all conductors have an internal resistance and therefore provide someresistance to electrical charge.

Question 488: EFirst, calculate the rate of petrol consumption:

erefore, the total power is:

Power =

Since efficiency is 20%, the power delivered to the wheels =Question 489: DBeta radiation is stopped by a few millimetres of aluminium, but not

by paper. In β- radiation, a neutron changes into a proton plus an emitted

1363

electron. is means the atomic mass number remains unchanged.Question 490: F

Firstly, calculate the mass of the caren using where v = 0 ms-1 and u = 15 ms-1 and t = 10 x 10-3 s

Question 491: EElectrical insulators offer high resistance to the ow of charge.

Insulators are usually non-metals; metals conduct charge very easily. Sincecharge does not ow easily to even out, they can be charged with friction.

Question 492: Fe car accelerates for the rst 10 seconds at a constant rate and then

decelerates after t=30 seconds. It does not reverse, as the velocity is notnegative.

Question 493: Be distance travelled by the car is represented by the area under the

curve (integral of velocity) which is given by the area of two triangles and arectangle:

Question 494: CUsing the equation force = mass x acceleration, where the unknown

acceleration = change in velocity over change in time.

1364

Hence:We know that F = 10,000 N, mass = 1,000 kg and change in time is 5

seconds.

So,

So change in velocityQuestion 495: Dis question tests both your ability to convert unusual units into SI

units and to select the relevant values (e.g. the crane’s mass is notimportant here).

0.01 tonnes = 10 kg; 100 cm = 1 m; 5,000 ms = 5 s

In this case the force is the weight of the wardrobe

us,Question 496: F

Remember that the resistance of a parallel circuit (RT) is given by:

1365

us, and therefore

Using Ohm’s Law:Question 497: EWater is denser than air. erefore, the speed of light decreases when

it enters water and increases when it leaves water. e direction of lightalso changes when light enters/leaves water. is phenomenon is knownas refraction and is governed by Snell’s Law.

Question 498: Ce voltage in a parallel circuit is the same across each branch, i.e.

branch A Voltage = branch B Voltage.e resistance of Branch A = 6 x 5 = 30 Ω; the resistance of Branch B =

10 x 2 = 20 Ω.

Using Ohm’s Law: I= V/R. us,Question 499: Cis is a very straightforward question made harder by the awkward

units you have to work with. Ensure you are able to work comfortably withpre xes of 109 and 10-9 and convert without difficulty.

50,000,000,000 nano Watts = 50 W and 0.000000004 Giga Amperes = 4A.

1366

UsingQuestion 500: BRadioactive decay is highly random and unpredictable. Only gamma

decay releases gamma rays and few types of decay release X-rays. eelectrical charge of an atom’s nucleus decreases after alpha decay as twoprotons are lost.

Question 501: D

Using

Now using Ohm’s Law:

So, each resistor has a resistance of .

If two more resistors are added, the overall resistance =Question 502: BTotal work done by engine = Force x Distance = Weight of Tractor x

Distance

us,

1367

Question 503: GElectromagnetic induction is de ned by statements 1 and 2. An

electrical current is generated when a coil moves in a magnetic eld.Question 504: DAn ammeter will always give the same reading in a series circuit, not in

a parallel circuit where current splits at each branch in accordance withOhm’s Law.

Question 505: DElectrons move in the opposite direction to current (i.e. they move

from negative to positive).Question 506: AFor a xed resistor, the current is directly proportional to the potential

difference. For a lament lamp, as current increases, the metal lamentbecomes hotter. is causes the metal atoms to vibrate and move more,resulting in more collisions with the ow of electrons. is makes it harderfor the electrons to move through the lamp and results in increasedresistance. erefore, the graph’s gradient decreases as current increases.

Question 507: EVector quantities consist of both direction and magnitude, e.g. velocity,

displacement, etc., and can be added by taking account of direction in thesum.

Question 508: C

e gravity on the moon is 6 times less than 10 ms-2. us, gmoon= ms-2.

1368

Since weight = mass x gravity, the mass of the rock =

erefore, the densityQuestion 509: DAn alpha particle consists of a helium nucleus. us, alpha decay

causes the mass number to decrease by 4 and the atomic number todecrease by 2. Five iterations of this would decrease the mass number by20 and the atomic number by 10.

Question 510: CUsing Ohm’s Law: e potential difference entering the transformer

(V1) = 10 x 20 = 200 V

Now use to give:

us, V2 = = 400 VQuestion 511: DFor objects in free fall that have reached terminal velocity, acceleration

= 0.us, the sphere’s weight = resistive forces.Using Work Done = Force x Distance: Force = 10,000 J/100 m = 100 N.

1369

erefore, the sphere’s weight = 100 N and since g = 10ms-2, thesphere’s mass = 10 kg

Question 512: Fe wave length of ultraviolet waves is longer than that of x-rays.

Wavelength is inversely proportional to frequency. Most electromagneticwaves are not stopped with aluminium (and require thick lead to stopthem), and they travel at the speed of light. Humans can only see a verysmall part of the spectrum.

Question 513: BIf an object moves towards the sensor, the wavelength will appear to

decrease and the frequency increase. e faster this happens, the fasterthe increase in frequency and decrease in wavelength.

Question 514: A

Using Newton’s second law: e Braking Force = Mass x Acceleration.

us, Braking Force =Question 515: CPolonium has undergone alpha decay. us, Y is a helium nucleus and

contains 2 protons and 2 neutrons.erefore, 10 moles of Y contain 2 x 10 x 6 x 1023 protons = 120 x 1023 =

1.2 x 1025 protons.Question 516: Ce rod’s activity is less than 1,000 Bq after 300 days. In order to

calculate the longest possible half-life, we must assume that the activity isjust below 1,000 Bq after 300 days. us, the half-life has decreased activityfrom 16,0 00 Bq to 1,000 Bq in 300 days.

After one half-life: Activity = 8,000 Bq

1370

After two half-lives: Activity = 4,000 BqAfter three half-lives: Activity = 2,000 BqAfter four half-lives: Activity = 1,000 Bqus, the rod has halved its activity a minimum of 4 times in 300 days.

300/4 = 75 daysQuestion 517: Gere is no change in the atomic mass or proton numbers in gamma

radiation. In β decay, a neutron is transformed into a proton (and anelectron is released). is results in an increase in proton number by 1 butno overall change in atomic mass. us, after 5 rounds of beta decay, theproton number will be 89 + 5 = 94 and the mass number will remain at 200.erefore, there are 94 protons and 200-94 = 106 neutrons.

NB: You are not expected to know about β+ decay.Question 518: C

Calculate the speed of the sound

us, the Wavelength =

Approximate 333 to 330 to give:Question 519: BFirstly, note the all the answer options are a magnitude of 10 apart.

us, you don’t have to worry about getting the correct numbers as long asyou get the correct power of 10. You can therefore make your life easier byrounding, e.g. approximate π to 3, etc.

1371

e area of the shell = πr2.

e deceleration of the shell

en, using Newton’s Second Law: Breaking Force = mass x acceleration=

Finally: Pressure =Question 520: Be fountain transfers 10% of 1,000 J of energy per second into 120

litres of water per minute. us, it transfers 100 J into 2 litres of water persecond.

erefore the Total Gravitational Potential Energy,

us,

Hence,

1372

Question 521: EIn step down transformers, the number of turns of the primary coil is

larger than that of the secondary coil to decrease the voltage. If atransformer is 100% efficient, the electrical power input = electrical poweroutput (P=IV).

Question 522: Ce percentage of C14 in the bone halves every 5,730 years. Since it has

decreased from 100% to 6.25%, it has undergone 4 half-lives. us, thebone is 4 x 5,730 years old = 22,920 years

Question 523: E

is is a straightforward question in principle, as it just requires you toplug the values into the equation: Velocity = Wavelength x Frequency – Justensure you work in SI units to get the correct answer.

Question 524: EIf an element has a half-life of 25 days, its BQ value will be halved every

25 days.A total of 350/25 = 14 half-lives have elapsed. us, the count rate has

halved 14 times. erefore, to calculate the original rate, the nal countrate must be doubled 14 times = 50 x 214.

214 = 25 x 25 x 24 = 32 x 32 x 16 = 16,384.erefore, the original count rate = 16,384 x 50 = 819,200Question 525: D

1373

Question 526: BEach three-block combination is mutually exclusive to any other

combination, so the probabilities are added. Each block pick isindependent of all other picks, so the probabilities can be multiplied. Forthis scenario there are three possible combinations:

P(2 red blocks and 1 yellow block) = P(red then red then yellow) +P(red then yellow then red) + P(yellow then red then red) =

Question 527: CQuestion 528: Cis is a rare case where you need to factorise a complex polynomial:(3x )(x ) = 0, possible pairs: 2 x 10, 10 x 2, 4 x 5, 5 x 4(3x - 4)(x + 5) = 03x - 4 = 0, so x =

1374

x + 5 = 0, so x = -5Question 529: C

Question 530: Ep α ∛q, so p = k ∛qp = 12 when q = 27 gives 12 = k ∛27, so 12 = 3k and k = 4so, p = 4 ∛qNow p = 24:24 = 4∛q, so 6 = ∛q and q = 63 = 216Question 531: A8 x 9 = 728 = (4 x 2) = 2 x 2 x 29 = 3 x 3(2 x 2 x 2 x 3 x 3)2 = 2 x 2 x 2 x 2 x 2 x 2 x 3 x 3 x 3 x 3 = 26 x 34

Question 532: C

Note that 1.151 x 2 = 2.302.Question 533: Ey2 + ay + b= (y +2)2 - 5 = y2 + 4y + 4 - 5= y2 + 4y + 4 - 5 = y2 + 4y - 1So a = 4 and y = -1

1375

Question 534: E

Question 535: C

Question 536: EAngles SVU and STU are opposites and add up to 180°, so STU = 91°e angle of the centre of a circle is twice the angle at the

circumference so SOU = 2 x 91° = 182°

1376

Question 537: E

Question 538: EQuestion 539: B

For the black ball to be drawn in the last round, white balls must bedrawn every round. us the probability is

Question 540: C

1377

e probability of getting a king the rst time is , and the probability ofgetting a king the second time is . ese are independent events, thus, theprobability of drawing two kings is

Question 541: B

e probabilities of all outcomes must sum to one, so if the probabilityof rolling a 1 is x, then: x+x+x+x+2x=1. erefore, .

e probability of obtaining two sixes

Question 542: Bere are plenty of ways of counting, however the easiest is as follows:

0 is divisible by both 2 and 3. Half of the numbers from 1 to 36 are even (i.e.18 of them). 3, 9, 15, 21, 27, 33 are the only numbers divisible by 3 thatwe’ve missed. ere are 25 outcomes divisible by 2 or 3, out of 37.

Question 543: C

List the six ways of achieving this outcome: HHTT, HTHT, HTTH, andTTHH, THTH, THHT. ere are 24 possible outcomes for 4 consecutivecoin ips, so the probability of two heads and two tails is:

Question 544: D

1378

Count the number of ways to get a 5, 6 or 7 (draw the square if helpful).e ways to get a 5 are: 1, 4; 2, 3; 3, 2; 4, 1. e ways to get a 6 are: 1, 5; 2, 4; 3,3; 4, 2; 5, 1. e ways to get a 7 are: 1, 6; 2, 5; 3, 4; 4, 3; 5, 2; 6, 1. at is 15 outof 36 possible outcomes.

Question 545: CQuestion 546: B

ere are two ways of doing it, pulling out a red ball then a blue ball, orpulling out a blue ball and then a red ball. Let us work out the probabilityof the rst: , and the probability of the second option will be the same.ese are mutually exclusive options, so the probabilities may be summed.

Question 547: A[x: Player 1 wins point, y: Player 2 wins point]Player 1 wins in ve rounds if we get: yxxxx, xyxxx, xxyxx, xxxyx.(Note the case of xxxxy would lead to player 1 winning in 4 rounds,

which the question forbids.)

1379

Each of these have a probability of p4(1-p). us, the solution is 4p4(1-p).

Question 548: F

Question 549: D

Question 550: A

Question 551: B

1380

1381

Question 559: BWhilst you de nitely need to solve this graphically, it is necessary to

complete the square for the rst equation to allow you to draw it moreeasily:

is is now an easy curve to draw (y = x2 that has moved 2 units leftand 10 units up). e turning point of this quadratic is to the left and well

1382

above anything in x3, so the only solution is the rst intersection of the twocurves in the upper right quadrant around (3.4, 39).

Question 560: CBy far the easiest way to solve this is to sketch them (don’t waste time

solving them algebraically). As soon as you’ve done this, it’ll be veryobvious that y = 2 and y = 1-x2 don’t intersect, since the latter has itsturning point at (0, 1) and zero points at x = -1 and 1. y = x and y = x2

intersect at the origin and (1, 1), and y = 2 runs through both.Question 561: BNotice that you’re not required to get the actual values – just the

number’s magnitude. us, 897653 can be approximated to 900,000 and0.009764 to 0.01. erefore, 900,000 x 0.01 = 9,000

Question 562: C

Question 563: A

1383

Question 564: AQuestion 565: C

Question 566: B

Question 567: D

Question 568: C

1384

Question 569: AQuestion 570: C

Question 571: C

1385

Question 572: Cis is fairly straightforward; the rst inequality is the easier one to

work with: B and D and E violate it, so we just need to check A and C in thesecond inequality.

C: 13 - 22 < 3, but A: 23 - 12 > 3Question 573: B

Whilst this can be done graphically, it’s quicker to do algebraically(because the second equation is not as easy to sketch). Intersections occurwhere the curves have the same coordinates.

Question 574: D

Question 575: EQuestion 576: D

1386

We can eliminate z from equation (1) and (2) by multiplying equation(1) by 3 and adding it to equation (2):

3x + 3y – 3z = -3 Equation (1) multiplied by 3

2x – 2y +3z = 8 Equation (2) then add both equations

5x + y = 5 We label this as equation (4)Now we must eliminate the same variable z from another pair of

equations by using equation (1) and (3):2x + 2y – 2z = -2 Equation (1) multiplied by 2

2x – y + 2z = 9 Equation (3) then add both equations

4x + y = 7 We label this as equation (5)We now use both equations (4) and (5) to obtain the value of x:5x + y = 5 Equation (4)

- 4x - y = -7 Equation (5) multiplied by -1

x = -2Substitute x back in to calculate y:4x + y = 74(-2) + y = 7- 8 + y = 7y = 15Substitute x and y back in to calculate z:x + y – z = -1-2 + 15 – z = -113 – z = -1-z = -14z = 14us: x = -2, y = 15, z = 14

1387

Question 577: Dis is one of the easier maths questions. Take 3a as a factor to give:3a(a2 – 10a + 25) = 3a(a – 5) (a – 5) = 3a(a – 5)2

Question 578: BNote that 12 is the Lowest Common Multiple of 3 and 4. us:-3 (4x + 3y) = -3 (48) Multiply each side by -34 (3x + 2y) = 4 (34) Multiply each side by 4-12x – 9y = -144

12x + 8y = 136 Add together

-y = -8y = 8Substitute y back in:4x + 3y = 484x + 3(8) = 484x + 24 = 484x = 24x = 6Question 579: E

Don’t be fooled, this is an easy question, just obey BODMAS and don’tskip steps.

Question 580: E

1388

Since there are 26 possible letters for each of the 3 letters in the licenseplate, and there are 10 possible numbers (0-9) for each of the 3 numbers inthe same plate, then the number of license plates would be:

Question 581: B

Question 582: C

Question 583: ALet x, y, and z represent the rent for the 1-bedroom, 2-bedroom, and 3-

bedroom ats, respectively. We can write 3 different equations: 1 for therent, 1 for the repairs, and the last one for the statement that the 3-bedroom unit costs twice as much as the 1-bedroom unit.

(1) x + y + z = 1240(2) 0.1x + 0.2y + 0.3z = 276(3) z = 2xSubstitute z = 2x in both of the two other equations to eliminate z:(4) x + y + 2x = 3x + y = 1240(5) 0.1x + 0.2y + 0.3(2x) = 0.7x + 0.2y = 276-2(3x + y) = -2(1240) Multiply each side of (4) by -210(0.7x + 0.2y) = 10(276) Multiply each side of (5) by 10(6) -6x -2y = -2480 Add these 2 equations

1389

(7) 7x + 2y = 2760

x = 280z = 2(280) = 560 Because z = 2x280 + y + 560 = 1240 Because x + y + z = 1240y = 400us the units rent for £ 280, £ 400, £ 560 per week respectively.Question 584: C

Following BODMAS:

Question 585: BConsider a triangle formed by joining the centre to two adjacent

vertices. Six similar triangles can be made around the centre – thus, thecentral angle is 60 degrees. Since the two lines forming the triangle are ofequal length, we have 6 identical equilateral triangles in the hexagon.

1390

Now split the triangle in half and apply Pythagoras’ theorem:Question 586: BLet x be the width and x+19 be the length.us, the area of a rectangle is x(x + 19) = 780.erefore:x2 + 19x – 780 = 0(x - 20)(x + 39) = 0x – 20 = 0 or x + 39 = 0x = 20 or x = -39Since length can never be a negative number, we disregard x = -39 and

use x = 20 instead.us, the width is 20 metres and the length is 39 metres.Question 587: Be quickest way to solve is by trial and error, substituting the

provided options. However, if you’re keen to do this algebraically, you cando the following:

Start by setting up the equations: Perimeter = 2L + 2W = 34us: L + W = 17

1391

Using Pythagoras: L2 + W2 = 132

Since L + W = 17, W = 17 - Lerefore: L2 + (17 – L)2 = 169L2 + 289 – 34L + L2 = 1692L2 – 34L + 120 = 0L2 – 17L + 60 = 0(L – 5) (L – 12) = 0us: L = 5 and L = 12And: W = 12 and W = 5

Question 588: CMultiply both sides by 8:Remove brackets:Simplify:Add 10:Subtract 8x:erefore:

Question 589: C

1392

Question 590: AQuestion 591: C

Question 592: D

Question 593: C

1393

Question 594: Dere are three outcomes from choosing the type of cheese in the

crust. For each of the additional toppings to possibly add, there are 2outcomes: 1 to include and another not to include a certain topping, foreach of the 7 toppings

us, the number of different kinds of pizza is: 3 x 2 x 2 x 2 x 2 x 2 x 2 x2 = 3 x 27

= 3 x 128 = 384

Question 595: AQuestion 596: CIf two shapes are congruent, then they are the same size and shape.

us, congruent objects can be rotations and mirror images of each other.e two triangles in E are indeed congruent (SAS). Congruent objectsmust, by de nition, have the same angles. \

1394

Question 597: B

Question 598: BQuestion 599: AIf X and Y are doubled, the value of Q increases by 4. Halving the value

of A reduces this to 2. Finally, tripling the value of B reduces this to ⅔, i.e.the value decreases by ⅓.

Question 600: C

e quickest way to do this is to sketch the curves. is requires you tofactorise both equations by completing the square:

1395

us, the rst equation has a turning point at (1, 2) and doesn’t crossthe x-axis. e second equation has a turning point at (3, -19) and crossesthe x-axis twice.

1396

MOCK PAPER ANSWERS

Mock Paper A: Section 1Question 1: Ce simplest solution is to calculate the total area at the start as 20 x

20 = 400cm2.en recognise that with every fold the area will be reduced by half

therefore the area will decrease as follows: 400, 200, 100, 50, 25, 12.5 –requiring a total of 5 folds.

Question 2: Dis is the only correct as it is the only statement that doesn’t

categorially state a fact that was discussed in conditional tense in theparagraph.

Question 3: BOff the 50% carrying the parasite 20% are symptomatic. erefore 0.5 x

0.2 = 10% of the total population are infected and symptomatic. Of which0.1 x 0.9 = 9% are male.

Question 4: De most important part of the question to note is the gure of 30%

reduction during sale time. Although A and B are possible the questionasks speci cally with regard to cost. erefore it is only worth waiting forthe sale period if the sterling to euro exchange rate does not depreciatemore than the magnitude of the sale. As such solution D is the only correctanswer as it describes anticipating a loss in sterling value less than 30%against the euro.

1397

Question 5: DBegin by calculating the number of childminders that can be hired for

a 24 hour period as 24 x 8.5 = 204. erefore a total of 4 childminders canbe hired continually for 24 hours with £184 left over – as the questionstates the hire has to be for a whole 24 hour period and therefore theremainder £184 cannot be used. As such D is the correct answer of 4 x 4 =16.

Question 6: Be simplest way to approach this question is to recognise that there is

a difference of £1.50 between peak and off-peak prices for all individualsexcept students. e total savings can therefore be calculated as (3 + 5 + 1)x 1.5 = 9 x 1.5 = 13.5.

Question 7: BRecognise that with each progressive frame the number of black

shapes decreases by one whilst the number of white shapes increases bytwo. erefore B is the correct answer with 9 white shapes and 0 blackshapes.

Question 8: DAnswers A and B are simply incorrect as the measurement taken is a

percentage increase (/decrease) which will normalise baseline diameterstherefore allowing for comparison over multiple time points. You shouldbe aware from your studies that ultrasound is an invaluable technique indistinguishing between adjacent tissue types. Any methodology isrepeatable if it is correctly chronicled and followed therefore leaving thecorrect answer of D.

Question 9: CIf both the ight and travel from the airport are delayed this will be the

longest the journey could possible take – producing a total journey time of20 + 15 + 150 + 20 + 25 = 230 minutes or 3 hours 50 minutes. ereforegiven all possible eventualities, to arrive at 5pm, boarding should begin at

1398

13.10pm. Answer D is incorrect as a delayed plan would add 20 minutes tothe journey whilst the transport to the meeting at the other end takes aminimum of 15 minutes – even if Megan could teleport instantaneouslyfrom the airport to the meeting she would be 5 minutes later than if therewasn’t a plane delay.

Question 10: Eis is almost a trick question and simply an application of

exponential decay. Recall that an exponential decay is asymptotic to 0 asno matter how small the volume within the cask becomes, only half of it isever removed. It could be argued that this process cannot continue once asingle molecule of whiskey is left – and when splitting that single moleculein half it is no longer whiskey. However the question does not ask “howlong till all the whiskey is gone” but rather “how many minutes will it takefor the entire cask to be emptied” and therefore the process can continuein nitely – even if the only thing left in the cask is a collection of quarks …or half that.

Question 11: EWith these questions it is important to only consider information

displayed in the graph and not involve any assumptions provided by yourprior knowledge. erefore this question is questioning your ability toconsider correlation as opposed to causation. e graph simply shows thatwaist size and BMI are positively correlated with one another and that is it.e nature of a scatter plot does not allow you to deduce which of thevariables (if any) drives the result observed in the dependent variable.However the fact that they are correlated is an important result andtherefore D is also incorrect.

Question 12: DObserve that in the depicted frames the total number of sides

increases by 8 each time independent of the colours used. erefore as the4th displayed frame has 33 sides, the correct answer is D with 41 sides.

1399

Question 13: Ee question can be expressed as (40 x 30) – x(50 x 30) = 200 = 1,200 –

x1,500. erefore x = 2/3.Question 14: De information given is very much a red herring in this question. is

can be solved using your own application of sequence theory. If a sequenceis 6 numbers long, the 6 numbers can be re-arranged into a total of 6!possible sequences = 1 x 2 x 3 x 4 x 5 x 6 = 720

Question 15: EAlthough this is an extremely abstract question, all the information

needed to answer it is provided. e key rule to have isolated from theinformation is that with each progressive sequence, a bell can only moveone position at most. erefore looking through each in turn – for examplewe can exclude A as the 3 goes from position 4 to position 2. is leavesthe only correct answer of B. Recall that the information states bells canonly move a maximum of one place – they can of course move 0 places.

Question 16: AAs the largest digit on the number pad is 9, even if 9 was pressed for an

in nitely long time the entered code would still average out at no largerthan 9. erefore it would be impossible to achieve a reference numberlarger than 9. Indeed this is an extremely insecure safe but not for thereason described in B ( for if the same incorrect number was pressedinde nitely it would never average out as the correct one) but ratherbecause the safe could in theory be opened with a single digit.

Question 17: DA is incorrect as it ignores the section of the text that states the

evolution of resistant strains is driven by the presence of antibioticsthemselves. e text states that the rate of bacterial reproduction is a largecontributing factor and therefore not wholly responsible – hence B isincorrect. Since this is just one example (and only the information in the

1400

text should be considered for these questions) for C to make such ageneral statement is complete unjusti ed.

Question 18: Be fastest way to solve this question is to calculate the quantity of

cheese per portion as 200/10 = 20. Which for 350 people would require 350x 20 = 7000g or 7kg.

Question 19: CCalculate the calori c content of 12 portions as 12 x 300 = 3,600kcal.

As this represents 120%, evaluate what the initial amount would be as(3,600/120) x100 = 3,000kcal.

Question 20: BBegin by calculating the initial weight of all the ingredients in the

Bolognese sauce which comes to a total of 3.05kg. erefore when cookingfor 10 people 3.05 x 4 = 12.2kg of pasta should be used. Which in turnmeans for 30 people 3 x 12.2 = 36.6kg should be used.

Question 21: DCalculate the new weight of ingredients in the Bolognese sauce

excluding garlic and pancetta which produces a total of 2.8kg. Note thatonions represent 0.3kg per 10 people and as such the ratio can berepresented as 0.3/2.8 or alternatively dividing top and bottom by 0.3 1/9.3

Question 22: EBegin with calculating total preparation time as 25 x 4 = 100 mins. e

fact that Simon can only cook 8 portions at a time is somewhat a redherring as it doesn’t impact the calculation. Total cooking time can becalculated as a further 25 x 8 = 200 mins. Producing a total time of300mins or 5 hours.

Question 23: DDue to the quantities colour t-shirts are priced at £5 and black and

white at £2.50. erefore the order will incur a total cost of 50 + (50 x 5) +(200 x 2.5) = £800.

1401

Question 24: EAnswers A and B directly con ict with information presented in the

text whilst C and D may well be true but there is insufficient informationin the text to address these points. is is an important reminder thatalthough you may well have been a scout and be able to comment onoptions C and D, you may only consider these arguments in terms of thetext provided.

Question 25: CNote that with each progressing frame, although the whole frame is

rotated 90 degrees clockwise, the black arrows are rotated 90 degreesanticlockwise. As such after 4 frames the sequence resets back to theinitial frame and C is the correct answer.

Question 26: Ce article seen here is a particularly good effort at a discursive text as

it is completely impartial. Note that the article simply states the facts fromeither side in equal measure. Nowhere does the author present theiropinion on the matter nor do they insinuate their beliefs in anyway.

Question 27: Ce journey time is rounded to the nearest hour (13). erefore the

longest it could possibly be is 13 hours 29 minutes or it would be round upto 8 hours. erefore the latest the ferry will arrive, assuming the traveltime estimate is accurate, is given as 20.29.

Question 28: Be correct answer is 28. Assume, although very unlikely, that you roll

a 2 every single go – you will never need to take a step back, only your twoforward. erefore rolling a lower number in this case is bene cial.

Question 29: Be simplest way to calculate this is to nd the lowest common

multiple of the given laptops which is 40 x 60 x 70 = 168,000 seconds =

1402

2,800 minutes = 46.67 hours – although they will not all be on the same lapnumber at this time.

Question 30: CA large amount of subtly different data is described here. Of note is the

rst experiment which describes how nerve conduction is faster in righthanded men than it is in left handed men. is result is not transferable towomen until it is proven! e experiment currently being conducted onlyconsiders dominant hand in men vs women. at could be either hand ornot necessarily the females’ right hand. For example all of the females inthis experiment could have been left handed and there is no informationin the text to say otherwise, therefore we cannot tell.

Question 31: CRecognise that two square based pyramids will comprise 8 triangles of

base width 5 and height 8; plus two 5 x 5 squares. us giving at total areaof 8(8/2 x 5) + 2(5 x 5) = 210 cm2

Question 32: BIn order to approach this question rst realise that in the rst well 1ml

of solvent is being combined with 9ml of distilled water producing 1eq ofsolute in 10ml – hence the rst well produces a dilution by a factor of 10.With each progressive dilution the concentration is reduced by a furtherfactor of 10 – hence by well 10 the concentration is at x/1010

Question 33: Ee compartments of the human body are occupied by numerous

uids, as the student is only interested in measuring the volume of blood,it is essential he chooses a solute that will only dissolve in blood. So as hisknown quantity of solute remains no, it must be neither removed noradded during its time in the body. Hence all of the written assumptionsmust be made and many more.

Question 34: D

1403

e fastest way to approach this question is by calculating the totalprice per head as 10 + (8/20) + (10/60) = 10.567. To make the mathssimpler this can be rounded safely to £11 a head at this stage. 2,300/11 isapproximately equal to 209 which when rounded to the nearest 10 is 210people.

Question 35: AWhilst this passage is attempting to weigh up two sides of an

argument, it has a clear one sided approach focussing heavily on theexcitement of dangerous sports. It even states that hunting is recognisedas exciting by some. Since the previous sentence discussed the linkbetween archery and hunting, the statement is a fair extrapolation tomake.

END OF SECTIONMock Paper A: Section 2Question 1: BLet tail = T, body and legs = B and head = H.As described in the question H = T + 0.5B and B = T + H.We have already been told that T = 30Kg.erefore substitute the second equation into the rst as H = 30 +

0.5(30 + H).Re-arranging reveals that -0.5H = 45Kg and therefore the weight of the

head is 90Kg, the body and legs 120Kg and as we were told the tail weighs30Kg.

us giving a total weight of 240KgQuestion 2: DRecall that kinetic energy can be calculated as E = 0.5mv2. erefore if

mass remains constant it is the v2 term that must be reduced to asixteenth. In other words v2 = 1/16 and therefore the correct velocity is1/4x.

Question 3: A

1404

An organ is de ned as comprising multiple tissue types. As blood andskeletal muscle are themselves tissues they cannot be classi ed as organs.

Question 4: Eis question is best considered in terms of the aerobic respiration

equation. With that in mind it becomes apparent that increased forwarddrive through the reaction will produce large amounts of water and CO2whilst demanding an increased supply of O2. Further from this equationwe realise that aerobic respiration produces large amounts of heat, and assuch it is expected – in the interest of thermoregulation – that the bodywill both perspire and vasodilate in attempt to increase heat loss.erefore E is the correct answer.

Question 5: DRecall that the nephron is the smallest functional unit of the kidney.

e question therefore is asking you what is the smallest basic functionalunit of striated muscle? To which the answer is the sarcomere. Note that amyo bril is a collection of many sarcomeres and is therefore not thecorrect answer.

Question 6: BInsulin is a polypeptide hormone released by the pancreas in response

to elevated plasma glucose levels. erefore it can be expected that plasmaglucose concentration will be proportional to the concentration of insulinin the blood. Furthermore recall that glucagon also released by thepancreas mobilises glucose stores. erefore the greatest concentration ofplasma glucose would be expected at the time when glucagon is highestduring a period of elevated insulin.

Question 7: DAnswers a and c are both nonsense and can be eliminated straight

away. You will know from your study of the immune system that it isplasma B cells that produce antibodies and that plasma T cells do notexist. Also recall that an immune response can be mounted as quickly as

1405

within a fortnight which leaves the only correct answer d. e passagestates that only once blood types are mixed is the immune responseinitiated, therefore answer d provides an explanation as to how thishappens but also why the rst born child is unaffected.

Question 8: ERecall that pH is a logarithmic scale of proton concentration and

therefore will have the largest effect on hydrogen bonding.Question 9: DAn organ consists of many cell types which once differentiated are

committed to that single cell line. erefore a totipotent stem cell isrequired to produce the multiple cell types required. In order to ensurethat the organ is an exact genetic match, stem cells from the individual inquestion must be used. Unless that individual is an embryo, adult stemcells must be used.

Question 10: DIsotopes of an element all contain the same number of protons but a

different number of neutrons. As atomic number refers solely to thenumber of protons it will not change. However as mass number is the sumof atomic number and neutron number – it would be expected to change.If an isotope contains one extra proton, then assuming that the charge ofthat isotope is 0, then it must also contain one extra electron. Chemicalproperties are the same for all isotopes. erefore the correct answer is D.

Question 11: Be transition metals are the most abundant catalysts – presumably

due to their ability to achieve a variable number of stable states. ereforethe correct answer is the d-block elements.

Question 12: EBegin by writing down the balanced equation that describes the

reaction of francium with water: 2Fr + 2H2O 2FrOH + H2. Next calculatethe moles of francium entering the reaction as 1338/223 = 6. We therefore

1406

know from the stoichiometry of the equation that this reaction willproduce 3 moles of hydrogen. Recall that 1 mole of gas at roomtemperature and pressure occupies 24dm3. erefore the hydrogenproduced in this reaction will occupy 3 x 24 = 72dm3.

Question 13: De simplest way to approach this type of question is to assume that

there are 10 atoms within the compound. In this case that produces thefollowing result: C3H4F2Cl. Next look to see if any of the subscript numbersare divisible by a common factor. Also if there are any decimals, multiplyup by a common factor until only integers are present. In this case thecorrect answer is achieved straight away.

Question 14: Cis question requires you to have a correct answer from the previous

question, although these questions are unfair in the fact that this currentquestion cannot be answered without success in the rst part – there arealways one or two of these per paper. Simply calculate the Mr of yourempirical formula: 113.5. And then divide 340.5 by this: 340.5/113.5 = 3.erefore multiply your empirical formula up by a factor of 3.

Question 15: B

e calculation in this question is simple: concentration =mass/volume, what this question is really testing is the manipulation ofunorthodox units. Begin by noting the use of g/dL in the nal answers andtherefore begin by converting the quantities in the question into theseunits. 1.2 x 1010 kg = 1.2 x 1013 grams and with 10 decilitres in a litre, 4 x1012 L = 4 x 1013 dL.

Question 16: A

1407

A catalyst is not essential for the progression of a chemical reaction, itonly acts to lower the activation energy and therefore increase thelikelihood and rate of reaction.

Question 17: ECationic surfactants represent a class of molecule that demonstrates

both hydrophilic and hydrophobic domains. is allows it to act as anemulsifying agent which is particularly useful in the disruption of greaseor lipid deposits. erefore cationic surfactants have applications in all ofthe products listed.

Question 18: BRecall that V = E/Q; therefore when substituting SI units into these

equation it is discovered that V = J/C = JC-1.Question 19: CRecall that voltmeters are always connected in parallel – and so that

they don’t draw any current from the circuit have an in nite resistance.Ammeters on the other hand are connected in series and therefore mustnot perturb the ow of given, meaning they have zero resistance.

Question 20: AMuch of the information in this question is not needed and is simply

put there to distract you. is question can be most quickly solved usingthe equation F=ma or force = mass x acceleration. As object A is the onlythings moving in this scenario it is the only source of energy to beconsidered. Its mass will be the same before and after the collision and sowe need only calculate the magnitude of retardation. Given as (15 – 3)/0.5= 24ms-2. erefore when plugging into the rst equation we realise that F= 12 x 24 = 288N of force dissipated. Alternatively this question could besolved by calculating the rate of change of momentum.

Question 21: CNote the atomic masses and numbers in the equation. Whilst the

atomic mass has remained constant the atomic number has increased by

1408

one and hence the element has changed. e only explanation for this isthat a neutron has turned into a proton (and an electron which isrepresented by x). erefore the correct answer is C – beta radioactivedecay.

Question 22: BBegin by calculating the velocity of the wave as speed = wavelength x

frequency = 3 x 20 = 60km/s. Which in a time period of one hour (3600s)would equate to a total distance of 60 x 3600 = 216,000km.

Question 23: Ce numerator of the fraction consists of 3 distinct terms or 3 distinct

dimensions. As all other functions within the equation are constants onewould consider this the volume of a complex 3D shape.

Question 24: BExpand the larger scienti c number so that it reads 10 to the power 6

like so: 4.2 x 1010 = 42000 x 106. Now that the powers are the same acrossthe numerators, a simple subtraction can be performed (42000 – 4.2) x 106

= 41995.8 x 106 which can be simpli ed to 4.19958 x 1010. Next consider thedivision which can be competed in a two-step process, rst divide thenumerator by 2 like so (4.19958/2) x 1010 = 2.09979 x 1010 and then subtractthe powers like so 2.09979 x 10(10-3) = 2.09979 x 107.

Question 25: ANote the triangle formed by the right angle lines and the tangent.

Recall that as this is a right angle triangle then the other two angles mustbe 45o. As angles along a straight line add up to 180o a must equal 180 – 45= 135o. Angles around the origin must add up to 360o and therefore b =(360 – 90)/2 = 135o. erefore the correct answer is A.

Question 26: Be probability of drawing a blue ball (1/21) and then a black ball

(1/20) is 1/21 x 1/20 = 1/420. However note that it is also possible that

1409

these balls could also be drawn out in the opposite order. erefore theprobability must be multiplied by two like so 1/420 x 2 = 2/420 = 1/210.

Question 27: Ce question states that the repeat experiment is identical to the rst

in all aspects apart from the result. erefore although a number of theoptions may be true like calibration bias, it would have been applied toboth experiments and therefore should not affect the result. As such thedifference in results is simply due to random chance.

END OF SECTIONMock Paper A: Section 3Doctors should be wearing white coats as it helps produce a placebo

effect making the treatment more effective.➢ is statement addresses the role of the patient’s personal experiencein his/her cure or treatment of their disease. It is an interesting topic sincethe role of psychological factors in the treatment of disease is largelyunexplored. ere is a growing body of evidence that supports theeffectiveness of placebo treatments for some diseases when it comes tomanaging patient symptoms, but there is very little that addresses the roleof attire and visual appearance of doctors.➢ It is also important to immediately question the truth of the statement.ere is some evidence to suggest that there is such a thing as a “whitecoat effect” that in uences patient’s behaviour and the perception of theirproblems when they are faced with a doctor. Questioning the statement isvery important as it demonstrates that you re ect on the issue.➢ When answering this question, there are several factors to consider.ere is the role that clothing plays in the de nition of professions. Howdoes the attire of an individual in uence the way he or she is perceived bythose receiving his/her service? Some examples here are police officers orjudges where the uniforms are heavily tied to the public perception of theirprofession. Police officers are a particularly good non-medical example

1410

since there are uniformed and non-uniformed officers that play differentroles playing on the different public perception of uniform and civilianclothing and the fact that without the uniform the police officer is notrecognisable. en the question arises if this should apply for doctors too.Does it make a difference if doctors have clothing that visually separatethem from other people in the hospital and does this have an in uence onthe patient’s experience of treatment. Other points to consider whenaddressing the role of attire is the depiction of doctors in the publicsphere. is includes TV shows, books, news etc.➢ Arguing against the statement is more difficult than it seems simplybecause you should make sure that you provide a diverse answer thataddresses several aspects. On one hand, there is the connection of aspeci c attire to a speci c professional role as described above. On theother hand, there is the question whether attire is relevant to in uence thepatient’s experience to improve health outcome. e whole point of aplacebo effect in this context is that it improves the outcome.➢ Another point to consider when arguing against this statement is thepower distribution that comes with the uniforms and whether that issomething that is bene cial for the patient-doctor relationship.➢ Arguing to the contrary, you can look at situations where aprofessionalization of the doctor-patient relationship can be bene cial.Now, to be clear, the relationship between doctor and patients shouldalways be professional, but in the context of this question, you can use therole of attire and its role in establishing this professionalization. Examplesfor this include conversations about life-style changes and the role thepatient can play in improving his/her own health, especially if this involvesgiving something up. In this case, attire can give the doctor legitimacy anda certain degree of authority.

“Medicine is a science of uncertainty and an art of probability.”

1411

➢ is statement basically addresses the fact that there is no such thingas certainty in medicine. People are different and individual and so is theirexperience of disease. For this reason, the statement argues that all adoctor can do in terms of approaching a sense of certainty, the doctorshould weigh up different probabilities and possibilities of disease. eargument also suggests, that weighing up the different options ofdiagnoses is an art, rather than acquired knowledge. is suggests somedegree of natural talent. It also provides a degree of contrast between theaspect of science that provides the theoretical basis for pretty much everydecision we make in medicine and the art of the application of knowledge.It also acknowledges that science always contains a degree of uncertainty,even when individuals believe in the absolute truth of theirtheory/knowledge.➢ Arguing to the contrary basically aims at increasing the perceived roleof science and certainty versus that of art and uncertainty or exibility.e main problem with this statement is the general perceptionassociated with the words “science” and art. ey naturally lie on differentends of a spectrum with science being associated with facts and certaintyand art being associated with softer skills and an absence of certainty.➢ If you choose to go into a more example oriented direction, there areseveral points you can raise to write a good and strong essay. On exampleis the treatment of infectious diseases with antibiotics, especially in severecases. Often you will nd that the disease is treated with a broad-spectrumantibiotic that is likely to target the causative agent based on localexperiences and local occurrence of diseases. is is then later adjusted ifnecessary once a precise identi cation of the causative agent was possible.Other examples include the strati cation of disease causes. One examplehere is smoking and lung cancer. Whilst it is generally accepted thatsmoking increases the risk of lung cancer, there are still non-smokers that

1412

get lung cancer and life-time smokers that do not. is pattern can beapplied to a variety of parameters to result in similar results.➢ In general, you can keep this essay very philosophical and abstract, oryou can aim more at direct examples to illustrate your points. Bot optionshave strengths and weaknesses. A theoretical essay will stay more with theoverall style of the statement, whilst a more example oriented essay will beeasier to write and to keep track of. However, it will also be more difficultto nd appropriate examples.➢ In the conclusion, when you give your opinion, it pays to be very directon one hand, but also to be very speci c. Depending on which route youtook for your main body arguments, this may be easier or more difficult.You can also pick up the idea of medicine being art again as this is aninteresting point and ties in with the idea that medicine cannot beexclusively learned from books but should also contain a component ofpatient interaction.

“e New England Journal of Medicine reports that 9 out of 10doctors agree that 1 out of 10 doctors is an idiot.”➢ is statement addresses how scienti c research will never ndcomplete acceptance in the eld. It also suggests that no matter what isbeing published by even the most highly acclaimed scienti c journals,there is always a risk of error. In the end, it illustrates that medicalresearch usually is a game of probabilities as there is never completecertainty when it comes to the pattern of diseases or the optimaltreatment of disease.➢ On a face value level, this question is simple. It is very vague in itsassertion, not de ning which one of the 10 doctors think that the other isan idiot. Do they all think the same person is an idiot or do they thinkdifferent persons are idiots? is an important issue to raise whenanswering this question as it presents a fundamental aw of the question,especially if one is to apply it to general medical research and research

1413

practice. Even in the sometimes uncertain realm of medical research,parameters such as populations of subjects are always clearly de ned,which is what gives any form of research value. If this was not the case,research would be completely arbitrary. Coming back to the question then,if all 9 doctors believe that that the one speci c one of them is an idiot andthey have no prior contact and no connection to each other, then chancesare that this single doctor is actually an idiot. If, however, there is nopattern whatsoever to the claim that one doctor is an idiot, then thatweakens the claim. Especially if the whole concept is then widened to apopulation level.➢ Arguing to the contrary has different obvious points. You can either stayvery close to the actual wording of the question which will lead you downa similar road as I have illustrated above, or you can use the question as aparable for the way we conduct scienti c research. is will require you tohave a good understanding of scienti c method.➢ If you decide to go down the scienti c method pathway, you will haveseveral things to point of. Firstly, is the de nition of populations, as this iscompletely ignored in the question. In any form of research de ning thepool of data you draw from is essential as only this will deliver accurateand usable information. It is all about reducing vagueness as much aspossible. Secondly you need to de ne the research criteria. What exactly ismeant by idiot in this case for example? Only if you formulate a clear-cutgoal can you then acquire the data needed to come to a meaningful result.e term ‘asking the right questions’ comes to mind. irdly you need toensure repeatability. For this you need to de ne your populations verybroadly and in appropriate sizes. You must make sure that there is as littleconnection between the subjects as possible as this will reduce and biasfrom personal relationships.➢ ese two options should help you write a strong essay, especially sincethy can be combined in essentially any way you choose.

1414

“My father was a research scientist in tropical medicine, so I alwaysassumed I would be a scientist, too. I felt that medicine was too vagueand inexact, so I chose physics.”➢ ere are several components to this question that you need to beaware of if you want to write a good essay. On one hand is the personStephen Hawking himself. Being a world renowned theoretical physicist,gives the whole quote an almost comical note. is is something youshould be aware of as you will always have to point out problems with thequestions. Moving beyond this, there are several other points you shouldbe aware of. One is the vagueness of the statement. is obviously is due tothe fact that it is taken from what probably was a whole speech, ratherthan this single passage. Again, something you should point out. enthere is the subject matter of tropical medicine. Tropical medicine is inpart still a very new eld and a eld which much room for explorationsimply because there is such a wealth of different life forms in tropicalareas that can cause diseases, some of which may never have beenobserved before. is necessarily adds to the perceived uncertainty. Inaddition, keep in mind that Stephen Hawking is now 75 years old, whichplaces his father’s professional career to the rst half of the 20th century, alot has happened in medicine since then. Secondly you should address thesubject matter of physics. Whilst some elds in physics have very littleuncertainty and vagueness, a lot of areas are very precise such as gravity ormechanics. So, it is important to make that distinction as physics is such abroad topic.➢ When it comes to arguing to the contrary, there are several perspectivesyou can take. On one hand, you can argue that tropical medicine is moreaccurate than Hawking gives it credit for. An easy way to do this is toenlarge it to general medicine as the clear majority of general medicalprinciples will still apply in tropical medicine, what will change will bedifferent pathogens and the environmental factors in uencing pathology

1415

and healing processes. If we accept that in general medicine is a fairlyexact science, we can use that to support the same claims about tropicalmedicine.➢ Another point of attack would be to point out the vagueness of someareas of physics. Easy targets here are String theory and relativity theory.Neither of those can be supported by non-mathematical evidence at thispoint and even the potential discovery of the Higgs boson in the Cernsuper collider does not provide enough answers to these questions yet.ere are many other examples in the eld of physics that are vague, that’swhy they have given rise to completely separate job description: thetheoretical physicist.➢ Finally, you can also consider Hawking’s personal history withmedicine. His suffering from ALS for decades and being bound to awheelchair after far outliving any suggested life-expectancy it isunderstandable that he considers medicine as somewhat vague. iscould well have in uenced him in this statement.

END OF PAPER

1416

Mock Paper B: Section 1Question 1: BWhen the total number of sides is even, the shapes are black; when the

total number of sides is odd, the shapes are white.Question 2: CStart by calculating the area of wall that may be painted per tin of

paint as 10 x 5 = 50m2. erefore to paint the whole area 1050/50 = 21 tinsof paint are required per coat. As such to complete 3 coats it will cost Josh3 x 21 x 4.99 = 314.37.

Question 3: CA is a correct assumption as procession is a function of rotational

motion. B is a necessary assumption or rather inference of the rstsentence. e second sentence only says that an asterism can be used, notthat it is the only possible method. Nothing is mentioned of navigating theSouthern Hemisphere and therefore C is not a valid assumption.

Question 4: DRecognise that “bank hours” refers only to hours that the bank is open

– which Mon to Fri is 8 hours whereas it is only 6 hours on a Saturday.Although John needs the money by 8pm the bank closes at 5 and that 3hours difference cannot be used. Hence working backwards John will need8 hours on the Tuesday, 8 hours on the Monday, Sunday is closed, 6 hourson the Saturday, 8 hours on Friday and 8 hours on ursday and 4 hourson the Wednesday. With a closing time of 5pm, the latest John can cash thecheque on Wednesday is 1pm.

Question 5: DFirst thing to recognise here of course is that individual diamonds can

be combined to form larger diamonds with the 5 x 5 diamond the biggestof them all. To avoid counting them all and risking losing count, insteaddeduced the number of triangles per corner and per side; then multiply upby 4.

1417

Question 6: BQuestion 7: DA is categorically wrong as the rst two paragraphs discuss how

aneurysms produce in ammation which in turn blunts endothelial NOaction. B is incorrect as it states aneurysms directly promote CVD, this isnot a direct process. It is the blunted NO which directly produces the CVD.C can be ignored as nowhere are aneurysms categorised like this. E isincorrect as the text states that aneurysms reduce NO which will reducevasodilation, thus increasing basal vasoconstriction and thus reducingblood ow. Leaving the correct answer of D which is of course true asobservations are not transferable between species until testedscienti cally.

Question 8: CAny statement which refers to national or global gures is instantly

incorrect as the text does not mention any statistical analysis has takenplace. In order to produce national statistics from a small sample size suchas this requires statistical analysis. Whilst E could possibly be true itcannot be stated as there are so many possibilities – perhaps the time ofthe survey was during rush hour in which case the majority of the trafficwould have been travelling in the same direction anyway to reach anindustrialised area.

Question 9: ABlack shapes demonstrate a reduction of 1 side and a transition into

white, whilst white shapes demonstrate a gain in 1 side and a transitioninto black.

1418

Question 10: Ais question can be solved quickly if you rst realise that there is no

need to calculate both volumes and subtract the larger from the smaller,instead only convert the television dimensions into metres and thencalculate 60% of that.

Question 11: EFrom the information provided all the aws listed are valid since

David’s main point is that he has chosen the cheapest. A could be true asthere is an additional cost of £3 for staying at Whitmore, therefore if thevehicle they are using achieves sufficient miles per gallon then travellingthe extra few miles could cost less than £3 in terms of petrol. B again ispossible which would argue against it being cheap, as would D. And if C istrue then David’s argument is awed altogether.

Question 12: DC is irrelevant as nowhere does the passage mention standards of

modern medical practice. A may be incorrect as nowhere does the articleexplicitly say that animal testing is the only accepted method of drugapproval. B categorically con icts with the rst sentence of the secondparagraph.

Question 13: ABegin by converting all the quantities into terms of items as that is the

terminology used on the graph axis. erefore 12 rugby balls = 6 items and120 tennis balls = 24 items. Reading from the graph reveals their respectiveprices as £9 and £5. erefore the total cost of products in the order is (6 x9) + (24 x 5) = 174. Since this is signi cantly more than £100 the deliverycharge is waived.

Question 14: DCalculate the cost of 10 of everything as (2 x 5) x (10 x 7) x (5 x 9) =

£125. Recall that delivery charge is waived at £100 and this therefore a trickquestion and no delivery charge is applied anyway.

1419

Question 15: ETennis balls are sold in the largest pack and so they must be

considered. Begin by dividing 1000/5 using the value from the rst column= 200. As this is above the range 0 -99 look up the item value in the 100-499 range where a £1 discount is applied per item. erefore in actuallyfact 1000/4 = 250 items can be purchased which equates to a total of 250 x5 = 1250 balls.

Question 16: BRecognise that 120% pro t is equivalent to 220% of the original price.

In which case the initial purchase price = (1,320/220) x 100 = £600.Question 17: ANote that here the question uses the term item and therefore simply

read the costs directly off the graph giving a total order cost of (2 x 2000) +(4 x 2000) + (6 x 2000) = 24,000. Recall though that he only pays tax on theamount over £12,000 which in this case is £12,000. erefore he pays12,000/4 = £3,000 tax.

Question 18: ANote that the total number of black shapes is equal to the number of

corners within the white shape. Hence with a ve pointed start and veblack shapes; A is the correct answer.

Question 19: EFirst calculate an average complete one way journey time as 40 + 5 + 5

= 50 minutes. Deducting his breaks he works a total of 7 hours 20 or 440minutes. Since the rst train is already loaded his rst run will only take 45minutes leaving 395 minutes to complete his working day. 395/50 = 7remainder 45. Note that 45 minutes is not enough to fully unload the trainbut it is enough to load the train and drive the distance. erefore thedriver will complete a total of 9 journeys equalling a distance of 198 miles.

Question 20: E

1420

A is not actually a valid assumption as we do not know what proposalconservationists might be bringing to the local councils, they have onlyexpressed their concern. ey may well be bringing a proposal to ask forfunding to rehome all the species in the affected environment. B isessential to the nal paragraph whilst C must be assumed otherwise thecouncils would not be presenting these proposals at all.

Question 21: DAs there is not really information in the question to calculate the

answer quickly. Instead consider each answer in term and calculate thedifferences to nd the correct price difference in the question:A) (3x 1) + (2 x 1.25) – (15 x 0.3) + (10 x 0.5) etc…

Question 22: EBased on the information in the question options A - D are simply

wrong. A is incorrect as antibiotic E has not affected growth at all. B isincorrect as the other antibiotics have signi cantly affected growth. E wasthe least effective antibiotic. C was not the most ineffective as it diddisrupt growth slightly whereas E had no effect at all. D will now be takenand the experiment repeated with D at numerous concentrations to ndthe optimum dose.

Question 23: A

Question 24: B

Quickly represent the question schematically as We can now observethat A in fact supports George’s argument, C also supports George’sargument and D may well be true but it would have no effect in disrupting

1421

the argument, simply only imply D and E are both also equal to 0. Howeveras E is equal to C it should therefore not equal B.

Question 25: Bis question is much less complicated than it sounds. Begin by just

considering a single hour. roughout the hour of 1 the hour hand willpointing at 1. Only during the 5th minute of that hour will the minute handpoint to the 1 whereas every 5th second of the minute the second handpoints to the 1. All these events will only coincide once. As there are 24hours in a day 00:00 through to 23:59 this event will happen 24 times.

Question 26: BAs there are already to crosses on the board to only one nought it is

noughts turn. James is forced to play B3 but then suppose crosses plays A1.James is then forced to block the diagonal by playing C3. However supposethen that crosses plays A3 leaving James to claim the winning move withC2. erefore depending on the play it is possible for James to winalthough he cannot adopt a strategy to guarantee that he wins.

Question 27: De total number of sides of all the white shapes is divided by the

number of sides of the black shape: 30/5 = 6 sides. erefore the answermust comprise a 6 sided shape but note the answer is always in black.

Question 28: CWhilst B may be true it is not a reason for dependence, only a

supporting factor. Dependence implies that we have no choice but to useelectricity. Hence A is wrong as gas is readily available; hence D is wrongfor the same reason. is leaves the correct answer C which is the onlystatement which truly describes our absolute necessity for electricity –since electrical appliance by de nition only function with electricity.

Question 29: BFirst note that 27 guests plus Elin herself means that 28 people will be

eating the 3 courses which will require a total of 28 x 3 = 84 glasses of wine.

1422

is is a total volume of 84 x 175 = 14.7L = 21 bottles. As wine is only soldin cases of 6, Elin will have to buy 24 bottles so as not to run out. Recall thebuy one get one free offer so she only pays for 2 cases.

Question 30: ERecognise that when rounded to the nearest 10 the shortest an

episode could last is 35 minutes. Hence a total of 7 x 12 = 84 episodeswould take a total of 84 x 35 = 2,940 minutes = 49 hours.

Question 31: EPoints A and B are the best exempli ed through this passage. Often

great discoveries come from accidental observations and then exactprocesses are re ned through many experiments in a trial and errorfashion until the correct methodology is achieved. e passagedemonstrates how as our understanding of the world around us advanceso too does our ability to provide healthcare. D can be observed in thepassage as the 50/50 split.

Question 32: CWhilst A and D are true they do not force the stranger to give him the

sapphire – remember Jack can be given any stone for a truthful statement.B and C are both lies and will earn Jack nothing. Instead if Jack states Ethen the stranger has no choice to hand over the sapphire else it would bea false statement.

Question 33: ADespite the enormous interest rate in Simon’s current account it is

only awarded twice, whereas in the saver account it is awarded 4 times.Hence earnings from the saver account = 100 x 1.54 = £506 whereasearnings from the current account would have stood at £361.

Question 34: De largest possible key can be obtained were the rst two numbers

are at a maximum because they are multiplied together 9 x 9 = 81. Subtract

1423

the smallest number to yield 81 – 1 = 80 and again divide by the smallestnumber which is 1 hence 80 is the largest possible key.

Question 35: Ce use of coloured shapes in this question is simply a distraction and

no pattern exists. Instead notice that when the total number of sides of allthe shapes is even, the shapes are found outside the speech bubble; whenthe number of sides is odd, the shapes are found within the speech bubble.

END OF SECTIONMock Paper B: Section 2Question 1: DAs the question states that GLUT2 is ATP independent then answer A)

active transport is instantly incorrect as it is ATP dependent.Osmosis is applicable only to water molecules and is therefore

incorrect.Exocytosis refers to the movement of molecules out of a cell and is

therefore incorrect.Simple diffusion is incorrect as the question states that GLUT2 is

essential for the process.is leaves the correct answer of facilitated diffusion.Question 2: EIn order to answer this question you must recall that anaerobic

respiration in humans produces only lactate and energy, whilst in yeastthe anaerobic respiratory process yields a molecule of ethanol and CO2 perglucose molecule. erefore there will be 0 mol of CO2 produced in thehuman cell culture and you need only work out the moles of CO2 producedby the yeast cell culture to calculate the difference. ere is a total of5.76/0.18 = 32 mol of glucose, of which half is supplied to the yeast cellculture. With a stoichiometric ratio of 1:1 in the anaerobic respirationequation a total of 16 mol of CO2 will be produced.

Question 3: E

1424

Firstly recall that endocytosis is a process of molecular transport intocells that result in vesicular formation. is question requires you torealise the special case of this which is phagocytosis – conducted by whiteblood cells in the ingestion of pathogens.

Question 4: EAll of the above statements are true of the Calvin cycle with regards to

the Krebs cycle. As the main driver of photosynthesis we know that theCalvin cycle requires both CO2 and light in order to conduct ATPdependent reactions. As opposed to the Krebs cycle in man however, theCalvin cycle adopts the use of NADPH as the intermediate in electrontransport.

Question 5: DOption d is one of only 2 graphs that demonstrate a quadratic

relationship with the peak enzyme activity correctly placed – pepsin fromthe stomach close to pH 1, and trypsin secreted by the pancreas andtherefore alkaline around pH 13. e curves traced in option c however arefar too broad over the pH range to represent enzyme activity. As the pHscale is logarithmic, even a change of 1 or 0.5 can be devastating to enzymeactivity.

Question 6: Ais question was taken directly from the BMAT syllabus where many

examples are listed for different principles. Reading through the BMATsyllabus and highlighting these is a very good idea as well as learning thede nitions listed.

Question 7: AInitially the electron con guration of Mg is 2,8,2. In binding to two

chlorine atoms it is effectively ionised to Mg2+ and it loses two electrons toleave a complete outer shell and thus the correct answer is 2,8.

Question 8: D

1425

e rst thing to note in this trace is that the m/z axis has been cutshort. From looking up the mass of calcium in the periodic table onewould expect to see the x axis centred around 40. However here the traceis only displaying those isotopes with valence 2 (z= 2) hence the values arehalf the size. erefore ( from the periodic tablE) when dividing the mostabundant isotope of chromium by two, 52/2 = 26, we con rm that theoutlier bar on the right is indeed the contaminant. erefore to calculatethe actual abundance of Mr 40 calcium ignore the chromium like so: 55/95= 11/19.

Question 9: ABegin by converting the total weight of arsenic into grams like so 15 x

106 = 1.5 x 107. en divide by the Mr of arsenic which is 75 (2sf) giving 2 x105. Don’t forget that the sample is at worst 80% pure. erefore there willbe a minimum of (2 x 105) x 0.8 = 1.4 x 105 moles of pure arsenic.

Question 10: DRecall that average atomic mass is calculated as the sum of (isotope

mass x relative abundance). erefore 28 = (26 x 0.6) + (30 x 0.3) + 0.1x.Rearranging this equation reveals that 0.1x = 3.4 and that the mysteryisotope therefore has an atomic mass of 34.

Question 11: AFirst recall that when a group 2 metal is reacted with steam a metal

oxide is formed and therefore the following chemical equation can bedrawn: Mg + H2O(g) MgO + H2. Note the stoichiometric ratio which issimply 1. Next calculate that there is 72/24 = 3 mol of hydrogen produced.erefore assuming that there is 3 mol of all other reactants and thereaction is complete one would expect 3 x 24.3 = 72.9g of magnesium and 3x 18 = 54g of steam. is is indeed the case and therefore the reaction iscomplete.

Question 12: B

1426

e reducing agent is the species which is itself reduced in thisinstance from looking at the oxidation states we can see that that speciesis S2-. As after the reaction has taken place it has an oxidation state of +6which would require a loss of negative charge ie electrons.

Question 13: Ce highly stable bonds between carbon atoms, and between carbon

and hydrogen atoms renders alkanes relatively unreactive. is isimportant to note as it highlights the major difference between alkanesand alkenes.

Question 14: ARecall that current = charge/time. e question provides both charge

and time in the correct units and so the calculation is relatively simplewith no unit conversions required. erefore current = 5/15 = 1/3 = 0.33A.As the question states that the balloon has a negative charge it hastherefore gained electrons. Given that a current is de ned as a netmovement of electrons, in this situation the current must be owing intothe balloon.

Question 15: DGiven that Power = IV it can be deduced that I = P/V. Recall that power

given in Watts is a measure of the energy transferred per second andtherefore has the alternative units Js-1. When substituting these units intothe power equation re-arranged for Amps it is revealed that I = (Js-1)/V = A.

Question 16: DFor a transformer that is 100% efficient power in must equal power

out, recalling that P=IV. erefore the transformer has a power output of24 x 10 = 240W which is 80% of the initial input. As such the initial powerinput was (240/80) x 100 = 300W.

Question 17: CBegin by calculating the energy required to hoist the mass, this is

calculated using the potential energy equation: mgh. Energy = mass x g x

1427

height = 20 x 10 x 30 = 6000N. e power output of the motor is calculatedas the joules dissipated per second = 6000/20 = 300W

Question 18: DIn order to solve this problem recall that activity = decay constant x

number of remaining atoms. erefore the decay constant can becalculated simply as 0.36/6 = 0.06.

Question 19: DRecall that household electricity is available in the UK at 240V. Begin

by calculating the wattage that the bulb is receiving as 0.5 x 240 = 120W.Given that the energy rating of the bulb is 80W, we can assume that thisbulb is only 80/120 = 66% efficient.

Question 20: EQuestion 21: C

Question 22: EBegin by drawing your line of best t, remembering not to force it

through the origin. Begin tting the general equation y = mx + c to yourline. Calculate the gradient as ∆y/∆x and read the y intercept off yourannotated graph.

1428

Question 23: EQuestion 24: DNon-normally distributed data doesn’t demonstrate a 50-50 split of

data points either side of the mean. erefore standard data analysistechniques like normal range are inappropriate (as the formula for normalrange is mean 1.96SD). Instead the interquartile range is used.

Question 25: DRandom chance is a large issue particularly in medicine. Clinical trials

are inherently awed as they only consider a very small percentage of thepopulation which is far outweighed by the genetic variation demonstratedwithin the human genome. erefore statistics must be used to transformsample data into data representative of the entire population.

Question 26: BBegin by calculating the speed of the innermost well as the

circumference of travel over time = 20 x 3.14 = 62.8m/s. Calculate theoutermost well speed in the same manner = 40 x 3.14 = 125.6m/s. 125.6 –62.8 = 62.8m/s faster.

Question 27: Ce question states that the repeat experiment is identical to the rst

in all aspects apart from the result. erefore although a number of theoptions may be true like calibration bias, it would have been applied toboth experiments and therefore should not affect the result. As such thedifference in results is simply due to random chance.

END OF SECTIONMock Paper B: Section 3“Progress is made by trial and failure; the failures are generally a

hundred times more numerous than the successes; yet they are usuallyleft unchronicled.”➢ is statement aims at several aspects of science. On one hand, it aimsat scienti c method. It demonstrates that science itself is based on trial

1429

and error and that to come to the right answer we should test theoriesrepeatedly, adjusting them all the time to become more precise and morein keeping with our results. In the end, it is very rare that a theory survivesunchanged. It also stresses that the progress of science is slow andlaborious as it requires a constant string of trial an error experimentsbefore providing any results. e second component the statementaddresses is the way the scienti c progress is seen in the public and evenamongst scientists. e common perception is that only success countsand if a theory cannot be proven it is a failure. is of course is a problemsince every failure provides a new angle to start from on the hunt forsuccess. Failure becomes necessary for success to be possible.➢ Since this statement basically has two components, when arguing tothe contrary, you will have to demonstrate either that failures stand in adifferent relationship to success or that the reporting of failures is equal tothat of successes. Either is going to be difficult as the stamen itself forms aself-ful lling prophecy. You cannot disprove it, provided it has some truthto it, since you will not nd any evidence for it. So, you will have to focuson a more theoretical level to fund support to argue against the quote.➢ You can argue that failures, being part of research arte always re ectedto some extend in the presentation of data in research papers. ey willalso appear in the analysis components of any piece of research as failuresare essential for the progress of research as it narrows the eld of possibleanswers.➢ Another perspective you can approach this topic from is to separate thefailure and the success. Failure of one theory, even if it had been thought tobe correct at some point will lead to evolution of a different idea thatbuilds on the conceptual failures of the previous idea. ereby, one ideafacilitates the other and the failing of one concept will directly result in anew concept that then in turn will either remain a success or become afailure at some point down the line.

1430

➢ You can also consider the role failure plays in our society. It is generallyseen as a bad thing and as something to be avoided. is of course doesalso apply to the scienti c community. But at the same time, failure canalso provide a new stepping stone for future success, provided lessons arelearned from the cause of the failure that can then be applied for futureprojects.

“He who studies medicine without books sails an uncharted sea, buthe who studies medicine without patients does not go to sea at all.”➢ is statement aims directly at the connection between science andsoft skills when it comes to the practice of medicine. It claims thatmedicine is more than just a science that can be learned by theories alonebut has a large human component that gives the scienti c aspect ofmedicine meaning. Without the application of the theoretical knowledge,the subject studied has no value at all, at least it cannot be calledmedicine.➢ When addressing this statement, there are several tension points youshould consider. Firstly, there is the uncharted sea. is symbol has severalaspects. On one hand, it is threatening and dangerous as the sailor cannotknow where difficult streams and lurking rocks are located. On the otherhand, it also has a component of excitement and adventure, just think ofthe old explores Cook and Columbus etc. Secondly the symbol of notsailing at all. In this again, you can use the sailor metaphor to become fullyclear on what he means. Imagine a sailor that has excellent navigationalskills but lacks the courage to apply them and so wanders to the harbourevery day to stare across the sea. All his skill is wasted as he never sets footon the waves.➢ In order to argue against this stamen, you should focus on the rst part,the sailing an uncharted sea. is is because the second part holds a deeptruth that is difficult to disprove. Even when it comes to medical research,you will have to interact with patients that you draw data from for your

1431

research. However, it is simple to argue why books play a vital role inmedicine. In this case books are synonymous to all forms of theoreticallearning. e main focus here should be that of safety of the patients.Without books it is only a matter of time until the doctor makes a mistake,crashing his proverbial ship on a proverbial sandbank. Since the focus ofmedical treatment is the improvement of the patient’s condition or qualityof life, uncertainty and adventurism have no role in this. At this point it isessential to keep the text on a general level, as medical progress does alsocome from ignoring the common wisdom. Remember the theory of 4humours from the Middle Ages, if it hadn’t been for somebody breakingwith this common wisdom and basis of teaching, modern medicine wouldnever have been born...➢ If you write an essay about this topic, make sure that you have a veryclear position that will give you a good basis to argue from. Also make sureyou have fully understood the statement. Due to the use of metaphors thiscan be tricky, but on the other hand, if you have understood the question,you can use similar metaphors and they will tie in nicely with thequestion. is will then give your whole essay a smoother appearance andmake it better to read.

‘“Medicine is the restoration of discordant elements; sickness is thediscord of the elements infused into the living body”➢ To understand this statement, you have to understand Leonardo daVinci. Being an Renaissance artist and scientist, artist and architect, hehad a very varied background but also lived in the late 15th and early 16th

century which will obviously have in uenced his perception of medicine.e idea of discordant elements that are infused in the body and that haveto be rebalanced is clear evidence of that since it basically rephrases thetheory of the 4 humours that was the basis of pretty much all schoolmedicine pretty much up until the 19th century, when pathogens werediscovered and described in their properties to cause disease. In general,

1432

however, the statement is to be understood in a sense that diseaserepresents a damaging in uence on the body, who’s default is health, andthe job of medicine is to rectify this in uence to restore health.➢ When arguing against this stamen, there are several possible angles ofattack. On one hand, there is the historical aspect of the 4 humoursmentioned above. is is pretty straight forwards since you can easilydemonstrate why da Vinci would be in uenced by this theory and howthis theory was inherently false. On the other hand, you can attack theidea of corruption through diseases causing in uence by a more generaldiscussion of disease patterns. Whilst it is true that infectious diseases arecause by the insertion of pathogens into the healthy organism, there are avast amount of diseases that are not. One good example are geneticdiseases. Especially those that are inherited in a recessive pattern meaningthat parent generations must be carriers and therefore be ‘corrupted’ aswell without displaying the actual disease. If you want to go down themutation route as well, you can point out that not all change causesnegative outcomes since mutations form the basis of evolution andthereby the basis of how we as a species came to be.➢ e second point of attack to argue against da Vinci here is the role ofmedicine. Whilst it is generally true that the aim of medicine is to cure thepatient, sometimes this is either not possible due to a lack of ability of themedical profession, i.e. we just don’t have a cure, or it is not desirable sincethe risk to the patient if undergoing treatment outweighs the risk of thedisease or the bene t of treatment. Good examples here are chemotherapyin the frail and elderly. Another example is that of mutations as the motorof evolution as mentioned above.➢ To the last part of the question, this statement still holds some truth tomodern medicine, consider for example cancer that can be caused bypoisonous external in uences such as smoking or radiation, in this casethe treatment will involve on one hand the removal of the negative

1433

stimulus, if possible, and on the other hand the treatment of the negativeimpact this stimulus has left.

“Modern medicine is a negation of health. It isn’t organized to servehuman health, but only itself, as an institution. It makes more peoplesick than it heals.”➢ With this statement, some background knowledge can be very helpful.Ivan Illich was a Croatian-Austrian Priest and philosopher that livedduring the 20th century. He is generally known for is critic of theinstitutions of Western culture such as schools or in this case modernmedicine. Looking at the statement itself, it is clear, that the basic messageis that medicine has no interest in curing humans but rather prolongstheir suffering to sell them as much treatment as possible to fund its owninterests.➢ In order to argue against this stamen, it can be helpful to detectcomponents of truth in it that can then be refuted. One point that can beraised in connection to this statement is that of medicalisation. Bylabelling everything that does not conform 100% with the ideal of health inmedical terms produces a population of sick people that then requiretreatment.➢ Another point where the statement holds true is in different health caresystems such as the one in the US where maintaining a sick statusprovides continued income to the doctor and the medical professionalsinvolved in treatment. is is less of an issue in publicly fundedenvironment such as the NHS where there is a stricter regulation ofresources and therefore less option for arti cial prolongation of treatmentrequirements.➢ Arguing against the statement is fairly easy, especially when arguingfrom the perspective of the NHS. In the NHS, healthcare is provided free ofcharge for residents and there are no direct barriers in place to blockaccess to health care. is in itself proves Illich wrong since it would serve

1434

the institution to make health care a luxury item that comes with theassociated price tag.➢ Arguing that the primary duty of the doctor is not to prolong life ismore difficult, since two of the ethical pillars of the medical profession callfor doctors not to do harm and to act in the patients’ best interest, both ofwhich aim at the prolongation of life in the majority of cases. ere aresome exceptions to the prolonging life idea, and it is probably safest toapproach this part of the question from that angle as it will ensure thatyou stay on the right track and don’t end up in a direction you didn’t wantto go.➢ Limitations to the idea of prolonging life are pretty much all the casesfalling under palliative care where the idea is to remove suffering andproviding symptomatic relief rather than curing the dieses causing thesymptoms. Common examples here are cancer in the elderly that are not

t enough to undergo chemotherapy or surgery. Other examples areincurable diseases such as inoperable brain tumours etc.➢ In this question again, it helps very much to have a clear idea of whatyou think about the issue. It will make it easier to structure your answerappropriately and it will ensure that you don’t navigate yourself intouncertain waters which is fairly easy with this topic, especially when theidea of prolonging life or not is being introduced.

END OF PAPER

1435

Mock Paper C: Section 1Question 1: Dere are three different options for staying at the hotel. ey could

either pay for three single rooms for £180, one single and one double roomfor £165, or one four person room for £215.

Subtracting the cleaning cost for one night would leave:£180-(3x£12) = £144£165-(2x£12) = £141£215-£12 = £203e cheapest option is one single and one double room and they want

to stay three nights, giving £141x3 = £423.Question 2: DGlass one starts with 16ml squash and 80ml water. Glass two starts

with 72ml squash and 24ml water. 48ml is half of 96ml so 8ml squash and40ml water is transferred to glass two. Glass two now contains (8+72 =80ml squash) and (24+40 = 64ml water). Glass two now has a total of 144mland half of this is transferred to glass one. Glass one now has (40+8 = 48mlsquash) and (32+40 = 72ml water). erefore glass one has 48ml squashand glass two has 40ml squash.

Question 3: BB is the main conclusion of the argument. Options A and D both

contribute reasons to support the main conclusion of the argument thatthe HPV vaccination should remain in schools. C is a counter argument,which is a reason given in opposition to the main conclusion. Option Erepresents a general principle behind the main argument.

1436

Question 4: Bis expression can be solved by multiplying each term by (12 kmh-1 x

15 kmh-1):15d km = 6 km +12d km3d km = 6 kmd = 2 kmerefore, the bus overtakes the bike after travelling 2 km.Question 5: BFirstly, determine who will move up to set one. Terry, Bahara, Lucy and

Shiv all have attendance over 95%. Alex, Bahara and Lucy all have anaverage test mark over 92. Terry, Bahara, Lucy and Shiv all have less than5% homework handed in late. erefore, Bahara and Lucy will both moveup a set. Secondly, determine who will receive a certi cate. Terry, Bahara,Lucy and Shiv have absences below 4%. Alex, Bahara and Lucy have anaverage test mark over 89. Bahara and Shiv have at least 98% homeworkhanded in on time. erefore, only Bahara will receive a certi cate.

1437

Question 6: CQuestion 7: DI need to make 48 scones, which makes up 8 batches.8 batches would take: 35+ 7(25+10) +25 = 305 minutesI need to make 32 cupcakes, which makes up 4 batches.4 batches would take: 15+ (4x20) =95 minutesI need to make 48 cucumber sandwichesis would take (8x5) = 40 minutesAdding 305, 95 and 40 minutes is 440 minutes in total. 440 minutes is

equivalent to 7 hours and 20 minutes. Adding 7 hours and 20 minutes to10:45am leads to 6:05pm so I will be nished at 6:05pm.

Question 8: De tallest pyramid is 12m and the smallest is 6m. Subtracting the

height of the tallest pyramid from the height of the smallest pyramidleaves 6m.

Question 9: A

1438

Work out the two wages by substituting the information provided intothe formula:

Jessica’s wage is: 210 + 42 - 3.2 = 248.8Samira’s wage is: 210 + 78 - 8.8 = 279.2Subtracting 248.8 from 279.2 leave 30.4 so the difference between their

wages is £30.4.Question 10: Ce main conclusion is C. A and B both represent reasons to support

the main conclusion of the argument. Option D represents an assumptionthat is not stated in the argument but is required to support the mainconclusion that research universities should strongly support teaching.Option E is a counter argument that provides a reason to oppose the mainargument.

Question 11: DD is the main conclusion of the argument. A is a general principle of

the argument, but the argument is more speci c to the use of helmetsrather than the wider concept of danger in sport and the responsibilities ofthe governing bodies to sports players. Options B and C are reasons tosupport the main conclusion. Option E is an intermediate conclusion,which acts as support for the next stage of the argument and as a reasonto support the main conclusion.

Question 12: DQuestion 13: E

➢ Some students born in winter like English, Art and Music

1439

➢ ere is not enough information to tell whether some students born inspring like both Biology and Maths.➢ We don’t know what the students born in spring think about Art.➢ We don’t know what the students born in winter think about Biology.➢ ere is not enough information to know whether this is true or not.

Question 14: Ae main conclusion is option A that some works of modern art no

longer constitute art. B is not an assumption made by the author as themain conclusion does not rely on all modern art being ugly to be valid. C isnot an assumption because the argument does not rely on artists studyingfor decades to produce pieces of work that constitute art. is point issimply used to support the main argument. Options D and E are stated inthe argument so are not assumptions. A is an assumption because it isrequired to be true to support the main conclusion, but is not explicitlystated in the argument.

Question 15: EReducing the price of the sunglasses by 10% is equivalent to

multiplying the price by 0.9. the price of the sunglasses is successivelyreduced by 10% three times and so the price on Monday is 0.9³ the price ofthe sunglasses on Friday. 0.9³ is equal to 0.729 and so the price of thesunglasses on Monday is 72.9% of the price of the sunglasses on Friday.

Question 16: CIt is easier to write out this calculation in the following format:

1440

a b 7 –a b5 6 5From the above subtraction it is clear that b must be equal to 2

because 7 minus 2 is equal to 5, which is the unit term of the answer. It isnow possible to rewrite the calculation with 2 substituted for b:

a 2 7 –a 25 6 5From the above calculation it is possible to gauge certain facts. A must

be greater than 5 because 1 is carried over to the second term:a 12 7 –a 25 6 5It is now clear than a must be equal to 6 because 12 minus 6 is equal to

6, which is the tens value of the answer.Question 17: ELook at the at cube net and note the shapes that are adjacent to each

other. Sides that are joining on the net will be beside each other on theformed cube. Work through to deduce option E can be formed from thecube net shown.

Question 18: Ee H shape is comprised of 12 squares. e shape’s area of 588 can be

divided by 12 to give 49, which is the area of each individual square. esquare root of 49 is 7 and so the side length of each individual square is7cm. e perimeter of the shape is comprised of 26 sides and the length ofeach side is 7 so the perimeter of the shape is 182cm.

Question 19: Ee information provided about the child needs to be inserted into the

BMI formula: BMI=35 ÷ 1.2²

1441

1.2 squared is equal to 1.44 and it may be easier to work out 3500divided by 144. e answer needs to be worked out to 3 decimal places foran answer required to 2 decimal places. e answer to 3 decimal places is24.305 and so the BMI to 2 decimal places is 24.31.

Question 20: CIt is important that the information is inserted into the formula given

for calculating the BMR of a woman rather than a man:BMR= (10 x weight in kg) + (6.25 x height in cm) – (5 x age in years)

-161BMR = (10 x 80) + (6.25 x 170) – (5 x 32) – 161BMR= 800 + 1062.5 -160 -161e BMR of the woman in the question is therefore 1541.5 kcalQuestion 21: Dis time, the information needs to be inserted into the formula for

calculating the BMI of a man:BMR= (10 x weight in kg) + (6.25 x height in cm) – (5 x age in years) + 5BMR= (10 x 80) + (6.25 x 170) – (5 x 45) +5BMR= 800 + 1062.5 -225 +5e BMR of the man in the question is therefore 1642.5 kcal. e man

does little to no exercise each week. It is therefore required to multiply1642.5 by 1.2, which gives a daily recommended intake of 1971 kcal.

Question 22: BSlippery Slope describes a series of loosely connected and increasingly

worse events that lead to an extreme conclusion. A is not a aw becausethe author does not predict a series of undesirable outcomes. C is not a

aw. It is unlikely that correlation has been confused with cause if theAmerican school did not change other aspects of the school day althoughthis is not explicitly stated in the argument. D is not a aw. A circularargument assumes what it attempts to prove and this is not the case inthis argument. E is a counter argument rather than a aw. B is the aw in

1442

the argument. Just because moving start times later worked in one schoolin America does not mean that it will work in all other cases.

Question 23: DOptions A and E, if true, would weaken the argument. If the class is

more disrupted this will be detrimental to learning, as will less effectiveteaching. B does not strengthen the main conclusion, which is based onimprovement in academic achievement levels rather than activity levels. Cdoes not strengthen the argument as the school curriculum makes nodifference to the argument about the science behind teenage brains. If D istrue then it suggests that the improvement in grades is a direct effect ofthe later school starts rather than a mere correlation.

Question 24: Be main conclusion is that EnergyFirst is expected to expand its

customer base at a rate exceeding its competitors in the ensuing months.A does not directly contradict the main argument. It demonstrates a awin the argument in that it ignores the fact that other companies may bestronger in other areas and attract customers by other means. However, itdoes not serve to weaken the main argument. C does not contradict themain conclusion; EnergyFirst could still expand its customer base at thefastest rate even if there is not much competition between energycompanies. D would not weaken the argument as it refers to the rate ofnew customer intake rather than the number of new customers attracted.E, if true, would strengthen the argument because it suggests that visualadvertising would attract new customers. B would weaken the mainargument because if it were true then investing the most money inadvertising would not serve to attract the most customers.

Question 25: AOption B points out a aw in the argument, which attributes the

healthier circulatory system of vegetarians to diet, but ignores otherpotential contributory factors to a healthy circulatory system such as

1443

exercise. C is not an assumption: the health bene ts of a vegetarian andomnivorous diet are not discussed; rather the argument is centred on thenegative health rami cations. D is stated in the argument so is not anassumption and option E is a counter argument, not an assumption.Option A is required to support the main conclusion but is not stated inthe argument so is an assumption made in the argument.

Question 26: AFirst, calculate the number of hours spent ying and waiting. It takes

24 hours in total from Auckland to London, 11.5 hours from London toCalgary and 8 hours from Calgary to Boston. In total this amounts to 43.5hours of ying and waiting. Boston is 16 hours behind Auckland and sowhen Sam arrives in Boston it will be 27.5 hours ahead of 10am. e timein Boston will therefore be 13:30 pm.

Question 27: Bis question requires you to nd the lowest common multiple. is is

the product of the highest power in each prime factor category.18= 32x233= 3x1127= 33

erefore, 33, 11 and 2 need to be multiplied together which equals 594seconds between simultaneous ashes. 5 minutes or 300 seconds needs tobe subtracted from 594 in order to nd the length of time until the next

ash. e time that they will next ash simultaneously is 294 seconds.Question 28: BFirstly, calculate the number of students who play each instrument. 21

students play piano, 12 play violin and 3 play saxophone. Point 1 is truebecause the sum of 21 piano students and 12 violin students is 33, which is3 more than the total number of students in the class. erefore, at least 3students must play both piano and violin. Point 2 is true because only 12students actually play the violin so there cannot be more than 12 students

1444

playing both piano and violin. Point 3 does not have to be true becausesome of the 9 students that do not play piano may play the violin.

Question 29: D

One way of answering this question is to set out the result after eachgame:

After game 5 Lucy has £128.13, however the question is asking howmuch money Lucy gains. e difference between 128.125 and 50 is 78.13,so Lucy gains £78.13.

Question 30: COption A may explain why young drivers are involved in more

accidents but does not need to be true for the main conclusion to hold. Bwould weaken the argument if true as drivers that spend more timedriving will have a greater chance of being involved in accidents regardlessof age. D is not an assumption, but if true may weaken the argument as itattributes the accidents to unsafe cars rather than unsafe driving. E isirrelevant to the main conclusion: it does not matter whether the youngdrivers are male or female; arguably steps should still be taken to reducethe number of accidents. Option C represents an assumption that is notstated in the argument but is required to support the main conclusion.

Question 31: De total weight of all of the apples is 6 multiplied by 180g, which

equals 1080g. e highest value the heaviest apple could take would occur

1445

if all of the other 5 apples weighed the same as the lightest apple. 5multiplied by 167g, the weight of the lightest apple, is 835g. e differencebetween the weight of all of the apples (1080g) and 835g gives the highestpossible weight of the heaviest apple, which is 245g.

Question 32:In A the banner has not been ipped. In C the crescent is black and the

square is grey. In D the teardrop has not been ipped and neither has the Lshape or the black staircase shape. In E the banner is grey, the teardropand crescent are white and the L shape is grey.

Question 33: Ee number of sheep sold on day 1 was therefore 42. Since twice the

number of sheep were sold on day 2 as day 1, then the total number ofsheep sold across the two days is equal to 42 multiplied by 3, which is 126sheep.

Question 34: Be main conclusion is that we should not wait for proof of climate

changeA and D are reasons to support the main conclusionC is an analogyE is a counter argumentQuestion 35: B

1446

e mean is the sum of all of the numbers divided by the number ofterms. From the information, we know that the sum of the rst 8 numbersdivided by 8 is equal to 44 plus the sum of the rst 8 numbers all divided by10. An expression for this can be written like this:

Two equations can be derived from the above expression:10y = sum of 8 numbers +448y = sum of 8 numbersIf we subtract the second equation from the rst, we are left with:

2y=44 y=22e value of y and the average of both sets of numbers is therefore 22.

END OF SECTIONMock Paper C: Section 2Question 1 : DStatement 1 is false. Sucrose is a disaccharide formed by the

condensation of two monosaccharides (glucose and fructose)Statement 2 is false. Lactose is a disaccharide formed by condensation

of a glucose molecule with a galactose moleculeStatement 3 is true. Glucose has two isomers: alpha-glucose and beta-

glucoseStatement 4 is trueStatement 5 is trueQuestion 2 : CStatement 1 is true. High temperatures and pH extremes cause a

permanent alteration to the highly speci c shape of the active site so thatthe substrate can no longer bind and the enzyme no longer works

Statement 2 is false. Amylase is produced in the salivary glands,pancreas and small intestine

Statement 3 is trueStatement 4 is false. Bile is stored in the gall bladder, but it does travel

down the bile duct to neutralise hydrochloric acid found in the stomach

1447

Statement 5 is true. Fructose is sweeter than glucose so smalleramounts can be used in food used in the slimming industry

Question 3 : Ce combining of food with bile and digestive enzymes occurs in the

duodenum of the small intestine. In the ileum of the small intestine, thedigested food is absorbed into the blood and lymph. e digested foodthen progresses into the large intestine. In the colon, water is reabsorbed.Faeces are then stores in the rectum and leave the alimentary canal at theanus.

Question 4: CStatement 1 is trueStatement 2 is true. For example the drug curare, a South American

plant toxin which is used in arrow poison, stops the nerve impulse fromcrossing the synapse and causes paralysis and can stop breathing

Statement 3 is false. e sheath provides insulation for the nerve axonand increases the speed of impulse transmission

Statement 4 is false. e peripheral nervous system includes motorand sensory neurons carrying impulses between receptors, effectors andthe central nervous system. e CNS consists of the spinal cord and thebrain

Statement 5 is true. A re ex arc travels from sensory neuron to relayneuron to motor neuron and is an innate mechanism designed to keep theanimal safe. For example, it allows a person to quickly draw their handaway from a ame.

Question 5 : CStatement 1 is trueStatement 2 is false. e transition metals are both malleable and

ductile, they conduct heat and electricity and they form positive ions whenreacted with non-metals.

1448

Statement 3 is true. ermal decomposition is a reaction whereby asubstance breaks down into two or more other substances due to heat.When a transition metal carbonate is heated, metal oxide and carbondioxide are produced. e carbon dioxide can be collected and will turnlimewater cloudy.

An example of this reaction is: CuCO3 -> CuO + CO2

Statement 4 is false. Transition metal hydroxides are insoluble inwater

Statement 5 is trueQuestion 6 : Eere are 9 Sulphur atoms on the left so there must by 9 on the right.

erefore, the values of B and C must add to make 9. is can be written asan equation: B+C=9

It is now useful to try to balance the Oxygen atoms: 4A+36 =10+4B+4C+14

Simplify to give: 12 = 4B+4C-4AEquation 1 can now be substituted into equation 2 to give: 12 =

(4x9)-4A24 = 4AA = 6ere are 6 Potassium atoms on the left. is means that there must

also be 6 potassium atoms on the right so B must by 3. As shown inequation 1, B and C add to make 9 so C must be 6.

5 PhCH3 + 6 KMnO4 + 9 H2SO4 = 5 PhCOOH + 3 K2SO4 + 6MnSO4 + 14 H2O

Question 7: BStatement 1 is true. Males have one X chromosome so if the allele is

present they will be affected. Females have two X chromosomes so both

1449

need to be affected to be red-green colour blind as the condition isrecessive

Statement 2 is true because according to the Punnett square belowhalf of the children will have the homozygous recessive tt genotype and sowill be non-rollers.

Statement 3 is true because all of the male children will inherit an Xchromosome from the mother which will carry the colour blind allele.

Question 8: B

Question 9: A

1450

Question 10 : CQuestion 11: DStatement 1 is false because the pulmonary artery carries

deoxygenated blood from the right ventricle to the lungsStatement 2 is true. is property of the aorta allows it to carry blood

at high pressure and is why it pulsatesStatement 3 is false because the mitral valve, otherwise known as the

bicuspid valve, is between the left atrium and left ventricleStatement 4 is trueQuestion 12: De Ar of Carbon is 12, Hydrogen is 1 and Oxygen is 16. erefore, 12g

of carbon is 1 mole of carbon; 2g of H is 2 moles of hydrogen and 16g of Ois 1 mole of oxygen. e empirical formula is therefore CH2O. emolecular weight is 30 gmol-1, which goes into 120 gmol-1 exactly 4 times.e empirical formula must therefore be multiplied by 4 to obtain themolecular formula so the molecular formula is C4H8O4.

Question 13: B

To win one game, Rupert must win one squash game and one tennisgame. In order to calculate the probability one winning one game it is

1451

necessary to add the probability of winning one tennis game and losingone squash game to the probability of losing one tennis game and winningone squash game. e following calculation must be performed:

Question 14: C

e numbers can all be written as a fraction over 36:

Question 15 : EQuestion 16 : CStatement 1 is trueStatement 2 is false because infrared has a longer wavelength than

visible lightStatement 3 is trueStatement 4 is false because gamma radiation and not infrared

radiation is used to sterilise food and to kill cancer cells

1452

Statement 5 is true because darker skins contain a higher amount ofmelanin pigment, which absorbs UV light

Question 17: Bis question requires the use of the equation:P=mv where p=momentum, m=mass and v=velocitye total momentum before the collision is equal to the sum of the

momentum of carriage 1 (12000 x 5) and carriage 2 (8000 x 0), which is60,000 kg ms-1. Momentum is conserved before and after the collision sothe total momentum after the event also equal 60,000 kg ms-1. ecarriages now move together so the combined mass is 20,000kg. Using theequation again, the total momentum (60,000 kg ms-1) divided by the totalmass (20,000 kg) gives the velocity of the train carriages after the crash,which is equal to 3 ms-1.

Question 18: CStatement 1 is false. In a nuclear reactor, every uranium nuclei split to

release energy and three neutrons. An explosion could occur if all of theneutrons are absorbed by further uranium nuclei as the reaction wouldescalate out of control. Control rods that are made of boron absorb someof the neutrons and control the chain reaction.

Statement 2 is false. Nuclear fusion occurs when a deuterium andtritium nucleus are forced together. e nuclei both carry a positive chargeand consequently, very high temperatures and pressures are required toovercome the electrostatic repulsion. ese temperatures and pressuresare expensive and hard to repeat and so fusion is not currently suitable asa source of energy.

Statement 4 is true. During beta decay, a neutron transforms into aproton and an electron. e proton remains in the nucleus, whereas theelectron is emitted and is referred to as a beta particle. e carbon-14nucleus now has one more proton and one less neutron so the atomicnumber increases by 1 and the atomic mass number remains the same.

1453

Statement 5 is false. Beta particles are more ionising than gamma raysand less ionising than alpha particles

Question 19 : EQuestion 20 : EStatement 1 is trueStatement 2 is true. Decomposers in the soil break down urea and the

bodies of dead organisms and this results in the production of ammonia inthe soil

Statement 3 is trueStatement 4 is true

Question 21: AQuestion 22: Ee question is asking for which of the statements are false

1454

Statement 1 is trueStatement 2 is trueStatement 3 is false. Ionic compounds do conduct electricity when

dissolved in water or when melted because the ions can move and carrycurrent. On the other hand, solid ionic compounds do not conductelectricity.

Statement 4 is true. Alloys contain different sized atoms, making itharder for the layers of atoms to slide over each other.

Question 23: Ae equation for a circle, with centre at the origin and radius r is x2 +

y2 = r2

e equation of this circle is therefore x2 + y2 = 25Solve the problem using simultaneous equations or by drawing the

line onto the graph.x2 + (3x-5)2 =25is simpli es to 10x2 – 30x = 010x(x - 3) = 0So x=3 or x=0 where the two graphs intersectQuestion 24: DStatement 1 is false. Heat energy is transferred from hotter to colder

places by convectionStatement 2 is trueStatement 3 is true. Radiation can travel through a vacuum like spaceStatement 4 is false. Shiny surfaces are poor at re ecting and

absorbing infrared radiation and dull surfaces are good at absorbing andre ecting infrared radiation

Question 25: CStatement 1 is trueStatement 2 is false. e melting and boiling points increase as you go

down the group

1455

Statement 3 is trueStatement 4 is false. Chloride is more reactive than bromine so no

displacement reaction occursStatement 5 is trueQuestion 26: CABC and DBE are similar triangles because all of the angles are equal.

erefore:

is is the case because the side length of the small and large trianglesare in proportion to each other. Substitute the side lengths into theexpression:

DE=6cmQuestion 27: Eis question requires the use of the equation:v2=u2+2ah where v= nal velocity, u=initial velocity, a=acceleration and

h=heightFrom the information provided in the question, we know that v=0ms-1,

u=40ms-1 and a=-10ms-2. Inserting these values into the equation gives:0=1600 + 2(-10h)e maximum height reached is therefore 80m

END OF SECTIONMock Paper C: Section 3‘e NHS should not treat obese patients’Explain what this statement means. Argue to the contrary. To

what extent do you agree with the statement?

1456

e statement argues that free health care should not be given topatients with a BMI of 30 or more. is essay will consider bothperspectives before arriving at a conclusion.

ere are several arguments to support the treatment of obesity by theNHS. e rst is that it is in accordance with the de nition of a disease;namely that is reduces life expectancy, negatively impacts normal bodyfunction and can be induced by genetic factors. Obesity often has a geneticbasis, for example the melanocortin-4 receptor polymorphism and leptinreceptor de ciency, which shift the homeostatic balance towards weightgain and are associated with hyperphagia and obesity. Obesity can also bea major feature of certain syndromes such as Prader-Willi syndrome,Bardet-Biedl syndrome and Cohen syndrome. If obesity is either classi edas a disease or is an unavoidable rami cation of certain syndromes thensurely it should be treated by the NHS just the same as any other disease.

Moreover, if the NHS refuses to treat obese patients, it will becomedifficult to decide where to draw the line. Should smokers or people whodrink alcohol also be denied free health treatment and how manycigarettes or units per week should qualify? Should all obese people bedenied free health treatment or just in cases where it is not an unavoidablesecondary result of certain syndromes? Obesity is often a consequence ofmental illnesses such as depression and it may be hard to differentiatecause from effect.

On the other hand, obesity in certain cases could be considered as aself-induced condition rather than an actual illness. Individuals arguablyexercise a degree of free will and are responsible for the amount of caloriesthat they consume and the amount of exercise that they do. ere is anargument that obesity is driven by structural changes in the environmentand is a mass phenomenon in uenced by advertising and propaganda.Perhaps societal changes in the outlook towards healthy living arerequired to address the obesity problem.

1457

Many NHS organisations already ration surgery for overweightpatients and will not for example pay for joint or hip replacements forpatients with a BMI of over 30. Surely NHS funds and taxpayers money isbetter spent on people who make an effort to maintain a good level ofhealth, for instance patients who are subject to largely unpreventable andserious diseases such as certain cancers.

I would suggest that each case should be considered on an individualbasis and that obesity treatments should be included on the NHS wherethey may act to signi cantly improve the patient’s life in the longer term.

‘We should all become vegetarian’Explain what this statement means. Argue to the contrary, that

we should not all become vegetarian. To what extent do you agreewith this statement?

is statement is saying that everyone should stop eating meat. isessay will consider both perspectives before arriving at a conclusion.

Some animals are raised in poor living conditions. Circumstances canbe cramped and due to growth rate maximisation, animals can developserious joint problems. Pig tails are cut, chickens have their toenails andbeaks clipped and cows are dehorned without painkillers. e slaughterprocess can also be stressful and inhumane. Halal meat is not stunnedbefore the jugular vein is slit and death is not instantaneous. It could beconsidered unethical to kill animals for food in this way when vegetarianoptions are available. Moreover, if farmers grew crops in place of livestock,this would generate more food and potentially alleviate world hunger.

Farming meat also has environmental implications. e overgrazing oflivestock entails signi cant deforestation, which destroys natural habitatsand endangers wild species. Enteric fermentation generates hugegreenhouse gas emissions and ammonia and hydrogen sulphide leachpoisonous nitrate into the water.

1458

A vegetarian diet also has notable health bene ts. Diets high in animalprotein can cause excretion of calcium, oxalate and uric acid, whichcontribute to the development of kidney stones and gallstones.Vegetarians absorb more calcium: meat has a high renal acid contentwhich the body neutralises with calcium leached from bones, which canweaken them. A diet rich in legumes, nuts and soy proteins can improveglycaemic control in diabetics. Moreover, growing crops instead of farminglivestock can reduce antibiotic use and minimise the development ofresistance.

However, there are advantages to eating meat. Meat contains healthysaturated fats that enrich the function of the immune and nervous system.Meat is the best source of vitamin B12 required for nervous and digestivesystem function and is a better source of iron than vegetables (the bodyabsorbs 15-35% of heme iron found in meat compared to only 2-20% of thenon-heme iron found in vegetable sources). Most plants do not containsufficient levels of essential amino acids.

Moreover, a vegetarian diet can actually have negative environmentalconsequences. For example, some herbicides utilised on geneticallymodi ed crops are toxic to wild plants and animals are often killed duringharvest. Eating meat could be considered as natural rather than cruel orunethical. Moreover, the problem of world hunger could partly beattributed to economics and distribution as opposed to insufficientamounts of food.

I would argue that it is ethically acceptable to eat meat so long as it israised in a satisfactory way. It provides important nutrients, especially forgrowing children. However, it would be better if we reduced the amount ofmeat we eat in order to reduce the environmental impact of entericfermentation and deforestation.

‘Certain vaccines should be mandatory’

1459

Explain what this statement means. Argue to the contrary. Towhat extent do you agree with the statement?

Vaccines are antigenic substances derived from the infectiousmicroorganism itself that provide immunity against a disease. estatement argues that some vaccines should be compulsory. is essaywill consider both perspectives before arriving at a conclusion.

Vaccines can protect the individuals that receive them against terribledebilitating diseases. Moreover, vaccines can also protect others in thepopulation. If a certain proportion of the population are protected, herdimmunity can be achieved. is means that people who cannot bevaccinated, for instance if they are immunocompromised or undergoingchemotherapy, will not contract the disease. Vaccines can also protectlater generations. For instance, mothers vaccinated against rubella reducethe chance of their unborn children acquiring birth defects such as loss ofvision, heart defects, cataracts and mental disabilities. Some vaccineshave completely eradicated diseases for example the last case of Smallpoxoccurred in Somalia in 1977. Rinderpest, a disease of cattle, has also beeneradicated and the instance of Polio has been substantially reduced.

Although many vaccines are available on the NHS and are funded bythe taxpayer, they ultimately cost less to administer than the expenseinvolved in time off work to care for a sick child, long term disability careand medical costs.

Nonetheless, vaccines sometime have serious and occasionally fatalconsequences. About one in a million children are at risk of anaphylacticshock. e rotavirus vaccination can result in a type of bowel blockageknown as intussusception; and the DPT and MMR vaccines have beenassociated with seizures, coma and permanent brain damage. Somephysicians have raised concerns over the ingredients used in vaccinations.For example, thimerosal has been linked to autism, aluminium taken in

1460

excess can cause neurological harm and formaldehyde is a carcinogen thatcan result in coma, convulsions and death.

It could also be argued that the decision to be vaccinated shouldconstitute a personal medical decision and individuals should be allowedto exert freedom of choice. ere are also religious objections tovaccinations. For example the Amish object to vaccines and mandatoryvaccinations. e Catholic Church is also opposed to the ingredients ofcertain vaccinations. For example, the MMR vaccine is cultivated in cellsderived from two foetuses aborted in the 1960s.

However, the chance of serious side effects is incredibly small andfurthermore the ingredients in vaccines are safe in the tiny amounts used:the exposure of children to aluminium is higher in breast milk than it is invaccines. e FDA ( food and drug administration) requires vaccines to betested for up to 10 years before they are licensed and even after licensing,they continue to be monitored. In my view, the wider bene ts of vaccinesoutweigh the minimal risk of poor side effects. In addition, it could beargued that personal decisions should be restricted when they affect thehealth of others. erefore, I am supportive of certain vaccinations beingmandatory.

‘Compassion is the most important quality of a healthcareprofessional’

Explain what this statement means. Argue to the contrary. Towhat extent do you agree with the statement?

e statement argues that in careers involved in caring for the sick;kindness and empathy for the patients is the most important professionalattribute. I will provide reasons for why this might be the case, whilst alsodiscussing the importance of a sound scienti c knowledge. I will thendecide which quality I believe to be the most important.

A lack of compassion in care homes and hospitals could be held partlyresponsible for inexcusable cases of patient neglect. For example, care

1461

home members have been mocked and tortured and in hospitals, patientshave been left surrounded in their own urine and forced to drink waterfrom ower vases. Arguably this neglect has arisen from a lack of care andcompassion from the healthcare professionals. However, at the same timeit must partly be attributed to understaffing, lack of resources andtraining. Moreover, it is difficult to assess someone’s level of compassionand it is uncertain whether this is something that can actually be taught.

A greater level of compassion would lead to better diagnoses. A largeaspect of healthcare involves listening and communicating to patients. If adoctor has more empathy, patients are more likely to trust their doctorand disclose more personal information. An empathetic manner has alsobeen shown to reduce patient anxiety and lead to faster patient recovery.

On the other hand, too much empathy could actually hamperhealthcare professionals. Doctors and other health workers often require adegree of objectivity in order to make optimal decisions that may goagainst the patient’s wishes. A level of detachment would also helpprofessionals to remain calm in stressful clinical situations. Clearly, it isdesirable for doctors and other healthcare professionals to have a detailedand comprehensive medical knowledge contributing to faster diagnoses,more skilled treatments and faster recoveries.

I would argue that scienti c knowledge is the most important qualityof a doctor especially because it is a necessity in order to practicemedicine. However, compassion is also a highly important quality in ahealthcare worker and is what separates an adequate doctor or nurse froman exceptional one.

END OF PAPER

1462

FINAL ADVICE

Arrive well rested, well fed and well hydratede BMAT is an intensive test, so make sure you’re ready for it. Unlike

the UKCAT, you’ll have to sit this at a xed time (normally at 9AM). us,ensure you get a good night’s sleep before the exam (there is little pointcramming) and don’t miss breakfast. If you’re taking water into the examthen make sure you’ve been to the toilet before so you don’t have to leaveduring the exam. Make sure you’re well rested and fed in order to be atyour best!

Move onIf you’re struggling, move on. Every question has equal weighting and

there is no negative marking. In the time it takes to answer on hardquestion, you could gain three times the marks by answering the easierones. Be smart to score points- especially in section 2 where somequestions are far easier than others.

Make Notes on your EssaySome universities may ask you questions on your BMAT essay at the

interview. Sometimes you may have the interview as late as March whichmeans that you MUST make short notes on the essay title and your mainarguments after the essay. is is especially important if you’re applying toUCL and Cambridge where the essay is discussed more frequently.

AfterwordRemember that the route to a high score is your approach and

practice. Don’t fall into the trap that “you can’t prepare for the BMAT”– this

1463

could not be further from the truth. With knowledge of the test, someuseful time-saving techniques and plenty of practice you can dramaticallyboost your score.

Work hard, never give up and do yourself justice.Good luck!

1464